You are on page 1of 321

2

Writing Team
Stan Jones
Ken Couchman
Keith Slinn
Frank Morgan
Pearson Australia
(a division of Pearson Australia Group Pty Ltd)
20 Thackray Road, Port Melbourne, Victoria 3207
PO Box 460, Port Melbourne, Victoria 3207
W\VW.pearson.con1.au

Offices in Sydney, Melbourne, Brisbane, Perth, Adelaide


and associated cmnpanies throughout the world.

First published by HarperSchools


A Division of J-larperCollinsPublishcrs (Australia) Pty Ltd 1983

Reprinted by Pearson Australia


(a division of Pearson Australia Group Pty Ltd)
1998, 2000,2001, 2002, 2003, 2004, 2005,2006, 2007, 2008, 2009,2010, 2011 (twice)

All rights reserved. Except under the conditions described in the Copyright Act 1968 of
Australia and subsequent amendtnents, no part of this publication may be reproduced,
stored in a retrieval system or transmitted in any form or by any means, electronic,
mechanical, photocopying, recording or otherwise, without the prior permission of the
copyright owner.

Produced by Pearson Australia


Printed in Malaysia, VP

National Library of Australia


Cataloguing-in-Publication data

Jones, S.B. (Stanley Bruce)


3 unit mathematics. Book 2.
For secondary school students

ISBN 978-0582-81054-9

1. Mathematics. I. Couchman, K.E. (Kenneth Edgar). II Title.

510
Pearson Australia Group Pty Ltd ABN 40 004 245 943

Every effort has been tnade to trace and acknowledge copyright. However, should any
infringement have occurred, the publishers tender their apologies and invite copyright
owners to contact them.
The graph and derivative of y = ax-- the exponential
function y = ex-- differentiation of exponential func-
tions - - integration - - the logarithm function - -
differentiation and integration - - the derivative of y =
ax-- derivative of log ax-- a value for e.

Radian measure of angles --length of circular arc--


area of a sector--graphs of the trigonometric functions
- - graphical solution of equations - - differentiation
of the trigonometric functions-- integration of the trig-
onometric functions.

Chord properties of circles-- angles within a circle--


angles in a segment of a circle-- cyclic quadrilaterals
and concylic points-- arcs and angles-- chords and
arcs-- tangents to circles-- tangents and the alter-
nate segment.

The derivative applied to velocity and acceleration - -


the use of primitives with motion in a straight line - -
rates of change-- exponential growth and decay--
further growth and decay.

Parametric equations of the parabola --equation of a


chord - - tangent and normal to a parabola - - the
chord of contact - - general geometrical properties of
the parabola - - locus problems.

Ratios of sums and differences - - ratios of double


angles-- integration of sin 2 x and cos 2 x-- the angle
between two straight lines-- trigonometrical ratios in
f
terms of tan --solution of trigonometrical equations.

Review of basic results - - area under a curve - -


volume of revolution - - integration using substitution
--definite integrals using substitution.
Motion in a straight line-- acceleration as a function of
position-- equations of motion under constant accel-
eration-- vertical motion under gravity-- the motion
of a projectile-- simple harmonic motion.

Inverse algebraic functions-- an important relationship


dy dx = 1 - - .mverse tngonometnca
dx · d . . I f unctrons
. --
the derivatives of the inverse trigonometrical functions
--integration of the inverse trigonometrical functions.

Definition-- graphs of polynomial functions-- divi-


sion of polynomials-- the remainder theorem-- the
factor theorem-- roots and coefficients of polynomial
equations-- numerical estimation of roots; halving the
interval and Newton's method of approximation.

Discovering patterns-- Pascal's triangle-- the ex-


pansion of (a + x)n --the general binomial theorem
--proof of the Pascal triangle relations-- the formula
for nck --relations between coefficients.

Tree diagrams-- permutations --factorial notation


- - permutations with repetitions or conditions - -
combinations-- games of chance-- selections with
conditions - - note on binomial coefficients - - the
binomial probability distribution.
Englishman Charles Babbage's Difference Engine was Photograph courtesy of I 8 M
based on the same principles as today's computers.
This machine could generate logarithmic and
astronomical tables to six decimal places.


I
IC s
CHAPTER 19

The word exponent is often used instead of the word index. Functions where the variable is in the
index (e.g. 3x, 10 2 x) are called exponential functions. One aim of this chapter is to learn how to use
exponential functions and in particular to discover two very important functions.

Exponential graphs have a particular shape. Here is


part of the graph of y = 2x and the table of values for 8
it.

6
X -2 -1 0 1 2 3

2x 1
4 2
1
1 2 4 8 4

You can learn more about exponential graphs by 2


thinking about the questions in the following exercise.

-3 2 3

1. (a) What is the value of 2x if x = -3 and if x = -4?


(b) What is the value of2x if x = 4 and if x = 5?
(c) What is the value of 2x if x = 10?
(d) How would you describe the behaviour of the graph of y = Y as x becomes larger and
larger?
(e) Can you think of any value of x which would make 2x negative?
(f) If x approaches negative infinity, what value does 2x approach?
2. Try these similar questions about y = Y.
(a) Do you expect Y ever to be negative?
(b) What is the behaviour of 3x as x becomes smaller and smaller (x ---+ - oo)?
(c) Where does the graph of y = 3x cut they axis?
(d) If xis any positive value, say x = 4, is Y larger or smaller than 2x?
(e) If a is negative which is larger 2a or 3a7
3. Complete the following table of values for y = Y and draw a sketch graph of the curve. For this
graph the domain is limited to lxl ~ 3. If necessary use a calculator.

3 2 1 0 15 2 25 3

1 3: = I - 1 - 1 - I I 1 I I I

2
4. On a different set of axes because of the scale difference sketchy = lOX and verify that the same
fundamental properties hold, namely:
(a) 10x is never negativt;:
(b) 10x --+ 0 as x --+ - oo
(c) 10x increases r~pidly and without limit as x gets larger
(d) the graph passes through the point (0, 1).
5. Without drawing up a table of values draw a rough sketch of y = 4x merely showing vital features
of shape. What is one important point the graph passes through?
6. Draw your own graph or copy the graph of y = 2x and draw a tangent at the point (0, 1). Check
if the gradient of this tangent is about 0·7.
7. Find rough values for the gradients (slopes) of the graphs y = 3X, y = 4x andy = wx at the point
(0, 1) on each.
8. Verify from your answers and from the shape of your curves that the gradient of the tangent at
(0, 1) on y = 3x is greater than for y = 2x and similarly it increases for y = 4x and y = 10x.
Given that the gradients of y = 2x andy = Y at (0, 1) are approximately 0·7 and 1·1 respectively,
guess what the gradient of y = 2·5x at (0, 1) might be.
9. Drawupatableofvaluesandsketchy = 2·5xfromx = -1 tox = 2andestimatethegradient
at (0, 1).

Consider any function y = ax where a is a constant greater than 1. Since a0 = 1 this graph passes
through (0, 1) for any value of a. Let us try to find the gradient at this point in the way we have used
in the calculus.

Take the point P(O, 1) and let Q be a point '


Ill
II
. on the curve near P so that OM= fix and let .;.,

QR = fly. Q, being on the curve y = aX, must


have coordinates (fix, a"x)
fly QR a"x- 1
Then - = - = - - -
fix PR fix
The limit of this expression as fix --+ 0 is the
gradient at (0, 1).
We will call this gradient m and use the ideas
above to differentiate ax.

Since Q is (L1x, a"x) then QM = a"x


3
Let P(x, y) and Q(x + L!x, y + L!y) be two points on the
curve y =ax.
Now Y =ax
and y + L!y = ax+tlx
so L!y = ax+Ax - ax
=ax. aAx _ax
= ax(a 6 x- 1)
L!y _ ax(a 6 x - 1)
thus
L!x - L!x
and dy = lim L!y
dx t.x-->0 L!x
. 6
ax(a x - 1)
= ltm -----'~---'- )(

t.x-->o L!x
=a x .ltm
. (at.x-
A
t.x->0 uX
1)
1
But we have previously found that the limit lim (at.x - ) is the gradient of the curve at the point
t.x->O L!x
(0, 1) and which for any curve we call m.
So dy = max where m is the gradient at (0, 1).
dx

A number of important results follow:


1. We can differentiate y = ax if we know its gradient at (0, 1). The gradient of y = Y at (0, 1) is
0·69 approx. so for that curve Jx = 0·69 x 2x.
2. On any exponential curve the gradient at any point is proportional to the value of the function at
that point (i.e. : = my) so when the function is small it increases slowly but when the function
is large its gradient is also large.
3. Looking at the expression : = max it follows that if we could find a function ax for some a where
the gradient m at (0, 1) was 1 then we would have found a function which is unchanged upon
differentiation. Such a function can be found, it is called the exponential function and it has great
importance in mathematics.

The gradient of y = 2x at (0, 1) is approx 0·69.


The gradient of y = Y at (0, 1) is about 1·1.
Somewhere between 2 and 3 is a value e such that ex has a gradient of 1 at (0, 1).
e turns out to be 2·71828 ... approximately.

y = ex is called the exponential function.


It has the property that if y = ex then: = ex.

Calculators and books of tables have values of this function.


4
Example:
Using tables or calculator find:
(a) e2·63

Solution:
(a) 13·87377 (b) 0·49659 (c) 0·01479

1. Evaluate:
(a) eo·z (c) e-o·s (e) 5e 3
6
(b) e1·67s (d) e-3·7 (f)~
6
2. Draw the graph of y = ex for -1 ~ x ~ 3 and check that the gradient of the tangent at (0, 1)
is 1.
3. (a) Find approximately the value of the function y = ex where x = 1· 5.
(b) Find the gradient of the graph y = ex drawn above when x = 1· 5. Is the gradient approx-
imately equal to the value of the function at this point?
4. Draw a rough sketch of y = e 2 x for -2 ~ x ~ 2.

= ax at (0, 1) could be found as the limit of a ~~


11

5. Earlier we said that the gradient of y


1
as ~x
becomes small. Using the calculator or tables for values of e11 x complete this table:

~X= 0·1 0·01 0·001

ellx = 1·1 0517

ellx- 1= 0·10517

ellx- 1
1·0517
~X

Example:
Sketch the graphs of y = 2x and y = rx and compare.
Solution:

X -2 -1 0 1 2

2x 1 1 1 2 4
4 2 4

2-x 4 2 1 1 1
2 4
2
The graphs are reflections of each other in they axis.

-2 0 2

5
1. On the same axes draw the graphs of y = ex andy = e-x for -3 ~ x ~ 3.
2. By adding ordinates of the above graphs or otherwise draw graphs of y = ex + e-x and of
ex +e-x
y=
2
3. Sketch the graphs y = Y andy= rx and compare them.
4. Draw a sketch of the graph y = e- 3 x for -2 ~ x ~ 2. Where does this graph cut they axis?
On the same axes also draw y = 2e- 3 x.
5. Draw graphs of y = !ex andy = ef on the same axes.

Remember

Examples:
Differentiate with respect to x.
(a) e6x+3

Solution:
(a) y = e6x+3 (b) y = e2x3
Let y = e" where u = 6x + 3 Let y = e" where u = 2x 3
dy = dy x du dy = dy x du
dx du dx dx du dx
= e" x 6 = e" x 6x 2
= 6e6x+3 = 6x2e2x3

1. Differentiate with respect to x:


(a) e3x (d) e-4x (g) 3x + _! + e-x
X
(b) esx-6 (e) e2x + 9 (h) !(ex - e-x)
(c) e?x+s (f) ex+ 5x 2 +2 (i) 2e ?-x

2. Differentiate with respect to x:


(a) 5e 6x (c) x- e7 -x (e) e-4x2
(b) ex2 3
(d) ex + 2x (f) e.fX
3. Find second derivatives of:
(a) e2x (b) e3x+2 (c) el-x

4. If f(x) = ~(e~ + e-~), find f'(a).

5. Find the gradient of the tangent to the curve y = ex at the point (1, e).
6. Show that the tangent to the curve y = 2eX, at the point where x = 2, is parallel to the tangent
to the curve y = e2 x at the point where x = 1.
6
7. Show that the curves y = e 3 x andy = 3ex each have the same gradient where they cross they axis.
Find the equation of the tangents to each graph at these points.

8. If y = ex + e-x, show that ~J = y.

9. If y = 4e-x + 5e- 3 X, show that~;;. + 4: + 3y = 0.


10. Show that the tangent to the graph y = ex at the point where it crosses they axis is inclined to
the x axis at an angle of 45°. Hence find the angle between the tangents toy = ex andy = e-x
at the point (0, 1) which is on both graphs.

11. Show that if y = e 7 "- 2 , then Jx = 7y and also that~~ = 49y.

12. Find the equation of the tangent to the curve y = e" + 1 at the point (1, e + 1).

We consider now the use of the product and quotient rules and the application of the calculus t~
maxima and minima.
Example (i):
(a ) ·r~ . y
D 11erentmte + 1 w1t. h respect to x.
= ---
ex
2x
(b) Find first and second derivatives ofy = x 2 ex.
Solution:
ex + 1
(a) y=-- (b) y = x2ex
2x Let y = u x v where u = x2
Let y = I!_ where u = ex + 1 and v = ex
v and v = 2x dy = vdu + udv
du dv dx dx dx
v--u-
dy dx dx = ex X 2x + x 2 ex
dx v2 = (x 2 + 2x)ex
2xe" - (ex + 1) 2 d2
X
and -{- = ex(2x + 2) + (x2 + 2x)ex
4x 2 dx = ex(x 2 + 4x + 2)
2xex- 2ex- 2
4x 2
xe"- ex- 1
2x 2
Example (ii):
Find the maximum and minimum values of y = x 2 ex. (Note: Results above may be used).
Solution:
if y = x 2 ex Given the turning points found opposite we use
then from above y' = (x 2 + 2x)ex the second derivative to examine their nature
as ex is never zero then y' = 0 From above y" = ex(x 2 + 4x + 2)
if x 2 + 2x = 0 if x = 0, y" = e 0 x 2 (positive)
if x(x + 2) = 0 if x = -2, y" = e- 2 (4 - 8 + 2)
if x = 0 or x = - 2 = -2e- 2 (negative)
Substituting x = 0 and x = - 2 So the point (0, 0) is a minimum and the point
the turning points are (0, 0) and (- 2, 4e- 2 ). ( -2, 4e- 2 ) is a maximum.
7
1. Differentiate with respect to x:
ex
(a) xex (d) 3x 2 ex (g)~
X

(b) 3xex (e) xe 2 x 1


(h) ex + 1
ex+ 1
(c) e-x(x - 1) (f)~ (i)
ex e x- 1
2. Find second derivatives of:
(a) ex2 (b) xex (c) 2xe-x
5
3. Rewrite ~ in such a way that the quotient rule is not needed and differentiate with respect to x.

4. Iff(x) = (1 + 2x)e 3 X, findf'(2) andf"(O).
5. Find the equations of the tangents to the curves y = e 3 x + 2, y = e2 x + 1, andy = ex at the
points where they cross the y axis. Show that these tangents all pass through the same point on
the x axis.
6. Show that the tangent to the curve y = ex - 2x at the point (1, e - 2) passes through the origin.

7. Show that, if y = Ae -ct where A and c are constants, then ~:; - c2 y = 0.

8. (a) Find the stationary value for the curve y = t(ex + e-x) and show that it is a minimum.
(b) What happens to the value of y when x becomes large?
(c) Show that the values of y when x = 10 and when x = - 10 are the same. Also show that for
any value a the values of y when x = a and when x = -a are the same. What does this tell
you about the shape of the curve?
(d) Draw a rough sketch of the curve.
N.B. This curve is called a catenary. It is the shape of a chain or rope hanging freely.
9. A sketch of the curve y = xe-x is drawn opposite.
Check these features:
(a) Show that the curve has one stationary point
which is a maximum. Find it. (See A) A
(b) Show that the curve passes through the origin.
(c) Find the point of inflexion where the curve
changes curvature. (See B)
(d) Find the point C where x = -1.
(e) Show that the curve approaches the x axis as
x becomes large. (See point D)

10. Consider the curve y = xex. Its graph is similar to


the above but with obvious differences due to the
positive sign. c
(a) Find the first and second derivatives.
3
(b) Find any maximum or minimum values.
(c) Find any point of inflexion.
(d) Check if the curve passes through the origin or not.
(e) What happens toy if x becomes large?
(f) Is y large or small, positive or negative when
(a)x=-3 (b)x=-10?
(g) Draw a sketch of the curve.
8
We need to use a little ingenuity in integrating exponential functions. Study these examples.

Example (i):
Show that if y = e5 x+t, then t= 5e 5 x+1.

Hence find fesx+ 1dx.

Solution:
If y = e 5 x+1, then dy = 5e 5 x+t
dx

Hence J Sesx+ldx = eSx+l + C

So esx+ldx _ .!_esx+l + C.
J -5

Find r
Example (ii):
2
xex dx.

Solution:
d 2 2
Note that -(ex) = 2xex
dx
So
J 2xex dx
2
= ex + c
2

Thus r xex dx
2
=t J:
= t[ex2Ji
2
2xex dx

= !(e4 - e).

Example. (iii):
Calculate the area under the curve y = !(e 2 x + e- 2 x) and between the lines x = -1 and x = 1.
Solution:
The area is given by

fl

-1
!(e2x + e-2x)dx
As the graph is symmetrical about the y-axis this is
f
2 t(e 2x + e- 2x)dx or f
(e 2x + e- 2x)dx.

Either this integral or the original one may be


calculated.
Area = f
(e 2x + e- 2x)dx -1 0

= [te2x _ !e-2x]6
= (te 2 - !e- 2) - (t- t)
= te2 - te-2
=i= 3·627 if simplified answer is required.
9
1. Integrate with respect to x:
(a) e4x (d) e3x + e-x (g) 6e4-x
(b) e-2x (e) e-f (h) e1-6x
(c) eax+b (f) 3x2 + e2x (i) e-2x
4
2. Integrate with respect to x:
2
(a) 4xex (c) xe7x2+1 (e) xex
2
+ xe-x 2

(b) ex - xex
2 (d) xzezx3+1 (f) 2:,
e
3. Evaluate these definite integrals to 3 decimal places:
(a) f e 3xdx (b) f 2
xe-x dx

4. Find the area bounded by the curve y = eX, the x and y axes, and the line x = 2.
5. Find the area under the curve y = i(ex + e-x) from x = -2 to x = 2.
6. The diagram shows the curves y = e 2x and y = e-x.
Calculate the area under each curve from x = 0 to x = 1
and hence find the area shaded between the curves.

7. Show that the area between the curves y = ex andy = e2x from x = 0 to x = 1 is given by
(ie 2 - e + i).

8. The diagram shows part of the graphs of y = ex and


2
y = xex • Show that these graphs both pass through the
point (1, e) and that one crosses they axis at y = 0 and
the other at y = 1. Find the area shaded from x = 0 to
x=l.

10
9. The shaded region in the diagram below the curve y = ex
is rotated about the x axis. Find the volume of the solid
of revolution formed.
Hint: Remember V = n r
y 2 dx.
(1, e)

10. The region under the graph y = !(ex + e-x) and between the lines x = -2 and x = 2 is rotated
about the x axis. Find the volume of the solid of revolution formed.
11. Find the area shaded which is bounded by the curve
y = ex, the line y = e and they axis.

12. Sketch the curve y = e- 2 x. Find the area between the curve, the x axis, and the lines x = 1 and
x = 2. If this region is rotated about the x axis find the volume of the solid generated.

13. Differentiate xex and show that d~ (xex) - ex = xex. Hence prove that fxex = xex - ex + c.

Connected with the exponential function is another very important function called the logarithm
function. You have already noted that an index can be called an exponent. It can also be called a
logarithm. More particularly, if y = ax then x is called the logarithm of y to the base a (written
X= logay)

then x = loga y
then x = loge y

You may have met logarithms before, including the common logarithms to the base 10. Logarithms
to base e are called natural logarithms or Napierian logarithms after a Scotsman John Napier who
published an article on logarithms in 1614.
Follow these examples to investigate the theory behind logarithms. Remember, a logarithm is
an index.
11
Example (i):
Given 2 6 = 64 and 34 = 81 write down
(a) log 2 64 (b) log 3 81 (c) log 10 100
Solution:
(a) log 2 64 = 6 because 2 6 = 64
(b) log 3 81 = 4 because 34 = 81
(c) log 10 100 = 2 because 10 2 = 100

Example (ii):
Find (a) log 2 16 (b) logs 125
Solution:
(a) Since 16 = 2 4 (b) Since 125 = 5 3
then 1og 2 16 = 4 then logs 125 = 3

Example (iii):
Find (a) log 3 J27 (b) log~ 32
Solution:
(a) J27 = 3.[3 = 3t (b) 32 = 2S = (J'i) 10
.'.1og 3 J27 = 1·5 so logv'Z 32 = 10

Example (iv):
Prove that: (a) 1oga xy = 1oga x + loga y
X
(b) loga- = loga X - loga y
y
(c) loga x" = 11 loga X

Solution:
Let ak = x and a"' = y
(a) xy = ak x a"'
= ak+m
Hence loga xy =k + m
= loga X + loga y.
X ak k-m
(b) -=-=a
y a"'
X
I.e. loga- = k - m = loga X - loga y.
y
(c) x" = (ak)" = ak"
loga x" = kn = n loga x.

Example (v):
Given loge 3 = 1·09861 and loge 2 = 0·69315, calculate loge 6 and loge 81 to 4 decimal places.
Solution:
(a) loge 6 = loge (3 X 2) (b) log 3 81 = loge 34
= loge 3 + loge 2 = 4loge 3
= 1·09861 +0·69315 = 1·09861 X 4
= 1·79176 = 4·39444
= 1·7918 (to 4 places) = 4·3944 (to 4 places)

12
Example (vi):
What is the natural logarithm of:
(a) e3·57? (b) 1·7? (c) 170? (d) 0·17?
Solution:
(a) By definition if y = e 3 ' 5 1, then loge y = 3·57.
Note: In general loge ea =a.
(b) From tables or calculator loge 1·7 ~ 0·5306.
(c) From calculator loge 170 = 5·1358 direct. Tables usually only have logarithms of numbers from
1 to 10. Hence if using tables proceed as follows:
loge 170 = loge (1·7 X 100)
= loge 1·7 + loge 100
= loge 1·7 + 2 loge 10
~ 0·5306 + 2 X 2·3026
~ 5·1358
(d) loge0·17 ~ -1·77196 fromcalculatordirect
1·7
= loge lO if using tables
=loge 1·7- loge 10
= 0·5306 - 2·3026
= -1·772.

1. Use calculator or tables to find to 4 places:


(a) loge 5 (c) loge 1·384 (e) loge 1
(b) loge 7 (d) loge 10 (f) loge 2·7
2. Given loge 10 = 2·3026 and loge 7 = 1·9459, without tables or calculator find:
(a) loge 1000 (c) loge 70 (e) loge 0·7
(b) loge 0·01 (d) loge 7000 (f) loge 0·007
3. Given loge 2 = 0·69315, and loge 3 = 1·09861, find without tables or calculator to four decimal
places:
(a) loge 4 (c) loge 1·5 (e) loge 18
(b) loge 27 (d) loge 12 (f) loge 13-5
4. Use your knowledge of indices (powers of 2 and 3) to find:
(a) log 2 128 (b) log 3 9J3 (c) log 2 J2
5. Use the fact that e0 = 1 to find loge 1.
6. What is loga 1 for any positive value of a?
7. Let a > 0. Can you think of any number c such that ac would be negative? (Think of the graph
ofy=2x.)
8. Is the statement loga( -4) meaningful if a is greater than zero?
9. Can you think of any number z such that ez = 0? Can there be any value for loge 0?
10. Draw the graph of y = loge x for values of x from 0·1 to 6. Use the same scale on the x andy
axes. From your graph estimate the value of x for which loge x = l.
11. On the same axes as for question 10 draw the graph of y = ex but for convenience of scale draw
for -2 ~ x ~ 3. On your graph draw also the line y = x. What do you notice about your two
graphs in relation to the line y = x?
N.B. y = ex andy = loge x are called inverse functions.
13
12. On the same set of axes in each case draw the following curves. (N.B. Unless otherwise stated
log x means loge x).
(a) log x and log 3x (N.B. log 3x = log 3 + log x).
(b) log x, log (x + 2) and log x + log 2.
(c) log x and log x 2 •
13. What is the maximum domain which can be assumed for log x? Assuming this domain, what is
the range?
14. Simplify log a3 - log a2 •
15. Simplify loge (e 2 + e) - loge (e + 1).
16. Solve: (a) ex= 3 (b) e- 2 x = 4.
17. (a) Whatislog 2 64? (b) Whatis2 1og2 64 ?
18. (a) What is log 3 81? (b) What is 31og, 81 ?
19. What do you think e 1og, 2 is?
N.B. If y = ax then X =loga y
So y =ax= alog,y.
20. Evaluate: (a) e 1og 3 (c) elog 10
(b) elog 7·5 (d) elog1

21. Solve the equations


(a) e 1og x = log 2 (b) elog 3 x = 6

22 .. Show that 3 log x + 2log y -!log z = log (x~{).

dy 1
We now prove that dx = dx
dy

Taking limits as ~x -+
. 't -~y
0 l lml = ---,-----
~x-+0 ~X 't ~X
l'l m l-
~x-->0 ~Y
1
(because if ~x -+ 0 then ~y -+ 0)
. ~X
It m -
~y-->0 ~y

Thus dy = - 1- or- x
dy dx
= 1
dx dx dx dy
dy
This result can also be arrived at geometrically.
14
Given y = logex
Then this means X= eY
dx _ Y
from which dy-e.
dy 1
But (previous result)
dx = dx
dy
1
eY

X
This is a very special result.

dy
If y. = log x
e '
then -
dx
= -X1

Hence also f~dx = loge x +C

Notice the importance of these results. Up to date we have had integrals for all other powers of x
1
except-.
X

Example (i):
Find the derivative of
(a) loge (5x - 2)

Solution:
(a) y = loge (5x - 2) (b) y = loge (7x 2 + 3x)
Let y = loge u where u = (5x - 2) Let y = loge u u = 7x 2 + 3x
dy dy du dy dy du
dx = du · dx dx = du · dx
=!u 0 5 =!.
u
(l4x + 3)
5 14x +3
5x - 2 - 7x 2 + 3x
N.B. Here is a quick method you may wish to use.
dy f'(x)
If y = loge f(x) then dx = f(x).

Check that this is so using the example above.

Notation:

15
1. Differentiate with respect to x:
(a) log 3x (e) log (3x + 1) (i) log x 4
(b) log x + 2x (f) log (2x - 3) (j) 8 log 3x
(c) log x 2 (g) log (x 2 - 5) (k) 2 log x + 5 log 2x
(d) e x + log 2x
2
(h) 5x - log 2x 3 (1) t log 3x
2. Show that log (x + 2)(x - 3) is the same as log (x + 2) + log (x - 3). Perform the differen-
tiation of this function in two ways. Which is easier?

3. Differentiate y = log x + 1 with respect to x by the way you consider easier.


x+4
4. Show that log JX+9 = t log (x + 9). Hence or otherwise differentiate this function with respect
to x.

5. Differentiate with respect to x:


(a) In (2x + l)(x - 5) (c) In (x + 6) 3 () 1
(x + 1)(x - 1)
e n (x + 2)
(b) l n - -
x +1
X
(d) In JX+4 (f) In (3x + 2) 2

6. Differentiate with respect to x:


1 1
(a) lnl (b) In (2x - 1) (c) In 2x3
X

7. Differentiate these products with respect to x:


(a) x In x (b) 2x 3 In (x - 2) (c) x In x - x

8. Don't confuse y = log x 2 andy = (log x) 2 • Differentiate each with respect to x.

9. Now think of y = log (log x). This can be written as log u where u = log x. Differentiate with
respect to x.

10. Find second derivatives of:


(a) log (3x + 1) (b) xlogx- x

11. If y = log x, show that dx 2 +


d2y (dy)2
dx = 0.

12. The line x = a cuts the curves y = log x andy = log 2x at P and Q. Show that the tangents to
the curves at P and Q are parallel. Also show that the distance PQ remains constant for all values
of a.

13. Find the equation of the tangent to the curve y = In x


(a) when x = 1 (b) when x = 3.

14. At what point on the curve y = In 2x is the gradient t? Find the equation of the tangent at this
point.

15. Show that the minimum value of y = x log xis_!. Sketch the graph from x = t to x = 5.
e

16. Find the maxima, minima, or points of inflexion for the following functions and sketch a graph
for each.
(b) y = In x 1
(a) y = X ln X - X (c) y =In x +-
X X

16
17. Differentiate with respect to x:
In x (c) ex. In x
(a ) x
e
(b) e-x (d) In ex + 1
In x ex- 1
18. Remember that if x = -3, then Ixi = 3 and draw the graph of log lxl.
2
19. Write ln (x - 9) as the sum of two logarithms and hence differentiate with respect to x.
20. What is the maximum assumed domain of y = log (x + 2)? Where does the curve cross the
x axis? What is its gradient at this point? Sketch the curve.
21. For what value of x is the ratio of the natural logarithm of a number to the number itself the
greatest.
Hint: This question really asks-when is the function log x a maximum?
X

Remember
d
dx (log x) =
1
x' hence f~dx = log x + C

d
dx log f(x) =
f'(x)
f(x) ' hence fj.~j dx = log f(x) + C

The art of integrating some fractional expressions is to try to put them into the form ff'(~)) as shown
below.
Example (i):
Find f_!_dx.
2x

Solution:
l = -21fl-dx =1 +
f-dx
2x x 2
-log x C

Example (ii):
6
Find f x 2 : 7 dx.

Solution:
Notice that if f(x) = x 2 + 7, then f'(x) = 2x.
So f-P-ax
x+7
=f 3x+x 2;dx =3f x+2x 7 dx =3log(x
2 2
2
+ 7) +C.

Example (iii):
Find J4
2x
x + 3 dx.
+1
Solution:
.
We notice that
4x +2= 2.
2x +1
17
Hence we break up the fraction into two parts as follows:

4x + 3 dx = f 4x + 2 + 1dx
f 2x+l 2x+l

= J(~~ : ~ + 2x ~ 1) dx

= f 2dx + ~ f 2x ~ 1dx

= 2x + -!- log (2x + 1) + C.

1. Integrate the following with respect to x:


1 4
(a)- (c) 4x- 3 (e) 3x2 + 2 x - -X5
4x
(b) _1_ 1 3
(d) 6x - 7 (f)
x+2 2x - 1
2. Integrate these expressions with respect to x:
2x x-2 ex
(a) (e)
2
x + 1 (c) x 2 - 4x + 1 ex+ 1
(b)
X
(d)~2 (f)~
x2 - 3 1- x x4 + 3
3. Simplify, if necessary, and integrate with respect to x:
2 x2e2x2 - 1
(a) 5x + ;x - 6
X (b) (x + ~2 Y (c)
X

4. Evaluate:
e-1 1 2 X
(a)
J X+
1
e
- - dx

2
1
(c)
J0
3
- 2- - dx
X + 1
1
(b)
J1
2t + -dt
t (d)
J1 (2t - 1) dt

X + 7
+ -8- and hence show that Je+ X + 7
1
5. Show that - - can be written as 1 - - = e + 7.
x- 1 x- 1 2
x- 1

6. Show that
1
+ x = - 1 + - 2 and hence find
1 -x 1 -x
J+ 1
x dx.
1 -x

x ++ 5 dx.
. J2x
7. Fmd
4

8. Show that -1- - -1- = - 2 -


x- 1 x+ 1 x - 1
2 - . Hence or otherwise
. show that
2
- 2-
X- 1
J 3
1 - dx = -!-log 1· 5.

i
9. Find the area under the curve y = (a) from x = 1 to x = 5 (b) from x = 1 to x = e.
10. Check the area under the curve y = i from x = 1 to x = e found above, by an approximate
numerical method.
11. Find the area under the curve y = ~ from x = 1 to x = 2.
12. Sketch the curve y = 1 -f. Where does it cross the x-axis? Find the area between the curve
and the x axis and bounded by the line x = 1.
18
13. Sketch the curve y = 2 - ~. Show that it cuts the x-axis where x = t and find the area between
the curve and the x axis and from x = t to the line x = 1.
1
14. The gradient at any point (x, y) on a curve y = f(x) is - - . The curve crosses they axis at the
x+ 1
point (0, 4). Find the equation of the curve.

15. The region under the curve y = ~from x = 1 to x = e is rotated about the x axis. Find the
'./X
volume of the solid of revolution

16. Find the area under the curve y =~from x = 1 to x = 3.


X +2
17. The region under the curve y = J~x2
1
2
and bounded by the lines x = 0 and x = 1 is rotated
about the x axis. Find the volume of the solid of revolution.

18. The region between the curve y = 1 - ~and the x axis


X
and from x = 1 to x = 2 is rotated about the x axis.
Show that the volume generated is n(3 - 4 log 2) cubic
units.

19. Find the area of the region shaded between the curve
y = In x and they axis and between the lines y = 0 and
y = 1. Do it this way:
(a) Rewrite y = In x as x = eY. (e. 1)
I
(b) Integrate eY with respect toy, fromy = 0 toy = 1. I
I
I
I
I
I

20. In a similar way (i.e. by first rewriting y = In x as x = eY), find the volume of the solid of revolu-
tion formed when the region described in question 19 is rotated about they axis.
21. Find the volume of the solid of revolution when the area between the curve y = In x and the
y axis and from y = 0 to y = 2 is rotated about the y axis.
22. Sketch the graph of y = In 2x. Rewrite this equation as x equal to some function of y. Find the
volume formed by rotating the region between this curve and they axis and from y = 0 toy= 1,
about they axis.
19
So far we have only differentiated exponential expressions such as ex or ef<x). We look now at general
expressions such as ax or af<x) where a is some constant.
Remember that if a = eY then y = In a by definition.
This means that a = e1"a (which is an important result)
Now ax = (e1nay
= exlna
d(ax)
So ~=Ina. e.dna (as In a is a constant)
=In a. ax.
Example:
lfy = a 2 x, find z.
Solution:
y = a2x
Let y = a" where u = 2x
dy dy du
dx = du. dx
=In a. a". 2
= 2ln a. a 2 x.

1. Find the slope of the tangents to the curves y = 2x and y = Y when x = 0 and compare with
the values of0·7 and 1·1 estimated from graphs drawn in exercise 19.1.
2. Find the slope of the curve y = 10x when x = 2.
3. Differentiate with respect to x. Leave logarithms unsimplified.
(a) a 4 x (c) 5x . 4x
(b) (6x 2 + 1)ax (d) y
X

4. If y = sx find:. Hence find I sx dx.

5. Find I wxdx.

6. Find the area under the curve y = 2x from x = 0 to x = 3 to 2 decimal places.

Before the advent of calculators, logarithms to base 10 were very useful. Logarithms are useful because
being indices they can be added or subtracted when numbers are being multiplied or divided and most
people find the first two operations easier. But why logarithms to base 10?
It is impossible to print a table with logarithms of all numbers shown separately because of space.
Lists in tables are restricted usually to logarithms of numbers from 1 to 10. Logarithms of other
numbers can then be calculated as, for example:
log 17 =log (1·7 x 10) =log 1·7 +log 10
log 0·017 = log (1·7 --;-. 100) = log 1·7 - 2log 10
20
If logarithms are to the base 10 these transpositions are easy because log 10 10 = 1 and
log 10 100 = 2 etc.
This all raises two questions
(a) How can logarithms be changed from one base to another?
(b) How can logarithms to different bases be differentiated or integrated?

CHANGE OF BASE DIFFERENTIATION OF y = loga x


Let y =log" x Let y =log" x
Then x = aY Before differentiating rewrite this as
Taking logarithms of both sides to base b loge X
y = -- (see opposite)
1ogb x = 1ogb (aY) loge a
= y. 1ogb a dy 1
= log" x . 1ogb a (by substitution) .. dx = loge a . ~ (as loge a is a constant)

This is usually written Remember

X=--
1ogb x
logb a

and a special case is : and a special case is :

loge x_
logloX = _
loge 10

Example (i):
Use the fact that loge 8·2 = 2·104 and that loge 10 = 2·3026 to find log 10 8·2 by change of base.
Solution:
log 8·2
By change of base, loglO 8·2 = 1 e 1
oge 0
2·104
Hence loglo 8·2 = 2·3026
= 0·914.
N.B. As a check log 10 8·2 = 0·914 to 3 places from calculator direct or from common logarithm
tables.

Example (ii):
Differentiate log 10 (4x - 1) with respect to x.
Solution:
y = log 10 (4x- 1)
= log 10 u where u = (4x - 1)
dy_dy du
dx- du · dx
=-1-.1.4
loge 10 u
4
loge 10. (4x- 1)'
21
1. Use tables or calculator to find the following logarithms to base e:
(a) log 1·95 (b) log 5·803 (c) log 9·61
2. Using the change of base rule, change each logarithm above to an equivalent logarithm with a
base of 10.
3. Show that log 10 x = 0·4343 ... x loge x.
4. Find the following logarithms to base e and then in each case change them to base 10. Answers
to 3 decimal places.
(a) log 84 (b) log 0·059 (c) log 10
5. Differentiate with respect to x (leave unsimplified).
(a) log 10 3x (c) log 10 x (e) log 10 (5x 2 - 7x)
X
(b) 2x 2 log 10 x (d) log 10 (x 2 )

6 . Show th a t !!_
d (1 og 10 x ) -- 0·4343 ...
X X
1
7. Prove that loge 10 = - - - . Look up both loge 10 and log 10 eon a calculator or in tables and
1oglo e
check that the result is correct.

Some interesting properties of the number e:


(a) The area under the curve y = _!from 1 toe is 1 square unit.
X
(b) The base for natural logarithms is e.
(c) If y = ex then k= ex (i.e. unchanged). Likewise I exdx = ex.
(d) At all points on the curve y = ex the gradient is equal to the value of the function at that point.
. dy
t.e. dx = y.

(e) If y = ekx, then;~ = ky. (i.e. the gradient is k times the value of the function at any point).
(f) The curve y = ex has a gradient of 1 where it crosses they axis.
(g) e = lim
u--+oo
(1 + _!_)".
n
An approximate value can be found fore by using this result which is proved

in the next section.

Take the point P(Ax, e"'x) on the curve y = ex and join ~


P to Q(O, 1). f:
PR
Slope of PQ = RQ p (f. X, e"X)

e"'x- 1 e"x-1
Ax
R
But because the slope of the tangent at Q is 1 t.x
t.x 1
lime- =1
t.x->0 Ax 0 M
. Ax
or 11m = 1 (by reciprocal)
t.x->0 e"'x - 1

22
Now let ~ = e11x - 1 from which e11 x = (1 + ~) and Ax = loge ( 1 + ~} By substituting these
results for e11x and Ax in the line above we have:

lim
loge(l + 11 l) = 1
llx->0 1
n
or lim
n---+co
11 loge (1 +n = l) 1 (because 11 ~ oo when Ax ~ 0)
So lim loge 1 +
IJ->00 ( -1)" 1 11
=

lim 1 + -1)" = e 1
= e.
or
( n
n-+oo

use your calculator to find the value of 1 ( + n1)" when


(a) 11 = 100 (b) n = 1000 (c) n = 10 000
Can you see that for n very large the value of 1
( + n1)" approaches 2·7182818 ... which is the
approximation given by the calculator for ex when x = 1?

1. Differentiate with respect to x:


(a) e-2x (e) loge (Sx + 2)
(b) loge (1 + x) (d) (3x + 2)ex (f) lnx
X

2. Find the equation of the tangent to the curve y = e 3 x at the point where x = 1. Find where
this tangent cuts they axis.
3. Find primitives for:
(a) e2x+3 1 4x
(b) (Sx - 4) (c) x2 + 1

4. Find the area under the curve y = e2 x from x = 1 to x = 3.

5. Find r 2
(ex - e-x)dx. What is the area under the curve y = ex - e-x from X = 1 to X = 2?

6. If y
d2 + 4~
= A + Be- 41 , show that _____11_ d = 0.
dt 2 dt
d 2y dy
7. If y = e2 x + e
4
x, show that dx 2 - 6 dx + 8y = 0.

8. Differentiate with respect to x:


(a) e.fX (c) 3x ln x (e) ex'+1
(b) ef (d) ln (x; l) (f) ln-2--1
X
X

+
23
(a) tl x2: 3dx (b) r
2(5x + ~)dx
9. Find these integrals (leave answers unsimplified)

10. Show that the tangent to the curve y = ex at the point where x = l passes through the origin.
Find the area under the curve from x = 0 to x = l and also the area between the curve, the
tangent mentioned above, and they axis.
11. Find the area under the curve y = f from x = 1 to x = 5. Also show that the area under this
curve from x = l to x = e is l unit of area.
12. Find the area shaded in the diagram between the curve
y = ln x and the y axis and bounded by the lines
y = 0 (the x-axis) andy = 0·5.
Then find the volume of the solid of revolution 0.5 - - - - -
formed if this region is rotated about they axis.
Hint: Note that y = ln x can be rewritten in another
form.

13. Find the area under the curve y = i(ex + e-x) from x = 0 to x = 3.
14. Calculate the volume of revolution if the region under the curve y = ex and between the lines
x = 0 and x = 2 is rotated about the x-axis.
1
15. Prove that 1ogb a = - b. You may assume the change of base rule.
1oga
16. Sum the series log 10 2 + log 10 4 + log 10 8 + · · · to 10 terms.
17. Consider the function y = ~ .
X + 1
(a) Find any stationary values.
1
(b) By writing the curve as y = - - , show that as x becomes large y becomes very small.
x+-x1
What happens when x ---+ - oo?
(c) Sketch the curve (it is called the serpentine curve).
(d) Find the area under the curve from x = 0 to x = 2.
1
18. Sketch the graph of y = . Find the area bounded by the curve, the x axis and the lines
2x - 1
x = land x = 3.

19. By noticing that_!__ - _!__ = ~ find


4 ~
Jx - 4
dx. (Leave answer without using
6

x- 2 x+ 2 x - 3
tables or calculator).

20. Show that :


I~ ~ dx = x + 6 loge (x + 1) + c.
21. Use numerical integration to find an approximate value for the area under the curve y = f from
x = l to x = 3 and check by formal integration.

22. Use Simpson's Rule to find the area under the curve y =_!_from x = 2 to x = 4 and check
2x
by formal integration.
24
I I

In designing the Sydney Opera House. the engineers Photograph courtesy of the Sydney Opera House Trust
made the surface of every shell part of a sphere.
CHAPTER 20

In earlier work involving trigonometry the degree was used as a unit of measurement. It was a con-
venient unit for practical, numerical measurement but when dealing with trigonometric functions
a new unit, called the radian is more appropriate. On your calculator there is a selector switch which
may be set to radians or degrees.
In a unit circle, with centre 0, two radii OP, OQ are drawn so that:
the arc length PQ = radius OP = radius OQ
= 1 unit.
The angle POQ subtended at the centre by the unit arc PQ is defined
to have a measure of one radian. P

: A radian is the measure of the angle which an arc of unit length subtends at the
of a unit circle.

If, in a circle of unit radius, an angle ROS has its vertex at the centre, its radian measure is equal to
the length of arc on which it stands.
If arc length RS = x units
then L ROS = x radians.

Examples:
(i) If the length of arc RS = 1· 3 units, then L ROS = 1· 3 radians.
(ii) If the arc RS measures 0·73 units, then LROS = 0·73 radians.
As in earlier trigonometry, if the rotation is anticlockwise the angle is taken as positive; if clockwise
as negative. Hence in the unit circles below:
Q

L POQ = ~radians. LPOQ = -2·25 radians.

26
In this way, any arc length, which may be measured by a real number, may be interpreted as an
angle measured in radians. When theoretical work is being done with the trigonometrical functions,
the radian measure is always used. The degree measure is still of vital importance in practical situations
such as carpentry and surveying.

It is important to be able to convert between radians and degrees. This may be done by means of
tables or on some scientific calculators there is a key to automatically carry out the conversion.
However, equivalent values may be found by means of a simple relationship.
The circumference of a circle is given by
C = 2nr
where n ts· the uratwna
. . 1 num ber def'me d as the ratiO. of circumference
d. an d IS
. approximate
. 1y equa1 to
mmeter
3·14159.
In the case of the unit circle, the circumference is
C = 2n (as r = 1).
Thus, the length of the semi-circular arc PQR = n units.
If follows that the straight angle PO R measures n radians.
R

c=2n

n is the length of the semicircular arc in a unit circle and hence is the measure in radians of the
straight angle PO R.

Now the number of degrees in a complete revolution is 360 degrees and this corresponds to an arc
length of 2n units and so the radian measure of a complete revolution is 2n.

It follows that
180°
1 radian =
7t
~ 57·2958°
and 1 degree = ;O radians
1
~ 0·01745 radians.
------------ -~ -----------~---------~------- --------~~~

27
Examples:
(i) Express 90° in radians. (ii) Express -450° in circular measure.
180° = n radians 180° = n radians
1o = 1~O radians 1o = 1~O radians
. o = 90 x n _ 4500 = _ 450n
. . 90 180 180
= n ra d'mns. 5n d'mns
= --ra
2 2
(on dividing numerator and
denominator by 90)
(iii) Find the equivalent of~ radians in degrees.
n radians = 180°
n . 180°
- rad1ans = - -
3 3
= 60°.

1. Express the following angles in radians, leaving your answers in terms of n.


(a) 30° (f) 210° (k) -60°
(b) 45° (g) 300° (1) 54°
(c) 60° (h) 420° (m) -270°
(d) 120° (i) 22!0 (n) 480°
(e) 135° (j) 18° (o) 262!0
2. State in degrees the following angles measured in radians.
(a) ~ (d) i"z (g) 9"o (j) 5s" (m) 29
(b) 3n (e) 45
(h) i~ (k) ?2" (n) s3n

(c) 43" (f) 4n (i) i~ (1) i (o) - 7s"


3. Using a calculator, express in degrees (correct to four decimal places) the following angles which
are given in radians.
(a) 1·5 (c) 1·2 (e) 2·75 (g) - 3·15
(b) 2·0 (d) 0·05 (f) 0·3 (h) 6·28
4. Give the circular measure of the following angles, correct to four decimal places.
(a) 28° (c) 113° (e) 116·5° (g) 298°30'
(b) 77° (d) 137° (f) 97·35° (h) -317·652°
5. For each of the following angles, give its radian measure in terms of n and write its sine and
cosine as surds.
(a) 60° (c) 300° (e) 135°
(b) 225° (d) 120° (f) 420°
6. For each of the following angles, give its measure in degrees and evaluate its sine and tangent
ratios, leaving the answers as surds.
(a) 23" radians (c) 83" radians (e) 1 ~" radians
(b) 3n radians 3
(d) 2" radians (f) 54 radians
7. With your calculator set to radian mode, find correct to 4 decimal places :
(a) sin 1 (d) sin 0·6 (g) cos i~ (j) tan (5n - 1)
(b) cos£ (e) cos 5·8 (h) tan 3·7 (k) sec 34
(c) tan 1·2 (f) 7
tan 5" (i) sin io (1) cot 3
28
8. (a) Two angles of a triangle are~ and 23n radians. What is the measure of the third angle in degrees?
(b) Three angles of a quadrilateral have as their measures -f,! and 56n radians. Express the size
of the remaining angle in degrees.
(c) What is the size of each interior angle of the following regular polygons? Give your answers
in radians.
(i) hexagon (ii) octagon (iii) dodecagon (12 sides).
(d) The sum of two angles is 233J' radians and their difference is 133J' radians. Find the size of each
angle in degrees.

In a circle of radius r units, an angle measuring e radians stands


on an arc of length l units.
Now, as the length l, of the circular arc increases so does the
angle at the centre.
Finally, the arc becomes the circumference of the circle and e
becomes a complete revolution (2n radians).

Hence,
length of arc, I units e radians
circumference of the circle (in the same units) 2n radians
and
l e
2nr 2n
. I= 2mB
·· 2n
and I= re.

l = re.
From this formula it follows that the measure of an angle ein radians is given by
. arc length
e radtans = .
length of radms

Now consider the sector AOB (shaded in the above diagram). As the angle e increases, so does the
area of the sector. Finally, when e = 2n radians the area of the sector becomes the area of the circle
(nr 2 ).
H ence,
area of sector AOB
.
e radians
area of the c1rcle 2n radians·
Now, let the area of the sector AOB be A square units.
Then,
A e

29
In a given circle of radius r units, a sector contains an angle of 8 radians at the centre. The area
of the sector is given by

Example (i):
Find the length of the arc in a circle of radius 6 em, subtending an angle
of 0·86 radians at the centre.
Find also the area of the corresponding sector.
Solution:
l = r8
= 6 X 0·86
= 5·16
length of arc PQ = 5·16 em
A= tr 8
2

=t X 6 X 6 X 0·86
= 15-48
area of sector POQ = 15·48 cm 2 •
Example (ii):
Find the angle, in radians and degrees, subtending an arc of length 9 em in a circle of radius 8 em.
Solution:
8={
r
_ _2_
- 8
= 1·125
:. angle 8 = 1·125 radians
1·125 X 180 d
egrees
n
~ 64·4578 degrees by calculator.

Example (iii):
2
A sector of a circle, which contains an angle measuring i radians at the centre has an area 4·7059 cm •
Find the radius of the circle correct to four decimal places.
Solution:
A= tr 8 2

4·7059 = tr 2 . i
4·7059 X 2 X 6 2
------=r
n
:. r = J12 x ;·7059

:. radius ~ 4·2397 em by calculator.

Example (iv): p
In a circle of radius r, show that the minor segment (shaded),
subtending an angle of 8 radians at the centre, has an area given by
tr 2 (8- sin 8). Find (a) the area of the segment and (b) the length
of the chord PQ when r = 10 em and 8 = 45°. 0

Solution:
Area of minor segment = area of sector 0 PQ - area of triangle 0 PQ
= tr 2 8 - tr 2 sin 8 (area of triangle = tab sin C)
= tr 2 (8 - sin 8).
30
of minor segment = !r 2 (8- sin 8)

(a) 8 = 45° = i radians.


area of minor segment = !x l0 2 (!f- sin !f)
= 50(!f - 0·707107)
~ 3·91456 cm 2 by calculator.
(b) Draw OR .1 PQ.
Then L ROQ = t L POQ = i radians p

sin ROQ = ~~
RQ = 10 sin i
chord PQ = 20 sin i
~ 7·6537 em by calculator.

1. A circle has a radius of 3·5 em. Find :


(a) its area and circumference.
(b) the length of arc subtending an angle of 1·75 radians at the centre.
(c) the area of the sector formed by the arms of this angle.
2. For given circles, the following values of the radius are given together with values of() for the
angle subtended by an arc at the centre. In each case find the corresponding arc length.
(a) r = 8 em 8 = 1 radian (d) r = 10 em () = 3 radians
(b) r = 5 em 8 = 2·5 radians (e) I' = 5·8 em 8 = 135°
(c) r = 6 em 8 = 1·76 radians
3. Copy and complete the following table, giving answers correct to three decimal places:

RADIUS (r) ANGLE AT CENTRE (8) ARC LENGTH (/)

(a) 6·2cm 3 radians


1-----------
(b) 2 radians 3·6 em
1-----------
(c) 2·8 em 8·21 em
1-----------
(d) 60 degrees 4·82 em
1-----------
(e) 4' radians
3
15·86 em
1-----------
(f) 134mm

4. The circumference of a circle is 198 em. Calculate the length of arc which subtends an angle of
120 degrees at the centre.
5. (a) Taking the radius of the earth as 6341 km, what angle would be subtended at the earth's
centre by an Equatorial arc of length 9000 km?
(b) What arc length would be subtended by an angle of 1 minute at the earth's centre?
31
6. Find the area of the sectors in a circle of radius 10 em where the angles at the centre are;
(a) 0·78 radians (d) 150 degrees (g) 34" radians
(b) 1·04 radians (e) -if radians (h) 37 degrees
(c) 60 degrees (f) 23" radians
7. A sector of a circle has an area of 28·86 cm 2 and contains a central angle of -if radians. Find:
(a) the radius of the circle (b) the arc length of the sector.
8. The area of a sector of a circle of radius 12 em is 188·5 cm 2 . Find the central angle contained in
the sector in :
(a) radians (b) degrees.
9. Complete the following table for a given sector of a circle:

RADIUS ANGLE AT CENTRE AREA

(a) 15 em 90 degrees
--------- ----------------- ----------
(b) 5·6 em -!f radians
--------- ----------------- ----------
(c) 6·4cm radians 47·9 cm 2
t--------- ----------------- ----------
(d) 10m radians 26·18 cm 2
t--------- ----------------- ----------
5
(e) 6" radians 61·14 cm 2
1--------- - - - - - - - - - - - - - - - - - - - - - - - - - - -
(f) 1·762 radians 88·10 cm 2
1--------- ----------------- ----------
2
(g) 2·8m 3" radians
--------- ----------------- ----------
(h) 63 degrees 48·06 cm 2
--------- ----------------- ----------
(i) 7·14 em degrees 100 cm 2

10. A pendulum of length 1·2 m swings through an angle of i radians. What is:
(a) the length of arc traced out by the end of the pendulum?
(b) the area of the sector formed by the swing of the pendulum?
11. What is the area of the sector of a circle of radius 5 em, if the length of arc bounding the sector is :
W5~? ~3~? ~9~?

12. Find the area of the minor segment in a circle with:


(a) radius 4 em and subtending an angle of 1·32 radians at the centre
(b) radius 5 em and subtending an angle of -j radians at the centre
(c) radius 6·2 em and subtending an angle of 135 degrees at the centre.
13. (a) What is the area of the major segment of a circle in which the minor segment subtends a
central angle of 56" radians and the radius is 7 em?
(b) What is the area of the minor segment cut off a circle of radius 10 em by a chord of length
12 em?
Check your answer by the approximate formula
area of minor segment ~ Ubase) x height.
14. A circular piece of wire has a radius of 12 em. This is cut and bent to form an arc of a circle whose
radius is 60 em. Find the angle subtended at the centre by this arc. Give your answer to the nearest
degree.
32
15. A circular pond has an area of 615·75 m 2 • It is divided into sectors, one of which contains a
central angle of 23'. Find:
(a) the length of the arc in this sector
(b) its area.
16. Find the shaded areas in the following figures where the arcs are quadrants of circles.
(a) l (b)

l
~------4cm------~

17. In an equilateral triangle with each side 8 em, circular


arcs, each of radius 2 em are drawn as shown below.
Calculat~ the total arc length of the curve.

If an angle is expressed in radian measure then the trigonometric ratios may be defined as functions
of a real variable. For example, sin xis defined as the sine of an angle of size x radians and for every
value of x there is one and only one value of sin x.
(a) The graph ofy =sin xfor -2n ~ x ~ 2n.

Period

Amplitude

- Period -~-

33
(i) The graph of y = sin xis continuous and periodic, resembling a wave motion. The period,
or wave length, is 2n, the distance between two successive crests or troughs on the curve.
It can be seen that the graph repeats at intervals of 2n.
sin x = sin (x + 2n)
(ii) The amplitude of the function is half the difference between its maximum and minimum
values. Hence the amplitude of y = sin x is 1.
(iii) y = sin xis an odd function, sin ( -x) = -sin x, and its graph has point symmetry about
the origin.
(iv) For the given domain:
-the solution to sin x = 1 is x = - 3d', -f.
-the solution to sin x = 0 is x = - 2n, - n, 0, n, 2n.

(b) The graph ofy = 2 sin xfor 0 ~ x ~ 2n.


For y = 2 sin x the period is 2n,
that is, it is the same as for
y = sin x. However the amplitude
is doubled.
What is the amplitude for
Jl 2n
y = 3 sin x?
---~-----~-----~------~-~---~(-----~~

Period

(c) The graph of y = sin 2x for 0 ~ x ~ 2n .


For y = sin 2x the amplitude
remains the same as for y = sin x.
However the period for this graph
is n, that is half the period of
y = sin x. We notice that the graph
shows two full wave motions from
0 to 2n. For the given domain, the
solution to sin 2x = 1 is x = !f, 54.

N.B. In this section of work it is sufficient to draw neat sketches of the functions which illustrate
their essential characteristics. It is not necessary to plot a great many points.

1. On the same axes draw sketch graphs of:


(a) y = sin x (b) y = 2 sin x (c) y = 3 sin x (d) y = 4 sin x
and give the amplitude and period of each function.
N.B. For functions of the form y = a sin x, a gives the amplitude of the graph.
2. What is the amplitude of the graph of:
(a) y =!sin x? (b) y = 5 sinx?
34
3. Draw sketch graphs of the following functions and give the amplitude and period for each.
(a)y=sinf (b)y=sin4x (c)y=sin3x
N.B. For functions of the form y = sin bx, the value of b affects the period but not the ampli-
tude of the graph.
The period of y = sin x is 2n.
2
The period of y = sin bx is bn.

4. What is the period of the graph of:


(a) y =sin 5x? (b) y =sin~? (c) y = sin nx?
5. Draw freehand sketch graphs of the following functions and give the amplitude and period of each.
(a) y = -sinx (b) y = 2 + sinx (c) y = 2sin2x (d) y = sin(x + }-)
6. Use the graphs drawn in question 3 to solve the following equations.
(a) sin f = 0 for the domain 0 :::;; x :::;; 2n.
(b) sin 4x = 1 for the domain 0 :::;; x :::;; n.
(c) sin 3x = 0 for the domain 0 :::;; x :::;; n.

The graph ofy =cos xfor -2n :::;; x :::;; 2n.

-J[
2n

-1

I~ Period

(i) The graph of y = cos x has the same wave shape as the graph of y = sin x. In fact if the sine
curve is translated a distance of 1- to the left it falls on the cosine curve.
(ii) The period of y = cos x is 2n and the graph repeats at intervals of 2n.
cos x = cos (x + 2n)
(iii) y = cos xis an even function, cos ( -x) = cos x, and its graph is symmetrical about they-axis.

1. What is the amplitude of the curve y = cos x?


2. Over the given domain, use the graph of y = cos x to write down the solutions to the equations:
(a) COS X = 1 (b) COS X = -1 (c) COS X = 0
3. Make freehand sketches of the following graphs. Give the period and amplitude of each.
(a) y = 2 cos x (d) y = cos 3x (g) y = -cos x
(b) y = 5cosx (e) y = 2 + cosx (h) y = cos(x + !t)
(c) y = cos 2x (f) y = cos f (i) y = 3 cos 2x
35
4. On the same axes sketch the graphs of y = cos x andy = sin x. By using the addition of ordinates
method, sketch the graph y = cos x + sin x.
5. On the same axes sketch the graphs of y = 4 cos x andy = 3 sin x from 0 ~ x ~ 2n. By adding
ordinates sketch the graph of y = 4 cos x + 3 sin x. What is the period and amplitude of the
new graph?

The graph ofy =tan xfor -2n ~ x ~ 2n.

Since tan x = sin x, tan xis not defined for cos x = 0. Hence the graph of y = tan xis discontinuous
COS X
in this domain for x = - 3:1', --I, -I, 32".
The period of y = tan x is n and it can be seen that the graph repeats at intervals of n.
tan x = tan (x + n).

Draw sketch graphs of the following functions and give the period of each.
1. y = -tanx 2. y =tan 2x 3. y =tan~

Here is the graph of y = sec x for - 2n ~ x ~ 2n. The graph of y = cos x is shown dotted.

)j ''
\

~I
I

-1 :I -n·--
__'-- _.-;:__ -- I:___
I I
I I
I I
I I
I I
I I

36
1
Since sec x = - -, the graph of y = sec x can be sketched by using reciprocal values of cos x. We
COS X
note also that sec x is not defined and its graph is discontinuous when cos x = 0. That is, when
x = - 3{, -1, 1, 32 the graph of y = sec x has vertical asymptotes.
The range of values for y = cos x is - 1 .!( y .!( 1.
The range of values for y = sec x is y .!( -1 andy ~ 1.
The period of y = sec x is ln.

1. (a) Sketch the graph of y = sin x for - ln .!( x .!( ln.


(b) For what values of xis cosec x not defined?
(c) Sketch the graph of y = cosec x on the same axes as used for y = sin x.
l. (a) For what values of xis cot x not defined?
(b) Draw in the asymptotes and sketch the graph of y = cot x for - ln .!( x .!( ln.
3. What is the period of the curve y = sec lx?

Example:
Find the graphically the number of solutions of the equation sin x = -f.
Solution:
The diagram shows the graphs y = sin x and
y = f drawn on the same axes.
The graph of y = f was drawn through (1, £)
and (0, 0).
The two graphs intersect in three points, show-
ing that the equation sin x = 1 has three solutions.

1. Show graphically that the equation


(a) cos x = x has one solution
(b) cos x = <f has three solutions.
l. Sketch the graphs y = sin lx and y = 1 and find how many solutions there are to the equation
sin lx =-f.
3. The graphs of y =sin x, y = -f, y = -f andy = i
are given. Read off approximate solutions to the
following equations in the domain 0 .!( x .!( n:
(a) sin x = 1
(b) sin x = -f
(c) sin x = i

X
0 2 3

37
4. (a) On the same axes, draw accurate graphs of y = 2 sin x, y = x, y = ~' y = 1 in the domain
0 :( x :( n.
(b) From the graphs find approximate solutions to the equations
(i) 2 sin x = x (ii) 2 sin x = ~ (iii) 2 sin x = 1
5. Solve the equation sin 2x = ~in the domain 0 :( x :( n.
6. In the domain 0 :( x :( n draw the graphs of y = sin x and y = cos x. Solve the equation
sin x = cos x.

1. (a) If 3 degrees equals x radians, find x, sin x and tan x to four decimal places. What is the value
of cos x?
(b) For small x, what can you say about sin x, x and tan x?
(c) Is it true to say for small x, that cos x =io 1?
2. In the diagram, 0 is the centre of the unit circle and PT is
the tangent at P. L PO A = x radians. It can be seen that:
LOPA <sector OPA < LOPT.
Find the areas:
(a) LOPA (b) sector OPA (c) LOPT
and show that
sin x < x < tan x.
If xis small
LOPA =io sector OPA =io LOPT.

J
As seen above sin x < x < tan x.
X 1
Dividing through by sin x, we have 1 < - - < - -
sin x cos x ·
Now as x-+ 0, cos x-+ 1, hence lim~= 1.
x-+0 Slll X

If follows that lim sin x = 1.


x-+0 X

1. DERIVATIVE OF sin x
In Appendix XI we prove that

~(sin x) = cos x

38
2. DERIVATIVE OF cos x
Using the above result we now find the derivative of cos x.
Since cos x = sin ( ~ - x)
:X (cos x) = :X [sin ( ~ - x)J
= cos ( ~ - x) x (- 1) Using the function of a function rule.

=sin x x ( -1)
= -sin x.

d
dx (cos x) =

3. DERIVATIVE OF tan x
Using both the above results and the quotient rule we find the derivative of tan x.
. sin x
Smce tan x = - -
cos X
-d (t anx ) -
dx
_ -d (sin
--
dx cos x
x)
cos x. cos x - sin x. (-sin x)
cos 2 x
cos x + sin 2 x
2

cos 2 x
1
cos 2 x
= sec 2 x

Example (i):
Differentiate sin 3x.
Solution:
Let y = sin 3x, then by the function of a function rule.
dy d
dx = cos 3x x dx (3x)
= 3 cos 3x.
Example (ii):
If y = cos (2x + 5) find dx.
Solution:
Using the function of a function rule.
dx -sin (2x + 5) x :X (2x + 5)
-2 sin (2x + 5).
39
Example (iii):
Differentiate tan (3 - 5x).
Solution:
Let y = tan (3 - Sx), then by the function of a function rule.
dy = sec 2 (3 - Sx) x !!_(3 - 5x)
dx dx
= -5 sec 2 (3 - Sx).
Example (iv):
If y = ex tan x, find ~.
Solution:
Using the product rule
dy = tan x . !!_(ex) + ex. !!_(tan x)
dx dx dx
= tan x . ex + ex . sec 2 x
= ex(tan x + sec 2 x).
Example (v):
Differentiate cos 2 4x.
Solution:
Let y = cos 2 4x = (cos 4x) 2 , then by the function of a function rule
dy d
dx = 2 cos 4x x dx (cos 4x)

= 2 cos 4x [-sin 4x x ~ (4x)J


= 2 cos 4x x ( -4 sin 4x)
= -8cos4xsin4x.
Example (vi):
Find~ (log sin x).
Solution:
Let y = log sin x, then by the function of a function rule
dy = - 1- x !!_(sin x)
dx sin x dx
1
= - . - X COSX
Sill X
=cot x.
Example (vii):
3
Differentiate___;_____ with respect to x.
smx
Solution:
x3
Let y = -.-, then by the quotient rule
smx
. d ( 3) 3 d ( . )
dy Sill X . dX X - X . dX Slll X

dx sin 2 x
sin x x 3x - x 3 x cos x
2

sin 2 x
2
_ x (3 sin x - x cos x)
- sin 2 x
40
Differentiate the following with respect to x.
·ex
1. sin 6x 17. cos 3x - sin 2x 33. -.-.
Slll X

2. 3 tan x 18. tan 2x + sin 4x 34. sin 3 x


3. cos 4x 19. 2 sin (3x - 4) 35. cos 2 2x
4. sin !x 20. 4 cos(~- x) 36. log sin 2x

5. sin nx 21. x sin x 37.~


COS X

6. cos (2x + 5) 22. sin x cos x 38. log cos x


2
7. tan (3x - 1) 23. x tan x 39. e 2 x sin 2x
8. sin 2x 24. ex sin X 40. e-x tan x
9. cos (3 - 2x) 3
25. x cos 2x 41. cos (2x + ~)
esinx 2
10. 4 tan 2x 26. 42. x sin 2x
1 - sin x
11. sin bx 27. 2x sin~ 43.
1 + sin x
4x
12. cos (bx + c) 28. ex COS X 44. _e_
tan x

13. sin x- cos x 29. tan 2x cos x 45 . sin~


X

1
14. 2 cos x + 3 tan x 30. - . - 46 . .Jcos 2x
Slll X

. sin 2x
15. sin 3x + 2 cos x 31. 47
cos 3x
COS X

16. tan 1- x 32. 48. log tan 3x


tan x

The following primitives, or indefinite integrals, are inverse results of the derivatives in the previous
section.

I cos x dx = sin x + C

I sin x dx = -cos x +C

I sec 2 x dx = tan x +C

Since £_(sin ax) = a cos ax


dx
then I
cos ax dx = ~ sin ax + C.
41
This leads to the following more general results.

f
cos ax dx = !; sin ax + C
f sinaxdx =-!;cos ax+ C

sec 2 axdx =!tan ax+ C


a

Example (i):
Find the primitive function of cos 3x.
Solution:
Jcos 3x dx = t sin 3x + C
Example (ii):
Find Jsec (4x + 3)dx.
2

Solution:
Jsec 2
(4x + 3)dx =!tan (4x + 3) + C

Example (iii):
Evaluate 1~(1 + sin 2x)dx.

Solution:
J: (1 + sin 2x)dx = [x- ~cos 2xJ:
= [~- ~c-1)]- [o- ~(l)J
n 1 1
=2+2+2
-~+1
-2 0

Example (iv):
Find the area bounded by the curve y = cos x and the x-axis between x = 0 and x= ~·

Solution:
A= L~cos xdx y

n
= [sin xl02
= (1) - (0)
=1.
The area is 1 square unit.

42
1. Find the primitives (indefinite integrals) of:
(a) sin 2x (f) sin (2x + 3) (k) 1 -cos 3x
(b) cos 5x (g) cos(~ - x) (l) sin (n - x)
(c) sec 2 x (h) cos x - sin x (m) x - sin (2x - 3)
(d) sin 3x (i) sec 2 ( 4x + 3) (n) 1 + sin (~ - x)
(e) sec f
2
(j) cos 3x + sin 2x (o) sec 2 2x - sin 4x
2. Evaluate the following definite integrals.
(a) 1*sec2 xdx (d) I' (1 +cos x)dx (g) f sin n~ dx
(b) L~sinxdx (e) J;cosxdx (h) f (cos ~x)dx

(c) J: sin 2x dx (f) f: (sin x + cos x)dx (i) f-~ 2


2(x + sec x)dx

3. Find the area under the curve:


(a) y = sin x between x = 0 and x = n
(b) y = cos x between x = 32 and x = 2n
(c) y = sin 2x between x = 0 and x = i
(d) y = 2 cos 3x between x = 0 and x = ~
(e) y = 3 cos x + 4 sin x between x = 0 and x = *·
4. (a) Show that the volume of the solid formed, when the area under the curve y = sec x between
x = 0 and x = i is rotated about the x-axis, is given by
2
V = n J:sec xdx.

(b) Find this volume.


5. Differentiate sin 3 x and use the result to evaluate
t~ sin 2 x cos x dx.

1. Differentiate:
nx
(a) cos (c) tan nx (e) log cos 3x
3
(b) sin 2 x (d) e2 x sin 2x (f) sin 3x
3x
2. Find the second derivative of:
(a) sin 5x (b) cos 4x
d2y
3. If y = 3 sin x - 4 cos x show that dx 2 + y = 0.

4. Ifj(x) =sin 2x, findf'(i) andf"(O).

5. If y = sin X
0
, express X
0
in radians and hence find 1x.
43
6. Find:
(a) f sin 3x dx (b) f sec 2
4xdx (c) fcos nxdx

7. Evaluate:
(a) L;j'cos2xdx (b) t;j'(l + sec 2 x)dx (c) LJ (sin 3x - x)dx

8. If y = tan 2x 0 , find Jx. First express 2xo in radian measure.


9. Use the identity sec 2 8 = I + tan 2 8 to show that
f tan ed8 =
2
tan e- e + c.
10. Differentiate esinx and hence find f: cos X eslnx dx.

II. Differentiate tan 3 x and hence find f: tan 2 x sec 2 x dx.

12. Differentiate loge sin x and hence find f cot x dx.

13. Sketch each of the following curves and hence find the area under the curve from x = 0 to x = ;f.
(a) y = cos x (b) y = sin 2x (c) y = 3 cos x (d) y = 2 sin !x.
14. On the same diagram sketch the curves:
(a) y =sin x andy= sin 2 x
(b) y = cos x andy = cos 2 x.
15. Find the gradient of the curve y = sin x at the point where
(a) X= 0 (b) X= -J (c) x = ~

16. Sketch the graph of y = 2 cos 3x for -n ~ x ~ n. What is the period of the function 2 cos 3x?

17. If dx = cos 2x andy = I when x = ;j', find y when x = ~·


18. Sketch the curve y = 2 sin ~x from x = -1 to x = I. Find the area bounded by this curve,
the x-axis and the ordinates at x = - 1 and x = 1.
19. Find the maximum and minimum values for y = sin 2x in the domain 0 ~ x ~ n.

20. The range of a shell fired from a gun having an elevation of 8 radians is given by R =
vz sin 28
g
where V and g are constants. For what angle of elevation will the range be a maximum?

21. Use Simpson's Rule with four equal subintervals to find an approximation for t 1
tan x dx.

44
The circle in architecture
M.L.C. Plaza, Sydney.
CHAPTER 21

In what follows we assume you know the meaning of the terms circle, radius, diameter, centre, cir-
cumference, chord, arc, sector and segment. The terms major arc, and minor arc refer to arcs which
are respectively more or less than half of the circle. Note that the words radius, diameter, and cir-
cumference are used sometimes to refer to the lines themselves and sometimes to the length of them.
Remember that a circle is defined as a set of points each of which is equidistant from a given fixed
point, so it is the (boundary) line which is the circle and not the region inside it.
Concentric circles are circles with the same centre. Circles which have equal radii are equal circles.

Listed below is a series of important chord properties.

PROPERTY 1
The perpendicular from the centre of a circle to a chord bisects the chord.

PROPERTY 2
The line from the centre of a circle to the mid-point of a chord is perpendicular to the chord.

PROPERTY 3
Equal chords in equal circles are equidistant from the centre.

PROPERTY 4
Chords in a circle (or equal circles) which are equidistant from the centre are equal.
In Questions 1 to 4 of Exercise 21.1 you will prove the properties listed above.

1. In the diagram, 0 is the centre of the circle and OX j_ AB.


=
Prove that 60AX 60BX and hence prove that X is the
mid-point of AB.
2. Using the diagram of Question 1, suppose only that 0 is the
centre and that X is the midpoint of AB. Using congruence,
prove that LOXA = LOXB and hence that OX .lAB.

Use these diagrams for A


Questions 3 and 4 below. Here
0 and Hare the centres of equal
circles (i.e. AO = CH) and
OX j_ AB and HY j_ CD. c
8

46
3. Suppose in the diagram AB and CD are equal chords. What follows about AX and CY? Using
congruence prove OX= HY.
4. Again in the diagram, assume that OX = HY. Prove AX = CY and hence prove that AB = CD.
5. A corollary of Property 1 states 'the per-
pendicular bisector of a chord will pass through
the centre.'
A, Band Care three points on a circle. Use
the corollary to describe how you would find B
the position of the centre.

6. The diagram shows two concentric circles with


centre 0. ACDB is a line cutting the two circles
as shown. OX has been drawn perpendicular
toAB.
Prove (a) CX = XD
(b) AX= XB.
What follows about AC and DB? A 8

7. 0 and Hare the centres of two circles which


intersect at A and B.
N.B. AB is called the common chord.
=
Prove (a) b.OAH b.OBH
(b) LAOZ = LBOZ
and hence prove that the line of centres 0 H
bisects the common chord AB at right angles.
N.B. This is a significant result worth re-
membering.

Example (i):
A chord 10 em long is drawn in a circle of radius 6 em. How far (perpendicular distance) is the chord
from the centre of the circle?
Solution:
In the diagram, 0 is the centre and AB is the chord. Draw OX per-
pendicular to AB.
Then AX =:o !AB (perpendicular from centre bisects a
= 5cm chord)
By Pythagoras theorem:
OX 2 = OA 2 - AX 2
= 36- 25
= 11
OX= Jff em (or 3·32 em by calculator)
47
Example (ii):
Two chords, of length 12 em and 16 em are drawn parallel to each other in a circle of centre 0. If
the chords are 2 em apart, what is the radius of the circle?
Solution:
Draw OX j_ CD cutting AB in Y.
Because ABIICD
LOYB = LOXD (corresponding angles)
= 90°
0

The diagram contains two right-angled triangles 60XD and 60 YB. It is impossible to use Pytha-
goras' theorem separately in each triangle on its own as there are not enough facts. The power of
algebra is needed.

0
rem
(x+2) em

x em

8 X 6em D

Let OY= xcm


Then as YX = 2 em (data)
Then OX= (x + 2) em
Let the radius be r em.
We form two equations
from 60YB r 2 = x 2 + 8 2
from 60XD r 2 = (x + 2) 2 + 62
.'. x 2 + 64 = (x + 2) 2 + 36
x 2 + 64 = x 2 + 4x + 4 + 36
4x = 24
x=6
So OY = 6 em, OX= 8 em and r = 10 em .
.·. the radius of the circle is 10 em.

1. Find the radius of a circle which has a chord 16 em long in it which is distant 6 em from the centre.
2. A chord 3·6 em long is distant 2·4 em from the centre of a circle. What is the radius?
3. A chord drawn in a circle of radius 6· 5 em is 2· 5 em from the centre. What is the length of the
chord?
4. In a circle of radius 17 em, what is the perpendicular distance from the centre of a chord which
is 30 em long?
48
5. Two circles of radii 10 em and 6·5 em intersect in a common chord 12 em long. How far apart
are their centres? (see Exercise 21.1 no. 7)
6. Two parallel chords of a circle are 24 em and 12 em long and they are 6 em apart. What is the
radius of the circle?
7. Two circles with centres 0 and H intersect in
the common chord AB. If AB = 16 em,
OH = 21 em and AH = 17 em, find AO.

8. The common chord of two intersecting circles is 24 em long and the centres are 25 em apart.
If the radius of one circle is 15 em find the radius of the other circle.
9. Find the length of the common chord of two intersecting circles whose radii are 15 em and 13 em
and whose centres are 14 em apart.
10. In the diagram ACDB is a line cutting two concentric circles.
OX is perpendicular to AB. If the radii are 34 em and 20 em
and if OX= 16 em, find AC.
11. Suppose that in the diagram of Question 10 the radii are 51 em
and 26 em and that AC = 35 em. Find OX.
Hint: Let ex= X em.
B

12. A chord of a circle is 20 em long and is 8 em from the centre. Find the length of a chord which
is 6 em from the centre in the same circle.
13. AB and CD are chords which intersect at right angles at X in a circle of radius 7 em and centre 0.
If AB = 10 em and CD = 6 em, find OX.
14. In the diagram, 0 is the centre of the circle and OX j_ AB.
Given AB = 12 em and CX = 2 em, find the radius AO.
Hint: Let the radius be rem. What is an expression for 0 X?

Draw diagrams where necessary and set out reasons for these proofs.
1. Two concentric circles whose common centre is 0 are cut by a third circle whose centre is H.
The common chords are AB and CD. Prove ABI!CD.
49
2. Draw a circle and mark a point P within it. Construct a chord of a circle passing through P which
is bisected at P. Give brief reasons for the method.
3. Two chords AB and CD bisect each other at X. Show that this is not possible unless X is the
centre.

4. Two circles are drawn with centres 0 and H. A line


ACDB is drawn perpendicular to OH, cutting the circles
at A, Band C, D respectively. Prove AC = BD.

5. AB and CD are two equal chords of a circle whose centre


is 0. If AB and CD meet at X and if ON and OM are
drawn perpendicular to AB and CD, as shown in the
diagram, prove that NX = XM and hence that
AX= XDand CX = XB.

6. Two equal circles intersect in P and Q. A chord APB is drawn through P parallel to the line joining
the centres 0 and Hand meeting the circles in A and B. From 0 and H perpendiculars OX and
HY are drawn to AB.
(a) What type of a figure is OXYH?
(b) Prove OH = XY. What is true about OH and AB?
(c) Prove OX= HYand also that the chords AP and PB are equal.

7. Two circles intersect in P and Q and APBand CQD


are parallel chords drawn through P and Q as
shown. 0 and Hare the centres and OL l. AB and
HK l. AB.
(a) What type of figure is LKNM?
(b) Prove LK = MN.
(c) What type of figure is ABDC?
(d) What is true about AC and BD?

8. AB is a chord in a circle of centre 0 and AB is double its distance from the centre. Find the size
of LAOB.
9. ABC is an isosceles triangle in which AB = AC. A circle is drawn with A as centre cutting BC
in X and Y. Prove that BX = CY.
50
10. The diagram shows two equal circles with centres
0 and H. The line AD is such that AB = CD. OX
and HY are drawn perpendicular to AD.
(a) What type of figure is OHYX?
(b) Prove that 0 H is parallel to AD.
Hint: Remember all the tests by which a figure is
proved to be a parallelogram.

11. AB and A C are chords in a circle of centre 0. A 0 D is a diameter and L DAB = L CAD. Prove
that AB = AC.
12. Two circles, centres 0 and M, intersect in P and Q. APB is a line through P parallel to OM and
cutting the circles in A and B. Prove AB = 20M.
13. AB is the diameter of a circle centre 0 and XY is a chord
which cuts it. AC and BD are perpendiculars drawn from
A and B to XY. Also, OM is a perpendicular drawn from
0 to XY. Prove that XC = D Y.
Hint: Remember that if three parallel lines cut off equal
intercepts from one transversal then they do so on any
other transversal.

14. In the diagram 0 is the centre of the semicircle and AP and


BQ are perpendicular to any chord XY (produced). Prove
that XP = YQ.
Hint: Draw an extra line in the diagram so that the
properties of intercepts and parallel lines can be used.
15. Show that in a circle a shorter chord is always further from
the centre than a longer chord.
16. AB, CD and XYare chords in a circle with centre 0. XY
cuts AB and CD in L and M, which are the midpoints of
AB and CD. Prove that XYis greater than either AB or CD.

A number of important results can be discovered and proved with regard to various angles that can
be drawn within circles. There is one fairly crucial result upon which several others depend or from
which they can be proved. So it will be examined first.

PROPERTY 5
The angle at the centre of a circle is twice the angle at the circumference standing on the same arc.
51
It is important in proving a general result not to give a restricted diagram. Check that the three
diagrams below between them cover every case and that the argument below applies to all.

Figure 1 Figure 2 Figure 3

Data: 0 is the centre of a circle; AB is an arc and C some other point on the circumference.
Aim: To prove LAOB = 2 x LACE.
Construction: Join CO and produce to D in Figures 1 and 3. In Figure 2 mark D as a point on CO
produced.
Proof: In all diagrams let L BCD = x and LA CD = y. Note that in Figure 2 LA CD does not exist.
In all figures, because OB = OC
L 0 BC = L OCB (angles in isosceles triangle)
=x
In /':::,.QBC, exterior LBOD = LOCB + LOBC
= 2x
In a similar way it can be proved that
LAOD = 2y (excluding Figure 2)
In Figure 1, by addition, LAOB = 2x + 2y
= 2(x + y)
= 2 x LACE
In Figure 2, LAOB = 2x
= 2 x LACE
In Figure 3, by subtraction, LAOB = 2y - 2x
= 2(y- x)
= 2 x LACE
Thus, for all figures, the angle at the centre, LAOB, is twice the
angle at the circumference, LACE, standing on the same arc.

Although there were three figures above there is one case still not
covered. What if LAOB is reflex (marked min the diagram)? Is
reflex LAOB also twice LACE?
The answer is yes.
Let CO be joined and produced to Don the circumference. We
now have angles LBOD and LAOD which are not reflex.
By the previous proof
LAOD = 2 x LACD
LBOD = 2 x LBCD
by addition reflex LAOB = 2 x LACE

52
1. In the diagram 0 is the centre of the circle and LAOB = 68°.
Find LAXB and LAYB.
Note: The word 'find' implies giving a reason for your
answer.
2. In the diagram of Question 1 suppose now that
LAXB = 42°. Find LAOB. Hence find LAYB.

3. In the same diagram now suppose LAYB = 26°. Find LAOB and LAXB.

4. AB is the diameter of a circle as shown. What is the size of


LAOB? If Cis a point on the circle find LACE.

5. Look again at the diagram for Question 1. Let LAXB be X 0 • In terms of x what are the sizes of
LAOB and LAYB? What can you say about any two angles at the circumference of a circle and
standing on the same arc?
6. In the diagram of Question 1 assume LAOB = t What is the size of LAXB and LAYB?
0

7. A, B, C and Dare four points on a circle whose centre is 0.


If LAOC = 110° find LADC. What is the size of reflex
LAOC and, hence, what is the size of LABC?
8. A, B, C and D are four points on a circle of centre 0 (as in
the diagram). Given LADC = 72o find obtuse LAOC and
hence reflex LAOC. Now find LABC. What is the sum
LADC + LABC?

9. The following questions refer to a diagram similar to that of Question 7 but, of course, with
different-sized angles.
(a) If LAOC = 126°, find LADC and LABC.
(b) If LADC = 38°, find LABC.
(c) If LABC = 113°, find LADC + LABC.
(d) Suppose LADC = X Find LAOC, reflex LAOC and LABC each in terms of x. Show
0

that LABC + LADC = 180° regardless of the value of x.


(e) Can you put into words a statement that is true of the angles of a quadrilateral ABCD which
is drawn in a circle?
53
DEFINITIONS
If LAXE or LAYE as shown stand on the arc AE they are said
to be subtended by the arc AE.

As the chord AE cuts off a segment from the circle (see Dia-
gram) these angles LAXE and LAYE are said to be in the same
segment.

If AE is a diameter of a circle (see Diagram) then the segment


it cuts off is in fact a semicircle. In this case LACE is called an
angle in a semicircle.

PROPERTY 6
Angles in the same segment of a circle are equal.
Data: AE is an arc of a circle of centre 0. LAXE and LAYE
each stand on arc AE.
Aim: To prove LAXE = LAYE.
Proof: Join AO, EO.
LAXE = !LAOE (angle at centre is twice angle
at circumference)
Similarly LAYE = !LAOE.
Hence LAXE = LAYE.

PROPERTY 7
An angle in a semi-circle is a right angle.
Data: AOB is a diameter and Cis a point on the circumference.
Aim: To prove LACE = 90°.
Proof: LAOE = 180° (straight angle)
But LACE= !LAOE (angle at centre is twice angle
at circumference)
Hence LACE= 90°.
54
1. 2. 3.

AB .l PQ. PR is a diameter. 0 is the centre.


LPAB = 28°. LRSQ = 38°. LOBA = 23°.
Find LABQ. Find LPRQ. Find LACE.
4. 5. E 6.

LEAD= 42°, A C is a diameter. 0 is the centre.


LBED = 36°. LBAC = 32°, LAOB = 70°,
Find LCBE. LBEC = 28°. LOBC = 10°.
Find LCBD. Find LBAC.
7. AB and CD are two chords in a circle; they intersect at X. If LAXD = 4T and LXAC = 25°,
find LABD.
8. AOD is the diameter of a circle with centre 0 and BC is a chord parallel to AD. If LACE = 30°,
show that OB and AC are perpendicular.
9. AB and CD are two parallel chords in a circle. AD and BC intersect at E (not the centre). If
LABC = 35°, show that AE = BE.
10. AB is the diameter of a circle with radius 5 em. AC is a chord 8 em long. Find the length of BC.
11. AB and DC are chords of a circle which meet when produced at X. The centre of the circle is 0.
If LAOB = 110°, LXBC = 68° and LXCB = 80°, find LCOD.

1. OP and OQ are two radii at right angles. PR and QS are parallel


chords. Find the size of LPRQ and LRQS. Hence or otherwise
show that PS .l RQ.

55
2. Two chords of a circle AB and CD intersect at X (not the centre). Prove that the triangles ACX and
DBX are equiangular.
3. Two chords AB and CD when produced intersect outside a circle at X. Show that the triangles
AXD and CXB are equiangular.
4. A, B, C and Dare four points in order on a circle and AC and BD intersect at E. If AD = AE,
show that BC = BE.
5. AB and CD are two parallel chords in a circle and AD and BC intersect at X. Prove that AX = BX.
6. Two concentric circles have a common centre 0. OP, OQ and OR are radii of the larger circle,
cutting the smaller circle in A, Band C, respectively. Prove LPQR = LABC.
7. 8. 9.

Given A C is a diameter. APC and BPD are AP is a diameter. AQB is a


Name all pairs of equal straight lines. straight line.
angles. Prove LAQB = LDQC. Prove PB is a diameter.
10. 11. 12.
D

c ~-~

AB _l_ AD and Given AX = CX. The two circles touch at B.


CD _l_ AD. Prove ACiiDB. AB and EB are each diameters.
Prove 6.ABD =
6.A CD. Prove AD/IEC.
Hence prove ABCD is a
rectangle.
23. Two circles intersect in P and Q, A and Bare such that AQ is the diameter of one circle and BQ
is the diameter of the other. Show that the points A, B and P are collinear.

A quadrilateral which is drawn in a circle (or through whose vertices


a circle can be constructed) is described as cyclic. In fact, any figure
whose vertices lie on a circle is called cyclic. Points which lie on a circle
are called concyclic points.
56
PROPERTY 8
The opposite angles of a cyclic quadrilateral are supplementary.
Data: ABCD is a cyclic quadrilateral. 0 is the centre of the circle.
Aim: To prove LADC + LABC = 180°
and similarly LEAD + LBCD = 180°.
Proof: Join AO, OC.
LADC = ~LAOC (angle at the centre is twice
angle at the circumference)
also LABC = ~reflex LAOC (same reason)
By addition LADC + LABC = ~LAOC +~reflex LAOC B
= ~ x 360° (angles at a point add to 360°)
= 180°.
Similarly by joining BO and DO it may be proved that
LEAD+ LBCD = 180°.

PROPERTY 9
The exterior angle at a vertex of a cyclic quadrilateral is equal to the opposite interior angle.
A sketched proof:
In the diagram call LBCD = X 0

Then LDCE = (180 - x)o (BCE is a straight line)


also LEAD = (180 - x)o (opposite angles of cyclic
quadrilateral)
B
Hence L DCE = L BAD.

PROPERTY 10
If two opposite angles of a quadrilateral are supplementary then the quadrilateral is cyclic (and its 4
vertices are concyclic points).
A
Note: This proof is a good example of what is called
indirect proof The property is assumed not to be true and
a contradiction is shown to follow.
Data: Let ABCD be such that LABC + LADC = 180°.
Aim: To prove that a circle can be drawn through B
A,B, CandD.
Proof: Assume a circle drawn through A, B and C cuts
AD in E. Join CE.
Now because ABCE is a cyclic quadrilateral
LABC + LAEC = 180° (opposite angles)
But LABC + LADC = 180° (data)
So LAEC = LADC
But this is impossible because
LAEC = LADC + LECD (exterior angle of t6.ECD)
So LECD has no measure and E and D must coincide. Therefore a circle which passes through
A, Band C must pass through D.
57
1. 2. 3.

Find x, y. Find x, y. Find x, y, z, t and k.

4. 6. B

Find x, y. 0 is the centre. ABCD is a parallelogram.


Find x, y. Given LADC = 65°.
Show CE = CB.

7. The diagonals of a cyclic quadrilateral ABCD intersect at X. If LAXD = ll0°, LBDC = 35°
and LADE = 55°, draw the diagram and fill in the size of all angles.
8. ABCD is a parallelogram inscribed in a circle (i.e. the points A, B, C and D lie on a circle.) Prove
that ABCD is a rectangle.
9. Two chords of a circle AD and BC meet atE when produced outside the circle. AB and CD are
joined. Prove the triangles EDC and EAB are equiangular.
10. Two circles intersect in P and Q. APB and CQD are
straight lines. If L PAC = X 0 , find an expression for
LPBD and show that AejiBD.

11. AB and DC are parallel chords of a circle and AD and BC when produced meet outside the circle
at E. Prove EC = ED.
12. ABCD is a trapezium inscribed in a circle with AB and DC the parallel chords.
Prove: (a) LBCD = LADC
(b) AD= BC.
13. ABCD is a cyclic quadrilateral and A C bisects the angles L BAD and L BCD. Prove that A Cis a
diameter.
Hint: First prove that L ABC = L ADC.
58
14. 0 is the centre of a circle and LAOC = LABC (see
Diagram). Let LADC = X 0 and prove that x = 60.

15. In the diagram opposite, the centre of the smaller circle


0 lies on the larger circle. Let L PBQ = X 0 • Find L POQ
and LPAQ in terms of x. If APBQ is a parallelogram,
show that x = 60.

16. APB and AQC are straight lines. (see Diagram). If AP is


a diameter, show that LPBC = 90°.
17. ABC is an isosceles triangle withAB = A C. DE is drawn
parallel to BC, meeting AB and AC in D and E. Prove
that BCED is a cyclic quadrilateral.

18. AB and AC are two chords of a circle of centre 0 drawn from a point A on the circle. X and Y
are mid-points of AB and AC. Prove that the points A, X, 0 and Ylie on a circle. Where would
the centre be?

19. In the diagram prove:


(a) EFIIAD
(b) the points A, G, Hand Dare
concyclic.

59
It is fairly obvious that if you had two equal-sized apple
pies and you cut sectors which had the same angles (i.e.
LAOB = LCHD) then the sectors would.be equal in all
respects and arc AB would equal arc CD.

Again, if you had a bike wheel with 10 spokes equally


spaced around the circumference (i.e. AB = BC = CD,
etc.) then it is intuitively obvious that all the angles
( LAOB, LBOC, L COD, etc.) at the centre would be
equal (and, in fact, each 36°).

These ideas can be formalised in the following assumptions:

ASSUMPTION 1
In equal circles, if two arcs are equal then they sub tend equal angles at the centre.

ASSUMPTION 2
In equal circles, if two arcs sub tend equal angles at the centre then the arcs are equal.

COR 0 LLAR I ES (obvious consequences)


1. The above properties hold in respect of arcs or angles in the same circle.
2. Equal arcs also make equal angles at the circumference because the angles at the circumference
are respectively half the centre angles.
3. In equal circles angles subtended at the centre are proportional to the arcs on which they stand.
4. In equal circles arcs are proportional to the angles which they subtend at the centre.
N.B. Corollaries 3 and 4 are generalisations which are easily proved.
For example, in the diagram opposite, suppose that D
arc CD = twice arc AB. Then arc CD can be divided at M so that
arc CM = arc MD = arc AB. It follows that
LCOM = LMOD = LAOB and hence
LCOD = twice LAOB.
This generalises to any proportion.
A

60
Example (i):
The circumference of a circle is 20 em. What is the length of an arc in this circle which subtends an
angle of 100° at the centre?
Solution:
The total circumference subtends an angle of 360° at the centre.
So arc required 100°
circumference = 360°
arc reqmre d
0
= 100 x 20 em
360
= 5·5 em
Example (ii):
What are the sizes of each interior angle of a regular pentagon drawn in a circle?
Solution: A
As there are five equal arcs around the circle then each arc subtends
an angle of 72° at the centre and hence subtends an angle of 36° at
the circumference.
Hence in the figure each angle marked x equals 36° as each stands
on one of the equal arcs ED, DC or CB.
Hence L EDC = 108°
All angles of the pentagon are 108°.

1. A, B, C, D and E are five points equally spaced around a circle with centre 0. If AC cuts BE at
X, find LAXB.
2. Nine points, A, B, C, D ... etc. lie on a circle whose centre is 0 and the arcs AB, BC, CD ...
etc. are all equal. Find the sizes of L AOB, L ADB, L AEC and L ABC.
3. P, Q, Rand S are four equally-spaced points on a semicircle drawn on AB as diameter. 0 is
the mid-point of AB. Find the size of L SAO and L OSA and show that AS bisects L RAB and
also LPSO.
4. Lines are drawn on a clock face, joining the numerals 12 with 7 and 3 with 10. Show that these
lines are perpendicular.
5. ABC is an equilateral triangle inscribed in a circle of r~dius 5 em. Find the length of the minor
arc AB.
6. If an arc 10 em long subtends an angle of 40° at the centre what is the length of the circumference?
Also find the radius.
7. A square is inscribed in a circle and each minor are cut off by the sides is 8 em long. Find the
radius of the circle.
8. ABCD is a square and AEF is an equilateral triangle inscribed in
a circle (see Diagram). A Cis an axis of symmetry of the triangle.
Find the sizes of the angles of the triangle ABE. Also, what is E
the ratio of arc BE to arc CE?

61
9. Find the angles of a triangle formed by joining the numerals 12, 4, and 7 on a clock face.
10. In the diagram arc BC is three times larger than arc AD and
LBEC = 80°. Find LABD.
0
Hint: Let L ABD = X You may need to find other angles in

terms of x first.

11. With the same data as Question 10 find what fraction arc AB + arc CD is of the circumference.
Hint: There is no need to find the separate fractions for each arc and, indeed, it may not be
possible to do so.
12. A piece of wire is in the form of an arc of a circle of radius 50 em subtending an angle of 80° at
the centre. It is bent to form a complete circle. What is the radius of this (smaller) circle?

l. Show that two parallel chords PQ and RS in a circle


cut off equal arcs P R and QS.

s
R

2. In the diagram it is given that L ABC = L BCD.


Show that arc AB =arc CD.
Hint: Begin by considering arcs ADC and BAD.

3. Two equal circles with centres 0 and H intersect in


A and B. Prove that LAOB = LAHB. Hence what
can you say about the arcs AXB and A YB cut off on
each circle?

62
4. Two equal circles intersect in A and Band XA Yand
P A Q are straight lines through A.
Show (a) arc PX =arc QY
(b) BP = BQ (see Question 3)
(c) LPBQ = LXBY.

5. BC is a chord in a circle and X is the mid-point of the minor arc and Y is the mid-point of the
major arc cut off by BC. Show that XY is a diameter.
6. AB is a diameter and AC is a chord of a circle whose centre is 0. D is the mid-point of the arc
BC. Show that OD is parallel to AC.
7. AB and CD are two diameters of a circle and X is the mid-point of the arc BC. Prove
LXCD = LXBA.
8. Two chords AB and CD intersect within a circle at X. Prove that L AXC is equal to the sum
of the angles subtended at the circumference by the minor arcs AC and BD.
Hint: Join AD.
9. If two chords of a circle, AB and CD, meet at right angles, show that the length of the arc AC
plus the arc BD is equal to half the circumference of the circle.
Hint: See Question 8.

PROPERTY 11
Equal angles at the centre of a circle stand on equal chords.
Data: 0 is the centre of the circle and LAOB = LCOD.
Aim: To prove AB =CD.
Proof: In the 1':-.'s AOB and COD,
AO= CO (radii)
BO=DO (radii)
included LAOB =included LCOD (data)
.'. 1':-.AOB = 1':-.COD
.'. AB= CD.

PROPERTY 12
Equal chords subtend equal angles at the centre of a circle.
The proof is left to the reader.

COROLLARIES
(a) Equal chords in a circle cut off equal major and minor arcs. This is proved by noting that equal
chords subtend equal angles at the centre which in turn stand on equal arcs.
(b) Equal arcs cut off equal chords.
(c) All of the above properties or corollaries apply to chords in equal circles.
63
SPECIAL NOTE:
Angles at the centre of a circle are not proportional to the chords on which they stand.

In the diagram, suppose chords AB, CD and DE are equal.


Then arcs AB, CD and DE are equal and
LAOB = LCOD = LDOE.
Hence LCOE = 2 x LAOB A
and arc CE = 2 x arc AB
But obviously chord CE =I= 2 x chord AB
D
because CE is less than CD + DE.

1. ·Prove Property 12 using congruence.


2. AB and CD are two equal chords in a circle. Show that
LDAC = LADE and hence or otherwise that AD and CB
are parallel.

3. PQ, RS and LM are three parallel chords in a circle. Prove that 6.PRL =6.QSM.
4. AB and A C are two chords of a circle and AX bisects L BA C, meeting the circle at X. Prove
that BX = CX. Are triangles ABX and ACX congruent? Are the angles L ABX and L ACX
equal?
5. In the diagram AC bisects L BAD and BX = AD. X
Prove: (a) 6.BCX =6.ACD
(b) AC =ex
(c) LBXC = LCAD

6. AB and CD are two parallel chords of a circle. CA and DB meet, when produced, at X. AD and
BC intersect at Y.
Prove: (a) YA = YB
(b) XA = XB
(c) XY (produced if necessary) will pass through the centre.
64
.
r-----~···---------------~------------------------------·-----------------------------~~__j
~-----~- ----------~---··------~--···

DEFINITION
A tangent to a ciwle j, a line which touch" a circle in one point only. I

PROPERTY 13
A tangent is perpendicular to the radius drawn from the point of contact.
Data: PTis a tangent to a circle with centre 0. PTtouches the
circle at P.
Aim: To prove OP .1 PT.
Proof: Suppose OP is not perpendicular to PT. Draw OX
perpendicular to PT. On PT take a point E such that
XE = PX. Join OE.
Now in the 6s OPX, OEX
LOXP = LOXE (being 90° by construction) E T
PX = XE (by construction)
OX is common
=
.'. 60PX 60EX (2 sides and included angle)
.'. OP = OE
Now, as OP is the radius this would mean that E lies on the circle.
This means a contradiction exists: PT, a tangent, cuts the circle in a second point E.
Hence the assumption that L 0 P X is not 90° must be false
.'. OP .1 PT.

PROPERTY 14
A straight line drawn perpendicular to a radius at its extremity is a tangent to the circle.

COROLLARY
A line drawn perpendicular to a tangent at the point of contact will pass through the centre.
Proofs of the above properties follow by indirect proof.

PROPERTY 15
Two tangents drawn to a circle from an external point are equal.
Data: TP and TQ are tangents from T to a circle of
centre 0.
Aim: To prove TP = TQ.
Proof: The proof is left to the reader.
Hint: Prove 60PT =60QT.

I. Complete the proof of Property 15 above.


2. A tangent is drawn from a point P to a circle with radius 5 em. If P is 11 em distant from the
centre 0, find the length of the tangent PT.
65
3. In the diagram PT and PQ are tangents to a circle
with centre 0. Prove TQ j_ 0 P. (This result is worth
remembering.)
4. In the diagram of Question 3 it is given that
OP = 13 em and OT = 5 em. Let OY = x em. In
terms of x what is PY?
Hence or otherwise find TY.

5. Prove that if two tangents to a circle are parallel then the line joining their points of contact is
a diameter.
6. TP and RQ are parallel tangents to a circle with centre 0. TR is T
another tangent touching the circle at X and cutting TP and RQ
at T and R. Prove that L POT= L XOT and hence or other-
wise prove that L TOR = 90°.
N.B. in Exercise 5 above you have proved that POQ is a
straight line.

7. 0 A and 0 B are perpendicular radii. PQ is a tangent touching p


the circle at T and meeting OA and OB produced at P and Q.
From P and Q tangents PS and QM are drawn. Join OS and
OM and remember that unless you prove OS and OM to be
part of one straight line you must not just assume it to be so. s
Mark LOPT = X 0 and LOQT =yo. Label all angles in the
diagram either as X 0 or yo or some other known value. What
relationship exists between x andy? Prove PS II QM.

8. AB is a diameter of a circle and Pis a point on the circle. AX and BY are perpendiculars from
A and B to the tangent at P. Prove PX = PY.
Hint: Join P to the centre.
A
9. AB and A C are chords of the larger of two concentric circles
with centre 0 and they are tangents to the smaller circle
touching it at D and E.
Prove: (a) AD = AE
(b) AB = AC
(c) DEIIBC
(d) the points A, D, 0 and E are concyclic. c

10. If a parallelogram is circumscribed about a circle (i.e. each side is a tangent) then by listing equal
line segments or otherwise show that the parallelogram is a rhombus.
66
11. In the diagram TS is a tangent to both circles
(called a direct common tangent). 0 and Pare
the centres and PX is drawn parallel to ST.
Prove PX = ST.
If OT = 10 em, PS = 6 em and
0 P = 20 em, find TS.

12. In the diagram TS is a tangent to both circles


(called a transverse common tangent).
Given radii OTand PS are 10 em and 6 em
and that OP = 20 em, find TS.
Hint: Again draw a line from P parallel to
ST, meeting OTin X and consider 60XP.

13. Two circles cut in A and B. Tangents AT and


AS at A are perpendicular. Show that AT and
AS pass through the centres of the respective
circles.
14. If tangents to two intersecting circles are at
right angles (see Diagram for Question 13)
find an expression for the distance between
the centres in terms of the radii (r and R).

15. Two circles intersect in A and Band the two tangents at A are perpendicular. Are the two tangents
at B also perpendicular? Give proof.

In many of the exercises above two circles intersect in two points. But sometimes two circles may touch
in one point only either externally or internally. When circles touch in one point only they have the
same tangent passing through this point.

In both diagrams it is easy to show that 0, I and Tare collinear where 0 and I are the centres.
67
1. Two circles with centres 0 and I touch externally at T. Prove that 0, I and Tare collinear. Would
the proof be much the same if the circles touch internally?
2. Two circles of radii 4·5 em and 2-4 em touch externally at P. What is the distance between their
centres?
3. Two concentric circles have radii of 8·5 em and 6 em. A third circle is drawn to touch them both
(externally with the smaller circle and internally with the larger circle). What is its radius?
4. The sides of a triangle ABC are 18 em, 12 em and 10 em. Circles are drawn with centres at A, Band
C so that they just touch externally in pairs. What is the radius of each circle?
5. Two equal circles touch externally at P. PX and PY are chords in the respective circles which are
perpendicular. If the centres are A and B, show that AX is parallel to BY and that AXYB is a
parallelogram.
6. Two circles, centres 0 and /, touch externally at T. PTQ is a straight line through T terminated
by the circumferences at P and Q. Prove that tangents drawn at P and Q will be parallel.
Hint: Consider triangles OPT and !QT.
7. Two circles with centres 0 and J touch at A. J lies on the
smaller circle. AJ is produced to B (i.e. AB is a diameter).
From B a tangent BCD is drawn touching the smaller circle
at C and meeting the larger circle in D. AD is joined cutting
the smaller circle inK. AC, OC and KJ are joined.
Prove: (a) OejJAD (c) KL = LM
(b) KDCM is a rectangle (d) BC = 3 x CD.

In the diagram PT is a tangent and P A is a chord. The chord PA


divides the circle into two segments on opposite sides of PA.
With respect to the LAPT between the chord PA and the
tangent PT, the segment ACP is called the alternate segment and
the LACP is called an angle in the alternate segment.

PROPERTY 16
The angle between a tangent and a chord through the point of contact is equal to the angle in the
alternate segment.
68
Da fa: PT is a tangent to a circle with centre 0. P A is a chord and D
the L PCA is an angle at the circumference, standing on
this chord.
Aim: To prove that LAPT = LACP.
Proof: Join PO and produce to meet the circle at D. Join AD.
c
Let LAPT = X 0 •

p T

The reader is asked to complete the missing spaces in this proof.


LOFT= D (OP is radius, PTis tangent)
as LAPT = X then LOPA = D
0
(answer in terms of X
0
)

L PAD = 90° (why?)


LADP =complement of LDPA (why?)
= D (in terms of x 0 )
LACP = LADP (why?)
=D
.'. LAPT = LACP.

1. PSis a tangent meeting the circle at T. p


(a) If LPTA = 116°, find LABT.
(b) If LSTA = 65° and LPTB = 40°, find LBATand LAPT.
(c) If LPTB = 54° and LETA = 37°, find LAPT.
(d) If LTBA = 74o and LAPT = 37°, prove that AT= PT. Which
other lines are equal?

2. Two circles intersect in A and B. TPU is a tangent


touching one of the circles at P. P A Q and PBR are
straight lines. QP is produced to S.
(a) If LTPB = 64°, find LBRQ.
(b) Prove that LSPT = LAQR.
(c) Are any pairs from the lines PT, AB and QR
parallel? (Don't guess from the diagram!)

3. Two tangents from an external point P touch a circle at Sand Q. A is any point on the major
arc SQ. If LSPQ = 36°, find LSAQ.
69
4. The diagram shows a circle inscribed in a triangle A
ABC (i.e. AB, BC and CA are tangents).
If LA = 76° and LB = 40°, find the size of each
angle of L:..DEF.
5. Using the diagram for Question 4, assume that
LDEF = 66° and LDFE = 80° and find LA,
LB and LC of L:..ABC.
c

6. Two tangents T A and TB are drawn from T to touch a circle at A and B. A third tangent touches
the circle at P and cuts TA and TB in D and E. Given LPAB = 36° and LPBA = 25°, find
LATE.
7. Two circles intersect in X and Y and Pis a point on one of them. PX and PY when produced
meet the other circle in M and N. Prove that the tangent at P is parallel to MN.
8. PQ is a chord of a circle whose centre is 0. Show that the angle between the chord and a tangent
at P is half the angle L POQ.
9. Two circles intersect in A and B. At A tangents are drawn to each circle, cutting the circles at P
and Q. Prove that the triangles PAB and QAB are equiangular.
10. PA and PB are tangents from an external point P to a circle with centre 0, Sis the mid-point
of the minor arc AB. Prove that AS bisects LPAB.

70
PAPER 43
1. Differentiate the following with respect to x.
(a) (2x + 1) 5 (b) .}2x - 5 (e) ecosx (d) loge (x 2 + 1)
2. Find the value of:

(a) f (x - x 2 )dx (c) 1 1


(2 sin x + 3 cos x)dx

3. a, b, 8 are in arithmetic progression while b, a, 2lt are in geometric progression. Find a and b.
4. The adjacent sides of a parallelogram are 10 em and 15 em respectively. If the shorter diagonal
is 16 em, find the angles of the parallelogram and its area.
5. Find the stationary values and point of inflexion for the function y = 2x 3 + 3x 2 - 12x + 8.
Sketch the corresponding curve and show that it cuts the x-axis at just one point.
6. ABCD is a quadrilateral and X is the mid-point of the diagonal BD. From X parallels are drawn
to BA and BC, meeting AD and DC in Yand Z respectively. Prove that YZ is parallel to A C.

PAPER 44

1. Find: if:
1
(a) y = sin 3 x (b) y = (x 2 + 1)ex (c) y = (3x - 1) 4 (d) y = sin x cos x

d 2y dy
2. Given that - 2 = 2 - 3x 2 , and when x = 0, y = 10 and -d = - 5, find y when x = 3.
~ X

3. If A(x- 1)(x- 2) + B(x- 1) +C =2x 2


- 3x + 5, find A, Band C.
4. Find the equation of the tangent to the curve y = x 3 + ~ at the point where x = 1.
X

5. Find the area enclosed between the curve y = 1 - x 2 and the x-axis. This area is rotated about
the y-axis. Find the volume of the solid of revolution.
6. ABC is an isosceles triangle, and PQ is drawn parallel to the base BC, cutting the sides in P and
Q. Show that the four points B, C, Q and P lie on a circle.

PAPER 45
1. The gradient at any point (x, y) on a curv~ is given by 1 + 3x 2 • If the curve passes through the
point (1, 4), what is its equation?
2. Evaluate:
30
(a) I; 3n +5 leave your answer in index form.
n=1 n=1

dd ~.
2
8 6
3. If y = - 3
x, find ddy and Find the value of x for which each of the functions y, ddy and
X X X X

~~is separately zero. Find a minimum value of y for positive values of x.


71
4. In a circle of radius 12 em an arc subtends an angle of 40° at the centre. Find:
(a) the length of the arc.
(b) the area of the sector bounded by the arc and the radii to its end-points.
(c) the area of the segment bounded by the arc and the chord joining its end-points.
5. In the quadrilateral ABCD, AD = BC and X, Y, Z are the mid-points of AB, AC, DC respectively.
Prove that the triangle XYZ is isosceles.
6. A circle is drawn on one of the equal sides of an isosceles triangle as diameter. Show that the
circle passes through the mid-point of the base of the isosceles triangle.

PAPER 46
1. Find the derivatives of the following functions.
2
1 1
(a) (x 2 - 1)(x + 5) (b) x - (c) -.-3- (d) log e2 x
2x + 3 sm x
2. Find primitives for :
2
(a) 3x 2 - 2x +t (b) JX + xJX (c) 5x- 3

3. (a) Complete the table of values for the function y = )2 + x 2 •

(b) Use Simpson's Rule to evaluate f )2 + x 2 dx.


4. If IX and f3 are the roots. of x 2 - 2x - 5 = 0, find the value of:
1
(a) IX + f3 (b) IX/3 (c) + 1
IX f3
5. Two circles intersect at A and B. Through A any two straight lines PAQ and XA Yare drawn
terminated by the circumferences. Show that the arcs PX and QY subtend equal angles at B.
6. With one throw of a die, what is the probability of:
(a) throwing a 3 or a 6?
(b) throwing an even number or a number less than 4?

PAPER 47
1. The first 3 terms of an arithmetic sequence are -2, 4, 10. Find the twentieth term and the sum
of twenty terms.
2. Sketch the graph of y = sin 2x for 0 :;;::; x :;;::; n. Find the area enclosed by the curve and the
x-axis from x = 0 to x = .q..
3. (a) Find the solution to the equation 2x 4 - 5x 2 - 7 = 0.
(b) If 3x 2 + 5x + 4 =
(x + 1)(ax + b) + c(x 2 + 1), find a, band c.
4. Two straight roads XY and XZ diverge from X at an angle of 37 degrees. Car A travels at 60 km/h
along road XY and car B travels at 72 km/h along road XZ. If both cars leave X at the same time
how far apart are they after 2 hours 20 minutes?
5. Evaluate:
(a) f2 x
1
- 1dx
X
(b) r(e 2 x - 1)dx (c) f~~ sec 2
x dx

72
6. AB is a chord of a circle and PAQ a tangent at A. If R is the mid-point of the major arc cut off
by the chord AB and perpendiculars are drawn from R to the tangent and the chord, prove that
the perpendiculars are equal in length.

PAPER 48
1. (a) Sketch the curve y = ex and find the area of the region enclosed by the curve the x-axis and
the ordinates at x = 0 and x = 2.
(b) If this area is rotated about the x-axis through 2n radians, find the volume of the generated
solid.
2. Evaluate:
50
(a) I 5n- 3
n=l

3. Find the equation of the tangent to the curve y = ,x 3 + 3x 2 at the point of inflexion.
4. A window consists of a rectangle surmounted by a semi-circle having as its diameter the width
of the rectangle. If the perimeter of window is 10 metres find the radius of the semi-circle which
will give the greatest possible area for the window.

5. The area under the curve y = loge x from x = 1 to x = 5 is given by 1 5


loge x dx. Since we do
not know the integral of loge x use Simpson's Rule to find an approximation to this area. Use
two sub-intervals 1 ~ x ~ 3 and 3 < x ~ 5.
6. ABCD is a parallelogram in which the diagonals AC and DB are drawn. LABD = L CBD = 30°.
Show that:
(a) LADB = L CDB
(b) LBAD is isosceles
(c) AC bisects LDAB.

73
Air pressure at an altitude of x metres is given by the Photograph courtesy of Qantas
exponential growth formula
P = P0 ekx, where k = -0.00012.

--~---------------------------------------------------------------------~
CHAPTER 22

··-~;~--- ----~------~---.-~

0 p

A particle P moves in a straight line. Suppose its distance, s metres, from a fixed point 0 after t seconds
is given by s = t 2 •

The distance-time graph would appear as shown with s


distance plotted on the vertical axis and time on the
16-
horizontal axis.
14 T

VELOCITY 12-
Now, velocity is the rate of change of distance with respect
to time. 10

Suppose we wish to find the average velocity of the a-


particle between the second and fourth seconds.
't distance travelled 6-
A verage ve1oc1 y = . k
time ta en 4- p
The average velocity is represented graphically by the
average gradient of the graph between the points P(2, 4) 2 -
and Q(4, 16)
. 16 - 4 12 2 3 4
i.e. Average velocity=---=-= 6.
4-2 2
The instantaneous velocity at timet, is given graphically by the gradient of the tangent to the curve
at timet.
Thus the instantaneous velocity at any point is given by the value of : at that point.
Now s = t2
ds
thus dt = 2t.
ds
Hence at P where t = 2, v = dt = 4.
The velocity after 2 seconds is 4 metres per second.
The symbol ds is sometimes written s, the dot denoting differentiation with respect tot.
dt

If a particle moves in a straight line and its distances from a fixed point in the line is expressed
as a function of time t, then its instantaneous velocity v at any time t is given by
ds
v = dt'

76
Example:
A particle moves on a line so that its distance, s metres, from the origin at time t seconds is given by
s = 4 - 3t + t 2 • Find:
(a) the distance of the particle from the origin at the start and after 2 seconds.
(b) the velocity of the particle when t = 2
(c) at what instant the particle comes to rest.
(d) the closest distance of the particle to the origin.
Solution:
(a) s = 4 - 3t + t 2
At the start t = 0
If t = 0, s = 4
.·. at the start the particle is 4 metres from the origin.
If t = 2, s = 4 - 6 + 4 = 2
.·. after 2 seconds the particle is 2 metres from the origin.
(b) v =
dsdt = - 3 + 2t
when t = 2, v = -3 + 4 = 1
.·. after 2 seconds the velocity is 1 metre per second.
(c) The particle is at rest when v = 0, i.e. when -3 + 2t = 0
giving t = I!
.'. The particle is at rest after It seconds.
(d) The closest distance to the origin occurs when sis a minimum
i.e. when~; = 0 and this occurs when t = I!
If t = 1!, s = 4 - 4! + 2! = lj:
.·. The closest the particle comes to the origin is lj: metres.

ACCELERATION
When velocity is not constant and changes with time the rate of change of velocity with respect to
time is called acceleration. Acceleration must therefore be measured in units of velocity per second.
For example, in metres per second per second.

If the velocity v of a particle is expressed in terms of timet, then the instantaneous acceleration
a at time t is given by:

a
dv
= dt or a = dt 2
. v = dtds)
(smce
Acceleration is sometimes written §where the two dots indicate double differentiation with
respect to t.

Example (i):
A particle moves in a straight line such that its distance, s metres, from a fixed point 0, in the line,
at the end oft seconds is given by s = 12 + 3t + 2t 3 . At the end of 4 seconds find:
(a) its distance from 0.
(b) its velocity.
(c) its acceleration.
77
Solution:
s = 12 + 3t + 2t 3
v = ds = 3 + 6t 2
dt
dv
a= = 12t
dt
(a) Using s = 12 + 3t + 2t 3
when t = 4, s = 12 + 3 x 4 + 2 x 43 = 152
.·. After 4 seconds the particle is 152 metres from 0.
(b) Using v = 3 + 6t 2
when t = 4, v = 3 + 6 x 4 2 = 99
.·. After 4 seconds the velocity of the particle is 99 metres per second.
(c) Using a = 12t
when t = 4, a = 12 x 4 = 48
.·. After 4 seconds the acceleration of the particle is 48 metres per second per second.

Example (ii):
A point moves in a straight line so that its distance s metres from a fixed point 0 at time t seconds is
given by s = 8 + 12t - 4t 2 + tt 3 .
(a) Find the position of the point when t = 0.
(b) Find the equations for velocity and acceleration.
(c) At what times is the velocity zero and what is the acceleration at these times.
Solution:
(a) s = 8 + 12t - 4t 2 + tr 3
when t = 0, s = 8
.·. The point is 8 metres from 0 when t = 0.
(b) s = 12 - 8t + t 2
S' = -8 + 2t
(c) If the velocity is zero 12 - 8t + t 2 = 0
(t - 2)(t - 6) = 0
t = 2 or 6
When t = 2, S' = - 8 + 2 x 2 = - 4
.·. When t = 2 the acceleration is -4 metres per second per second.
When t = 6, S' = - 8 + 2 x 6 = 4
.'. When t = 6 the acceleration is 4 metres per second per second.
N.B. A negative velocity means that the point is moving in the direction of s decreasing. A negative
acceleration means that the velocity is decreasing. For example, a negative acceleration could
cause a velocity of 8 metres per second to decrease to 6 metres per second or a velocity of
- 8 metres per second to decrease to - 10 metres per second.

1. A particle is moving in a straight line so that at time t its distance from a fixed point is given by
s = 2t 3 - t 2 . At what time is its velocity zero?
2. An object travels in a straight line with velocity given by v = 4t 5 • Find the acceleration at time
t = 2 if tis in seconds and v in metres per second.
78
3. A particle is moving in a straight line so that at time t seconds, the distance from a fixed point
is given by s = t 3 - 2t + 1. At what time is the acceleration equal to 12 units per second per
second?
4. A particle moves in a straight line so that its distance, s metres, from a fixed point 0, on the line,
at time t seconds is given by s = t 2 - 6t + 8. Find:
(a) the initial position of the particle.
(b) the position of the particle when t = 3 and t = 4.
(c) the distance travelled by the particle in the fourth second.
(d) the initial velocity and the velocity when t = 4.
(e) at what time the velocity is zero.
(f) the acceleration of the particle.
5. A body falling under gravity falls s metres in t seconds, where s = 4·9t 2 •
(a) Find the distance the body falls in the third second.
(b) Find the velocity after 3 seconds.
(c) Find the acceleration of the falling body.
(d) If the body hits the ground after 4 seconds from what height did it fall and what was the velocity
at impact?
6. A body moves in a straight line; and s, the number of metres from a fixed point 0 after t seconds,
is given by the formula s = 3 - 4t + t 2 •
(a) How far is the body from the fixed point at zero time?
(b) What is the velocity equation? Sketch its graph.
(c) What is the velocity when t = 1 and when t = 3. What interpretation do you give to the
different signs of these two velocities?
(d) When the velocity is positives is increasing. When does this first take place?
(e) When does the body pass through the point 0? What interpretation do you give to the two
answers?
7. If the velocity of a particle, after t seconds, moving in a straight line is given by v = 10 + 6t - t 2 •
Find the acceleration when t = 2 and the greatest positive velocity attained by the particle.
8. The distance xmetres of a body from a fixed point 0 after t seconds is given by x = 2t - 3t 2 + 2t 3 .
Find the velocity and acceleration after 3 seconds.
9. A particle moves in a straight line in such a way that its distance, x metres, from a fixed point 0,
in the line, after t seconds is given by x = 4t - 5t 2 + 2t 3 • Find:
(a) the position of the particle when t = 0, t = 2, t = 3.
(b) the difference in position from when t = 2 to when t = 3.
(c) the velocity when t = 2.
(d) the time when v = 0.
(e) the value of x when v = 0.
(f) the acceleration equation and calculate the velocity when the acceleration is zero.
10. A particle is moving in a straight line so that its distance, x metres, from a fixed origin 0, after
t seconds, is given by x = 12t - t 3 .
(a) Show that the particle is at the origin initially and find its initial velocity.
(b) Show that the particle moves away from the origin 0 for 2 seconds and that when it returns
to 0 its velocity is 24 m/s in a negative direction.
11. A body rolls down an inclined plane so that after t seconds its distance x metres from the top,
measured along the incline, is given by x = t + 4t 2 •
(a) Find the velocity of the body when t = 2.
(b) Show that the acceleration is constant and find its value.
(c) If the velocity of the body is 25m/sat the bottom of the inclined plane, find the time taken
for the body to reach the bottom and the length of the inclined plane.
79
Example (iii):
The distance x metres at time t seconds of a particle moving in a straight line, from a fixed point 0
is given by x = 2 cos 2t.
(a) Sketch the graph of this equation.
(b) Find the times when the particle is at rest and where the particle is at those times.
(c) What is the maximum distance of the particle from 0?
Solution:
(a) X (b) The particle is at rest when~~ = 0.
X= 2 COS 2t
V =tix
- = - 4 Sill
. 2t
tit
·r·-· v = 0 when sin 2t = 0
1 i.e. when t = 0, "I, rc, 32", 2rc. At these
times x = 2, -2, 2, -2, 2.
(c) The maximum distance from 0 is 2 m.
-2

1. A particle moves along a straight line, its distance from the origin being given as a function of
time by x = sin 2t, where xis measured in metres and tin seconds.
(a) Sketch the graph of this distance equation.
(b) Find the velocity of the particle when t = 0.
(c) Show that the particle is initially at 0 and find when next it returns to 0.
(d) Find the first two times when the particle changes direction and find its acceleration at these
times.
2. The distance x metres at time t seconds of a particle moving in a straight line from a fixed point
0 is given by x = 2 - 2 cos 2t.
(a) Sketch the graph of this equation for 0 ~ t ~ 2rc.
(b) Find the times when the particle is at rest and where the particle is at these times.
(c) What is the maximum distance of the particle from 0?
3. The displacement of a particle in a straight line is given by x = cos t + sin t, where x is the
displacement in metres after t seconds.
Find: (a) the initial displacement.
(b) the displacement after ;j: seconds.
(c) the velocity when t = 0, t = ;j:, t = ~·
(d) the acceleration when t = :X, t = 34'.
(e) sketch the graph of the displacement equation for 0 ~ t ~ 2rc.
4. The displacement of a particle moving in a straight line is x metres after t seconds where x =e 1
- t.
(a) Show the particle is at rest when t = 0.
(b) What is the velocity of the particle after 1 second?

Let us now consider the problem of finding the distance equation of a point as a function of time,
when given its velocity or acceleration equation as a function of time. This problem is solved by
using primitive functions.
80
dv
Since a= dt' v =I adt

ds
and since v = dt' s =I vdt

Example (i):
A point initially at the origin, at rest, has an acceleration which is given by a = 2 + 5t, where tis in
seconds and a is in metres per second per second. Find its velocity and distance equations as functions
oft.
Solution: Alternative set out using integra/notation
dv a = 2 + 5t
a=-= 2 + 5t
dt
v = 2t + ~t
2
+c v = J
(2 + 5t)dt
= 2t + ~t + c
2

but v = 0 when t = 0, hence C = 0 but v= 0 when t = 0, hence C =0


thus v = ds = 2t +~ thus v = 2t + ~t 2
+ ~t 2 )dt
dt 2
giving s = I (2t
= t2 + it3 + c
but s = 0 when t = 0, hence C = 0 but s = 0 when t = 0, hence C = 0
Thus s = t 2 + it 3 Thus s = t 2 + it 3

Example (ii):
A particle, which was initially at rest at the origin 0, has an acceleration after t seconds of (6 + 2t)
metres per second per second. What is its velocity at the end of 3 seconds, and what is the distance
tra veiled in this time?
Solution:
d2s
a = -2= 6 + 2t
dt
. ds 6 2
.. v = dt = t + t +c
Since the particle was initially at rest v = 0 when t = 0, thus C = 0.
ds
Hence v = - = 6t + t 2
dt
When t = 3, v = 6 x 3 + 32 = 27.
That is, after 3 seconds the velocity of the particle was 27 mjs.
Since ~; = 6t + t2
2 t3
s = 3t + 3 +c.
As the particle was initially at the origins = 0 when t = 0, thus C = 0
3
therefore s = 3t 2 + t
3
' 3
3
When t = 3, s = 3 x 32 +
= 36.
3
That is, after 3 seconds the particle has travelled 36 m.
81
1. The velocity of a particle is given by v = 6t 2 • If initially the body starts from the origin, find:
(a) the velocity after 3 seconds.
(b) the distance equation as a function of time.
(c) the displacement after 3 seconds.
(d) when the particle is 16m from the origin.

2. A particle moves along the x axis with an acceleration given by a = 6t + 3. When t = 0, x = 5


and the velocity of the particle is 1 unit per second. Find the position of the particle after 3 seconds.

3. A particle starts from rest and moves so that its acceleration at time t is given by a = - 6. If the
particle is initially 27 m from the origin, find:
(a) velocity v in terms oft. (b) distances in terms oft.
(c) the time at which the particle reaches the origin and its velocity at that time.

4. Given that~:~= 12t - 2, that~; = 0 when t = 0 and that s = 4 when t = 1, finds when t = 5.

d2s
5. Given that dt 2 = 2t, v = 0 when t = 0 and s = 10 when t = 1, finds when t = 2.

6. Given that§= 12- 2t and s = 16 when t = 0, find:


(a) the velocity of the particle after 3 seconds.
(b) the distance travelled by the particle in the first 3 seconds.

7. A particle starts from rest and moves so that its acceleration at timet is given by§= -4. If it
is initially 18 m from the origin find:
(a) sin terms oft (b) sin terms oft
(c) the time at which the particle reaches the origin and its velocity at that time.

8. A point, with initial velocity of 20 mjs, starts from the origin and has an acceleration given by
a = 8 - 2t. When does it first come to rest? How far has it moved in this time?

9. The velocity, after t seconds, of a particle is given in metres per second by v = 3t 2 + 1.


(a) Find the distance travelled by the particle in the first 4 seconds.
(b) Draw a velocity time graph for this motion and show that the area under the curve from
t = 0 to t = 4 gives the distance travelled by the particle in this time.
(c) By evaluating L 3
(3t 2 + 1)dt, find the distance travelled in the third second.

10. The velocity time graph for the equation v


v = t 2 - 2t + 2 is given.
By evaluating the area of the shaded region, v = t'-21+2
find the distance travelled in the first 3 seconds
by a particle moving according to this velocity
equation.
2

0 3

82
11. A train is moving with a velocity of 10 mjs as it v
passes a signal. Two seconds later the brakes are
applied and the train comes to rest after a further
3 seconds. The velocity-time graph of this motion
is shown.
Find the distance travelled by the train after
passing the signal.

12. The calculus can be applied to situations other than motion in a straight line, where a quantity
such as volume or area changes with respect to time. For example, suppose a valve on an oil
pipeline is opening such that the volume flow rate in cubic metres per second is given by the
equation
-dV = 1·5t
dt
where t is the number of seconds elapsed from the time the valve began to open. Calculate the
total volume of oil to flow through the valve in the first 20 seconds.
13. The rate of change of a certain area, in square metres per second, after t seconds is given by the
equation
dA 1
I 2
dt = J(f + 0

If the area is 6 square metres when t = 0, find the area after 3 seconds given that k = 2-4.
14. When a stone is dropped into a pond the area enclosed by the largest of the concentric circles
produced is given by
A 1 t 2
= 20 + 10 + t
A being the area in square metres t seconds after the stone hits the water. At what rate is the
area increasing when t = 3?

If y = f(x) the rates of change of y and x with respect to time can be compared by using the result.

Example (i):
A metal cube being heated has its edge length expanding at a constant rate of 0·01 mmjs. At what
rate is its volume increasing when its edge length is 60 mm?
Solution:
For a cube of side x units V = x 3•
Now dV dV dx and from the data dx = 0·01.
~ ~·~ ~

Hence dV = 3x 2 x 0·01.
dt
If X = 60, dV = 3 X 60 X 60 X 0·01
dt
= 108.
83
Therefore, the voh.Jme is increasing at the rate of 108 mm 3 /s when the edge length is 60 mm.
Example (ii):
A sphere is expanding so that its surface area is increasing at the rate of 0·04 cm 2 fs. Find the rate
of increase of its volume at the instant when the radius is 10 em.
Solution:
The surface area of a sphere is given by A = 4nr 2 •
Now dd:A = dd:A . ddr and from the data dd:A = 0·04.
t r t t
dr
Hence 0·04 = 8nr x -
dt
dr 0·04
dt 8nr
Now the volume of a sphere is given by V = 4nr 3 •
dV dV dr
Using
dt dr · dt
0·04
= 4m· 2 x - -
8nr
= 0·02r.
dV
lfr = 10, - = 0·02 x 10 = 0·2.
dt
Therefore, the volume is increasing at the rate of 0·2 cm 3 /s when the radius is 10 em.

1. A square plate of metal is being heated so that its side is expanding at the rate of 0·03 mm/s. At
what rate is the area increasing when the side length is 80 mm?
2. An ice-cube is melting so that its edge length is decreasing at the rate of 2 mm/s. At what rate
is the volume decreasing when the edge length is 200 mm?

3. Find~~ when A = nr 2 , r = 8 and:·= 0·04.

4. Find the rate of change of V with respect to t when V = 4nr3, r = 10 and ~ = 0·1.

5. A spherical balloon is being inflated so that the radius increases at a constant rate of 20 mm/s.
Calculate the rate of change of volume when the radius of the balloon is 500 mm.
6. A vessel is of such a shape that when the depth of water is x em the volume is V cm 3 , where
V = 30x + x 3 • Water is poured into the empty vessel at the rate of 25 em 3 /s. At what rate is the
level of the water rising when the depth is 5 em?

~x
2

7. Show that the area of an equilateral triangle of side x is given by A = • The sides of an
equilateral triangle are increasing at the rate of 5 mm/s. At what rate is the area increasing at the
instant when the sides are 10 em long?
8. A point moves along the curve y = -!-x 2 . The abscissa of the point changes at the rate of 4 units
per second. At what rate is the ordinate increasing when x = 3?
9. A spherical bubble is expanding so that its volume increases at the rate of 40 mm 3 /s. When the
radius is 10 mm what is the rate of increase of:
(a) its radius?
(b) its surface area?
84
10. The volume of a cube is increasing at the rate of 30 cm 3 js. Find the rate at which:
(a) the edge (b) the surface area
are increasing when the edge is 15 em long.
11. A spherical balloon is being deflated so that its surface area is decreasing at the rate of 0·1 cm 2 js.
Find the rate of decrease of:
(a) its radius (b) its volume
at the instant when its radius is 20 em.
12. The volume of a spherical ball of ice t hours after it has begun to melt is V cm 3 , its surface is
A cm 2 and its radius is r em.
dV . dr
(a) If dt = -3A, fmd dt.

(b) Show that~~ = -24n cm 2 per hour when the radius is 1 em.

13. The volume of a closed cylinder of constant height 8 em is given by V = 8m· 2 • The radius of its
base is 6 em and is increasing at the rate of 0·04 cmjs.
(a) At what rate is the volume of the cylinder increasing?
(b) At what rate is the total surface area of the cylinder increasing?
14. Sand falling from a funnel forms a conical pile such that the height of the pile is one and a half
times the radius.
(a) Show that the volume of the pile is given by V = n; .
3

(b) If the sand is falling at the rate of to m 3 per minute, find the rate at which the height is in-
creasing when the pile of sand is 3 m high.
15. A water trough is 200 em long and has a cross-section of a right-angled isosceles triangle. Show
that when the depth of the water is x em, the volume of water in the tank is 200x 2 cm 3 • Water
is poured in at a constant rate of 5 litres per minute. Find the rate at which the water level is
rising when the depth is 30 em.

EXPONENTIAL GRAPHS
Exponential functions are readily graphed using theEjkey on a calculator to find ordered pairs on
the graph.
Example:
Draw the graphs of the following exponential functions which are of the form y = ekx.
(a) y =ex (b) y = etx (c) y = e 2x (d) y = e-zx
Solution: TABLE OF VALUES

X -2 -1 0 0·5 1 2 3

y =ex 0·14 0·37 1 1·65 2·72 7·38 20·08

y = etx 0·37 0·61 1 1·28 1·65 2·72 4·48

y = e2x 0·02 0·14 1 2·72 7·38 54·60 403·4

y = e-2x 54·60 7·38 1 0·37 0·14 0·02 0·002

85
N.B. 1. As k increases in value the curve steepens.
2. y = e2 x andy = e- 2 x are reflections in they axis.
3. All these curves pass through the point (0, 1).

1. (a) Copy and complete the X -2 -1 -0·5 0 0·5 1 2


given table of values for
functions of the type Y =ex
y = Aex.
(b) Sketch the curves. y = 2ex
(c) What is the effect of
changing the value of y= 1·5ex
A iny = Aex?
y= -2ex

2. (a) Copy and complete X -2 -1 -0·5 0 0·5 1 2


the given table of
values for functions y = 2etx
of the type y = Aekx.
(b) Sketch the graph of y = 1etx
each function.
y = -tetx
y = -1e-tx

86
THE GROWTH AND DECAY FUNCTION
The exponential function has many practical applications. It is involved in natural examples of growth
and decay, such as population growth, growth of bacteria in a culture, the rate of cooling of a body,
the rate of decay of radioactive substances and the rate of change of pressure with altitude, to name
a few.
The exponential function N = Aek1 is known as the growth function. Note that A and k are con-
stants and N is a function oft. The independent variable is given as t since the growth function usually
varies with time.
Note that if N = Aek1 then dN = kAek 1
dt
= kN.

Thus the growth function N = Aek1 satisfies the equation ~~ = kN.


This equation states that the rate of change of N is proportional toN. For any practical situation
for which this is true the growth function will be of the form N = Aek1 •
If k is negative then N is decreasing continuously and the equation represents the decay function.
Substituting t = 0 in the formula N = Aek1 gives N = Ae 0 i.e. N = A.
Thus A represents the initial value of Nand the formula is often given as N = N0 ek1, etc.
--- - - - - - - ---- ---- ---- - -------- ---------~---

The exponential growth and decay formula is

[ _
N = Noekt
where (i) N0 is the initial value of N when t = 0.
(ii) k is the growth or decay constant for a particular population.

Example (i):
The number of bacteria found in a certain culture is given by N = N 0 e0 ' 121 , where tis in hours. Initially
the culture population was 240 000 bacteria. How many were present after 5 hours?
Solution:
The growth formula is N = N 0 e0 ' 121 .
From the data N 0 = 240 000
.'. N = 240000e 0 ' 121
when t = 5, N = 240000e 0 ' 12 x 5
= 240 000e 0 ' 60
~· 437 308·48 (by calculator)
After 5 hours there are approximately 437 000 bacteria present.

1. The number of people in a town is given by N = 12 000e 0 ' 091 , where t is the number of years
since 1980.
(a) What was the initial population in 1980, i.e. when t = 0?
(b) What would be the expected population in 1990, i.e. when t = 10?
2. The number of bacteria found in a certain culture is given by N = N 0 e0 ' 241 , where t is in hours.
Initially the culture population was 50 000 bacteria. How many bacteria would be present in the
culture after: (a) 2 hours? (b) 3! hours?
3. The number of people in a town is given by N = 2000e 0 ' 31 , where tis the number of years since
the end of 1970.
(a) Draw the graph of the number of people in the town against time for t = 1 to t = 10. Use
your graph to find the year in which the population first exceeded 15 000.
(b) Use your calculator to find the expected population of the town at the end of 1990.
87
4. In a certain chemical reaction the amount P grams of a 'substance which remains unchanged
after t minutes is given by the equation P = P0 e- 0 ' 151 • If the initial mass of the substance is
200 grams, find the amount remaining after (a) 1·5 minutes (b) 4·5 minutes (c) 6 minutes.
5. The temperature K (in degrees Kelvin) of a certain star in t million years time is given by the
equation K = 9000e- 0' 138 '.
(a) What is the present temperature of the star?
(b) Find what the temperature of the star will be in 2 million years time.
6. A piece of radioactive material having a mass of 500 grams is decaying according to the formula
M = 500e- 0 ' 251 , where M is the mass in grams of material remaining after t years.
(a) Draw a graph of this equation for values oft from 0 to 4.
(b) Use your graph to find:
(i) what time elapsed before there was only 400 g of material remaining. How long after
this before there was only 300 grams remaining?
(ii) how much material remained after 15 months? Check this answer using your calculator.

Example (ii) :
The city of Lismore had a population of 20 904 in 1971 and this had grown to 21 650 in 1975. Find
the annual growth rate as a percentage, correct to two decimal places, assuming that the growth rate
is proportional to the population.
If the growth rate is assumed constant, what would be the population in 1987?

Solution:
Since the rate of change of population is proportional to the population (i.e. ~ = kP) we can use
the exponential growth equation P = P0 ek'.
From the data P0 = 20 904
.'. P = 20 904ek1
When t = 4 (1975) 21650 = 20904e 4 k
. 21650 4k
t.e. 20 904 = e
Taking logarithms of both sides to the base e
lo 21650 = 4/.
ge 20 904 (;
.'. k ~ 0·0087662 (by calculator)
~ 0·88%
Thus the annual growth rate is 0·88%.
In 1987, t = 16. Using this value fort and taking k = 0·0088 the population is given by
p = 20904e0·0088xl6
~ 24064·567 (by calculator)
Thus population in 1987 would be approximately 24 100.

Example (iii):
A ship is travelling at 6 mjs when its motors are stopped. Its deceleration is proportional to its velocity
(i.e. ~~ = kv) and after 100 seconds its velocity is 3 mjs. Find when it~ velocity will be 1 mjs.
Solution:
We can use the decay equation v = Voekt
Substituting v0 = 6, v = 3 and t = 100 3 = 6e1ook
0 . 5 = elOOk
88
Taking logarithms of both sides to the base e loge 0·5 = lOOk
k =i= 0·0069314
When v = 1, 1 = 6e-o·oo69314r
1, = e-o·0069314r
Taking logarithms of both sides loge 1, = -0·00693141
.'. t =i= 258·49893 (by calculator)
Thus the speed will be 1 m/s after approximately 258 seconds.

1. The mass of an oi·ganism growing in a pond after t days is given by M = M 0 ek1• If the initial
mass is 1· 5 kg and k = 0·05, what would be the mass of organism after:
(a) 5 days
(b) 10 days
and (c) how long would it take the mass to double?
2. The population of a town has its rate of growth proportional to the population. If the annual
growth rate is 0·02 and the original population is 2400, what would be the population in:
(a) 4 years?
(b) 10 years?
(c) How many years would it take for the population to reach 3400?
3. A radioactive material decays according to the exponential function M = M 0 ek1• If the initial
mass is 50 g and the mass after 10 years is 40 g, find :
(a) k
(b) the amount present after 15 years.
(c) the half-life of the material. (The half-life of a radioactive material is the time taken for the
material to halve its mass.)
4. (a) Show that the half-life of any radioactive element is t(loge !) and so is independent of the
initial mass.
(b) The rate of decay of radium 226 is -0·0004278.
(i) Find its half-life to the nearest year.
(ii) How long would it take for one third of the original to decay?
5. If money is invested to earn interest that is compounded continuously, the amount after t years
is given by A = Pekr, where Pis the principal invested, k is the interest rate expressed as a decimal
and t the number of years. If $2000 is invested for 5 years at 10% interest compounding contin-
uously, what amount would be due.
N.B. Because interest is not usually compounded continuously the formula generally used is
A = P 1+
( nr)"N, h
W ere
p . th
IS
. . . t d
e pnnc1pa1 mves e , I' =
percentage interest rate
l OO ,
N = number of years and n = number of times paid each year.
6. An object travelling through a gas is subjected to a retardation, proportional to its velocity. If
initially the object has a velocity of 100 mjs, while after 4 seconds the velocity is 20 mjs, find its
velocity after:
(a) 8 seconds. (b) 10 seconds. (c) 20 seconds.
7. In a colony of bacteria, the number present initially is 8000. If the number doubles in 5 days,
how many would be present after 12 days?
8. In a sample culture at zero time there were 4·6 x 10 6 bacteria. At the end of 5 days there were
7·2 x 106 bacteria. Find:
(a) the growth rate.
(b) the number expected to be present at the end of 12 days.
89
9. Air pressure at an altitude of a metres is given in kilograms per square metre by the formula
P = P0 eka, where k = -0·00012. If the pressure at the earth's surface is 10000 kgfm 2 , what is
the pressure at an altitude of:
(a) 5000 m? (b) 15 km?
10. The chemical nitrogen pentoxide is a solid which has a decay rate of - 0·0005 when time is mea-
sured in seconds. Ifthe initial mass is 10 g:
(a) how much remains after 500 seconds?
(b) what is its half-life?
II. In 1961 the aborigine population in Australia was 75 309 and in the 1966 census was 80 207.
Assuming that the rate of population growth is proportional to the population, find:
(a) the growth rate.
(b) the estimated aborigine population in 1981.
12. To historically date objects less than 50 000 years old, carbon 14 dating is used. Carbon 14 has a
half-life of 5730 years. Before death animals and vegetation have a reading for carbon 14 of 12·5
counts per minute on a Geiger counter. Show that the decay rate for carbon 14 is -1·21 x 10- 4 •
If a piece of wood from an excavation site has a reading of 7 counts per minute, show that the
wood's age is approximately 4800 years.

In practical applications more complicated exponential functions are often used to describe the process
of growth and decay.
For example, if a population N has a rate of change proportional to the difference between Nand
a constant P, this may be expressed as
dN
dt = k(N - P) where k, P are constants.
As in earlier work, k is called the growth constant.
This equation has a trivial solution given by
N = P (where P is the given constant)
as in the above equation L.H.S. = ~~ = d~) = 0
and R.H.S. = k(N- P) = k(P - P) = 0.
Thus, both sides are equal to zero and so N = P is one solution.
A more general solution to this equation is given by N = P + Aekt where A is an arbitrary constant.
To verify this, substitute for N in
~~ = k(N- P)

:r (P + Aek') = k{ (P + Aek') - P}
kAekt = kAekt and so the equation is satisfied by N = P + Aek'.

If the rate of change of N is given by


dN
- = k(N - P), where k, Pare constants
dt
then solutions are given by
N = P (the trivial case)
or N = P + Aekt where A is a constant.

If a given set of initial conditions is provided the numerical values of the constants A and k may
be determined.
90
Example (i):
In a given population study the population N is given by N = 100 + Aek
1
• Initially N = 250 and
after two seconds N has grown to a 1000.
Find: (a) the values of A and k
(b) the population after 5 seconds.

Solution:
(a) N = P + Aekr
= 100 + Aekr
When t = 0, N = 250
250 = 100 + Ae0
A= 250- 100
= 150
When t = 2, N = 1000
1000 = 100 + 150e 2 k
900 = 150e 2 k
e2k = i~g
=6
Take logarithms to base e:
loge e 2 k = loge 6
2k =loge 6
.'. k = 1 loge 6
~ 0·896 correct to three decimal places .
.'. N = 100 + 150e 0 ' 8961
(b) When t = 5
N = 100 + 150e 0 ' 896 x 5
+
= 100 150e4-4 80
~ 13 335 by calculator.

Example (ii):
The rate of change of a population is given by dN = 0·8(N- 100).
dt
(a) Write down a solution for N.
(b) If N = 80 when t = 0, find A.
(c) Find Nwhen t = 2.

Solution:
(a) Since dN = 0·8(N- 100)
dt
N = P + Aekr where P = 100, k = 0·8
N = 100 + Ae 0 ' 81
(b) When t = 0, N = 80
80 = 100 + Ae 0
-20 =A
and
N = 100 - 20e 0 ' 81
(c) When t = 2
N = 100 - 20e 0 ·sxz
= 100 - 20el-6
= 100 - 99·060 646 by calculator
~ 1

91
Example (iii):
If N = P + Aekr where P = 500, A = 10 and k = -0·2, find Nwhen t = 1; t = 5; t = 10; t = 20.
Sketch the graph of N = P + Aekr.
Solution:
N = 500 + 10e- 0 ' 2 '
when t = 1
N = 500 + 10e- 0 ' 2
= 508·187 by calculator
~ 508
when t = 5
N = 500 + 10e- 0 ' 2 x 5
= 500 + 100e+ 0
= 503·678 79 by calculator
~ 504
when t = 10
N = 500 + 10e- 0 ' 2 x 10
= 500 + lOe- 2 ' 0
= 50 1· 353 35 by calculator
~ 501
when t = 20
N = 500 + 10e- 0 ' 2 x 20
= 500 + 1oe-4-0
= 500·183 15 by calculator
~ 500

510

508

506

504
The graph of N = 500 + 1oe- 0 ' 2 '
502
501
500

5 10 15 20

Thus, as t becomes larger and larger, N becomes closer and closer to 500 (the value of P). This may
be expressed as
limN= 500.
The graph of N = 500 + lOe- 0 ' 21 is asymptotic to the lineN= 500. In general, this happens for all
cases where the growth constant k < 0.

THE GENERAL CASE FORk< 0


Consider N = P + Aek1 where k < 0 (i.e. k is negative)
Put p = - k so that p > 0
N = P + Ae-p1 ask= -p
=P+~
ePI

92
Remembering that, as t increases to infinity, y = eP1, where p > 0, also tends to infinity,
A
then, as t -+ ro, eP1 -+ ro and - - + 0 (becomes very small)
ePt

and as N = P + e~~, N -+ P.
Thus, whenever k < 0, N-+ Past-+ oo

THE GENERAL CASE WHEN k > 0


Consider N = P + Aekt when k > 0.
Thus, as t increases, ekt will also increase and as t -+ ro, ekt -+ oo.
If A is positive, Aekt will increase to a large positive number, and so N = P + Aekt will become a
large positive number.
Thus, when k > 0, A > 0
as t -+ ro, N -+ oo (becomes infinitely large)
If A is negative, Aekt will decrease to a large negative number as t -+ ro and so N will also become
a large negative number.
Thus, when k > 0, A < 0
as t -+ ro, N-+ - oo (becomes infinitely small)

Given that~~= k(N- P), then N = P + Aek 1


If k < 0, as t -+ ro, N-+ P.


If k > 0 and A > 0, as t-+ ro, N-+ oo.
If k > 0 and A < 0, as t -+ ro, N-+ - ro.

1. Given that N = P + Aekt


(a) If P = 500, k = 0·5 and N = 1000 when t = 0,
(i) find A
(ii) find N when t = 3.
(b) If P = 750, A = 125 and N = 1000 when t = 1,
(i) find k correct to three decimal places
(ii) find N when t = 4.
(c) If P = 250, initially N = 650, and after 2 seconds N = 1500;
find: (i) A
(ii) k, correct to two decimal places
(iii) N after 4 seconds.
(d) If k = -0·6, P = 625, N = 1275 when t = 0,
find: (i) A
(ii) the time (to the nearest second) for N to decrease to 657.
(e) P = 120, k = 0·75 and initially N = 600.
Find: (i) the value of A
(ii) the time needed for N to double in value
(iii) the values of N when t = 3, t = 5, t = 10.

2. (a) The rate of change of a population is given by~~= 0·75(N- 200).


(i) State a general solution for N.
(ii) If N = 500 when t = 0, find the value of N after three seconds.
93
(b) If dN = -0·5(N- 200) and if, initially, N = 1000, find N after:
dt
(i) 1 second (iii) 5 seconds (v) 20 seconds.
(ii) 2 seconds (iv) 10 seconds
Sketch the graph of the function N.
3. A population N has its rate of growth proportional to the difference between Nand 150. This
dN
may be expressed as dt = A.(N- 150), where A = 0·8.
If initially N is 21 0, find N after :
(i) 1 second (iii) 5 seconds
(ii) 2 seconds (iv) 10 seconds.
What happens toN as t approaches infinity?
4. Given that N = 200 - 25e 0 ' 51 ,
(a) Find N when (i) t = 1; (ii) t = 2 and (iii) t = 5.
(b) What value of t would make N equal to zero?
(c) What happens toN as t ~ oo?
5. N = 50 - lOe- 0 '251 . Copy and complete the following table of values.

0
r---- ---------
1
f---- ---------

2
---- ---------
5
---- ---------
10
---- ---------
20

Explain the behaviour of N as t ~ oo.


Sketch the graph of N = 50 - lOe- 0 ' 251 •
6. Newton's Law of Cooling states that the rate of change of the temperature of an object is propor-
tional to the difference between its temperature and the temperature of the surrounding matter.
The temperature of the surrounding matter is assumed to be constant.
. This may be expressed as ~= k(T - P)
where P = constant temperature of the surrounding medium
T = temperature of the object which is a function of time t
k is a constant.
The atmospheric temperature is assumed constant at 30°C. A metal object, heated to 125°C is
placed in the air to cool and after 1 hour it is found to have cooled to 80°C.
What will its temperature be after 1! hours?
7. An oven has been heated to a constant temperature of 250°C. An object with a temperature of
10°C is placed in the oven and after 15 minutes its temperature is measured as 70°C.
(a) What will be the temperature of the object after 30 minutes?
(b) How long will it take to reach 200°C?
(c) What would be the limiting temperature which could be achieved by the object?
94
8. A saucepan of water, initially at 100°C, is placed in the open where the air temperature is constant
at woe. After 20 minutes it is found that the water has cooled to 75°C.
(a) Find the temperature of the water after 50 minutes.
(b) What would be the temperature of an identical saucepan of water, initially at 50°C, after 50
minutes?

9. An object heated to 90°C is placed in the open air where the temperature is 26°C. If the object
cools to 70°C in 30 minutes:
(a) What will its temperature be after 1 hour?
(b) How long will it take for the temperature to reach 30°C?

W. A hot piece of iron with a temperature of 150°C is placed in a freezer maintained at -10°C. After
30 seconds the temperature of the iron is 60°C.
(a) Find the temperature of the iron after 1 minute.
(b) When will its temperature be ooc?

11. A thermometer recording 2°C is brought into a room the temperature of which is set at a constant
22°C. After 5 minutes it registers woe.
(a) Show that T = 22 - 20e- 0 ' 1021 .
(b) How long will it take to record 20°C?

12. For a body falling under gravity in air, the rate of change of velocity is given by~= -k(V- P)
where P and k are constants. This has as a solution: V = P + Ae-k1•
If the rate of change is given by ~= - 0·02(V - 490), find:
(a) an equation for V, remembering that initially V = 0;
(b) the velocity after W seconds;
(c) the maximum velocity which could be achieved, assuming no outside interference.

13. A tank contains a brine solution for curing hams. Initially the tank contains 80 kg of dissolved
salt. The amount of salt is known to change at a rate given by ~ = - 0·01 (x - 50).
(a) Write down an equation from which x, the amount of salt, may be calculated.
(b) Find the amount of salt in the tank after 60 minutes.

14. As a part of a learning model, the rate of growth of habit, H, with respect to the number ofrein-
forcement exercises, x, is proportional to the difference between Hand a constant, S, representing
the maximum habit strength achievable.
dH
Thus dx = - k(H - S).
Given that S = 80; when x = 0, H = 0 and when x = 10, H = 30:
(a) show that H = 80(1 - e-o·0 4 7x)
(b) find H after 20 reinforcements.

15. In actual population studies the rate of growth is given by~= kP(1 - RP) where k, R are
constants.
This reflects limitation on growth, such as by lack of food and the existence of predators. This is
called the logistic growth equation.
(a) Show by differentiation that P = I kt, where I is the initial population
RI + (1 - RI)e-
(a constant), is a solution of the logistic equation.
(b) If k = 1·25; R = 0·00004; I= 10, find P after five years.
95
16. An isolated insect population is attached to a single plant which has a carrying capacity of 100
insects. Its logistic law of growth is given by c::; = 0·001P(l00 - P).

(a) Show by substitution that P


. = (!~O
1 + k .eo·lt'
(b) If initially P = 10, find k and hence the population when t = 40.
(c) Sketch the graph of the function P(t).

96
The Gladesville Bridge is the longest concrete arch Photograph courtesy Department of Main Roads NSW
span bridge in the world. The arch is in the shape of
a parabola and has a span of 305 metres. The
maximum clearance of the underside of the arch is
t10.8 metres at high water.

I
CHAPTER 23

PARAMETRIC EQUATIONS OF THE PARABOLA


As has been stated earlier, the locus of a point P which moves so that it is equidistant from a fixed
point S(O, a), called the focus and a fixed line y = -a, called the directrix is the parabola x 2 = 4ay.
The single equation x 2 = 4ay is called the Cartesian equation of the parabola. Often in Mathematics
it is more convenient to express the equation of the curve by means of two equations involving a
third variable called a parameter.
Consider the parabola given by the Cartesian equation
x 2 = 4ay.
Substitute x = 2at, where tis a parameter
(2at) 2 = 4ay
4a 2 t 2 = 4ay
at 2 = y.
Thus the parabola x 2 = 4ay may be represented by the pair of parametric equations
x = 2at} where t ts
. any real number.
y =at 2

Parabola: Cartesian: x 2
4ay =
2
parametric: x = at} where the domain is the set of reals.
y = at 2
Each value oft gives a unique point on the parabola and each point on the parabola corresponds
to a unique value oft.

Example (i):
Given the pair of parametric equations x = 2t}
y = t2
(a) Tabulate values of x andy for values oft so that -3 ~ t ~ 3.
(b) Draw the graph of the function y = f(x).
(c) Find the corresponding Cartesian equation.
(d) What is the focus of the parabola?

Solution:

(a) t -3 -2 -1 -z1 0 1
2 1 2 3

X= 2t -6 -4 -2 -1 0 1 2 4 6

y 1 1
= t2 9 4 1 4 0 4 1 4 9

98
(b) y

X' -6 -5 -4 -3 -2 -1 0 2 3 4 5 6 X

(c) As x = 2t
t = ~ and y = t2
2

= (~Y
x2
4
.·. x 2 =
4y which is the Cartesian equation of the parabola.
(d) x 2 = 4 x 1 x y, which is of the form x 2 = 4ay where a = 1
.'. the focus is (0, 1)

Given the parametric equations


x = 2at}
y = at 2
to find the Cartesian equation of the parabola, eliminate t, giving a resultant equation in
x andy.

Example (ii):
If x = 2at} represent a parabola, find
y =at 2
(a) the Cartesian equation
(b) the focal length
(c) the coordinates of the focus
(d) the equation of the directrix.

Solution:
(a) x = 2at (1) (2)
. X
.. t = 2a
Substitute in (2)
99
y = aC:Y
ax 2
- 4a 2
xz
4a
. ·. x 2 = 4ay is the Cartesian equation of the parabola.
(b) the focal length = a
(c) the focus is the point (0, a)
(d) the directrix is the line y = -a.
The general point T(x, y) on the parabola x 2 = 4ay is represented as the ordered pair (2at, at 2 )
and is often referred to as the point 't'.
For example, the point t = 1 is (2a, a)
t = -1 is (- 2a, a)
t =pis (2ap, ap 2 )
t = -pis ( -2ap, ap 2 )
~ -~--~~-- I
l
On the parabola·x-
- 2at}
2 the point 't'
y =at
reads as the point with parameter t, namely (2at, at 2 ).
t and - t are symmetric points about the y axis.
-------------------------------- ------------~---·--------------- --- ---------~

Example (iii):
Find the parametric equations for the parabola
x 2 = -12y.
Solution:
x2 = -12y
= 4( -3)y
:. a = - 3 and the focal length is 3.
The general parametric equations are x = 2at andy = at 2 .
.·. The parametric equations are
X=
y = -3t 2
-6t}

1. For the following parabolas, written in the standard form x 2 = 4ay, find the value Of 'a' and
hence the parametric equations for each parabola:
(a) x 2 = 4y (c) x 2 = y (e) x 2 = -4y
2
(b) x = 8y 2
(d) x = 2y (f) x 2 = -16y
2. Find the Cartesian equations for the following parabolas:
(a) X = 4t } (d) X = !t } (g) X = -±t }
y = 2t 2 y = 2 ±t y = -it2
(b) X= 6t} (e) X= 8t } (h) x = 2ap }
y = 3t 2 y = - 4t 2 y = -apz
(c) X = 8/ } (f) X = t }
y = 41 2 y = -!t2
100
3. For the parabolas (a) x = t, y = !t 2
(b) X = 6t, )' = - 3t 2
(i) tabulate the values of x andy for values oft in the domain -3 ~ t ~ 3
(ii) sketch the curve
(iii) find the Cartesian equation
(iv) state the coordinates of the vertex and of the focus
(v) state the equation of the directrix.
4. Eliminate 't' from each of the following pairs of parametric equations and so form the corre-
sponding Cartesian equation. Make a rough sketch of each curve.
(a) x = 2t } (b) x = 3 cost} (c) x = I + 2t }
y = 2 - 1 y = 3 sin 1 y = 4t 2 + 4t

Example (i):
(a) Find the equation of the chord joining P(t = 2) to Q(t = -1) on the parabola x 2 = 8y.
(b) What is the length of PQ?
Solution:
y (a) x 2 = 8y has parametric equations
X= 4t}
y = 2t 2

t 2 -I

X 8 -4

y 8 2
X
8- 2
Gradient of PQ =- -
8+ 4
_.!_
- 2
Equation of PQ is (y - 8) = !(x - 8)
2y- 16 = X - 8
X - 2y + 8 = 0

(b) Length of PQ = J(8 - 2) 2 + (8 + 4) 2


= J36 + 144
= JI80
~ 13·416 units.

I Dej;,1i~ions: 1. A chord whic-h--;asses through th;~focus


is called a focal chord. ~
L 2. The focal chord perpendicular to the axis of the parabola is called the latus
rectum.

Example (ii):
Find (a) The gradient.
(b) The equation of the chord through the points p, q on the parabola xy =at2al}.
=

(c) Show that if the chord passes through the focus pq = - I.


101
Solution:
y (a) When t = p; x ~ 2ap, y = ap 2
t = q; x = 2aq, y = aq 2
ap2- aq2
.·. gradient of PQ = _o_-~~
2ap - 2aq
- a(p + q)(p - q)
- 2a(p- q)
p+q
2
(b) The equation of PQ is (y - ap 2) = p ; q (x - 2ap)

X
2y - 2ap 2 = px + qx - 2ap 2 - 2apq
2y - px - qx + 2apq = 0
i.e. y t<P + q)x + apq = 0
(c) Now, if PQ passes through the focus S(O, a) then the
equation is satisfied by (0, a)
a 0 + apq = 0
apq = -a
pq = -1 or p =
q
--------------------------------- ---------------------------------- - - - - - - - - - - - - - - - - - - - - - - - - - - - - - -

The chord which joins the points (2ap, ap 2) and (2aq, aq 2) on the parabola x 2 = 4ay has
1. gradient= p; q
2. equation y t<P + q)x + apq = 0
3. if a focal chord pq = -1 or p = _!
q
i.e. t and _!are the parameters of the end points of a focal chord.
t
------ ------------

I. Find, from first principles, the equation of the chord joining:


(a) t 1 = 2, t 2 = - 1 on x = 2t, y = t 2
(b) / 1 = 2, ! 2 = -tonx = 6t,y = 3t 2 .
Show that this chord passes through the focus.
2. (a) (i) Find the equation of the chord joining the points t 1 = 2 and t 2 = - t on the parabola
x2 = y.
(ii) Show that the coordinates of the focus satisfy the equation.
(b) One extremity of a focal chord in the parabola x 2 = 8y is t = 3.
Find: (i) the gradient of the chord
(ii) the equation of the chord
(iii) the Cartesian coordinates of the other extremity of the chord
(iv) the parameter t at this point.
3. (a) Show that the chord joining t 1 = 1 and t 2 = -1 on the parabola x = 2at, y = at 2 is:
(i) a focal chord
(ii) is parallel to the x axis.
(b) Find the length of this chord.
(c) Show that the gradient of a focal ·chord joining the points (2ap, ap 2) and (2aq, aq 2) on the
p2 - I
parabola x 2 = 4ay, is~·

102
4. For the parabola x 2 = 4ay.
(a) Show that the gradient of the chord joining the vertex to a point P(2ap, ap 2 ) is~.
(b) If Q is the point (2aq, aq 2 ) state the gradient of OQ and hence show that the chords OP, OQ
are perpendicular if pq = -4.
(c) What is the condition for PQ to be perpendicular to the axis of the parabola?
5. (a) Findtheequationofthechordjoiningthepointst = 3andt = -tontheparabolax 2 = 4ay.
Show: (i) that it is a focal chord
(ii) that it cuts the directrix at the point (- 32a, -a).
(b) Prove that the chord joining the points p, q on the parabola x 2 = 4ay cuts the directrix at
the point ( 2a(pq - 1), -a).
p+q
Use this result to check your answer to part (a).
6. A secant which passes through the point (0, -a), cuts the parabola x 2 = 4ay at P, Q with para-
meters p, q.
(a) Prove that pq = 1.
1 1 1
(b) If S is the focus show that - + -Q = -.
PS S a

1. THE EQUATION OF THE TANGENT TO THE PARABOLA x 2 = 4ay AT


THE POINT (x 1 , y1 )
y Now x2 = 4ay
x2
.'. y = 4a

The gradient of a tangent to y = :a is given by


2

dy 2x
dx = 4a
X
2a
.'. gradient of the tangent at (x 1 , y 1 )
- xl
- 2a
.·. the equation of the tangent is
X
(y- Y1) = 2 ~(x- x 1)
2~(Xv\:- X1 )
2
Y- Y1 =

But (x 1 , y 1) is on the parabola


.'. x/ = 4ay 1
Thus the equation of the tangent is
1 1
y- Yl = 2axlx- 2a. 4ayl
1
= 2axlx- 2yl
. 1
.. Y + Y1 = 2
axx 1
or xx 1 = 2a(y + y 1) (1)

103
2 THE EQUATION OF THE TANGENT TO x = 2at, y = at 2 AT THE POINT
WITH PARAMETER 't'
This equation may be derived in various ways, each of which uses an important mathematical process.

Method 1: As x 1 = 2at, y 1 = at 2 substitute these in (1) above:


x . 2at = 2a(y + at 2 )
2atx = 2ay + 2a 2 t 2
Dividing both sides by 2a:
tx = y + at 2
or y - tx + at 2 = 0

Method 2: x = 2at, y = at 2
dx dy = 2at
.·. dt = 2a,
dt
. dy dy. dt
· · dx dt dx
dy
dt
dx
dt
2at
2a
=t
Thus in parametric form the gradient of the tangent at a point with parameter 't' is the
value of' t' .
.'. the equation of the tangent at t is:
y - at 2 = t(x - 2at)
= tx- 2at 2
or y tx + at = 0
2

Method 3:
y Suppose P is the point t 1
Q is the point t 2 •
The equation of the secant (the chord PQ) is
Y t(t1 + t 2 )x + at 1 t 2 = 0.
Now, as Q approaches P, t 2 approaches t 1 and the
secant becomes a tangent.
Thus, finally Q coincides with P and t 2 = t 1 , giving
the equation of the tangent as
Y - t(t 1 + t 1)x + at 1 . t 1 = 0
y- t 1 x + at 1 2 = 0
which leads to the tangent at the point 't' as:
X y- tx + at 2 = 0

3. EQUATION OF THE NORMAL AT THE POINT (x 1 , y 1 ) ON x 2 = 4ay


N.B. A normal at a point (x 1 , y 1 ) is the line perpendicular to the tangent, and so to the curve, at
that point.
104
y
Now the gradient of the tangent at P = m1 = x1
2a
2
.·. gradient of the normal is m 2 = - a as m 1 m 2 = -1
xl
.·. equation of the normal is:
2a
Y - Y1 = -x;-(x - x 1)

4. EQUATION OF THE NORMAL TO x 2 = 4ay AT THE POINT WITH


PARAMETER 't'
Gradient of the tangent at the point 't' = t
.·. gradient of the normal = _ _!_, as they are perpendicular
t
. ·. the equation of the normal is:
y - at 2 = _l_(x - 2at)
t
ty - at 3 = - x + 2at
.". x + ty = at 3 + 2at

On the parabola x 2 = 4ay (or x = 2at, y = at 2 ) the equation of the tangent at:
(x 1 ,y 1 )is: xx 1 =2a(y+y 1 )
t is: y - tx + at 2 = 0.
The equation of the normal at:
2
(xl,y 1)is: y- y 1 = - a(x- x 1 )
xl
t is: x + ty = at 3 + 2at.
The gradient of the tangent at a point 't' is t.
The gradient of the normal at a point 't' is _l.
t

N.B. Rather than memorise the above formulae students should master the methods used to obtain
them.

Example (i):
(a) Find the equations of the tangent and normal to x 2 = 4y at
(i) the point (2, 1)
(ii) the point t = -2.
(b) Find (i) the points of intersection of the two tangents
(ii) the points of intersection of the two normals.
105
Solution:
(a) (i) x 2 = 4y (ii) a = 1 and t = -2
xz .·. gradient of the tangent = -2
y=-
4 x = 2at
dy = 2x = 2 X 1 X -2
dx 4 = -4
X y = at 2
-
2 = 1 X ( -2) 2
=4
.'. gradient at (2, 1) = ~ .'. equation of the tangent is
= 1 y - 4 = - 2(x + 4)
.·. equation of the tangent at (2, 1) is 2x + y + 4 = 0 (2)
y - 1 = 1(x - 2) 1
gradient of normal = ---
X - y- 1 = 0 (1) -2
gradient of the normal at (2, 1) is = ~1 -
2
= -1 equation of normal
equation of normal at (2, 1) is y- 4 = t(x + 4)
y - l = - 1(x - 2) 2y- 8 =X+ 4
X+ y- 3 = 0 (3) X - 2y + 12 = 0 (4)

(b) (i) Intersection of tangents:


Solve equations (1) and (2)
x-y-1=0
2x+y+4=0
Adding
3x +!: ~ 1 }
.'. y = -2
.·. point of intersection of tangents is ( -1, - 2)
(ii) Intersection of normals:
Solve equations (3) and (4)
x+y=3
X - 2y = -12
Subtracting
3y = 15
y = 5 }
.'.X= -2
.·. point of intersection of the normals is (- 2, 5)

Example (ii):
Show that the line x - y - 2 = 0 is a tangent to the parabola x 2 = 8y.
State the coordinates of the point of contact.

Solution:
To show that the line is a tangent it is necessary to show that it cuts the parabola at only one point.
Thus there will be only one solution to the simultaneous equations
x 2 = 8y (1)
X- y- 2 = 0 (2)
xz
From(1) y =-
8
106
x2
Substitute in (2) x - S - 2= 0
8x- x 2 -16 = 0
x 2 - 8x +16 = 0
(x - 4) (x - 4) = 0
Thus the only solution is at x = 4 and y = 2
:. x - y - 2 = 0 is a tangent to x 2 = 8y at the point (4, 2)

1. Find the equations of the tangents and normals to the following parabolas:
(a) x 2 = 4y at ( -2, 1) (f) x = 6t, y = 3t 2 at t = 1
(b) x = 8y at (2, !)
2
(g) y = x 2 + 1 at (1, 2)
(c) x = 2yat(2,2)
2
(h) y = x 2 - x- 2at(-2,4)
2
(d) x = 2t, y = t at t = 2 (i) x = 2at, y = at 2 at t = 2
2
(e) x = 4t, y = 2t at t = -1 (j) x = 2at, y = at 2 at t = !.
2. Find: (i) the equation of the tangents
(ii) the equations of the normals
(iii) the point of intersection of the two tangents
(iv) the point of intersection of the two normals
for the following parabolas at the points indicated:
(a) x 2 = 2y at (- 2, 2) and (2, 2) (c) x = 2t, y = t 2 at t = 1 and t = -1
(b) x = l6y at (4, 1) and ( -8, 4)
2
(d) x = 6t, y = 3t 2 at t = 2 and t = 0.
3. (a) Prove that the line x - 2y - 2 = 0 is a tangent to the parabola x 2 = l6y and state the
coordinates of the point of contact.
(b) Prove that the line x + y + 3 = 0 is a tangent to the parabola x 2 = 12y. What is the point
of contact?
(c) Prove that the line x - y - 5 = 0 is tangential to the parabola x = lOt, y = 5t 2 •
Find the coordinates and the value of the parameter 't' at the point of contact.
4. Given the parabola x 2 = !y, for what values of m does the line y = m(x + 1):
(a) have two points of intersection?
(b) one point of intersection?
(c) no points of intersection?
5. For the parabola x = 6t, y = 3t 2 find:
(a) the equation of the tangent at t = 3
(b) the equation of the normal at t = -t.
What can be said about the two lines?
6. The line x - 2y + 8 = 0 cuts the parabola x 2 = 8y at P and Q. Find:
(a) the coordinates of P and Q
(b) the equations of the tangents at P and Q
(c) their point of intersection.
7. (a) Prove that the tangents at t = p and t = q on x = 4t, y = 2t 2 meet at (2(p + q), 2pq).
(b) Prove that the normals at these points meet at (- 2pq(p + q), 4 + 2(p 2 + pq + q 2 ) ).
8. Show that for the points 'p' and 'q' on the parabola x = 2at, y = at 2 :
(a) the tangents meet at (a(p + q), apq)
(b) the normals meet at ( -apq(p + q), a(2 + p 2 + pq + q 2)).
(c) If pq = 2, show by substitution into x 2 = 4ay that the point of intersection of the normals
lies on the parabola.
107
(d) Write down the points of intersection of:
(i) the tangents
(ii) the normals
to the parabola x = t, y = !t 2 at the points t = 2 and t = -1.
9. Find the equations of the tangents and normals to the parabola x = 2at, y = at 2 at the points
t = 1 and t = - l.
Show that these four lines form a square with diagonals intersecting at the focus.
10. The parabola x 2 = 4y is cut by the straight line 3x - 2y - 4 = 0 at P and Q.
Find: (a) the coordinates of P and Q
(b) the equations of the normals at P and Q
(c) show that the normals intersect on the parabola.
11. The points A, B, C and Dare the points t = -2, t = -1, t = 2 and t = -!respectively on the
parabola x = 2t, y = t 2 •
(a) (i) Find the equations of the tangents at C and D.
(ii) Show that their point of intersection lies on the directrix.
(b) (i) Find the equations of the normals at A and B.
(ii) Show that their point of intersection lies on the parabola.

y Consider the parabola x 2 = 4ay which divides the x, y plane into


three sets of points: points within the parabola, points on the
parabola, and points outside the parabola.
Let P(x 1 , y 1 ) be an exterior point to the parabola.
Let the line x = x 1 (through P, parallel to the y axis) cut the
parabola at T.
At T the x value is x 1 and let the ordinate be y 2 •
As T lies on the parabola x 2 = 4ay, at T
X 2
Y2 = - 1-
4a
X X 2
As y 1 < J'2, y 1 <
4~
2
4ay 1 < x 1 , a ;:::, 0
.'. x/ > 4ay 1
Thus any point (x 1, y 1 ) is external to the parabola x 2 = 4ay if x/ > 4ay1.
Let P(x 1 , y 1 ) be an external point to the parabola x 2 = 4ay and let Q be a point on the parabola
with parameter t.

y Now, the equation of the tangent at Q is


y - tx + at 2 = 0.
If Plies on this tangent then (.Xu y 1 ) satisfies the equation
so that
y 1 - tx 1 + at 2 = 0
or at 2 -tx1+Y1=0. (l)
This is a quadratic equation in 't' and x 1 , Y1 are constants
for the given point P.

108
4
Now, this quadratic has roots given by t = x 1 ± .Jx2a12 - ay 1 and will have two distinct roots when
x 1 2 > 4ay 1 , which is the condition for (x 1 , y 1 ) to be an external point.
Thus, from any external point two tangents can be drawn to the parabola.
Let the points of contact have t = t 1 at Q and t = t 2 at T, as in the diagram.
Then the equation of the chord, or secant, TQ is
2y- (t 1 + t 2 )x + 2at 1 t 2 = 0 (2)
Now, from Equation (1)
the sum of the roots= t 1 + t2 = x 1
a
the product of the roots = t 1 t 2 = y 1 .
a
Substituting in Equation (2)
2y- x1x + 2a. Y1 = 0
a a
2ay - x 1 x + 2ay 1 = 0
xx 1 = 2ay + 2ay,
= 2a(y + Y1)
and this is the equation of the chord of contact of the two tangents from an external point P(x 1 , y 1).

The chord of contact of the tangents to the parabola x 2 = 4ay from an external point (x 1 , y 1 ) is
xx 1 = 2a(y + Yt)

N.B. If P(x 1 , y 1) is on the parabola this gives the equation of the tangent at P.

Example:
Tangents are drawn from an exterior point P(2, -4) to the parabola x 2 = 8y.
Find (a) the equation of the chord of contact
(b) the points of contact of the tangents with the parabola.
Solution:
(a) The equation of the chord of contact is
xx 1 = 2a(y + Y1 ) where x 1 = 2; y 1 = -4
.'. 2x = 4(y - 4) and a = 2 as 4a = 8
2x = 4y- 16
2x- 4y + 16 = 0
x - + 8= 0 (dividing throughout by 2)
(b) To find the points of contact solve simultaneously
x 2 = 8y (1)
X - 2y + 8 = 0 (2)
x2
From (1): y = ~
2
~ +8=
2

Substitute in (2): x - 0
x2
x- +8=0
4
4x- x 2 + 32 = 0
x 2 - 4x- 32 = 0
(x - 8)(x + 4) = 0
.'. x = 8 or x = -4
and from (1) y = 8 or y = 2.
Thus the points of contact are (8, 8) and (- 4, 2).

109
1. In each of the following:
(a) check that the given point is external to the parabola
(b) find the equation of the chord of contact of the tangents drawn to the parabola from the
given point.
(i) x 2 = 4y from (3, 1) (iv) x = 4t, y = 2t 2 from (2, -1)
(ii) x 2 = 12y from (6, 0) (v) x = 6t, y = 3t 2 from (4, 1)
(iii) x = y from (3, - 2)
2
(vi) x = 2t, y = t 2 from (0, -1)
2. Tangents are drawn to the parabola x 2 = 8y from an external point P( 4, 0). Find:
(a) the equation of the chord of contact
(b) the coordinates of the points of contact
(c) the equations of the two tangents from P.
3. Show that the point P(6, -4) is exterior to the parabola x 2 = 16y. Also find:
(a) the equation of the chord of contact of the tangents from P
(b) the points of intersection of this chord with the parabola
(c) the coordinates of the mid-point of the chord
(d) show that the chord of contact is a focal chord.
4. Tangents are drawn to the parabola x 2 = 16y from the points P(O, -4) and Q(2, -2).
(a) Find: (i) the equation of the chord of contact of the tangents from P
(ii) the equation of the chord of contact of the tangents from Q.
(b) Show that the two chords intersect at a point on the given parabola.
(c) Show that the chord of contact of the tangents from Pis bisected at the focus.
(d) Show that the normals from the ends of this chord meet on the axis of the parabola.
5. (a) Show that the point R(- It, -1) is external to the parabola x 2 = 4y.
(b) Find the equation of the chord of contact of the tangents from R.
(c) If this chord meets the parabola at P and Q, find the coordinates of P and Q.
(d) Find the coordinates where PQ (or PQ produced) cuts
(i) the x axis
(ii) the y axis
(iii) the directrix (at the point T).
(e) If Sis the focus of the parabola show that
LTSR = LPRQ = 90°
6. Tangents are drawn to the parabola x 2 = 8y from the point P(l, -3).
(a) Find the equation of the chord of contact.
(b) If the points of contact are Q and R, find their coordinates.
(c) If M is the mid-point of the chord QR show that MP is parallel to the axis ofthe parabola.
(d) (i) Find the coordinates of the mid-point of PM and
(ii) show that it lies on the given parabola.

The parabola has many geometrical properties which may be proved by means of analytical geometry.
When this method is used, coordinate axes are chosen so as to make the equation of the parabola
as simple as possible.
Thus, when proving these general properties, use the standand parabola, x 2 = 4ay (or x = 2at,
y = at 2 ), where
110
the vertex is at the origin, 0 y
the y axis is the axis of the parabola
the x axis is the tangent at the vertex.
In addition, the focus Sis (0, a)
the directrix is the line y = - a
the focal length is a.
General points are expressed in terms of parame-
ters p, t, q, etc.
A diagram should be sketched for each
example.

~--------a -------~
y =-a

Example (i):
Prove that the tangent at an arbitrary point P on the parabola is equally inclined to the axis of the
parabola and the focal chord through P.
Solution:
y Let the axis be chosen so that the parabola has the
equation
x 2 = 4ay.
Let P(2ap, ap 2 ) be a general point on the parabola.
S(O, a) is the focus
PSP' is the focal chord through P.
Let the tangent at P meet the axis at T.
It is necessary to prove that
LSPT = LSTP.
It is thus sufficient to show that
L
X
SP =ST.

K-- Let PK be drawn perpendicular to the directrix


y = -a, cutting the x axis at L.

Now from the definition of the parabola, the distance of P from the focus equals the distance of P
from the directrix
i.e. PS = PK
= PL + LK (from diagram)
2
= ap +a (distance is always positive)
i.e. PS = ap 2 + a (1)
Now the equation of the tangent PT is
y- px + ap 2 = 0
This cuts the axis of the parabola when x = 0:
.'. y = -apz
Thus Tis the point (0, -ap 2 )
Now ST= SO+ OT
=a+ ap 2
111
.'. from (1) ST = PS
LSPT = LSTP (opposite equal sides),
and so the tangent is equally inclined to the focal chord, PP', and the axis of the parabola, ST.

N.B. This property has great practical application in parabolic reflectors for telescopes, searchlights
and other instruments requiring the use of parallel rays.
y Consider a cross-section through a parabolic reflector.
L
A ray of light LP, parallel to the axis yO meets the surface
of the parabola at P and is reflected in relation to the tangent
at P.
Now as LPIIYT
the angle of incidence LPA = corresponding angle
STP = angle of reflection SPT by the previous result.
Thus, the reflected ray passes through the focus S.
As the same result applies for all such light (or radio) waves,
all the rays will concentrate at the focus. Another use would
be to collect heat rays from the sun to give an intense heat
source at S.
T For a searchlight or spotlight, the light source would be at
the focus and the light rays would be reflected as a cylinder of
parallel rays.

Example (ii):
Prove that the tangents to a parabola at the extremities of a focal chord intersect at right angles on
the directrix.
Solution:
y Let the parabola be x 2 = 4ay.
P(2ap, ap 2 ) and Q(2aq, aq 2 ) are the extremities of a
focal chord PQ.
Let the tangents at P, Q meet at T.
It is necessary to prove
(i) T lies on the directrix y = -a
(ii) LPTQ = 90°.
Now the equation of the chord PQ is :
Y - t(p + q)x + apq = 0.
As PQ is a focal chord it passes through S(O, a)
i.e. (0, a) satisfies the equation
.'.a+ apq = 0
apq = a
pq = -1 (1)

The equation of the tangent from P is: y - px + ap 2 =0 (2)


The equation of the tangent at Q is: y - qx + aq 2 =0 (3)
To find the co-ordinates ofT solve these equations.
equation (3) - equation (2):
(p - q)x + a(q 2 - p 2 ) = 0
(p - q)x = a(pz - qz)
x = a(p + q), p -=1- q.
112
substituting in equation (2):
y px- ap 2
=
ap(p + q) - ap 2
=
= ap 2 + apq - ap 2
= apq
but from equation (1), pq = -1
.'. y =-a
Thus T(a(p + q), -a) lies on the directrix y = -a. (i)
Also the gradient of PT = p
the gradient of QT = q
and pq = - 1, from equation (1)
.'. PT is perpendicular to QT (product of gradients = -1)
i.e. LPTQ = 90°. (ii)
Thus, the tangents at the ends of a focal chord intersect at right angles on the directrix.

Example (iii):
P and Q are points on the parabola x 2 = 4ay with parameters p, q. The tangents at P and Q meet
at T and M is the mid-point of PQ. MT is drawn to cut the parabola at N. Prove that
(a) MT is parallel to the axis of the parabola
(b) N is the mid-point of MT
(c) the tangent at N is parallel to the chord PQ.
Solution:
Let P(2ap, ap 2 ) and Q(2aq, aq 2 ) be two points on the parabola x 2 = 4ay.
PT, QT are tangents meeting at T.
M is the mid-point of PQ and MT meets the parabola at N.
y From Example (ii)
Tis (a(p + q), apq)
Now, the mid-point M has coordinates
2ap + 2aq ap + aq )
2 2

( 2 ' 2
= ( a(p + q), a(pz 2+ qz))

Thus M and T have the same abscissa


and so MTjjyO, the axis of the parabola. (1)
Now at N, x = a(p + q) and this satisfies the
equation of the parabola x 2 = 4ay .
.'. 4ay = a 2 (p + q) 2
a(p + q)z
y=
4
2
i.e. N is ( a(p + q), a(p : q) )

a(pz + qz) + apq )


2
Also, the mid-point of MT is ( a(p + q),
2
= (a(p + q), a(pz + ~2 + 2pq))

= (a(p + q), a(p : q)z)

= coordinates of N
Thus N is the mid-point of MT. (2)
113
xz also, gradient of the chord PQ
Now,y = a
4 apz- aqz
and dy = 2x 2ap- 2aq
dx 4a
a(p + q)(p - q)
= ~which is the gradient of the tangent at (x, y) 2a(p- q)
2a
=p;q, p=l-q
.·. gradient of the tangent at N = a(p + q)
2a
=p+q
2
.·. gradient of the tangent at N = gradient of the chord PQ
_p+q
- 2
i.e. chord PQIItangent at N. (3)

1. Pis the point (2ap, ap 2 ) on the parabola x 2 = 4ay. The tangent at P meets the axis of the parabola
at T and P N is drawn perpendicular to the axis, meeting it at N. The directrix meets the axis at A.
(a) Prove: (i) OS= OA (0 is vertex, S the focus)
(ii) ON= OT
(iii) the x axis bisects PT at a point, say B.
(b) On the diagram for part (a), PM is drawn perpendicular to the directrix, meeting it at M and
cutting the x axis at C.
(i) Find the equation of the line SM.
(ii) Show that B lies on SM.
(iii) Show that B is the mid-point of OC.
(iv) Show that SB 2 = SO . SP.
(v) Show that triangle TOE is congruent to the triangle PBC.
(vi) What type of figure is TMPS?
2. At a point P(2ap, ap 2 ) on the parabola x 2 = 4ay a tangent is drawn to cut the axis of the parabola
at T and the normal from P meets the axis in G.
If S is the focus:
(a) prove that ST = SG and
(b) if a line PN is drawn perpendicular to the axis, meeting the axis inN, prove that the length
of NG is a constant length for all positions of P.
3. Prove that if the tangent from a point P on a parabola meets the directrix at A, then the angle
ASP is a right angle.
4. If the chord of contact of the tangents to a parabola x 2 = 4ay, from an external point T(x 1 , y 1 )
meets the directrix at R, prove that RT subtends a right angle at the focus.
5. (a) If P(2ap, ap 2 ) and Q(2aq, aq 2 ) are points on the parabola x = 2at, y = at 2 and Sis the focus,
prove that PS + SQ = a(p 2 + q 2 + 2). Hence if PQ is a focal chord, show that the length

of PQ = a(p +~ Y.
(b) If the focal chord PQ is parallel to the x axis (the latus rectum) prove that PQ = 4a.
2
1
(c) Show that the gradient of a focal chord through Pis P -
2p
6. A straight line through T(O, -a) cuts the parabola x 2 = 4ay at P and Qwith respective parameters
p and q.
(a) Show that the equation of TP is 2py = x(p 2 + 1) - 2ap.
114
(b) Prove that, for TP to pass through Q, pq = 1.
1 1 1
(c) Hence prove t h a t - + - = - .
SP SQ a

7. Pis the point (2ap, ap 2 ) on the parabola x 2 = 4ay with focus S. The normal from P meets the
axis of the parabola at G and the latus rectum produced at A. PN, perpendicular to the axis of the
parabola, meets it at N.
Prove that:
NP (b) SA= NP
(a) NG = p SP NG.

8. The tangent from P, at one extremity of a focal chord PQ in the parabola x 2 = 4ay, meets the
latus rectum produced at L.
SP SL
Prove that SL =SQ.

9. Prove that the normals at the ends of a focal chord in a parabola are perpendicular to each other.

10. Pis a point on the parabola x = 2at, y = at 2 • From Q, the mid-point of OP, (0 is the vertex),
QA is drawn parallel to the axis of the parabola and meets the x axis at A.
Prove:
(a) AP is a tangent at P.
(b) K, the mid-point of AQ, lies on the given parabola.

11. The straight line drawn from a point P on a parabola to the vertex intersects the directrix at A.
Prove that AS (where Sis the focus) is parallel to the tangent at P.

12. PQ is a focal chord of the parabola x 2 = 4ay. Perpendiculars PA, QB are drawn from P, Q to
meet the directrix at A, B respectively.
Prove that P B, QA intersect at the vertex 0.

13. On the parabola x 2 = 4ay, the normal from a point P (parameter p) meets the parabola again
at R (parameter r). By using the fact that R(2ar, ar 2 ) satisfies the equation of the normal:
(a) Prove that p 2 + pr + 2 = 0.
(b) If the chord P R subtends a right angle at the vertex show that:
(i) pr = -4 (ii) p 2 = 2.
(c) The normal from another distinct point Q (parameter q) on the parabola also passes through R.
(i) What relationship exists between q and r?
(ii) Show that (p 2 - q 2 ) + r(p - q) = 0 and hence that p + q + r = 0.
(iii) Show that (p + q) 2 + r(p + q) + 4 = 2pq and hence thatpq = 2.
(iv) Prove that PQ intersects the axis of the parabola at an independent point.

14. Pis a given point with parameter p on the parabola x 2 = 4ay, with focus S. A line is drawn from
S, perpendicular to SP and meets the normal at Pin the point Q. P N, QM are drawn perpendicular
to the axis of the parabola.
(a) Find the ordinate at Q.
(b) If D is the intersection of the directrix and the axis of the parabola, prove that DM = 2DN.

15. (a) If the normal at P (2ap, ap 2 ) on the parabola x 2 = 4ay meets the parabola again at Q:
(i) Show that the coordinates of Q are ( -2a( p + 2) , a( p +
2 2 2
2) ) .
2
3 p p
2
4 1
(ii) Prove that the length of PQ is a( +2 p )>.
p
(b) If PQ subtends a right angle at the focus Sand M is the foot of the perpendicular from S to
the tangent at P
(i) find the coordinates of M; (ii) find the lengths SM, SP.
115
The problem is to find the locus of some given point as it moves in relation to a given parabola. For
example, find the locus of the mid-point of focal chords in a parabola.
The aim is to form an equation in x and y by eliminating other variables, usually the parameters
of given points.
To do this use is made of
(a) The general results known for the parabola
• equation of a tangent
e equation of a normal
• equation of chord of contact
e if PQ is a focal chord then pq = -1
e equation of a chord PQ.
(b) general algebraic results
• p2 + q2 = (p + q)2 - 2pq
• p3 - q3 = (p - q)(p2 + pq + q2)
• p3 + q3 = (p + q)(p2 - pq + q2).
(c) any information given in the question.

Example (i):
The parabolax 2 = 4ay has focus Sand vertex 0. At a pointP(2ap, ap 2), which moves on the parabola,
tangents are drawn and from S, a perpendicular SM is drawn to meet the tangent at M.
Find the locus of M.
Solution:
y Let M have coordinates (x, y).
Equation of PM is y- px + ap 2 = 0 (1)
Now the gradient of PM= p
1
.·. the gradient of SM =
p
Thus the equation of SM is
y- a= _ _!_(x- 0)
p
py- ap = -x
py + x- ap = 0 (2)

To find the coordinates of M, solve equations (1) and (2):


equation (2) x p: p 2y + px - ap 2 = 0 (3)
equation (1) +equation (3): (1 + p 2 )y = 0
y = 0.
This is independent of p (and of x) and so the locus of M has equation y = 0.
.·. The required locus is the x axis.

Example (ii):
Show that the locus of the mid-point of a set of parallel chords in a parabola is a line parallel to the
axis of the parabola.
116
Solution:
Consider the parabola x 2 = 4ay. Let PQ be one
position of the chord and let M be its mid-point.
Now the gradient of PQ is
ap 2 - aq 2 _ a(p + q)(p - q)
2ap - 2aq - 2a(p - q)
= p ; q as p =F q.
As this is constant for all the parallel chords, let
p; q= c,say (a)

2
.
Now M has coordmates (2ap + 2aq , ap + aq 2 ) = ( a(p a 2 + q 2 ))
+ q), 2(p ·
2 2
Thus at M, x a(p + q) =
2ac = from (a)
i.e. the mid-point of the parallel chords has abscissa given by x = 2ac where a and c are constants.
Thus the required locus is the line x = 2ac; a line parallel to the axis of the parabola.

Example (iii):
The normals to the parabola x 2 = 4ay at points p, q intersect at R. If the chord PQ varies so that
(when produced) it always passes through a point T(O, - 2a), show that the locus of R is the given
parabola.
Solution:
Consider the parabola x 2 = 4ay. Let PQ be one such chord passing, when produced, through
T(O, - 2a). The normals at P and Q meet at R.

The equation of PQ: 2y - (p + q)x + 2apq = 0

-----
\\
\
y
As it passes through T(O, - 2a)
- 4a - 0 + 2apq = 0
2apq = 4a
(a)
P(2ap, ap') pq = 2
\ Equation of the normal PR:
\ x + py = ap 3 + 2ap (1)
\ equation of the normal QR:
\
\ x + qy = aq 3 + 2aq (2)
\ To find R solve Equations (1) and (2):
\ Equation (1) - Equation (2):
\ (p - q)y = a(p 3 - q 3 ) + 2a(p - q)
\ (p - q)y = a(p - q)(p 2 + pq + q 2 ) + 2a(p - q)
\
Dividing both sides by p - q, p =F q.
y = a(p 2 + pq + q 2 ) + 2a ({3)
X
Substitute in Equation (1)
x = ap 3 + 2ap - py
2 2
= ap 3 + 2ap - p(ap + apq + aq + 2a)
= ap + 2ap- ap - ap q- apq 2 - 2ap
3 3 2

= -apq(p + q) . (y)
117
Now for all chords PQ through T, from Equation (o:)
pq = 2
.'. x = - 2a(p + q) }
and y = a(p 2 + pq + q 2 ) + 2a
= a(p 2 + 2pq + q 2 ) + 2a - apq
= a(p + q) 2 + 2a - 2a, as pq = 2
= a(p + q)z
.'. Y = a(p + q)z
x = -2a(p + q)
~~~
} the coordinates of R .
Square Equation (4):
xz = 4az(p + q)z
= 4ay as y = a(p + q) 2 from Equation (3)
Thus the locus of R is the parabola x 2 = 4ay.

1. From a point P on the parabola x 2 = 2y, a tangent is drawn. From the focus S, a perpendicular
is drawn to meet the tangent at R.
(a) Find the equation of SR.
(b) Find the locus of R.
2. Show that the locus of the mid-points of chords in the parabola x 2 = 4ay, and which pass through
the vertex, is another parabola, x 2 = 2ay.
3. Two points P, Q move on the parabola x 2 = 4ay so that the x coordinates of P and Q differ by
a constant value, 2a. What is the locus of M, the mid-point of PQ?
4. Prove that the locus of the mid-point M of focal chords in the parabola x 2 = 4ay is the parabola
x2 = 2a(y - a).
5. P 1 , P2 are points on the parabola x 2 = 4ay with parameters p and{,. If the tangents at P 1 and P2
intersect at R, prove that the locus of R is the line y = a.
6. At a point P on the parabola x 2 = 4ay, a normal PK is drawn. From the vertex 0 a perpendicular
OM is drawn to meet the normal atM. Show that the equation of the locus of Mas P varies
on the parabola is
x 4 - 2ax 2y + x 2 y 2 - ay 3 = 0.
7. The chord PQ in the parabola x 2 = 4ay subtends a right angle at the vertex of the parabola.
The normals to the parabola at P and Q meet at R.
(a) Prove that pq = -4, where p, q are the parameters at P and Q.
(b) Show that as P, Q take various positions on the parabola, the locus of R is the parabola,
x 2 = 16a(y - 6a).
8. (a) P and Q are points on the parabola x = 2at, y = at 2 • The tangents at P and Q intersect at R
so that the angle P RQ equals 90°. Show that the locus of R is the directrix.
(b) If PQ is a focal chord and the tangents at P and Q meet at R, show that the locus of R is the
directrix, y = -a.
9. If PQ is a focal chord of the parabola x 2 = 4ay; QR is the tangent at Q and RP is parallel to
the axis of the parabola, prove that the locus of R has as its equation
x 2 (y + 2a) = -4a 3 .
10. PQ is a focal chord in the parabola x 2 = 4ay. Normals are drawn at P and Q, at the ends of the
focal chord to meet at a point R. Find the locus of R for varying focal chords PQ.
118
11. Tangents are drawn to a parabola x 2 = 4y from an external point A (x 1 , y 1), touching the parabola
at P and Q.
(a) Prove that the mid-point, M, of PQ is the point (x 1 , tx/- y 1 ).
(b) If A moves along the straight line y = x - 1, find the equation of the locus of M.
12. Pis a variable point (6p, 3p 2 ) on the parabola x 2 = 12y, with focus S. SL is drawn perpendicular
to the tangent at P, meeting it at L. SM is drawn perpendicular to the normal at P, meeting it
atM.
(a) Find the equation of the locus of L.
(b) Find the equation of the locus of M.
13. If PN is a normal to the parabola x 2 = 4ay at a variable point P and SN is drawn through the
focus S parallel to the tangent at P to cut the normal at N.
Prove that the locus of N is x 2 = a(y - a).
14. Pis a point on the parabola x 2 = 4ay with vertex at the origin. A straight line through 0, parallel
to the tangent at P intersects the parabola again at Q. If the tangents at P and Q meet at T, show
that the locus ofT as P moves on the parabola is given by x 2 = ~ay.
15. PQ is a focal chord in the parabola x 2 = 4ay.
(a) PT is drawn parallel to the tangent at Q and QT is drawn parallel to the tangent at P.
Show that the locus ofT is x 2 = a(y - 3a).
(b) If M is the mid-point of the focal chord PQ and a line through M, parallel to the axis of the
parabola, meets the normal at Pin A, find the locus of A.

119
Sine and cosine curves are often seen on the screen
of an oscilloscope.
CHAPTER 24

Some earlier trigonometrical results are of importance for the work in this chapter.
1. Consider a unit circle with centre 0. A point P(x, y) is
e
taken on the circle and OP makes an angle with the x
aXlS.
Then, X= 1 . cos e and y = 1 . sin e
=cos e = sine
Thus, Pis the point (cos e, sin 8) (1)

2. The cosine rul~, used to find the side of a triangle, given


two sides and the included angle, is given by
d 2 = a 2 + b2 - 2ab cos e (2)

3. Some basic trigonometrical relationships:


(a) sin (-e)= -sine
(b) cos (- e) = cos e
(c) tan (- e) = - tan e
(d) cos (90° - 8) = sine (3)
(e) sin G - e) = cos 8
(f) sin 2 8 + cos 2 8 = 1
4. From coordinate geometry, if dis the distance between two points P(x 1 , y 1 ) and Q(x 2 , y 2 )
then d 2 = (x 1 - x 2 ) 2 + (y 1 - Y2) 2 (4)

Consider a unit circle, with centre 0. P, Q are two points on


the circumference so that PO makes an angle 8 with the x axis;
QO makes an angle¢ with the x axis .
.'. LPOQ = LPOX- LQOX
= (8- ¢).
p has coordinates (cos e, sin 8); Q has coordinates
(cos¢, sin¢).
OP = OQ = 1 unit.
Let the length of PQ = d.
Then in the l::,.POQ, by the cosine rule, result (2)
d 2 = 12 + 12 - 2 X 1 X 1 X COS (8 - cp)
d 2 = 2 - 2 cos (8- ¢) (a)

122
Also, using the distance formula, result (4)
d 2 = (cos() - cos ¢) 2 + (sin() - sin ¢) 2
= (cos 2 () - 2 cos() cos¢ + cos 2 ¢) + (sin 2 () - 2 sin() sin¢ + sin 2 ¢)
= (cos 2 () + sin 2 ()) + (cos 2 ¢ + sin 2 ¢) - 2 cos() cos¢ - 2 sin() sin¢
2
."o d = 1 + 1 - 2(cos ()cos¢ + sin() sin¢)
= 2 2(cos ()cos¢ + sin() sin¢) ({J)
Comparing (o:) and ({J):
2 - 2(cos e cos¢ + sine sin¢) = 2 - 2 cos(() - ¢)
On simplifying:
cos(() - ¢) = cos() cos¢ + sin() sin¢ (I)
Now, if¢ is negative,
let¢= -1/1, say where 1/J > Oo
Substitute in (I)
cos [ () - (- tjJ)] = cos () cos ( -1/1) + sin () sin ( -1/1)
°0 cos (e + 1/J) = cos ()cos 1/J - sin () sin 1/J
0 (II)
as cos ( -1/1) =cos 1/1 and sin ( -1/1) = -sin 1/1
If in (I), let e = 90°, then:
cos (90° - ¢) = cos 90° cos ¢ + sin 90° sin ¢
= sin¢, as cos 90° = 0, sin 90° = 1
."o sin(() + ¢) = cos [90° - (() + ¢)]
=cos [(90° - ())- ¢]
= cos (90° - ()) cos ¢ + sin (90° - ()) sin ¢
0 from (I)
sin (() + ¢) = sin ()cos ¢ + cos () sin ¢
." 0 (III)
If¢ is negative, let¢ = -1/1 where 1/J > 00
Substitute in (III):
sin [ () + (- 1/J) J = sin () cos ( -1/1) + cos () sin ( -1/1)
."o sin(() - 1/J) = sine cos 1/1 - cos() sin 1/1 (IV)

Now, tan(() + ¢) = sin(() + ¢)


cos(()+¢)
sin () cos + cos () sin
from (II) and (III)
cos e cos ¢ - sin () sin ¢
Divide numerator and denominator by cos () cos ¢ :
sin () cos + cos e sin ¢
cos () cos ¢ cos () cos ¢
cos () cos ¢ sin () sin ¢
cos () cos ¢ cos () cos ¢
tan (e + ¢) = tan () + tan ¢
.. 0
(V)
1 - tan () tan ¢
Finally,
tan (e - ¢) = sin (() - ¢)
cos(() - ¢)
- sin () cos ¢ - cos e sin ¢
from (IV) and (I)
- cos () cos ¢ + sin () sin ¢ '
Divide numerator and denominator by cos () cos ¢
sin () cos ¢ cos () sin ¢
cos () cos ¢ cos () cos ¢
cos e cos ¢ + sin () sin ¢
cos () cos ¢ cos () cos ¢
."o tan(() _ ¢) = tan() - tan¢ (VI)
1 + tan e tan¢
123
These results are true for all values of the angles and so are identities.
Thus, the symbol =, meaning 'is identically equal to', may be used.
Thus,
sin (A + B) = sin A cos B + cos A sin B
sin (A - B) = sin A cos B - cos A sin B
cos (A + B) = cos A cos B - sin A sin B
cos (A - B) = cos A cos B + sin A sin B
tan (A + B) = tan A + tan B
l -tan A tan B
tan (A _ B) = tan A - tan B
1 +tan A tan B

Example (i):
(a) Without using tables, find the exact value of sin 75°.
(b) Show, using the 'addition' formula, that sin 75° = cos 15°.
Solution:
(a) sin 75o =sin (45° + 30°)
=sin 45° cos 30° + cos 45° sin 30°
1 J3
1 1
=-·-+-·-
J22 J22
= \,(J3 + 1)
2-y2

= f(J3 + 1)
_J6+J2
4
(b) cos 15° =cos (45 - 30t
= cos 45o cos 30° + sin 45° sin 30°
=-1 .J3 +-1 .!
J22 J22
= 1!'l(J3 + 1)
2-y2
= sin 75° from (a).

Example (ii):
1
Prove that (a) tan (45° + A) = + tan A
1 - tan A
(b) sin (90° + A) = cos A.
Solution:
(a) tan (45 o + A) = tan 45° + tan A
l - tan 45o tan A
=
1 + tan A as tan 45° = 1
l -tan A
(b) sin (90° + A) = sin 90° cos A + cos 90° sin A
= 1 x cos A + 0 x sin A
=cos A.

Example (iii):
Prove that sin (A + B) sin (A - B) =sin 2
A - sin 2 B.
124
Solution:
sin (A + B). sin (A - B) =(sin A cos B + cos A sin B)(sin A cos B- cos A sin B)
=sin A cos B - cos A sin B, as (a + b)(a - b) = a b
2 2 2 2 2
-
2

=sin A(1 - sin B) - (I - sin A) sin B, as cos A = 1 - sin


2 2 2 2 2 2
A
=sin A - sin A sin B - sin B + sin A sin B
2 2 2 2 2 2

= sin 2 A - sin 2 B
Example (iv):
If A, Bare acute angles and sin A =!,cos B = g evaluate sin (A + B).
Solution:
As sinA =!, As cos B = g
and cos A= t and sin B = 5
13

12
4

.·. sin (A + B) = sin A cos B + cos A sin B


=!. g + t ·l3
- 63
-65

1. Write down the expansion for each of the following:


(a) sin (a + /3) (e) tan (2x - y)
(b) cos (a - f3) (f) tan (3A + 2B)
(c) sin (2x + y) (g) sin (3A + B) + sin (3A - B)
(d) cos (3x - 2y) (h) cos (2x - y) - cos (2x + y)
2. Write down an equivalent expression consisting of one term for each of the following:
tan 3A + tan 2B
(a) cos a cos f3 + sin a sin [3 (g)
1 - tan 3A tan 2B
tan 2a - tan 40°
(b) sin a cos f3 + cos a sin [3
(h) 1 + tan 2a tan 40°
(i) tan 30° - tan 20°
(c) sin 2a cos [3 - cos 2a sin [3
1 + tan 30° tan 20°
(d) cos 2A cos 2B - sin 2A sin 2B (j) sin 15° cos 60° - cos 15° sin 60°
(e) cos E sin 2F- sinE cos 2F (k) cos 5A sin 2A + cos 2A sin 5A
(f) sin A cos 40° + cos A sin 40° (1) cos 3A cos 2A + sin 3A sin 2A
3. Find exact values for each of the following expressions:
(a) cos 75° (f) tan 105°
(b) sin 15° (g) sin 10° cos 20° + cos 10° sin 20°
(c) tan 75o (h) cos 50° cos 10° - sin 50° sin 10°
(d) tan 15° (i) sin 75° cos 15° - cos 75° sin 15°
(e) sin 10 5o (j) tan 20° + tan 25°
1 - tan 20° tan 25°
125
4. Simplify each of the following expressions:
(a) sin (A + B) + sin (A - B)
(b) cos (A + B) + cos (A - B)
(c) cos (x + y) cos (x - y)
(d) sin (2x + y) - sin (2x - y)
(e) sin (P + Q) cos (P + R) - cos (P + Q) sin (P + R)
(f) cos (x + y) cos x + sin (x + y) . sin x
(g) sin (A + B - C) cos (A B + C) + cos (A + B - C) . sin (A - B + C)
(h) sin (A + B)
cos A cos B

5. (a) If sin rx =!,sin f3 = t3 , find:


(i) sin (rx + {3) (iii) cos ((X + {3)
(ii) cos ((/. - {3) (iv) sin ({3 + {3),
when 0 < rx < ~ and 0 < f3 < ~.
(b) If rx, f3 are acute angles with sin rx = ~, sin f3 = ! , evaluate:
(i) sin (rx + {3)
(ii) cos (rx - {3).
Use your calculator to find the size of the angle (X + {3, (X - {3. Give answers in degrees,
correct to two decimal places.
(c) The angles A, Bare both acute and sin A = !, cos B = ft' Find exact values for:
(i) cos (A + B) 2
(ii) sin (B - A).
Giving answers in degrees, correct to two decimal places, use calculators to find the size of the
angles (A + B), (B - A).
t
(d) Given that cos rx = 187 , 0 < rx < ~;tan f3 = 2 , 0 < f3 < ~;find exact values for:
(i) sin (rx - {3)
(ii) cos (rx + {3)
(iii) tan (rx + {3).
Use these results to find the values of ((X + {3), ((X - {3) in degrees, correct to two decimal
places. Use these solutions to calculate values for (X and {3. Check these answers directly from
the given values for cos rx and tan {3.
(e) IfsinA = -~andn <A< 32;cosB = -t3 and~ < f3 < n:
(i) find exact value~ for cos A, tan A, sin Band tan B;
(ii) calculate the size of the angles A and Bin degrees, correct to two decimal places. Hence,
write down the size of the angle (A - B).
(iii) Find the exact values for sin (A - B); cos (A - B); tan (A - B). Check the magnitude
of the angle (A - B) in each case.

6. Prove the following identities:


(a) sin(45° +(X)= ft(cos(X +sin (X)

(b) cos (A - 60°) =~(J3 sin A + cos A)


(c) cos ( ~ + x) =ft(cos x sin x)

(
n) _J3 tan x+ 1
(d) tan x. +- = Pi
6 v 3- tan x
(e) sin ( ~ + x) - sin ( ~ - x) =J2 sin x
(f) tan ( 8 _ !!_) =tan 8 - 1 =sin 8 - cos 8.
4 1 + tan 8 cos 8 + sin 8
126
7. Prove the following identities:
(a) cos (A + B) cos B + sin (A + B) sin B =cos A
(b) sin (A - B)
cos A cos B
= tan A - tan B

sin (A - B)
(c) .
smA smB
. = cot B - cot A

(d) cos (A + B) =cot A cot B - 1


sin A sin B
(e) cos (A + B). cos (A - B) =cos 2
A - sin 2 B
(f) tan A - tan B =sin (A - B)
tan A + tan B sin (A + B)
(g) sin (A + B) + sin (A - B) = t A
- w .
cos (A + B) + cos (A - B)
8. (a) Prove that:
=
(i) sin (30° + 8) + sin (30° - 8) cos 8
(ii) cos (30° - 8) - cos (30° + 8) = sin 8.
(b) (i) Write down exact values for sin 120° and cos 120°.
(ii) Prove that cos rx + cos (120° + rx) + cos (120° - a) = 0.
(iii) Prove that sin rx + sin (120° + a) - sin (120° - rx)= 0.
(c) (i) Iftan rx = !, 0 < rx < -I; tan f3 = t 0 < f3 < -I; show that rx + f3 = lj.
(ii) If the angles a, f3 are acute and cos IX = -t;
cos f3 = U, show that rx - f3 = 1-.
(d) Find an exact value for tan B if tan (A + B) = i and tan A = t.
9. (a) By using two expansions show that:
sin (A + B + C) =
sin A cos B cos C + cos A sin B cos C + cos A cos B sin C
- sin A sin B sin C.
By using the fact that tan A sin A and so sin A = cos A tan A, show that:
=
cos A
sin (A + B + C) =cos A cos B cos C (tan A + tan B + tan C- tan A tan B tan C).
(b) Similarly, show that cos (A + B +C)
= cos A cos B cos C - sin A sin B cos C - sin A cos B sin C - cos A sin B sin C
= cos A cos B cos C (1 - tan A tan B- tan B tan C - tan C tan A).
(c) Hence show that:
_ tan A + tan B + tan C - tan A tan B tan C
tan (A + B + C) = .
1 - tan A tan B - tan B tan C - tan C tan A
10. (a) In any triangle ABC, write a relationship between the angle A and the sum of the angles B
and C. Thus prove, that in the triangle ABC
sin A = sin B cos C + cos B sin C.
Deduce that
a = b cos C + c cos B.
(b) Using the formula for the area of a triangle, A = !ab sin C, and referring to the diagram:
(i) express the area l:J.PQR in terms of r, q
p
and the angle (a + /3);
(ii) write similar area formulae for the tri-
angles PQM, P RM using respectively
the angles rx, f3;
(iii) from the right-angled triangles PQM,
P RM express:
t in terms of r and rx; Q R
tin terms of q and /3.
(iv) By using the relationship between the areas of the three triangles mentioned above,
prove that:
sin (rx + /3) = sin rx cos f3 + cos rx sin f3.
127
sin 2A
The identity, sin (A + B) =
sin A cos B + cos A sin B, holds for all real values of A and B.
Let B =A
.'. sin (A +A) =
sin A cos A + cos A sin A
. ·. sin 2A =
2 sin A cos A (I)

cos 2A
In the identity, cos (A + B) =
cos A cos B - sin A sin B,
let B =A
.'. cos (A +A) = cos A cos A - sin A sin A
.'. cos 2A = cos 2 A - sin 2 A (II)
Now, as cos 2 A + sin 2 A = 1, cos 2 A = 1 - sin 2 A
sin 2 A = 1 - cos 2 A.
2
Put cos A = 1 - sin A 2
from (II)
cos 2A = 1 - sin 2 A - sin 2 A
.'. cos 2A
2
=
1 - 2 sin 2 A (Ila)
Put sin A = 1 - cos 2 A from (II):
cos 2A = cos 2 A - (1 - cos 2 A)
.'. cos 2A = 2 cos 2 A - 1 (lib)

tan 2A

tan (A +B) = 1tan- A+ tan B


tan A tan B
Put B =A:
tan (A + A) = 1tan-tan
A + tan A
A tan A
2 tan A
.'.tan 2A = (III)
1 -tan 2 A

Formulae for double angles:


sin 2A =2 sin A cos A
cos 2A =cos 2 A - sin 2 A
=1 - 2 sin 2 A
= 2 cos 2 A - 1
tan 2A =1 2- tantanA A
2

One important application of these results is in the field of integration.

1. Isin 2 xdx.
Now as cos 2x = 1 - 2 sin 2 x
2 sin 2 x = 1 - cos 2x
sin 2 x = !(1 - cos 2x)

.'. I 2
sin xdx = ~ J(1 - cos 2x)dx
= !(x -!sin 2x) + c
128
2. Jcos 2
xdx.

As cos 2x = 2 cos 2 x - 1
2 cos 2 x = 1 + cos 2x
cos 2 x = !(1 + cos 2x)
.". Jcos 2
xdx = ~ f(1 + cos 2x)dx
= !(x +!sin 2x) + c

.·. f sin 2
xdx = ~J (1 -cos 2x)dx = !(x-! sin 2x) + c

.·. f cos 2
xdx = ~ f (1 + cos 2x)dx = !(x + ! sin 2x) + c

Example (i):
If sin 8 = g, 0 < () < ~, find exact values for sin 28, cos 28.
Solution:
As sin 8 = g
cos() = l3
.". sin 28 = 2 sin 8 cos 8 cos 28 = 2 cos 2 8 - 1
= 2 X g X 153 = 2U3) 2 - 1
= U8. = l6°9 - 1
119
-169·

Example (ii):
Find exact values for each of the following:
(a) sin 15° cos 15°
(b) 1 - 2 sin 2 22! 0

Solution:
(a) sin 15° cos 15o = !(2 sin 15° cos 15°) (b) 1 - 2 sin 2 22!0 = cos 2(22!0 )
=!sin 30° = cos 45°
- 1_ 1_ 1
- 2 ° 2
=i- Jl"
Example (iii):
Prove that
sin 2A
1 + cos 2
A =tan A.
129
Solution:
sin 2A
L.H.S. =1 + cos 2A
2 sin A cos A
= 1 + (2 cos 2 A - 1)
2 sin A cos A

= sinA
cos A
= tanA
= R.H.S.
Example (iv):
Find an expression for tan 3A in terms of tan A.
Solution:
tan 3A = tan (2A + A)
tan 2A +tan A
1 tan 2A tan A
2 tan A +tan A
1 - tan 2 A
1- 2 tan A . tan A
1 - tan 2 A
2 tan A + tan A - tan 3 A
1 - tan 2 A
1 - tan 2 A - 2 tan 2 A
1 - tan 2 A
3 tan A - tan 3 A
1- 3 A
Example (v):
(a) Evaluate the definite integral
2
J:cos xdx
(b) Find the derivative of x sin 2x. Hence find the primitive off(x) = x cos 2x.

I"
Solution:
(a) 4 If"
cos 2 xdx = - 4 (1 + cos 2x)dx
2 0
0
= l.[x
2 + 1.2 sin 2x]~0 n
=![(*+!sin~)- (0 + !sinO)J6
=i+t
(b) : (x sin 2x) = x. 2 cos 2x + sin 2x
x = 2x cos 2x + sin 2x
:. 2x cos 2x = !(x sin 2x)- sin 2x

.'. I
x cos 2xdx = ~ [ f ! (x sin 2x)dx - f sin 2xdx]
Remembering that integration and differentiation are inverse operations:
I
x cos 2xdx = tx sin 2x - !( -t cos 2x) + c
= !x sin 2x + t cos 2x + c
= i(2x sin 2x + cos 2x) + c
130
1. In each of the following find exact values for
(i) sin 2A; (ii) cos 2A; (iii) tan 2A,
using the given value when 0 < A < -!
(a) sin A = ~ (e) tan A = S15
12 7
(b) cos A = D (f) cos A =
25
(c) tan A =
3
4 (g) sine= f
(d) sinA =
1~ (h) tan e =m-l
2. Find, without using tables, exact values for:
(a) 2 sin 15° cos 15° (f) 1 - 2 sin 2 75°
0
(b) sin 22! cos 22! 0
(g) cos 2 {'2 - sin 2 {'2
0 0 2
(c) cos 67! sin 67! (h) (cos 22! 0
sin 22!
-
0
)
0

(d) cos 2 15° - sin 2 15° (i) 2 tan 22!


1 - tan 2 22! 0

(j) 2 sin 75° cos 75°


cos 2 75° - sin 2 75o

3. By setting 38 = 28 + e, prove that:


(a) sin 38 = 3 sine - 4 sin 3 e
(b) cos 38 = 4 cos 3 e - 3 cos e.

4. Simplify each of the following in terms of the indicated angle:


(a) 1 + cos 2e; e (f) (cos e + sin e) 2 ; 2e
(b) 2
2 cos 2A- 1; 4A (g) cos e;!
(c) tan lOA; 5A (h) sine;!
(d) (cos e - sin e) 2 ; 2e (i) tan e;!
(e) 1- 2 sin 2 3A; 6A (') 1 - cos e. ~
J 1 +cos e' 2

5. Simplify each of the following:


(a) 2 sin (45° - e) cos (45° - e) (e) tan 7A - tan 5A
1 + tan 7A tan 5A
(b) 2 cos 2 (45° - e) - 1 (f) cos A (cos 4A + 2 sin 2 2A)
sin 2A 1 +cos 2e
(c) --A-- cos 2A
tan (g) 1 - cos 2e
sin 2x
(d) cos 5A cos 3A + sin 5A sin 3A
(h) 1 +cos 2x

6. Prove the following identities:


(a) (cos x + sin x)(cos x - sin x) =cos 2x (e) sin 2e
1 +cos 2e
=tan e
(b) (sine + cos e) 2 =1 + sin 2e (f) 1 + 0

Slll
cos 2e
2e =
-co
t e
(c) 1 - cos 2e =tan e (g) cos 2e + si~ 2e=4 cos e - 1 2

sin 2e cos e sm e cos e


(d) cos 4 e- sin 4 e =cos 2e (h) cos e + s~n e =sec 2e + tan 2e
cos e - sm e
131
7. (a) (i) Write down expressions for tan 3e and cos 3e in terms of the angle e.
(ii) Hence prove that tan 3e - tan e
2 sine
cos 3
e. =-
(b) (i) Prove that tan 2A - 2 tan A = tan A . tan 2A.
2

(ii) Show that tan (1 - A) = cos A - s~n ~.


cos A+ sm
(c) Prove the following identities:
(i) sin A + sin 2A tan A =
1 + cos A + cos 2A
(ii) sin 2 SA - sin 2 3A sin 8A sin 2A =
.. ')
(m
sin 2A - sin A
cos 2A - cos A + 1
tan
A
=
(iv) ~ -sm
+ s~n ~~ =tan 2
(45° +A)
(d) Write sin 3e and cos 3e in terms of the angle e. Hence prove that
sin 3e _ cos 3e _ 2 O. =
sine cos e
8. Evaluate the following definite integrals:
(a) t* sin 2 xdx (d) J *(cos 2 x- 1)dx
0

(b) J: 2
cos xdx (e) I~ (sin 2 x - 1)dx

(c) f !~ 2 sin 2
xdx (f) I~ sin 2 3xdx
9. Find the primitives of the following functions:
(a) f(x) = sin 2 -r (c) f(x) = cos 2 (ax + b)
(b) f(x) = sin 2 2x - ! (d) f(x) = sin 2 (ax + b)
10. (a) The area bounded by the graph of y = sin x, the x axis and the ordinates x = 0, x = n is
rotated about the x axis. Find in terms of n the volume of the resultant solid of revolution.
(b) Find the volume of the solid generated when the area of the region bounded by the graph
y = sin 2x, the x axis and the lines x = 0, x = n is rotated about the x axis.
(c) Find the area bounded by the curve y = sin 2 -r, above the x axis and between the lines x = 0,
x = 2n.
(d) Evaluate J:(sin e + sin 2 e)de.
(e) Find the area bounded by the x axis and one arch of the curve y = sin 2 3x.
11. (a) Write down the expansion for sin 2x. Hence find:
(i) f sin x cos xdx

(ii) f sin 2
x cos 2 xdx

(iii) f; (sin x + cos x) 2 dx.


(b) Write an expression for sin 2 2x in terms of cos 4x.
Hence show that fsin 4 2xdx = ix - ! sin 4x + l 4 sin 8x + c.

(c) Find f sin 4


xdx.

132
(d) Show that ![sin (5x + 2x) + sin (5x - 2x)] = sin 5x cos 2x.
Hence, find I: sin 5x cos 2xdx.

12. (a) Write down a formula for sin 3e in terms of the angle e.
Hence, show that sin 3 x = !(3 sin x - sin 3x). Use this to evaluate I: sin 3 xdx.

(b) The area bounded by the curve y = 2 - sin x, the x axis and the ordinates x = 0, x = n is
revolved about the x axis. Find the volume of the resultant solid.
(c) (i) Find the area bounded by the graphs of y = sin x, y = cos x and the lines x = i, x = -!.
(ii) Find the volume of the solid formed by rotating this area about the x axis.
(d) Find the derivative of f(x) = cos x + x sin x. Hence, evaluate the definite integral

I: xcos xdx.
(e) Find the derivative of j(e) = sin 2e - 2e. cos 2e. Use this result to find

I: e sin 2ede.

Let the straight line p with equation y = m 1 x + b1 , make


an angle a with the positive direction of the OX axis at R.
Letanotherstraightlineqwithequationy = m 2 x + b 2 meet
the line pat P. q meets the x axis at Sand makes an angle f3
with the positive direction of the OX axis.
Then m 1 = gradient of the line p = tan a.
m 2 = gradient of the line q = tan f3.
If e is the angle between the lines p, q at P, then from the X
triangle PSR, exterior angle PRX =angle PSR +angle SPR X
(sum of the interior angles)
i.e. a= f3 + e
:.e=r:x-/3
.. 0 tane = tan (a - /3)
tan a- tan f3
1 + tan a tan f3
= m 1 - m 2 , provided that m 1m 2 =/:. -1.
1 + m1m2
This may also be written as e= tan- 1 ( m 1 - m 2 )'where e is the acute angle between the two lines
1 + m1m2
at their point of intersection.
N.B. 1. If m 1 m 2 = -1, the lines are perpendicular.
e = 90°
e
and tan is infinite.
2. If m 1 = m 2 the lines are parallel
e = oo
1
tan e = 0 (= 7 - mi °
+ m1 = 1
+ m1 2) 0

3. Between any pair of intersecting lines there is one acute and one obtuse angle
e and 180- e.
When tan e is positive, the acute angle results and when tan e is negative, the obtuse angle
results.
133
The acute angle is always obtained by letting
tan e = m 1 -m 2 • I
I
1 + m 1m2

The acute angle between two lines with gradients m 1, m 2 is given by:
tan e =I m1 - m21
1 + m 1 m2

or e=tan-11 m1- m21·


1 + m 1m2
If m 1 m 2 = - 1, the lines are perpendicular.
If m 1 = m 2 , the lines are parallel.

Example (i) :
Find the size of the acute angle between the lines
X - 2y- 1 = 0
X+ 3y +2 = 0.

Solution:
X- 2y- 1 = 0 X+ 3y +2= 0
X - 1 = 2y 3y =-X- 2
y =h-t y = -tx- i
:. m1 =t .'. m2 = -t
... tan e= I1m1+ -m m21
m
1 2

=I t-+ -ttl
1
1(
2( -3)
= 1
:. e = tan- 1 1
= 45° (or l radians).

Example (ii):
A triangle PQR has its vertices with coordinates P(7, 6); Q( -1, 4) and R(4, 1). Find the size of each
angle of the triangle.
Solution: P(7, 6)
Let m 1 = gradient of PQ
6-4
7 - ( -1)
2
8
1
4

0 X

Let m 2 = gradient of QR Let m 3 = gradient of PR


1-4 6 - 1
4+ 1 7-4
3 5
-
5 3
134
Now tan LPQR = J m 1 - mzl
1 + mlmz
.l+l. I
=
=
l 14_!~~
1
LPQR = 45°
As m2 • m3 = -~ . t
= -1
LQRP = 90°
and remaining L QPR = 180° - 90° - 45°
= 45°

1. Find the tangent of the acute angles between the pairs of lines whose gradients are:
(a) !, i (d) -t,!
(b) t, 4 (e) -~, -%
1
(c) J3, J3 (f) 1~, -!.

2. Find the acute angle, to the nearest degree, between the pairs of lines with gradients:
(a) ~' t (d) J3,
1
(b) J, t (e) 2·2, 0·8
1
(c) 1, f'i (f)
J3- .
J3, J3 1
v3 3+1
3. Find the acute angle between the following pairs of lines. (Give answers in radians, correct to
two decimal places.)
(a) y = 2x; y = x (e) x- 3y + 1 = 0; 2x + 3y- 2 = 0
(b) y = 3x + 2; y = 2x + 5 (f) x - 2y + 3 = 0; 4x - y - 1 = 0
(c) y = 2x- 1; y = +1-tx (g) x + 2y - 1 = 0; 2x - 3y + 2 = 0
(d) !x + y = 5; 3x + y = 4 (h) 4x - 3y - 2 = 0; y = 3x - 2
4. In each of the following the line p joins the points P and Q; the line r joins the points R and S.
Find the acute angles between p, r, giving answers in degrees, correct to two decimal places.
P Q R S
(a) (3, 7) (2, 3) (5, - 4) (1, - 5)
(b) (2, 1) (4, 3) ( -1, 2) (3, 4)
(c) (2, 3) (5, -1) (4, -2) (3, 0)
(d) (2, 3) (8, -1) (1, 5) (4, 3)
(e) (5, 3) (1, 4) (7, 5) (6, 1)
5. Find the angles, to the nearest degree, of the triangles formed by the lines joining the given vertices
A, B, C. (It is suggested that a diagram should be drawn.)
A B C
(a) (2, 9) (4, 1) ( -3, -2)
(b) ( -4, 2) (3, -4) (1, 3)
(c) (2, 2) (0, 1) (3, 0)
(d)(3,2) (3,0) (4,0)
(e) (3, 3) (1, 1) (5, 1)
6. Find the three angles, to the nearest degree, of the triangles formed by the following lines:
(a) 2x- 3y + 4 = 0; 3x- 2y- 20 = 0; x + 5y- 11 = 0.
(b) X+ 2y + 4 = 0; 3y =X+ 4; 3x + y- 1 = 0.
135
(c) 2x+y-I =0;3x+2y+2=0;3y-x+ I =0.
(d) 7x + 3y + 5 = 0; 3x + Ily- 29 = 0; x- 2y- 4 = 0.

7. (a) Calculate the angles of the triangle formed by joining the points (- 2, 1), (3, 13) and (10, 6).
Hence show that the triangle is isosceles.
(b) P(6, 11); Q(O, 3) and R(8, 9) are the vertices of a triangle. Find the tangent ratios of the
angles QPR, QRP. What type of triangle is this? If M is the mid-point of P R, find the tangent
ratios of the angles PQM, RQM. What can you say about the line QM?
(c) A(2, 1); B(7, 3); C(9, 6); D(4, 4) are the vertices of a parallelogram.
Find (i) the angles BCD, BAD in degrees, correct to one decimal place. What can you
deduce about the opposite angles of a parallelogram?
(ii) the acute angles between the diagonals, correct to one decimal place.

8. (a) (i) Show that the acute angle between the lines x - 2y = 0 and 3x - y - 15 = 0 is !f
radians.
(ii) Show that the line 3x - y - 15 = 0 cuts the x axis at (5, 0).
(iii) If the gradient of the line x - 2y = 0 is m 1 , use the formula tan 8 = m 1 - mz , to
I + m 1m2
find the gradient and hence the equation of a second line through (5, 0) which makes
an angle of !f radians with the line x - 2y = 0.
(b) Find the equations of two lines through the point (5, 3) which make acute angles of !f radians
with the line 2x- y + 2 = 0. (Consider that if the acute angle is![:, the obtuse angle is 34".)

9. If two curves intersect at a point, the angle between the curve is taken as the angle between the
tangents at the point of intersection.
(a) (i) Find the point of intersection of y = x 2 andy = ~·
(ii) Find the acute angle, in degrees correct to two decimal places, between the two curves
at the point of intersection.
(b) (i) Show thatf(x) = sin x and g(x) = cos x intersect at x = !f.
(ii) Find the acute angle between these curves at x = !f. Give your answer in radians, correct
to two decimal places.

10. (a) In the plan of a set of railway yards, two railway y


tracks are represented on the plan as shown opposite.
They pass through the points indicated. If the maxi-
0
mum angle of intersection allowed is 22! would ,

this plan satisfy? If not, by how much is the angle


too great? What would be the angle between the
tracks if track 2 passed through the point (1·75, 1)
instead of the point (2, 1)?

0 X

11. If 8 is the angle between the lines


ax+ by+ c = 0 and a 1 x + b1 y + c1 = 0,
'
prove that tan 8 =I abaa 1
a b
1 + bb 1
-
1

Thus find the acute angle between the lines
2x + 3y- 5 = 0
-2x- y + 3 = 0,
giving the answer to the nearest degree.
136
12. Given two straight lines y = m 1 x andy = m 2 x, show that the bisector of the angle between
these lines is the line y = mx where
(m- m 1)(1 + mm 2 ) = (m 2 - m)(l + mm 1 ).
(Keep the 'm' in cyclic order. i.e. use m - m 1 and m 2 - m).
Thus show that the gradients of the two bisectors is given by the quadratic in m
(m 1 + m 2 )m 2 + 2(1 - m 1 m 2 )m- (m 1 + m 2 ) = 0.
Write down the product of the roots. What relationship is indicated between the two bisectors.

Sin 8
As sin 28 = 2 sin 8 cos 8
sin 8 = 2 sin ! cos!
This may be expressed as
2 8 0 8
sin 8 = 2
Sill 2 cos 2
2
(as sin 2 ! + cos 2 ! = 1).
sin ! + cos !
Divide numerator and denominator by cos 2 !
• • 0 2 tan!
. , Sln u = •
1 +tan 2 28
It is common practice to let tan ! = t
ll 2t
1 + t2'
0

. Sill o =
Cos 8
As cos 28 = cos 2 8 - sin 2 8
cos 2 ! - sin 2 !
cos 8 =
_ cos 2 ! - sin 2 !
(as cos 2 ! + sin 2 ! = 1).
- cos 2 ! + sin 2 !
Divide numerator and denominator by cos 2 !
1 - tan 2 !
cos 8 =
1 +tan 2 28
1 - {2
1 + {2
Tan8
ll sin 8
t ano =- -
cos 8
2 tan!
1 + tan 2 !
1 - tan 2 !
1 + tan 2 !
2 tan! 2t
1 - tan 2 ! 1 - t2

Thus, if t = tan !
2t
0

Sillu
ll
= ---
1 + {2
1-
cos8 =---
1+
{2

{2

tan8 =- -
2t
1- {2

137
Example (i):
Prove that
1 + sin e- cos
ee =t where t = tan -.
e
1 + sin e+ cos 2
Solution:
LHS = 1 + sin 8 - cos 8
. . . 1 + sin e+ cos e
1 + _2_t- - 1 - t 2
1 + t2 1 + t2
2
1 + _2_t- + 1 - t
1 + t2 1 + t2
1 + t 2 + 2t - 1 + t 2
1 + t2
1 + t 2 + 2t + 1 - t 2
1 + t2
2
2t + 2t
2t +2
- 2t(t + 1)
- 2(t + 1)
= t
= R.H.S.

Example (ii):
r.-e---------.,--
1 -sin x.
Express . m terms oft.
1 + Slll X
Solution:

1 - sin x
1 +sin x

1+ t 2 - 2t
1+ t 2 + 2t
t2 - 2t + 1
t2 + 2t + 1
(t - 1) 2
(t + 1) 2
t- 1
t + 1

Example (iii) :
Solve the equation t(2t - 1) = 0 for all values of e over the range oo < e < 360° (t = tan!).
Solution:
t(2t- 1) = 0
.'. t = 0 or t =!
i.e. tan! = 0 or tan! = ! where oo < ! < 180°
.'.! = 0°, 180° or ! = tan- 1 0·5
~ 26·57°, by calculator.
or 53·14°.
138
(In this exercise t = tan !)

1. Write each of the following as one term:


2t 2 tan 15°
(a) 1 - t2 (d)
1 - tan 2 15°
0
2 tan 22!
(b) 2 tan! (e)
1 + tan 2 ! 1 + tan 2 22! 0

1 - t2 1 - tan 2 75°
(c) 1 + t2 (f)
1 + tan 2 75°

2. Find an expression for each of the following in terms oft.


1 +cos e
(a) 1 - sine (f)
1 - cos e
sine+ cos e
(b) cos e+ sin e (g) sin e - cos e
(c) sine e
cos
1 +cos e (h) 1 - sine
(d) sine - cos e (i) 1 + tan e . tan !
(e) 3 sin e - 4 cos e (j) e
3 cos + 4 sin + 5 e
3. (a) Write expressions for
(i) cosec e
(ii) sece
(iii) cot e
in terms oft.
(b) Simplify the following in terms of t:
(i) tan x + sec x
(ii) ! cot! - cot e.
(c) Prove the following identity:
cote = !(cot! - tan!)
(d) Express
a+ cos
; e in terms oft.
(e) Prove that:
(i) sin 2e(1 - cos e) = 1
cos 8(1 - cos 2e)
(ii) 1 + s~n e + cos e =_!.
1 + sm e - cos e t
(f) Prove that 2t, 1 - t 2 , 1 + t 2 are the sides of a right-angled triangle.
(g) P rove t h at sin 2e e+ sin e e ='= - - · l'f'
2t- . H ence state a Slmp · ctOrth'lS expressiOn
1 1cat10n · 1ll
·
1 + cos + cos 2 1 - t2
terms of e.
(h) (i) If a sec e + b tan e = c, find an expression for tan! in terms of a, band c.
(ii) If b sine = a(1 - cos e), show that tan! = %.

4. Solve the following equations for 0 ~ x ~ 2n. t = tan f. Give answer in radians, correct to two
decimal places when necessary.
(a) t(t - 1) = 0 (e) t 2 - t - 2 = 0
(b) (t- 1)(J3t - 1) = 0 (f) t 2 - 5t + 6 = 0
(c) (j'it - 1)(t - J3) = 0 (g) 6t 2 - 5t + 1 = 0
(d) (3t + 2)(2t- 1) = 0 (h) 3t 2 + 4t - 4 = 0
139
Various trigonometrical equations have already been
studied in this course. The basic work on angles of any
(180 0)
magnitude is important.
S A

T C

Example (i) : (I + 0) 0)
Solve the equation 2 cos x + 1 = 0, 0 ~ x ~ 2n.
Solution:
2 COS X+ 1 = 0
2 COS X= -1
COS X= -t
.·. x = cos- 1 ( -t)
= n - ~ and n + ~, as cos is negative in second and third quadrants.
- 2tt 4tt
- 3' 3 (radians)
Example (ii):
Solve, over the interval 0 < 8 < 360°,
6 cos 2 8 - cos 8 - 1 = 0
Solution:
6 cos 2 8 - cos e-
1 = 0
(2 cos 8 - 1) (3 cos 8 + 1) = 0
2 cos 8 = 1 or 3 cos 8 = -1
.'. cos 8 = t or cos 8 = -t
.'. 8 = 60° or (360° - 60°) or 8 = 180°- 70·53° or 180° + 70·53°
.'. 8 = 60°, 300°, 109·47°, 250·53°.
In some cases it is necessary to simplify by means of trigonometrical identities before solving.
Example (iii) :
Solve 6 sin 2 8 - cos 8 - 4 = 0 for 0 ~ 8 ~ 2n.
Solution:
6 sin 2 8 - cos 8 - 4 = 0
6(1 - cos 2 8) - cos 8 - 4 = 0
-6 cos 2 8 - cos 8 + 2 = 0
.'. 6 cos 2 8 + cos e-
2 = 0
(2 cos 8 - 1) (3 cos 8 + 2) = 0
.'. cos 8 = t01' cos 8 = -1
.'. 8 = ~, 2n - ~ or 8 ~ n - 0·841, n + 0·841
= ~, \", 2·30, 3·98 (in radians).
Example (iv):
Solve sin 28 = cos 8 where 0 ~ 8 ~ 2n.
Solution:
sin 28 =cos 8
2 sin 8 cos 8 - cos 8 = 0
cos 8 (2 sin e - 1) = 0
.·. cos 8 = 0 or sin 8 = t
.'. 8 = -I, 32" or ~ and n - ~
8 =~,"I, sl, 3z"
140
1-

METHOD 1
One method of solving such equations is to make use of the identities.
. 2t 1 - t2 ()
sm 8 = - - -2 ; cos () = - - -2 where t = tan -.
1 +t 1 +t 2
Example (v):
Solve 8 cos () - sin () = 4 when 0 ~ () ~ 2n.
Solution:
8 cos () - sin () = 4
1 - t2 • 2t
Put cos()=--· sm () = - -
1 + {2' 1 + {2
2
8(1 - t ) 2t = 4.
1+ t 1+
2

Multiply both sides by 1 + t 2


8 - 8t 2 - 2t = 4 + 4t 2
.. . 12t2 + 2t - 4 = 0.
Divide both sides by 2
6t 2 + t- 2 = 0.
Factorise:
(2t - 1)(3t + 2) = 0
.·. t =! or t = ~
i.e. tan!= ! or tan! = -~
(!in first or third quadrant) or (!in second or fourth quadrant)
.'.! = tan- 1 (0·5) or ! = tan- 1 ( -~)
~ 0·4636 or n + 0-4634 or ~ n - 0·5880 or 2n - 0·5880
= 0-4636, 3·6052 or 2·5536 or 5·6952
.'. () = 0·9272 or 5·1072 radians.
(other values are neglected as greater than 2n)
N.B. 1. If 0 ~ () ~ 360° or 0 ~ () ~ 2n it is only necessary to consider the cases.
0 ~ ! ~ 180° or 0 ~ ! ~ n.
2. This method does not give the solution where() = 180° (n radians) as it results from! = 90°
and tan 90° does not exist.
Example (vi):
Solve over the interval 0 ~ () ~ 360° the equation 4 cos () + 3 sin () = 1.
Solution:
4 cos () + 3 sin ()= 1
1 - {2 . () 2t
Put cos 8 = + 12 ; Sln = - - -2
1 1+ t
4(1 - t ) + 3(2t) = 1
2

1+ t 1 + t2
2

Simplify 4 - 4t + 6t = 1 + t 2
2

5t 2 - 6t 3 = 0.
This is now solved by means of the quadratic formula
= -b ± )b - 4ac
2
1
2a
. _ -(-6) ± )(-6) 2 - 4 X 5 X (-3)
.. t - 2x5
-6 ± J%
- 10

141
.'.tan!= 1·57979 or -0·37979
.'. ! ~ 57·67° (or 180° + 57·67°) or 180° - 20·80° or (360° - 20·80°)
= 57·67° or 159·20°
.'. 8 = 115·34°, 318·40° (other values neglected as greater than 360°).

METHOD 2
This method involves expressing
a cos 8 + b sin 8 in the form A cos (8 - (X)
where A is positive
(X is called a 'subsidiary angle' in the range 0 ~ (X ~ 2n.
=
Let a cos 8 + b sin 8 A cos (8 - (X)
=A cos 8 cos (X + A sin 8 sin (X.
Equate coefficients
a = A cos (X ( 1)

b = A sin (X (2)
A sin a b
··A cos a a
b
tan a=-. (3)
a
Square and add equations (1) and (2)
a 2 + b 2 = A2 (sin 2 (X + cos 2 a)
= Az
.'. A = ..j"~2~+-b"'2 ( 4)

(the positive square root is taken as A > 0).


The quadrant in which a lies is determined from the signs of sin a and cos a in equations (1) and (2).

Thus
a cos 8 + b sin 8 =A cos (8 - a)
where A = )a 2 + b 2
b
tan a=-
a

and satisfies sin a = ~; cos a = ~.

Similar forms could be used:


b sin 8 - a cos 8 =
A sin (8 - a)
=
A sin 8 cos a - A cos 8 sin a
where b = A cos a
and a= A sin a
a cos 8 - b sin 8 =
A cos (8 + a)
=
A cos 8 cos a - A sin 8 sin a
where b = A sin a
and a = A cos a.
In examples any form may be used but for ease of working it is best to select the identity which gives
the same sign between the terms on the right as exists between the terms on the left.
lt is most important to learn the process rather than learn the formulae.
1

Example (vii):
Solve 2 cos 8 + sin 8 = J5 when 0 < 8 < 360°.
142
Solution:
Put 2 cos () + sin() =A cos(() - IX) =J5
A= .}2 2
+ 1 2

JS. =

Divide throughout by J5 = A
1
_l_ cos() + - - sin() =cos(() - =1 IX)
J5 J5 =cos () cos IX + sin () sin IX
,',COS IX= ~}
5
. V . ·. IX is in the first quadrant.
1
Slll IX = J5
and tan I X = !
• •• IX =i= 26· 565°, by calculator.
Also cos (() - IX) = 1
.'. () - IX = oo or 360°
.'. () = oo + 26·565° or 360° + 26·565°
.'. () = 26·565° (the other solution is beyond the range).

Example (viii):
Solve 4 sin() - 3 cos() = 2, 0° ::::; () ::::; 360°.
Solution:
Method 1. Put 4 sin() - 3 cos() A cos(() - IX) 2. = =
A= .}4 + 3
2 2

= 5.
Divide throughout by A = 5
~sin () - ~cos () cos (() - IX) i = =
=
cos () cos IX + sin () sin IX
.'. c?s IX = ;~ } ••• IX is in the second quadrant.
Slll IX = 5
.'. tan IX= -1
,',IX =i= (180- 53·1301t
=i= 126·8699°.
N.B. Care is needed in the setting up of the correct equations.
Also, cos (() - IX) = i = 0·4
.'. () - IX = 66·4219° or (360 - 66·4219t
i.e. () - 126·8699 = 66·4219 or 293·5781
.'. () = 193·291 o or 420·447°
.'. () = 193·291° (other value >360°),
Method 2. Let 4 sin() - 3 cos() A sin(() - IX) =
2. =
Again, A= 5
.'. ~sin () - ~cos () sin (() - IX) i = =
=
sin () cos IX - cos () sin IX.
Equating coefficients
COS IX
. - ~
Slll IX = 5
-:!:} .'. IX is the first quadrant.
.'. tan IX= i
=i= 36· 870°.
IX
Also, sin (() - IX) = = 0·4 i
.'. () - IX =i= 23·5782° or 156·4218°
.'. () =i= (23·5782 + 36·870t or (156·4218 + 36·870t
=i= 60·448° or 193·2918°
143
N.B. The first solution in the second method is equivalent to the discarded answer in the first method.
(420·447 - 360t = 60-447°
~ 60·448°.

1. Solve the following equations for 8 over the range 0 ~ 8 ~ 360°. Give answers in degrees.
(a) sin fJ = ~ (f) sin 38 = _!
y2 2
1
(b) 2 cos 8 - J3 = 0 (g) cos 8 2
= 4
(c) sin 28 = f (h) J2 sin~- 1= 0

~=
()
(d) tan = 1 (i) 2 sin 2 1
2
1 1
(e) tan 28 = J3 (j) cos 48 = - --
J2
2. Find all the solutions to the following equations between 0 and 2n radians. Give answers correct
to two decimal places.
(a) 4 cos 8 - 3 = 0 (e) tan x + cot x - 2 = 0
(b) 3 sin 8 = - 1 (f) sec 2 8 + tan 2 8 = 7
(c) 2 sec 8 + 3 = 0 (g) 3 tan 2 x - sec 2 x = 1
(d) 3 sec x = 4 cos x (h) (3 sin x - 2)(3 sin x - 1) = 0.
3. Solve the following equations as quadratics over the range oo ~ 8 ~ 360°.
(a) 3 cos 2 () - 2 cos 8 = 0 (f) 2 tan 2 8 ~ tan 8 - 1 = 0
(b) sin 8 + 3 sin 8 - 4 = 0
2
(g) sec 2 8 + 5 tan 8 - 5 = 0
2
(c) 4 sin 8 = sin 8 cos 8 (h) 15 cos 2 8 - 7 sin 8 + 13 = 0
2
(d) 2 sin 8 - sin 8 - 1 = 0 (i) 2 cos 2 8 = 1 + sin 8
(e) 6 cos () - 7 cos 8 + 2 = 0
2
(j) 2 - sin 8 - 3 cos 2 8 = 0.
4. Find the solutions to the following where 0° ~ x ~ 360°. Give answers correct to two decimal
places.
(a) sin 2x - cos x = 0 (f) sin 2x = 4 cos 2 x - 2 sin 2 x
(b) cos 2x - cos x = 0 (g) tan 2x + 3 tan x = 0
(c) cos 2x - sin x = 0 (h) 2 cos 2x = 1 + sin x
(d) cos 2x + cos x = 0 (i) cos 4x - 7 sin 2 2x = 0
(e) 5 sin 2x = 2 sin x !
(j) cos 2 x + sin 2 x - 1 = 0.
5. Use the' t' method to solve the following equations over the range oo ~ 8 ~ 360°. Give answers
correct to three decimal places.
(a) 4 cos 8 + 3 sin 8 = 1 (e) 2 sin 8 - 3 cos 8 = 1
(b) 2 cos 8 + sin 8 = 1 (f) cos x + sin x + 1 = 0
(c) 12 cos() - 5 sin 8 + 6 = 0 (g) 3 cos x + 5 sin x = 2·5

(d) sin 8 + 2 cos 8 = f (h) 5 cos 8 + 12 sin 8 - 13 = 0.

6. Solve the following equations in the range 0 ~ x ~ 2n. Give answers in radians, correct to three
decimal places.
(a) 4 cos x - 3 sin x = - 1 (f) cos x + J2sin x - J3
=0
(b) 5 sin x + 12 cos x = 2 (g) 12 cos x - 5 sin x = 1·3
(c) J3 cos x - sin x = 2 (h) J2 sin x + J2
cos x = 2
(d) 8 cos x 6 sin x = 1· 5 (i) 3 cos 2x - 2 sin 2x = 1
(e) 12 sin x + 5 cos x = 3 ·5 (j) 2 cos 2x + sin x = 5.
144
PAPER 49
1. In an arithmetic series of n terms, the first term is a and the last is b. Write a formula for the sum
of n terms.
Prove that the sum of the odd integers from 1 to 55 inclusive is equal to the sum of the odd
integers from 91 to 105 inclusive.
2. Find the derivatives of:
(a) 2x (b) log (cos 2x)
ex
3. A cube is expanding in such a way that its edge is changing at a rate of 5 cmfs. When its edge is
10 em long, find the rate of change of its volume.
4. Find the value of:

(a) f: 1
2
e xdx (b) f~"(sin x + cos x)dx.

~
2
5. Find the equation of a curve y = f(x), given that dd { = 12x 4 and that at the point (1, -1)
X
on the curve the gradient is 3.
6. The points P(2ap, ap 2 ) and Q(2aq, aq 2 ) lie on the parabola x 2 = 4ay. The chord PQ passes
through the point (0, a), find the relationship between p and q.
If the normals at P and Q intersect at R, find the equation of the locus of R.

PAPER 50
1. (a) Express 140°25' in radian measure.
(b) An arc of a circle of radius 20 em sub tends an angle of 140°25' at the centre of the circle. Find:
(i) the length of the arc
(ii) the area of the sector formed by this arc and the bounding radii.
2. Given that the sum of the first n terms of the series 5 + 7 + 9 + 11 + · · · is 117, find n. Find
also the smallest number of terms that must be taken for the sum of the above series to exceed 300.
3. Solve the equation tan 2 () + 5 sec 2 () = 11 for oo ~ () ~ 360°.

4. Find the domain of x for which -J:3--


X +1
increases with x. Find its maximum and minimum values
and sketch the curve.
5. The number of bacteria in a certain culture increases at a rate proportional to the number present,
i.e. dN = kN. How long will it take 1 000 000 bacteria to increase to 10 000 000 if it takes 12
dt
minutes to increase to 2000000?
6. A body starts from 0 with a velocity of 20 mjs and moves in a straight line with an acceleration
(3 + 2t) mjs 2 after t seconds. Find its velocity after 4 seconds and the distance it travels in the
4 seconds.
145
PAPER 51
1. Write the sum and the product of the roots of the equation ax 2 + bx + c = 0. If the sum of the
squares of the roots of the equation x 2 + 7x + 12 = 0 is equal to the sum of the squares of the
roots of the equation x 2 + kx - 8 = 0, find the possible values of k.
2. The point P(x, y) is any point on the curve y = e 2 x and N is the foot of the perpendicular from
P to the x axis. PTis the tangent to the curve at P and meets the x axis in T. Show that the distance
NT is l for all positions of P.
3. (a) Find the stationary points of y = x 4 - 4x 2 + 10 and sketch the curve.
(b) Sketch the curve y =cos 2x for -n ~ x ~nand find the area enclosed by the curve and
the x axis between x = -t and x = t.

X + 4
4. A point moves along the graph of y = ~ so that its x coordinate changes at the rate of
4 units per second. At what rate is they coordinate changing when x = 2?
5. (a) Write down the general formulae for the sum of an arithmetic series and for the sum of a
geometric series.
(b) Sum the following series ton terms:
n-l n-2 n-3
--+--+--+oo•
n n n
Find the sum of 15 terms in this series.
6. Prove that the angle between the bisector of the vertical angle of a triangle and the perpendicular
from the vertex to the base is equal to half the difference of the base angles.

PAPER 52
1. A number is selected at random from the integers 3 to 30. Find the probability that it is:
(a) a perfect square (b) a prime (c) either a prime or a perfect square.
2. Prove that the exterior angle of a cyclic quadrilateral is equal to the interior opposite angle.
ABC is a triangle inscribed in a circle. AB = A C, BC is produced to D and AD is joined to cut
the circle at E. Prove that LACE= LADE.
3. P(2ap, ap 2 ) is any point on the parabola x 2 = 4ay with focus S. A straight line through S perpen-
dicular to SP meets the tangent to the parabola from P in the point R. Prove that R lies on the
directrix of the parabola.
4. Show that the lines 2x + y = 1 and x + y = 2 intersect at the point ( -1, 3) and find the acute
angle between the lines at this point of intersection.
5. The section of the curve y = ex - 1 from x = 0 to x = 1 is rotated about the x axis. Find the
volume of the solid of revolution generated.
6. Sugar in water decomposes at a rate proportional to the amount still unchanged. If 20 kg of sugar
reduces to 14 kg in 4 hours, when will 95% of the sugar be decomposed?

PAPER 53
1. The volume of one litre of a certain liquid decreases to W litres after n days where
W = (1 - r)"
find the value of r to three significant figures if W = 0·56 after four days.
2. A particle is moving in a straight line, its displacement from a fixed origin after t seconds is given,
in metres, by x = t 3 - 3t 2 + 4t - 7. What is the velocity when the acceleration of the particle
is zero?
146
3. If a =
1
b Sbx , express x in terms of a and b. Hence express
3 + Sx
J 3
b - a in terms of a and b.
5x- a
4. (a) Differentiate (i) x sin 2x (ii) .Jx 2 - 2x.
(b) Show that :X (2x - sin 2x) = 4 sin 2 x.

5. Write down the condition for ax 2 + 2bx + c to be a perfect square. Prove that there are always
two real values of k which will make
ax 2 + 2bx + c + k(x 2 + 1)
a perfect square, unless a = c and b = 0.
2
=~
6. (a) Sketch the curve y
2 and give the coordinates of the focus and the equation of the directrix.
(b) A tangent to the curve y = 2x 2 is parallel to the line y = 4x - 3. Find the equation of this
tangent.

PAPER 54
1. (a) If JA .J52 .JTI7,
= + find the exact value of A.
(b) Find the values of A, Band C for which
=
2x 2 - 3x + 5 A(x- 1)(x- 2) + B(x- 1) +C
2. Find the area of the triangular region enclosed by the three lines
3x- y = 6
2x + y = 14
y=O
3. Find primitives of (a) 6x 2 - 3
(b) e3x
(c) sin 4x
4. The following table gives values of f(x) = JX log x
X 1 2 3 4 5

f(x) 0 0·98 1·90 2·77 3·60

Use Simpson's Rule with these five values to find an approximate value of

f JXlogxdx

5. The tangent at the point (8, 2) to the parabola x 2 = 32y meets the tangent at the vertex of the
parabola at T. Find the coordinates of the point T.
6. The number of bacteria in a culture is given by the equation N = N 0 ekt, where N 0 is the initial
number of bacteria, k is the growth rate per hour of the bacteria population and t is the time
in hours. If the number of bacteria in a culture was initially 50 000 and the growth rate is 15%
per hour, find the number of bacteria present after 5 hours.

PAPER 55
1. (a) If tan (A + B) = x and tan B = t, express tan A in terms of x.
(b) Find all the angles between oo and 360° which satisfy the equation 3 cos 2 e= 7 sine + 5.
2. A ball is dropped from a height of 10m. After each bounce it rises to half the height from which
it fell. Find to the nearest centimetre the distance it has travelled when it strikes the ground for
the seventh time.
147
3. A is the point ( -2, 0) and B the point (2, 0). A point P moves such that AP = 3PB. Prove that
the locus of Pis a circle whose equation is x 2 + y 2 - 5x + 4 = 0 and find the centre and radius
of this circle.
A point T moves such that the length of the tangent from T to the above circle is equal to the
distance of T from the y axis. Find the equation of the locus of T.
4. The acceleration of a body after t seconds is 2t + 2 m/s 2 • If initially the body was at rest 8 m from
the origin, find the displacement after 3 seconds.
5. For the curve y = f(x) it is given that f'(x) = 2x - 1 and that the curve passes through the
origin.
(a) Find the equation of the curve and show that it is a parabola.
(b) Find the focal length, coordinates of the vertex and the equation of the directrix of this
parabola.
6. Show that for all real values of a, b, c the equation
(x - a)(x - b) = c2
has real roots.

PAPER 56
1. Find the length of the common chord of two intersecting circles whose radii are 17 em and 10 em
and whose centres are 21 em apart.
2. Sketch the curves y = x and y = x + sin x between x = 0 and x = n and find the area bounded
by these two curves in this domain.
3. In a certain chemical reaction the amount P grams of a substance which remains unchanged after
t minutes is given by the equation P = 20e- 0 ' 151 • Find the number of grams of the substance:
(a) at the beginning of the reaction.
(b) remaining after 4·5 minutes.
4. (a) Find the stationary points on the curve y = x 3 - 4x 2 - 3x and establish their nature.
(b) Find the volume enclosed by the surface generated when the curve 4x 2 + y 2 = 64 is rotated
about the x axis.
5. (a) Differentiate x sin x.
. l'f
(b) SImp Iy
cos x + sin. x + cos x - sin
. x.
COS X - Sill X COS X + Sill X
6. A ship travelling at a speed of 8 mfs stops its engines and subsequently moves with a deceleration
proportional to its velocity.
That is, dv = kv.
dt
If its velocity 30 seconds after stopping engines is 7 mjs, when will its velocity be 4 m/s?

148
Sir Isaac Newton (1642-1727) was a mathematical
genuis. He developed his version of calculus in 1665.
CHAPTER 25

Already in this course integration has been studied as:


I. finding the primitive function for a given functionf(x) (also called the indefinite integral).
For example,
the primitive function of x

= Jxdx
2
=~ + c, where 'c' is the constant of integration

r
2. evaluation of the definite integral.
f(x)dx = F(b) - F(a),
= [F(x)]~
where F(x) is a primitive function ofj(x)

3. the application of the definite integral to find


(a) the area under a curve (or between two curves),
(b) the volume of the solid revolution when an area is rotated about one of the axes.

BASIC RULES:
If a

I. r
~ b ~ c then,

f(x)dx + f f(x)dx = r f(x)dx

2. r {f(x) ± g(x)}dx = r r f(x)dx ± g(x)dx

f ± g(x) }dx f ±

r
and {f(x) = f(x)dx f g(x)dx

3. f f(x)dx = - f(x)dx

(reversal of the limits; changes the sign of the definite integral).


4. If k is a constant
(a) fkdx = kx + c (b) fk .f(x)dx = kf f(x)dx

ALGEBRAIC:
x"+l
1.
fx"dx =; +
1
+ c,
+C
where n is rational, n of. -1

f
"d kx"+l
n +
l
2. 1CX X =
1
1 (ax + b)"+ 1
3.
f(ax + b)"dx = -.
a
+
n +1
c

150
LOGARITHMIC AND EXPONENTIAL:
dx
+c
4.
I X

dx
= loge X

1
= + +c
5.
I ax +
b -loge (ax
a
b)

6. Ij.(~; dx = loge (f(x)) +c

7. I exdx = ex +c

8. I eax+bdx = ~eax+b + c

TRIG 0 N 0 METRICAL:

9. Isin xdx = -cos x + c

10. Isin (ax + b)dx = -~cos (ax + b) + c

11. Icos xdx = sin x + c

12. Icos (ax + b)dx =~sin (ax + b) + c

13. Isec 2 xdx = tan x + c

14. Isec 2 (ax+ b)dx =~tan (ax+ b) + c

15. I 2
sin xdx = ~I (1 - cos 2x)dx

= ~(x - t sin 2x) + c

16. Jcos xdx


2
=~I (1 + cos 2x)dx

= ~(x + t sin 2x) + c

INVERSE TRIGONOMETRICAL FUNCTIONS:

17.I dx =sin- 1 x+c


.J1- x 2

18. I.J a2
dx
x2
= sin- 1 (~) + c
a

I
-

19. dx = 1 sin-1 (k(x + b)) + c


Ja 2 - k 2 (x + b) 2 k a
20. I~=
1 + x2
tan- 1 x + c

151
21. Ja2 dx+ x 2 .!tan-
a
1 (~) + c
a
=

22. J dx _.!__ tan-1 (k(x + b)) + c


=
a 2 + k 2(x + b) 2 ak a

1. Write down the primitives of the following:


3
(a) 3x 2 + 2x + 1 (e) (2x - 5)2 (i)
2x +3
1 1
(b) x 3
+ 3 + x -, 2
(f) (3x - 2)- 3 (j) esx
X
1
(c) 2x
2
+ JX + JX (g) X+_!_
X
(k) e2x-3

(d) (5x + 3) 2 (h) _.!__


4x
(I) e-4x+3

I~
2. Evaluate the following indefinite integrals:
J ~os xdx
(a)
I b - dx
- 2-
X + 1
(d) -
X -
J- 2-3x-5dx
--
5
5
dx (g)
Sill X+1
Ic?s x-+
(b)
I 2x- 3
X2 - 3x+ 5
dx (e)
X+
Jex ex+ 1dx
(h)

(i) ISill X

ex + 2e-2x dx
sin x dx
COS X

(c)
I x2 - 1dx
X
(f) ex - e 2x

3. Find the primitive functions for each of the following:


(a) sin (3x) (e) sin 2 x - ! (i)
x + 9
2

(b) cos (!x) (f) 2(cos x + sin x)


2 2
(j) (25 + x 2 )- 1
1
(c) sec 2 (5x + 4) (g) )9 - x 2
(k) 1
)9- (x + 2) 2
r (~ -
1
(d) 2 cos x - cos 2x (h) (16 - x 2 2
(I) 25 + 3) 2

4. Evaluate the following definite integrals:


1+ J1 a d
(a)
J o
(1 x) 2dx (f)
0

I-/2
)2-
dx
'
x 2 (k)
fo )a2 ~ x2
e2 t (1) L1 dt
(b)
I -dx (g) dt t-0·6
1 X 0
2t 2 + 1

(c) ~~cos (2x)dx (h) J2 (2x + 1) 2dx (m) J:e xdx


2
Jo -2

(d)
J1o ~
3

9 +x
(i) J -J
1

o
(
4 - x2
1
+-
1
-)dx
1 + x2
(n) L~ (cos e _ sin 28)de

(j) Is (25 - x 2f!dx (o)


2
2x
J x~ + 5x- 2
1
+ 5 dx

152
5. (a) Find the derivative of f(x) = x loge x - x. Hence write down the primitive function of
loge X.
(b) Differentiate y = loge {x + Jx 2 + 9} and hence evaluate f 6

4
dx
)x2 + 9
, correct to three

decimal places.
l + ~.

r
(c) Show that y = x - satisfies the equation dy = 1
X dx X

Evaluate (1 + : 2) dx.
% sin 8d8 .
(d) Evaluate e correct to three decimal places.
I
0 2 + 3 cos
(e) Show that, ifj(x) = x sin- 1 x + )1 - x 2 , thenf'(x) = sin- 1 x.
Hence evaluate J: sin - 1 xdx, correct to three decimal places.

In this work it is important to remember that area is always regarded as being positive.

CASE A

y A= Ib f(x)dx
f(x)

0 a b X

CASE B
y If the graph is below the x axis the integral yields
a b
a negative area, and the actual area is taken as the
0
absolute value.
A =I Ib f(x)dx I

153
CASE C
y If the graph cuts the x axis between the two limits
x = a, x = c then the area must be calculated in
two parts.
A= A1 + A 2

= rb f(x)dx +1f f(x)dx 1

0 X

CASED
y The area between a curve and the y axis is given by
d
g(y)
A = ld g(y)dy, where the given function ~ust be

expressed in terms of y.
A
c

0 X

CASE E
y jY = g(x) To find the area between two curves it is necessary
to:
(i) find the coordinates of the points of intersection.
(ii) calculate the difference between the two areas.
Here f(x) ;;::: g(x).

A = rb f(x)dx - rb g(x)dx

= rb {f(x) - g(x) }dx

N.B. To overcome the decision as to which curve lies above the other, absolute value could be used.
A = /lb {f(x) - g(x) }dx I= llb {g(x) - f(x) }dx I

Example (i):
(a) Evaluate I' sin 2xdx.

(b) Find the area bounded by the curve y = sin 2x, the x axis and the lines x = 0, x = n.
154
Solution:

(a) I' sin 2xdx = ![-cos 2x] 0


= ![-cos 2n - (-cos 0)]
= ![ -1 + 1]
=0
(b)
y.
The graph of y = sin 2x cuts the x axis at x = ~· As A 2 is
negative the total area
A= A 1 + /Az/
X
=fa~ sin 2xdx +I J; sin 2xdx I
= [-~cos2x]! +l[-~cos2x];l
= [-!cos n +!cos OJ + /-!cos 2n + ! cos n/
= (! + !) + /-! - !/
= 1+1
= 2 units 2 •
N.B. From the symmetry of the figure A 1 = /A 2 / and so A = 2 fa~ sin 2xdx.

Example (ii):
Find the area in the first quadrant between the graphs of y = x 3 andy = 4x.
Solution:

y I
Solving y
y
=
=
x3}
4x
8
x3 = 4x
y =4xl x3 - 4x = 0
6 I x(x 2 - 4) = 0
I x(x - 2)(x + 2) = 0
I .'. x = 0,2or -2.
4 I Thus, in the first quadrant points of intersection are
I
I
I X=
y=O
0} X=
y=8
2}
I
IL t
2
I A=
2
4xdx-
2
3
x dx I
2 X

= 11 2
3
(4x - x )dx I
= I[2x2 - ~4J: I
= / [ (s - ~6) - o] I

155
1. In each of the following, calculate the defined area. Give answers correct to three decimal places.
(a) y = !, x axis, x = 0, x =e. (f) y = 25 - x 2 , x axis, x = -2, x = 5.
X
1
(b) J' = -X+- 1' x axis ',x = 0 ' x. = 1. (g) )' = COS X, y = 0, X = 0, X = n.
(c) y = loge x, y axis, y = 0, y = l. (h) y = ex, x axis, x = -1, x = l.
1 1
(d) y = - -- , x axis, x = 0, x = 1 (i) y = , x axis, x = -!, x = !,
1 +X 2 yf1 - x 2
(e) y = - - . x = 0 , x = 3.
2x- , x ax1s, (j) y = sin 2 x, x axis, x = 0, x = 2n.
1 +x 2

2. For the following functions, find where the graph cuts the x axis within the given interval and
hence calculate the required areas.
(a) y = x 3 , x axis, x = 1, x = !. (d) y = x 2 - x - 2, x axis, x = -1, x = 3.
(b) y = x 2 - 4, x axis, x = 0, x = 4. (e) y = 2x 2 - x - 1, x axis, x = -1, x = 2.
(c) y=cos2x,xaxis,x=0,x=~. (f) y = cos x - ! , x axis, x = - n, x = n.

3. Find the area between the following pairs of curves. Where appropriate, leave answers in terms
ofn.
(a) y = 2x 2 , y = 2(x + 2). (f) y = x 2 , y = - x 2 from x = 0 to x = 3.
Sketch the figure. ·
(b) y = x 2 + 1, y = 3 - x. (g) y = cos 2x, y = -cos 2x from x = -l to
x=i-
(c) y = 2jX,y = x. (h) y = 3 - x, y = ~. Give answer correct to
three decimal places.
(d) y 2 = !x3 , y 2 = 3x in the first quadrant. (i) y = eX, y = e-x and the line x = 3. Give
answer correct to three decimal places.
1 2 1 (j) y _2189'y l.
( e) y = zX ' y = -2--1 . = =
X + X +
4. (a) Find the area in the first quadrant bounded by y = x 2 , y = x 3 , using:
(i) the x axis
(ii) they axis.
(b) Find the derivative of f(x) = x loge x - x. Hence write down the primitive function of
g(x) = loge x.
(c) Differentiate y = loge [x + y! x 2 + 9] and thus find f dx
yfx 2 + 9
.

(d) For the given function y = x 2 - x, find:


(i) the area bounded by the curve and the x axis
(ii) the area between the curve, the x axis and the ordinates x = 0, x = 2
(iii) the equation of the tangent to the curve at x = 1
(iv) the area between the curve and its tangent from x = 1 to x = 2.
(e) (i) Find the equation of the tangent tof(x) =sin x at x = %.
(ii) Show that the area between the graph of y = sin x and its tangent at x = %, between
the ordinates x = %, x = ~is 1 ! 4 (n 2 + 12.J3n - 72) units 2 .
(iii) Find the area bounded by the above curve and tangent from x = 0 to x = n. Give your
answer correct to three decimal places.
(f) Find the area between the graphs of y = sin x and y = cos x between two successive points
of intersection. Leave your answer as a surd.
(g) The rate of sales of a new cosmetic product is given by the equationf'(t) = 100- 80e- 1,
where t is the number of years since the product appeared on the market.
Find the total sales for:
156
(i) the first three years
(ii) the fourth year of selling.
(h) In a free economy the Producers' Surplus (PS) is given by
PS = y*x* - J:· S(x)dx, where:

y = S(x) is called the supply curve and gives the pricey at which a producer is prepared to
sell; y = D(x) is called the demand curve andy is the price at which a consumer is prepared
to buy; the intersection of y = S(x) andy = D(x) is called the point of equilibrium denoted
as (x*, y*); y* is called the market price, x* the quantity sold.
If D(x) = 50 - 9x
S(x) = llx + 20, find:
(i) the point of equilibrium
(ii) the producers' surplus.

CASE A
The volume generated when an area is rotated about the x axis.
y

V =nIb 2
y dx

=nIb [f(x)] 2 dx.

0 X

CASE B
The volume generated when an area is rotated about the y axis.
y
V = n1dx dy 2

= n f [g(y)]Zdy.

0 X

157
Example:
Find the volume when the area between y = x 3 andy = 4x, in the first quadrant, is revolved about:
(a) the x axis
(b) they axis.
Solution:
y (a) y = x 3 andy = 4x intersect at (0, 0) and (2, 8).
.'. Volume about the x axis is given by:
8
V =n 1 2
2
(4x) dx - n 1 2
(x
3 2
) dx

= n 1 2
(16x
2
- x 6 )dx

= n[16x3 - x?]2
3 7 0

= n[ c6 3x 8 - 1~8) - o]
_ 3)
= 128n(7
21
512n
X 21
. V 512n . 3
. . o1ume = umts .
21

(b) y = 4x ~ x = :!::
4
y = x3 ~ x = yt
.'. Volume generated about they axis is

v = n f (y~ dy2
_ n r (~Ydy
=n r (yt - t~) dy
= n[~y~- ~~J:
=n[ex5 32_~4)-o]
= n [9 6 _ 32]
5 3
128n
15
. V 128n .
. . o1ume = ---ys- umts 3 .

1. Find the volumes of the resultant solids when the given areas are rotated about the x axis. Give
· exact answers.
(a) y =JX, x axis, x = =
1, x 2. (c) y
1
4 + x2
=-.}
, x axis, x 0, x 2. = =
(b) y =sec x, x axis, x =0, x =~· (d) y =eX, x axis, x =0, x =!.

158
. . n n
()
e y = sm x, x ax1s, x = S' x = . (h) y = 4x 2 , y = x 3 .
4
(f) y = x 2 - 2x, x axis. (i) y = J5x 2 , y = JS.
(g) y = mx, x axis, x = 0, x = h. (j) y = JX, y = ]x + JX, X= 1, X= e.

2. The following areas are rotated about they axis. Find in terms of n, the volumes of the solids of
revoluton.
(a) y = x 2 + 1, y = 1, y = 5 (e) x + y = 3, y = 3 - x 2
(b) y = 4JX, y = 4, y = 8 (f) y = JX, y = !x
8
(c) y = , y = 1, y = 2 (g) xy = 2, 2x +y - 5 = 0
X
2
+4
(d) y = )9 - x 2 , y = 0, y = 3 (h) y = 3x 2 , y = 16 - x 2 (Draw a diagram).
2
3. (a) A parabola x = 4ay is rotated about they axis.
(i) Find the volume of the resultant paraboloid of height h.
(ii) A glass bowl is made by revolving the parabola x 2 = 4y about they axis. Find the volume
of water in such a bowl when the water depth is 12 em.
(b) The equation of a circle with diameter 2a may be expressed as y 2 = x(2a - x), where the
x axis is a diameter with end points (0, 0) and (2a, 0).
(i) Show that the volume of the segment of the sphere, with height h, formed by revolving
the above circle about the x axis, is given by:
V = nh 2 (a - th).
(ii) Hence, by setting h = 2a, establish the formula for the volume of a sphere.
(iii) What would be the volume of liquid in a spherical bowl of radius 15 em when it is 6 em
deep?
4. (a) Find the volume generated when the area bounded by y = 4x - x 2 , above the x axis, is
rotated about the x axis.
(b) (i) From the above equation show that x may be expressed as x = 2 + ~ or
x=2-~.
(ii) Hence, by using these two results, show that the volume of the solid formed by revolving
the above area about they axis is given by

8nr~.dy.
(iii) Evaluate this integral correct to two decimal places.

5. Prove that the volume of the solid formed by generating y = b sin x about the x axis from x = 0
a
to x a; is half that of the circumscribing cylinder (formed about the xaxis).
=

As has already be seen in the earlier examples of integration, some functions may be changed to the
standard forms by easy mathematical manipulation.
159
Example (i) :
x 3 - 5x 2 + 3x - 2
J ---~---dx
x2

= I (x - 5 +~- ~2) dx, on dividing each term by x 2

x2
= -
2
- 5x + 3 log x
e
+ -2X + c

Again, examples have been rearranged to give the form


Ijgj dx, which integrates as

loge [f(x)] + c
Example (ii):
x2 + 2
~---~dx
x 3 + 6x + 2
_ 1
-3
I 3(x
x3 +
2
+ 2) d
+2 x6x
2 3
1
=- I 3x + 6 dx, wh ere { f(x) = x + 6x +2
3 x 3 + 6x + 2 f'(x) = 3x 2 + 6
1
= 3 loge (x 3 + 6x + 2) + c
To aid the process of changing to a recognisable form, use is also made of substitution, which
results in a change of variable.
In mathematics an important use is made of what are called differentials.
If u = f(x), so that the derivative ddu exists then the differential of u, written as du, is defined as
X

du
du = dx. dx.

The differential
du = (~~). dx, where
u = f(x) andf(x) may be differentiated.

This result is used to change a variable from x to u.


For a more detailed justification see Appendix XII.

Example (i):

2 IJ2x + 1dx

Solution:

2 IJ2x + 1dx

= I J2x + I . 2dx

160
Now put u = 2x + 1
du
du = ~d . dx
X

= d(2x + 1). dx
dx
d(2x + 1) _
= 2dx as dx - 2.

.". fJ2x + 1 . 2dx

= f idu as u = 2x + 1 and 2dx = du


2 ;>
= -u 2 + c
3
2 3
= -(2x + 1)2 + c on resubstituting u = 2x + 1.
3

Example (ii):

f x.J1 + x 2 dx
Solution:

f x.J1 + x 2 dx Put u = 1 + x 2
du
= ~ f (1 + x 2 )t. 2xdx
~= 2x
dx
.". du = 2x. dx

= ~futdu
1 2 ;>
= 2. 3. u2 + c
1 3
= 3 (1 + xz)l + c as u = 1 + x2
An important result to consider is:

ff(ax + b)dx
Put u = ax + b
. du
· · dx =a
du
and du =~ . dx = adx
dx

.". f f(ax + b)dx

= ~ ff(ax +b). adx

= ~f f(u). du

= ~f f(ax + b) . d(ax + b)

161
Thus, ff(ax + b)dx
= ~ I f(ax + b) . d(ax + b)
= ~I f(u) . du where u = ax + b.

This result may be used to find some of the results in the earlier list of basic integrals. For example:
(i) I eax+bdx = ~ fe"du

1
= -e" +c
a

1 ax+b
= -e +c
a

(ii) fsin (ax + b)dx = ~I sin u. du


1
= --cos u +c
a
1
= --cos (ax + b) + c
a

(iii) f r=:;;===d=c=x==~ = b1 I-Jazdu_ uz


1 . -1 u 1
= b sm ~ + 1C
-_ -sm - -+-c)+ k
1 . _1 (bx
b a

(iv) f a2 +
dx
(bx + c) 2
=!
b
f a2
du
+ u2
(where u = bx + c)
1 -1 u
=-tan - + 1C1

ab a

=~tan-1 (bx +c)+ k


ab a

Example (iii):
Integrate (2x + 1) 10
Solution:

J(2x + 1)
10
dx Let u = 2x + 1
du = 2dx
=~I u 10
du

2 ·uu
1 1 11 1
= +I(

1
= -(2x
22
+ 1) 11 +k
162
Example (iv):
By using the substitution t = u
2
- 2 find the primitive of~.
2+ t

Solution:

IJ2+i
Put
dt

t = u2 - 2
dt dt
du = 2u and dt = du . du
= 2udu

. . IJ!-+r
II
-
=
.)2 +
2udu
2udu
(u 2 - 2)

R
= Iu:u
2

= 2 I du

= 2u +c
To return to the original variable t,
t = u2 - 2
.·. u = t + 2
2

u = jt+2
. . IJ!-+r 2 + t = 2jt+2 + c

Example (v):
Integrate JXFxX 1- X
by substitution of x = u
2

Solution:

IJXh Put x = u 2
dx = 2u

-
_ 2
IfN.Jl- 2udu

udu
u2
du
.'. dx = 2udu

- u.jl- u2

-2
-
I du
.)1 - u2
= 2 sin- 1 u +c As x = u2
= 2 sin- 1 JX + c u = JX
163
1. Use the given substitution to integrate the following functions:
(a) cos (2x + 3), u = 2x + 3 (d) e 5 x+ 2 , u = Sx + 2
1
(b) sec 2 (Sx + 1), u = Sx + 1 (e) , u = 4(x - 1)
16(x- OZ Jg-
1 1
(c) u = 4x- 1 (f) 16 + 25(x + 7) 2 ' u = S(x + 7).
4x- 1'

2. Establish the following standard integrals by using the given substitutions:


(ax+ b)"+ 1
(a)
I (ax + b)"dx = a(n + )
1
+ c, (u = ax + b)

(b) f cos (ax + b)dx = ~sin (ax + b) + c, (u = ax + b)

(c) I sec 2 (ax+ b)dx =~tan (ax+ b)+ c, (u =ax+ b)

(d) fJ a2 -
dx
k 2 (x + b) 2
= l
k
sin- 1 {k(x +b)}
a
+ c, [u = k(x +b)]

()I
e
a
2
+ ''
dx
'2(
x +
b)2 -_ 11 tan - {k(x + b)}
atC
1

a
+ c, [u -_ K
1 ( ,
x + b)] .

3. In each of the following use the given substitution to find the primitive functions
3
(a) 3J3x + 1, u = 3x + 1 (e) x(l + x 2 )2, u = 1 + x2
2
(b) , u = 2x + 1 (f) xJx 2 + 4, u = x2 + 4
J2x + 1
2
(c) , u 2 = 2x + 1 (g) J2x - 1, u 2 = 2x - 1
J2x + 1
(d) 3x 2 ~,u = x 3 1 cos 2x
(h) sin3 .
-
2x' u = sm 2 x.

4. Integrate each of the following, using the given substitution.

(a)
I vx
cos JX 1_
;-:_ dx, u = x 2 (f)
xdx 2
~· u = x - 2
I vx-2
(b) I cos 2 2t. sin 2tdt, u = cos 2t (g) f xJ.X+T dx, x = u
2
- 1

(c) I+(x 3)-tdx, u2 = x + 3 (h) Itan xdx, u = cos x

(d)
I x dx
-;=~=,
~3+3
2
l/ = X
3+ 3 (l') I (4- x)-!dx ,
JX X = U
2

(e) I ~ (x
2

2
)4 dx, x = u + 2 (j) fJcos x. sin xdx, u =cos x.

5. (a) By substituting t = 1 + cos x, show that


Jsin x. J1 +cos xdx = -i(l +cos x)t + c

(b) (i) Show that sin x - cos 2 x sin x = sin 3 x.


(ii) Thus, by using the substitution u =cos x, show that I sin 3 xdx = icos 3 x - cosx + c.

164
(c) Using the substitution u = sin 3t, show that
f (1 2
sin 3t) . cos 3tdt = t sin 3t - ~ sin 3 3t + c
(d) (i) If X= 4 sine, show that cos e = )16 - x 2 .
(ii) By substituting x = 4 sin 8, show that
8 sin- 1 ~ _lx)16- x 2
2
x dx
f )16- 2
= + c.
x 4 2
. . x
(e) U set he su bstltutwn . 8, to ven'fy t h at
= 3 sm f dx . . -1 -X + c.
= sm
)9- x 2 3

(f) (i) By use of the substitution x = 3 sin 8, find f )9 - x 2 • dx.


(ii) By a similar substitution show that

IJa· 2
- x
2
. dx = ~
2

sin-
1
~ + ~>/a 2 - x
2
+ c.

dx 1 x
(g) Show that
f (x 3 = -. + c.
2
+ 16)2 16 )16 + x 2
Use the substitution X = 4 tan e and rememl5et that tan 2 e + = sec 2 d.

The method of substitution may also be applied to the evaluation of definite integrals.
When limits are involved, a change in variable always means a change in the limits of integration.
For example, if the limits of integration are from x = 2 to x = 4 and the variable is changed by
the substitution
u = 2x + 1, then :
when x = 2, u = 2 x 2 + 1
= 5
X= 4, u=2x4+1
= 9.
Thus the new limits of integration are from u = 5 to u = 9.
--~- - --- --~- - - - - - - - - - - - ---------------------------------1

In evaluating a definite integral by use of change of variable, there must be a corresponding


change in the limits of integration.

Example (i):

f~
Evaluate
1 x 2 )1 + x 3 dx, using the substitution u = 1 + x 3 •
Solution:
When u = 1+ x3
du = 3x2
dx
.'. du = 3x 2 dx
When x = -1, u=1+(-1) 3
= 1- 1
=0
When x = 1, u=1+1 3
=2
165
t
Example (ii) :
1
Evaluate, correct to three decimal places,
2
x . ex + 1 dx, using the substitution u = x 2 + 1.

Solution:
u = x2 + 1 and when x = 0, u = 1
du = 2xdx X= I, u = 2

1
= -(e2 - e1)
2
~ 2·335, by calculator.

Example (iii):
By means of the substitution x = sin 8, evaluate the definite integral 1 1
)1 - x 2 dx, leaving your
answer in terms of n.
Solution:
Put x = sin 8 and when X = 0, sin 8 = 0
:. dx =cos ede :. e = o
when X = 1, sin 8 = 1
. n
.. 8=2
.'. f )I- 2
x dx

= f~JI-sin 2 8.cos8d8
= t~ .Jcos 2 e . cos 8d8
= t~ cos 2 ede
166
= -1 11!_2 (1 + cos 28)d()
2
=-
1[e + 1· 2e]~
2
0

-sm
2 0

=~[(~+~sin n)- (o +~sin o)J


n
4'

1. Using the given substitution, evaluate the following definite integrals:

(a) LJ.X+1
3
dx, x = u2 - 1 (f) t2
x 2)x3 + 1 dx, u = x 3 + 1

(b) J~ 4

1 .s
dt, u2 = 2t + 1 (g) J -J 10

16
x
+ 2x
dx, u 2 = 16 + 2x
flo
0

(c)
I 4
s .
x~ dx, u2 = x - 4 (h)
)16
x
+ 2x
dx, u = 16 + 2x

J
0

(d) Js x~
4
dx, u = x - 4 (i)
2
4
x
(3x- 4) 3
dx, u = 3x - 4

(e) J4 )2xdx
o + 1
' (i) uz = 2x + 1
(ii) u = 2x + 1
(j) Is )3xxdx+ 1
'u2 = 3x + 1.
1

2. Using the given substitution, evaluate each of the following correct to three decimal places:

(a) f 0
1
2x)x
2
+ 4dx, u = x
2
+4 (f) f Fx'
1

0 2 X
u = 2- x

(b) J./3 x)x 4 + ldx, u = x 2 + 1 (g) J4


JX dx
x(l + x)
, u = JX

f -J f
0 1

(c) 1 dx 'u = 3 + 2x (h) 1 3x2dx 'u2 = 1 + x3


0 3 + 2x 0 2) 1 + x 3

(d) fl -J dx
3 - 2x
,u = 3- 2x (i) fl l(ldx
+ X4
~)'u = 1 +xi

f(
0 X4
0

(e)
f o
3 ·xdx
)x + 1 2
, U
2
= X
2
+ 1 (J') 2

o 5
_dx ) 2 ,
2x
U =
5 - 2X.

3. (a) Evaluate L 4
1
+]xJX dx, using the substitution u = 1 + 2JX.

(b) Use the substitution, u = 1 - x, to evaluate the definite integral 1 1


(1 - x) 8 dx.

Hint: Remember that -

(c) (i) By using the substitution u = 1 - x show that


Io f(x)dx = r f(x)dx.

x
f J1=X dx = j-(1 - x)t- 2(1 - x)! + c.

167
(ii) Hence, evaluate the definite integral fo Fxdx, correct to three decimal places.
-1·5 I - X

f
~ sec (sin x)
2
. . . · ·
(d) Evaluate dx, usmg the substltutton u = sm x and remembermg that
0
sec x
I
secx = - - .
COS X
(e) (i) Show that (u - W = u3 - 9u 2 + 27u - 27.
(ii) Hence, by means of the substitution u = x + 3, evaluate correct to three decimal places,
1

f-2
x\jx + 3dx.

(f) Find the area under the graphf(x) = xJ3x - 2, above the x axis and between the ordinates
X= 1, X= 6.
Use: (i) u = 3x - 2
(ii) u2 = 3x - 2.

4. (a) (i) By division show that ~I


1
= u2 - u + 1- - - .
u+ u+ 1
1
(ii) Hence, by using the substitution x = u 2 , evaluate the definite integral I x JX dx,
Jo 1 + x
correct to three decimal places.
. Use the substitution u = x + I, to find the area bounded by the curve,
2
(b) f(x) = x
(x 2 + I) 2
the x axis and the ordinates x = I, x = 2.
(c) In this question use the substitution u = 3x - 1.
(i) Find the area bounded by the graph of f(x) = -J3x - 1, the x axis and the ordinates
X= t, X= 3.
(ii) Find the volume when this area is rotated about the x axis to form a solid of revolution.
(d) (i) Find the area bounded by the graph of f(x) = ~'the x axis and the lines x = 3,
X +1
x = 8. Use the substitution u 2 = x + 1.
(ii) Show that the volume of the solid formed by revolving this area about the x axis is given by

loge(~J}·
2
V= } + n{

168
Calculus plays a part in the designing of modern
structures.
M.L.C. Building, Sydney.


SIC
CHAPTER 26

In Chapter 22 when considering motion in a straight line the following formulae for position x,
velocity v and acceleration a were derived.

dx
v =-
dt
or v=x x = fvdt
dv
a=-=-
dt
d 2x
dt 2
or a=x v= fadt

1. A particle moves in a straight line such that its distance x metres from the origin at time t seconds
is given by x = t 3 - 9t 2 + 15t - 9. Find when and where the particle comes to rest.
2. A particle is moving in a straight line. At time t seconds its position x metres is given by
1
X = 2t + 1 + - - .
1 +t
Find: (a) the initial position at timet = 0.
(b) the velocity at time t = 2.
(c) the acceleration at timet = 1.
3. A particle is moving in the x axis. It started from rest at time t = 0 from the point x = 5. If its
acceleration at timet is 4 + 6t, find the position of the particle when t = 3.
4. The velocity of a particle moving along a straight line is given in metres per second by:
x = 2 + 2t- 3t 2
where t seconds denotes the time. If the particle is at the origin when t = 0, find the distances of
the particle from the origin when t = 1 and when t = 2. Also find the acceleration of the particle
when t = 3.
5. The displacement, x metres, of a particle moving in a straight line at time t seconds is given by
x = 12t - t 3 . Find:
(a) the time when the particle comes temporarily to rest.
(b) its displacement and acceleration at that time.
6. A particle moves from rest at a point 0 in a straight line, in such a way that its velocity, v m/s, t
seconds after leaving 0, is given by
v = 20t- 5t 2 •
The motion continues until the particle comes to rest at P. Calculate:
(a) the distance OP
(b) the greatest velocity of the particle.

I I
Previously velocity and acceleration have been expressed as functions of time. If the velocity is
expressed as a function of position we need to find a formula for acceleration.
170
dv dv dx
a=-=-- (function of a function rule)
dt dx · dt
dv . dx )
=v- ( smce dt = v (1)
dx
2
!!_(1. 2) _ d(!v ) dv
Now, dx zV - dv · dx
dv
= v · dx (2)
d(l.v 2 )
From Equations (1) and (2) a = ~x .

Hence:

Example (i):
The velocity of a particle moving in a straight line is given by v = 2x + 1, where x metres is the
distance from a fixed point 0 and v is the velocity in metres per second. Find the acceleration of the
particle when it is 3m from 0.
Solution:
2
a = d(!v ) = d !(2x + 1) 2
dx dx
d(2x 2 + 2x + !)
dx
= 4x + 2
When x = 3, a = 14.
When the displacement from 0 is 3m the acceleration is 14 mjs 2 •
Example (ii):
The acceleration of a particle is defined in terms of its position by the equation a = 2x + 4. If v = 5
when x = 2, find the velocity when x = 4.
Solution:
a=2x+4
v2 = f
2 adx = f (4x + 8)dx
i.e. v2 = 2x 2 + 8x + C
when x = 2, v = 5
therefore 25 = 8 + 16 + C
thus C= 1
and v2 = 2x 2 + 8x + 1
.". v = ±-./
2x 2 + 8x + 1.
The condition x = 2 when v = 5 is only satisfied by the positive solution.
:. v = -./2x 2 + 8x + 1 is the required velocity equation.
When x = 4, v = -}32 + 32 + 1
=J65
~ 8·06.
The velocity of the particle at x = 4 is approximately 8·06 mjs.
Example (iii):
A particle moves such that when it is x metres from the origin its acceleration is given by a = -!e-x.
Find its velocity when x =
3, given that v 1 when x 0.= =
171
Solution:
a= -!e-x
J
v2 = 2 adx = f -e-xdx
i.e. v2 = e-x + C
But v = I when x = 0, .'. C = 0.
Hence v 2 = e-x
v= ±J?X.
The condition v = I when x = 0 is only satisfied by the positive solution .
.'. v = JC_ is the velocity equation required.
When x = 3, v = ~
~ 0·22.
Hence the velocity at x = 3 is approximately 0·22 mjs.

I. Given x = J6X, find the acceleration x.


2. A particle moves so that when it is x metres from the origin its velocity is given by v = 2x - 1.
Find the acceleration of the particle when x = 4.
3. A particle moves with velocity v metres per second where v2 = 2x + 4 and x metres is the dis-
placement from the origin.
(a) Find the acceleration of the particle.
(b) Find the displacement of the particle when the velocity is zero.
(c) Find the velocity of the particle when the displacement is 6 m. How do you explain the two
answers?
4. A particle is moving in a straight line with a constant acceleration of 2 mjs 2 • If it starts from the
origin with a velocity of -6 mjs find:
(a) an expression for the velocity in terms of the displacement.
(b) its position when the velocity is zero.
(c) the velocity of the particle when it returns to the origin.
5. A body starts 15m from the origin, having a velocity of I2 mjs towards the origin. If the body
is subjected to a constant acceleration of 3 mjs 2 in the opposite direction to its initial motion
find:
(a) an expression for the velocity in terms of the displacement.
(b) the velocity when the displacement is 2·5 m.
6. A particle moves such that when it is x metres from the origin its acceleration is x 2 mjs 2 • If initially
the particle is at rest 2 m from the origin, find its velocity when it is 4 m from the origin.
7. A particle moves such that its acceleration when it is x metres from the origin is given by x= ~.
If the velocity is 2 mjs when x = 1, find the velocity when x = e2 •
8. The motion of a particle is such that when it is x metres from the origin its acceleration is given
by a = -e-x. Given that v = 2 when x = 0, find v when x = 2.

9. A particle moving in a straight line has an acceleration given by x = ~ when its displacement
X - 8
is x metres from the origin. Find its velocity when x = 4, given that x = 0 when x = 3.
IO. A particle moves such that when its displacement is x metres from the origin its velocity is given
by dx
dt-- v = y f2x + 4.
172
(a) By using the result that :~ = :"\:, show that t = f (2x + 4)-tdx.
dt
(b) Gtven that t = 0 when x = 0, show that t = .J2x
0

+4 - 2 and hence that x =


t2 + 4t .
2
(c) Find an expression for the velocity in terms oft and find the velocity when t = 5.
11. If the velocity of a particle is given by v = -/ 4x + 25 where x metres is the displacement from
the origin and if t = 0 when x = 0 find:
(a) an expression for x in terms oft
(b) an expression for v in terms oft.

Suppose a particle moves with constant acceleration a mjs 2 with initial displacement x 0 and initial
velocity u mjs.
dv
Now -=a
dt
Integrating v =at+ C, but v = u when t = 0, therefore C = u
Hence v = u +at (1)
dx
That is dt = u +at
Integrating x = ut + 1at 2 + k, but x = x 0 when t = 0, therefore k = x 0
Hence x - x 0 = ut + 1at 2
Put s = x- x 0 (the change in displacement)
s = ut + 1at 2 (2)
Squaring Equation (1) v2 = (u + at) 2
= u 2 + 2uat + a 2 t 2
= u + 2a(ut + !at )
2 2

Since s = ut + !at we have v = u + 2as.


2 2 2
(3)

Equations of motion under constant acceleration


v = u +at (1)
s = ut + !at 2 (2)
v2 = u 2 + 2as (3)

When a body is in vertical motion under gravity it has a constant acceleration of g mjs 2 downwards,
where g ~ 9·8.
Hence the equations of motion v = u + at, s = ut + !at 2 and v2 = u 2 + 2as apply to problems
concerning vertical motion under gravity, and if we take the upwards direction as positive then
a= -g.
Example:
A body is projected vertically into the air with a velocity of 49 mjs.
(a) How high will it rise?
(b) How long will it be before it returns to its starting point?
173
Solution:
(a) At the greatest height the velocity is zero
u = 49 Using v2 = u2 + 2as
v= 0 0 = 49 2 + 2 x (- 9·8)s
a = -9·8 s = 122·5
s = ?
The greatest height reached is 122·5 m.
t =-
(b) When the body returns to the starting point its displacement is zero
u = 49 Using s = ut + iat 2
v =- 0 = 49t + i(-9·8)! 2
a = -9·8 0 = t(49 - 4·9t)
s = 0 = 0 or t = 10
t = ? The body returns to the starting point after 10 seconds.

1. A body is projected vertically with a velocity of 24·5 mjs. What time elapses before it returns to
its starting point and what is the maximum height attained?
2. A stone is thrown vertically upwards with a velocity of 4·9 mjs. What time elapses before its
velocity is 2 m/s?
3. A steel ball is projected vertically and reaches its maximum height in 6 seconds. What height was
attained and what was the velocity of projection?
4. With what velocity must an object be projected vertically to reach a height of 50 m? How long
will it take to reach this height?
5. A stone is thrown vertically into the air with a velocity of 10 mjs.
(a) How high will it rise?
(b) At what times after projection will it be 3m above ground and what will be its speed at those
times?
6. A body is let fall from a height of 150 m. What is its height and velocity after falling freely for
5 seconds?
7. A body is projected vertically upwards with a velocity of 80 mjs. What will be its position and
velocity after 4 seconds?
8. A body is projected vertically upwards with a velocity of 27 mfs from a position 60 m high. How
long will it be before it reaches the ground below and what will be its velocity just before it hits
the ground?
9. A stone is let fall from a tall building and 1 second later another stone is projected vertically
downwards with a velocity of 20 mfs. When will the second stone overtake the first?
10. An object is projected vertically upwards with a speed of 49 mfs. Two seconds later another
object is projected vertically upwards from the same spot at the same speed. Find when and where
the two objects meet.
11. From a stationary helicopter a steel ball is projected vertically upwards with a velocity of 4·9 mjs.
If the ball reaches the ground in 5 seconds, how high was the helicopter when the ball was projected?
12. An object falling in a vertical line passes a window 3m high in-! second. Find the distance above
the top of the window from which the object was let fall?
174
Suppose a particle is projected with a velocity V at an
angle of a to the horizontal, then:
the horizontal component of this velocity is V cos a
the vertical component of this velocity is V sin a V sin a

Now let us consider the motion of a projectile fired on a horizontal plane .

.I
If air resistance is neglected the only acceleration acting on the body is that due to gravity, which
is acting vertically downwards.
There is no horizontal acceleration and hence throughout the motion the horizontal component
of the velocity is constant and equal to the initial horizontal velocity of V cos a.
We now consider separately the vertical and horizontal components of the motion.

VERTICAL MOTION
Since the vertical motion is under constant acceleration we can use the relevant formulae where
a = - g and u = V sin a.
Time of flight-the time of flight is twice the time to attain greatest height.
-at the highest point the vertical velocity is zero.
The time to attain highest point is given by:
u = V sin a v = u + at
v = 0 0 = V sin a - g t
a= -g Vsin a
t =---.
s =- g
t = ?
T 1me o fl'1ght = 2t = -
. f 2 V-
sin-a
g
Greatest height-use the formula v2 = u 2 + 2as
u = V sin a 0 = V 2 sin 2 a - 2gh
v=O V2 sin 2 a
h = --=---
a= -g 2g
s = h
t =-
175
HORIZONTAL MOTION
Range = horizontal distance travelled
= horizontal velocity x time of flight
2Vsin IX
V cosiX x ~-­
g
2
V 2 sin IX cos IX
g
V 2 sin 21X
g
The maximum range is obtained when sin 21X = 1. That is when IX = 45°.
vz
Maximum range = - .
g

Example (i):
A stone is projected with a velocity of 29·4 mjs at an angle of 30° to the horizontal. Find:
(a) the maximum height attained
(b) the range on a horizontal plane.
Solution:
(a) Considering the vertical motion
u = 29-4 sin 30° Using v2 = u2 + 2as
v= 0 0 = (29·4 sin 30°) 2 - 2 X 9·8 X h
a= -9·8 h = 11·025
s = h
t =~

Maximum height is approximately 11 m.


(b) Time to reach highest point.
u = 29·4 sin 30° Using v = u + at
v = 0 0 = 29·4 sin 30° - 9·8t
a = - 9·8 29·4 sin 30°
s = ~ 9·8
t = ?
. ffl" h 2 x 29·4 sin 30°
T Ime o Ig t = 2 t = .
98
Range = horizontal velocity x time of flight
29·4 cos 30° x 2 x 29·4 sin 30°
9·8
= 76·38
The range is approximately 76·4 m.
N.B. The formulae for greatest height and range could be used but it is probably safer not to rely
on memory and to work each question from the beginning. Also, in some questions the cir-
cumstances change and the fortrjulae cannot be applied so easily.

Example (ii):
A projectile is fired from the edge of a cliff 100 m high
with a velocity of200 mjs. If the angle of projection is
20° above the horizontal, find the time of flight and
the range of the projectile.
100m

176
Solution:
Consider first the vertical motion to find the time of flight.
u = 200 sin 20° Using s = ut + !at 2
v =- -100 = (200 sin 20°)! 4·9t 2
a= -9·8 4·9t 2 - (200 sin 20°)t- 100 = 0
s = -100 200 sin 20 ± J (200 sin 20°) 2 + 400 x 4·9
t = ? t = 9·8
The time of flight will be given by the positive solution.
Hence t = 15·29
Consider the horizontal motion.
Range = horizontal component of velocity x time of flight
= 200 cos 20° x 15·29 m
= 2874m.

Example (iii):
A golf ball is hit with a velocity of 40 mjs at an angle of 38° to the horizontal. If it just clears a tree
20m away, find the height of the tree.
Solution:
· Consider first the horizontal motion to find the time to reach the tree.
.
T 1me to tree =
20 osecond s
40 cos 38
Consider the vertical motion to find the height of the ball on reaching the tree.
u = 40 sin 38° Using s = ut + !at 2
v =
a
__ --9·8 h = 40 sin 38o x 4 20
0 COS 38°
- 4·9(40 20 3
COS 8°
)2
s = h = 13·65
20
40 cos 38°
The height of the tree is 13-65 m.

l. A stone is projected with a velocity of 34·5 mjs in a direction making an angle of 30° with the
horizontal. What is the maximum height attained and what is the range? What would be' the
maximum range possible with the same initial velocity?
2. A cricketer can throw a ball 40 m vertically upwards. What is his maximum throwing range on
a flat cricket field?
3. A body is projected with an initial velocity of 45 mjs and reaches maximum height after 4 seconds.
Find the angle of projection.
4. A body is projected from a point 80 m above the ground with a velocity of 20 mjs at an angle of
60° above the horizontal. Assuming the ground horizontal, find when and where it will strike
the ground.
5. A bullet is fired horizontally with a velocity of 800 mjs from a height of 3m above the horizontal
plane. What is the range of the bullet?
6. A boy can throw a stone a maximum horizontal distance of 80 m. How long is the stone in the
air and to what height does it rise?
7. A body is projected with a velocity of V mjs at an angle a to the horizontal. If the body reaches
the highest point in its path after 2 seconds~ find the height at that instant.
177
8. A golf ball is hit with a velocity of 35 mjs at an angle of 40° to the horizontal. If the ball just clears
a shrub 5 m away, find the height of the shrub.
9. A cricket ball is hit so that it reaches a height of 25m and strikes the ground 100m from the bat.
Find the magnitude and direction of the initial velocity.
10. At the highest point of its path a body has a velocity of 10 mjs and is 8 m above the ground. Find
the angle of projection and the initial velocity.
11. The highest point of the trajectory of a body is 300 m and the horizontal range is 5000 m. Find
the initial velocity and angle of projection.
12. A projectile reaches a maximum height of 100 m above the point of projection at a horizontal
distance of 250m from this point. Show that it was fired at an angle of elevation of approximately
38°40' and calculate the maximum range for the same velocity of projection.
13. Find an expression for the maximum range in the horizontal plane for a particle with initial
velocity V mjs. Find the two angles of projection to give a range equal to half the maximum range.
14. A projectile has a trajectory such that its range on a horizontal plane is four times the maximum
height attained during flight. Find the angle of projection.

Although most problems concerning projectiles can be solved using the techniques so far employed,
it is of interest to show that, neglecting air resistance, the trajectory of a projectile is parabolic in shape.
Problems can, of course, be solved using the mathematics developed in this trajectory.
By considering the vertical and horizontal components of the motion of a projectile we can establish
the equation of its trajectory.
We use the x andy axes for horizontal and vertical axes and origin as the point of projection.

y
v

VERTICAL MOTION
The initial velocity in a vertical direction is V sin a. If we take the upwards direction as positive the
particle experiences a uniform acceleration of -g due to gravity.
d2y
Thus dt 2 = -g
dy-
Hence dt - -gt +c
But dy V sin a when t = 0, thus c = V sin a
dt
Thus dy = V sin a - g t
dt
Hence y = V sin a . t - !g t 2 + k
But y = 0 when t = 0, thus k = 0
Giving y = V sin a . t - !g t 2
178
HORIZONTAL MOTION
The initial velocity in a horizontal direction is V cos IX and since no forces are acting in a horizontal
direction, this velocity will remain uniform.
dx
That is V cos IX
dt
thus x = V cos IX • t + c
But x = 0 when t = 0, thus c = 0
Hence x = V cos IX • t
Thus the pair of equations: x = V cos IX • t ( 1)
y = V sin IX. t - !gt 2 (2)
gives a parametric representation of the flight path of the projectile.
Eliminating t
X
From (1) t = --
V cos IX
2
Substituting in (2) y = V sin IX • X - 1
2g
( X )
V COS IX V COS IX
gx2
i.e. y = x tan IX -
2V 2 cos 2 IX
This Cartesian equation is of the form y = ax + bx 2 (since IX, V and g are constants), thus the equa-
tion represents a parabola.

1. A projectile is launched at an angle of30° to the horizontal with a velocity of 20 mjs. Find equations
for x andy in terms of time t and find the position coordinates of the projectile after 2 seconds.
2. A steel ball is projected at an angle of 35° to the horizontal with a velocity of 40 mjs. Find:
(a) equations for x andy in terms oft
(b) the Cartesian equation relating x and y
(c) by putting y = 0 in the Cartesian equation, find the range on a horizontal plane for the
given initial conditions.
3. A projectile is fired at an angle of 40° to the horizontal with an initial velocity of 60 mjs. Find
equations for height and horizontal distance in terms of time and use the height equation to
determine the time of flight.
4. A cannon ball was fired at an angle of 35° to the horizontal and hit a target 400 m distant on the
same level 8 seconds later. Find the initial velocity of the cannon ball and the maximum height
it reaches.

When moving in simple harmonic motion, a particle is oscillating backwards and forwards along a
straight line about a central position. This occurs in practice in the motion of a mass attached to a
spring, in the motion of a pendulum bob or in the vertical motion of a float as it rises and falls with
the tide.
179
One way to investigate simple harmonic motion is
to start by considering the motion of a point Q (see
Diagram) moving with uniform speed around a circle
with centre 0 and radius a.
As Q moves the angle echanges with respect to time.
The rate of change of e is called the angular velocity
ofQ.
Now, if we take n to be the angular velocity of Q
then:
de
dt = l1.

Integrating, we have e = 111 + a, where a is the constant of integration.


Now let us consider the motion of the point P which is the projection of the point Q on the x axis.
As the point Q moves around the circle we can see that the point P will oscillate along the x axis
between A and A', the diameter.
The motion of the point P is said to be simple harmonic motion.
Now, if Q is the point (x, y)
then OP = a cos e.
That is, the distance x of the point P from the centre 0 is given by:
X= a COS e.
But since e = nt + a, the distance x at timet is given by:
x = a cos(nt + a) (1)

a ~,

0 p
A' A
X

A point moving in a straight line is describing simple harmonic motion if:


x = a cos(nt + a)
where a, nand a are constants, a > 0, 11 > 0.

(i) If t = 0, then x = a cos a, which gives the initial position of the point in its motion along AA'.
(ii) nt + a is called the phase of the motion and a is called the initial phase.

AMPLITUDE
The maximum value of x is called the amplitude of the motion.
Since the maximum value of cos(nt + a) is 1, the maximum value of x = a cos(nt + a) is a. Thus
the amplitude is a.
From Equation (1) it follows immediately by differentiation that:
x = v = -an sin(nt + a) (2)
x = v = -an 2 cos(nt + a) (3)
where_\; and x are the velocity and acceleration at time t.

PERIOD
The period of the motion (T) is the time taken for the particle to complete one oscillation. That is to
return to the same position travelling in the same direction.
2
The period is given by T = n.
n
180
2
This can be seen by noting that in Equations (1), (2) and (3) above if tis increased by n the values
- 11
of x, x and x are unaltered. The result can also
be seen from the motion of Q around the circle of
2
reference since if n is the angular velocity of Q, the time for a full revolution must be n.
11

FREQUENCY
Iff oscillations take place in 1 second and each oscillation takes time T then we have
fT= 1
and f = _I_ = !!_
T 2n
f is called the frequency of the simple harmonic motion and is equal to the reciprocal of the period.

SPEED AND ACCELERATION AS FUNCTIONS OF POSITION


Beginning with xa cos (nt + a)
= (1)
x=
-an sin (11t + a) (2)
x = -an 2 cos (nt + a) (3)
Squaring (1) x 2 = a 2 cos 2 (nt + a) (i)
Squaring (2) x2 = a211 2 sin 2 (11t + a)
·2
(ii)
From (ii) x2 = a 2 sin 2 (11t + a) (iii)
11

x2 +~
x2 a2
Adding (i) and (iii) 112
=

x2 = 2(a 2 - x 2) or v2 = n 2(a 2 - x 2)
11 (4)
Also, since _\' = -an 2 cos (nt + a)
x= -n 2 [a cos (nt + a)]
x= -11
2
x (5)

USEFUL PROPERTIES OF SIMPLE HARMONIC MOTION


We now investigate Equations (4) and (5) to derive some further properties of simple harmonic
motion.
From v2 = n2(a 2 - x 2) we can see that:
(i) v = 0 when x = a or x = -a. That is, at the extremities of the motion the velocity is zero.
(ii) When x = 0, the velocity takes its maximum value of ±an. That is, at the origin or centre of
motion the velocity is a maximum.
From x = -n 2 x we can see that:
(iii) When x = 0, x = 0. That is, the acceleration is zero at the origin.
(iv) When x = ±a, x = + an 2. That is, the acceleration is a maximum at the extremities of the
motion.
An alternative definition of simple harmonic motion is:
If a particle moves in a straight line such that its acceleration towards a fixed point in the line is pro-
portional to its distance from the point, then the particle is moving in simple harmonic motion.
Equation (5) above, x = - n 2 x, satisfies this definition and hence an acceleration equation in this
form indicates that the particle is moving in simple harmonic motion.
Example (i):
A particle moving with simple harmonic motion starts from rest at a distance 6 m from the centre of
oscillation. If the period is 4n seconds:
181
(a) find the time taken to move to a point 3m from the origin
(b) find the velocity and acceleration at this point.
Solution:
2n
(a) From T = -
n
2n 2n 1
n=-=-=-
T 4n 2
In simple harmonic motion the velocity is zero at maximum amplitude. As the point starts from
rest 6 m from the centre of oscillation the amplitude is 6 m.
Thus, using x = a cos (nt + c.:)
X = 6 cos Ctt + c.:)
When t = 0, x = 6
6 = 6 cos (0 + c.:)
.', COS IX = 1
Thus c.:= 0
Hence x = 6 cos tt
When x = 3, 3 = 6 cos tt
cos Ctt) = t
tt
= cos- 1 t
tt = ~
t = 237t
2
The time taken to move to a point 3 m from the origin is 3" seconds.
(b) When x = 3, the velocity is given by
vz = nz(az - xz)
= !(36- 9)
v =J27
--
2
Therefore when x = 3, the velocity is approximately 2·6 mfs.
When x = 3, the acceleration is given by
.X= -n 2 x
-! x 3 = -imfs 2
The acceleration is -0·75 mfs 2 •

Example (ii):
A point moving with simple harmonic motion starts from a point 5 em from the centre of the motion
with a speed of 1 cm/s. The period is 8 seconds. Find the maximum speed and the maximum accelera-
tion.
Solution:
From T= 2n
n
2n 2n n
We have n = - = - = -
T 8 4
Substituting v = 1, x = 5, n = !fin the equation
vz = nz(az - xz)
nz
1 = (a 2 - 25)
16
16 = az- 25
nz
2 16
a = 25 + 2
n
a ~ 5·16
The maximum speed occurs when x = 0.
182
Using v2 = n2(a2 - x2)
2
= ~(5·16 2 )
16
The maximum speed is 4·05 cmjs.
The maximum acceleration occurs at maximum amplitude, i.e. when x = 5·16.
x = -n 2 x
n2
x= - -16X 5·16
-3·18
The maximum acceleration is 3·18 mjs 2 acting towards the origin.

1. A particle moves in simple harmonic motion with n = 2. If its rest position is distant 4 m from
the origin, find its velocity as it passes through the origin.
2. A particle moves in simple harmonic motion. Find the period of the motion if:
(a) the acceleration is 4 cm/s 2 , when the particle is 2 em from the centre of motion.
(b) the acceleration is 9 mjsec 2 , when the particle is 3 m from the centre of motion.
3. A particle is moving in simple harmonic motion. Find the speed of the particle when it is passing
through the centre of its motion given:
(a) the amplitude is 10 em and the period is -I seconds.
(b) the amplitude is 2 metres and the period is 3 seconds.
4. A particle moves in a straight line with simple harmonic motion; the amplitude of the motion is
8 em and the period is 2n seconds. Find the velocity and acceleration of the particle when its
distance from the origin is 6 em.
5. A point moving with simple harmonic motion has a velocity of 4 mjs when passing through the
centre of its path and its period is n seconds. What is its velocity when it is 1 m from the position
in which its velocity is zero.
6. A particle moving with simple harmonic motion starts from rest at a distance of 8 m from the
centre of oscillation. If the period is 4 seconds, find the initial acceleration and find the speed of
the particle when it is 6 m from the centre.
7. A particle moving with simple harmonic motion starts from rest at a distance 3 m from the centre
of oscillation. If the period is 2n seconds:
(a) find the time taken to move to a point 2m from the origin and
(b) find the velocity and acceleration at this point.
8. A particle vibrating in simple harmonic motion starts from a point 6 em from the centre of the
motion with a speed of 2 cmjs. The period is 10'seconds. Find the maximum speed and the max-
imum acceleration.
9. A body is vibrating in simple harmonic motion, the period being 12 seconds and the amplitude
10 em. What is the velocity and acceleration of the body when the displacement is 6 em?
10. A particle travels in simple harmonic motion such that its distance in metres from the centre of
motion is given by x = a cos (3t + o:) at time t seconds. If the particle starts from rest at a distance
of 5 m from the origin, find the distance from the origin after 3 seconds.
11. A particle is vibrating in simple harmonic motion. Show that the time required to travel from the
centre of motion to a point at a distance from the centre equal to half the amplitude is / 2 of the
period.
183
12. A point moves with simple harmonic motion; if when at distances of 3 and 4 em from the centre
of its path, its velocities are 8 and 6 cmfs respectively, find its period and its acceleration when at
its greatest distance from the centre.
13. A particle moving with simple harmonic motion makes 10 complete oscillations per minute. Its
maximum speed is 5 mjs.
(a) Find the amplitude of the motion.
(b) What is the time taken to move from full amplitude to half amplitude?
14. A particle moves with simple harmonic motion. At the extremities of the motion the absolute
value of the acceleration is 1 cmjs 2 and when the particle is 3 em from the centre of motion the
speed is 2J2 cmjs. Find the period and amplitude for this motion.
15. A particle is vibrating in simple harmonic motion, making 20 complete oscillations per minute. If
the amplitude is 6 em, calculate the velocity and acceleration when the particle is:
(a) at the centre
(b) at the extremities
(c) at a distance of 4 em from the centre of its path.
16. The rise and fall of the tide approximate simple harmonic motion. Suppose the interval between
successive high tides in a certain harbour is 12 hours 30 minutes and that in the entrance the
depth of water at high tide is 10 m and at low tide 6 m. If on a certain day low tide occurs at
6 a.m., at what time will the depth of water in the harbour entrance first reach 9 m?
17. The motion of a pendulum may be approximately represented by the acceleration equation
.. g
X = - LX

where x metres is the displacement from the central position at time t seconds, g is the gravita-
tional constant (g ~ 9·8) and L metres is the length of the pendulum. By comparing the equation
with .X = - n 2 x, find the time for a complete oscillation by a pendulum of length 1 m.

184
The graph of y = tan-'x on a computer monitor.
CHAPTER 27

Consider the simple linear function y f(x)


= y

=
2x. 6
This function is continuous and increasing over any y
5
domain within the set of real numbers. For convenience
select the domain as 0 ~ x ~ 3. It is satisfied by an infinite 4
set of ordered pairs such as
{(0, 0), (1, 2), (2, 4), (3, 6), (1~, 3)} 3
The function/: y = 2x takes any value of x in the domain 2
and transforms it to a unique value of y. For example:
OHO
1 H2
2H4. 3 4 5 6 X

The range of the function is the set of y values


{0 ~ y ~ 6}.

As often happens in mathematics we may need to use the reverse or inverse operation. For example,
the inverse of multiplication is division; the inverse to finding the square root is squaring and the
inverse of finding loge x is taking the exponential, ex.
In the above case, a function is required which reverses the second set of elements back to the original
set.
OHO
2H1
4H2
Thus, for each y in the range off there is a new function which reverses the sets of ordered pairs.
For example, it would be satisfied by the set of ordered pairs.
{(6, 3), (4, 2), (3, 1!), (2, 1), (0, 0)}
and the general element (x, y) becomes the element (y, x).
Thus for f: y = 2x the inverse function may be expressed as
x = 2y or on solving for y
X
y = g(x), say.
2=
Now the domain of g(x) = {0 ~ x ~ 6}
= range of f(x)
and the range of g(x) = {0 ~ y ~ 3}
= domain ofj(x).
Consider the point P(I, 2) satisfying f(x) = 2x
and the point Q(2, 1) satisfying g(x) = ~·
1 2 2 1
The mid-point of PQ is M = ( ; , ; )

= (~, ~)
which lies on the line y = x. Similarly for other corresponding points. Thus the graphs of f(x) and
its inverse g(x) are reflections in the line y = x.
186
y
Thus f(x) transforms 3 H 6
6
g(x) transforms 6 H 3.
For the domain 0 ~ x ~ 3
g{f(x)} = g{2x} asf(x) = 2x
_ (2x) X
2 - as g(x) =
2
= x. 3
Also for the domain 0 ~ x ~ 6
X 2
f{g(x)} as g(x) = -
2
asf(x) = 2x
=X. 2 3 4 5
f(x), g(x) are called mutually inverse functions so
that if both are applied in succession to any point
in the domain, the original value results.
For example, when x = 2: f(x) = 2x
f(2) = 4
X
and g(x) =
2
g(4) = 2, the original value.
The special notation used to show the inverse of a functionfisf- 1 .
Thus if f: y = 2x or f(x) = 2x
-1 X
f :y =2

N.B. (i) f- 1(x) is not the same as {f(x)} - 1 = f/Y).


(ii) A function has an inverse if a horizontal line cuts the graph of the function at only one
point.

If y = f(x), with domain a ~ x ~ band rangef(a) ~ y ~ f(b) has an inverse y = f- 1 (x) then
the domain of y = f- 1(x) isf(a) ~ X ~ f(b) and the range is a ~ y ~ b.
For y = f(x) to have an inverse each x in the domain must have a unique value y and eachy must
correspond to a unique value of x. (This is called a 1-1 correspondence.)

Example (i):
Find the inverse off(x) = x 3 . State the domain and range of each function. Sketch the curve.

Solution:
y = f(x) = x 3 Domain off= D1 =all real x.
Range off= R 1 = all real x
To find the inversef- 1
Intercharge (x, y): x = y 3
Solve for y: f- 1 : y = xi or f- 1(x) = xi
Dr1 = all real x
Rr1 = all real x.
187
Example (ii):
Sketch the graph of y = x 2 •
(a) Does y = x 2 have an inverse?
(b) Show that y = x 2 with domain 0 :::;; x :::;; 4 has an inverse.
(c) Find the inverse function stating its domain and range.
Solution:
y

\ 16
\
\ 14
\
\ 12

' \
\
\
10

'
\
\
6

~1\
I ' 1..
-4 -3 -2 -1 ' 0 2 3 4 X

(a) y = x 2 has no inverse as each y value has two possible x values.


For example, when y = 4, x = 2 or -2.
(b) f: y = x 2 domain 0 :::;; x :::;; 4
range 0 :::;; y :::;; 16

domain 0 :::;; x :::;; 16


range 0 :::;; y :::;; 4
188
1. For the following functions, f, find f- 1 and state its domain and range. Sketch the graphs off
andf- 1 .
(a) f: y = !x, 0 ~ x ~ 8 (d) f: y = -x 3 , -2 ~ x ~ 2
(b) j: )I = X + 1, - 2 ~ X ~ 2 (e) j: X + )I - 2 = 0, -3 ~ X ~ 3
(c) f: y = - 2x, - 4 ~ x ~ 4 (f) f: y = Jx
+ 1, -1 ~ ~ 3 x
2. Find the inverse functionf- 1 for each of the following. State the domain ofj- 1 .
1
(a) f(x) = 3 - 2x (c) f(x) = -
X
1
(b) f(x) = x2 - 2 (d) f(x) = --
x+2
3. (a) fis a function defined by y =ex. What is the inverse functionf- 1 ?
Sketch both graphs together with the graph of y = x.
(b) Ifjis defined by {(x, y): y = lOx} what is the inverse functionf- 1 ? State its domain and range.
(c) y = x 2 - 1 when x >
0. Find the inverse function and state its domain and range.
3
(d) Sketch the graph of y = ; x. By reflecting this graph in the line y = x, graph the inverse
function.
Find the equation for the inverse function.
4. Ifjis defined by y = 2x + 4, find:
(a) /-1
(b) the point of intersection ofjandf- 1
(c) show that this point lies on the line y = x.
5. (i) State a restricted domain so that the following functions will have an inverse.
(ii) Find the inverse function.
(iii) State the domain of the inverse function.
(a) y = 3x 2 (d) y = x 4
(b) y = x + 1
2
(e) y = x(x - 2)
(c) y = (x - 2) 2

AN IMPORTANT RELATIONSHIP
Consider a function y = f(x) which is differentiable at a point
P(x, y).
Let the tangent at P meet the x axis at Q and they axis at R.
Let the tangent make an angle ()with the x axis and an angle a
with the y axis.
Now the gradient of the tangent PQ toy= f(x) is
dy = tan () (i)
dx
This graph may also be expressed in the form x = g(y)
and its gradient (with respect to they axis) is
dx (ii)
dy =tan a
Now LOQR = () (vertically opposite angles)
n n
.'.8+a=
2 as LQOR = l

189
tan a = tan
= cot 8
0-8) as the tan of an angle is equal to the cot of its complement.
1
tan 8
From equation (i) tan 8 = ddy and from equation (ii) tan a = ddx
X y
. dx 1
.. dy dy
dx
or dy. dx = 1
dx dy
(For a formal proof see Appendix XIII.)

If y is differentiable then
dy. dx = 1
dx dy
dx 1
or
dy dy
dx

Example:
If y = x 3 , find
(a) x as a function of y
(b) 1x in terms of y
dx
(c) dy.

Solution:
(a) y = x3
.·.X= yt
(b) dy = 3x 2
dx
= 3(y1)2
= 3y32
(c) dx 1
dy dy
dx
1
--2
3y'
=
1 -~3
-y
3
. d(y1) 1 -~
l.e. -----;jJ = 3y 3

190
1. For each of the following
(i) find 1x
(ii) express x in terms of y
dx m
"') fim d dy
(111 · terms of x

(iv) show that 1x ·~; = 1.


(a) y = x! (c) y = J2=X
(b) y = x 5 (d) y = x2
- 1
2. If y = x! find (a) x as a function of y
dx
(b) dy and
dy 1 2
(c) hence show that dx = x- 3 .
3
3
3. If y = e x,
(a) find the inverse function as a function of y
(b) find ~; and hence find 1x in terms of x
(c) write down the inverse function as a function of x and hence find 1x.
4. For each of the following functions
(i) express each inverse function as a function of y
(ii) for the functionf- 1 , find~; and hence 1x.
(a) f: y x + 1 (give 1x in terms of x)
=
2

(b) f: y x + x (give 1x in terms of y)


=
5

(c) f: y 27x - x (give 1x in terms of y).


=
3

5. If xy 2 = 9, by expressing y as a function of x, find


(a) 1x in terms of x and hence
(b) :in terms of x.

(c) By expressing x as a function of y find~; directly and so check the answer to Part (b).
6. For the following functions find f- 1 as a function of y and hence the derivative of f- 1 at the
origin (0, 0)
(a) y = x(9 - x 2 )
X
(b) y = -1--2.
-X

The function y = sin x means that for any given real value of x it is possible to find a unique real
number y called the 'sine of x'.
191
To find an inverse function, which reverses this operation, is to interchange the x andy elements.
Thus if, f: y = sin x (i)
f- 1 : x = sin y (ii)
It is now necessary to find yin terms of x, i.e., to find a function so that y is a real number whose
'sine is x'.
Unfortunately, it is not possible to express y as a function of x by manipulating Equation (ii).
Now, f: y = sin x and f- 1 : x = sin yare reflections of each other in the line y = x and so it is
possible to draw the graph of x = sin y.

f: y =sin x
f7r /

•: x ,~sin y

From the graph it is obvious that x = sin y is not a function as for each value of x there is more
then one value of y.
To overcome the two problems
1. The inverse sine function is defined as a new function, which is written as
y = sin- 1 x.
This is read as 'sine to the minus one x'.
(i) This also appears on many calculators an in many books as y = arc sin x.
1
(ii) sin- 1 x does not mean the reciprocal (sin x)- 1 = -.--.
Slll X

2. To overcome the second problem, the domain of y = sin x is restricted so that the inverse
y = sin- 1 x exists as an inverse function.
It is usual to restrict the domain of y = sin x to the interval -~ ~ x ~ ~which still allows
the full range -1 ~ y ~ 1. The inverse function y = sin- 1 x now exists and has as domain
-1 ~ x ~ 1 and range-~~ y ~ ~·These functions are represented by the continuous lines on
the previous graph.
192
Thus if
f: y = sin x with D 1 = {x: -~ < x < ~}
R1 = {y: - 1 < y < 1}
then the inverse function is
f- 1 : y = sin - 1 x ( = arc sin x) with Dr 1 = {x: - 1 < x < 1}
Rrt = {y: -~ < y < ~}
and its graph is y

-1 X

Now the general equation y = sin x has many solutions.


By referring to the graph of y = sin x on Page 192, if y has the value 1, then x could have the values
~ or - 3{. If the graph were extended in both directions other values would also satisfy the equation,
e.g., s;, 92n, _ 7{, _ 1~".
The equation y = sin x, with changed symbols could be written as
sine= b.
It is now necessary to find a general solution for e, for any given b.
Take b =!
i.e. sine =!
As sine is positive in the first and second quadrants and sing = !,
for 0 < e < 2n the solutions are~ and n - ~·
Taking the next revolution,
for 2n < e < 4n the solutions are 2n + ~and 3n - ~,and so on,
for 4n < e < 6n the solutions are 4n + ~and 5n - ~,
for 6n < e < 8n the solutions are 6n + ~and 7n - ~,

for a negative revolution,


-2n < e < 0, the solutions are -2n +~and -n- ~·
From these results, if 11 is even
e = 111t + ~
if 11 is odd
e= 111t- *
These can be combined as a general solution
e = 111t + ( -1)" . ~where 11 is any integer.
Now as~= sin- ! 1

e = 111t + ( -1)" sin- 1 (!)


In the general case
e = 111t + (- 1)" sin - 1 b.
[-~-h~s~ t~-;~~~;~~~l~ti~~-:L~~ e ~-b is e : 11n + ( -1 )" sin - 1 b, where 11 is any integer.

193
THE INVERSE COSINE FUNCTION
As for y = sin- 1 x, a similar inverse function may be found for
f: Y =COS X
Then f- 1 : x = cos y
and by definition this is expressed as
y = cos- 1 x (cos to the minus 1, x)
To ensure the existence of an inverse a restriction must be placed on the domain off If the graph of
y = cos xis considered a suitable restriction, which covers the full range of the function, -1 ::::; y ::::; 1
would be to take the domain as
0::::; x::::; n.
This is shown as the unbroken line.
y

-COS X

-~ hr X

Hence, on interchanging the x and y values the domain of y = cos- 1 x is -1 ::::; x ::::; 1 and its
range is 0 ::::; y ::::; n.
Its graph is the reflection of y = cos x, 0 ::::; x ::::; n in the line y = x.
The graph cuts the y axis at (0, -f), which is the reflection of the point y

(-f, 0). It is concave upwards when -1 < x < 0 and concave down-
wards from 0 < x < 1.

'x

-1

Thus, iff: y = cos x with D 1 = {x: 0 ::::; x::::; n}


R1 = {y: -1 ::::; y ::::; 1}
then the inverse function is
f- 1 :y = cos- 1 x withD1 -, = {x: -1::::; x::::; 1}
Rr' = {y: 0 ::::; y ::::; n}.
Its graph is
y

-1 0 X

194
Consider the equation
cos e = b
Let b = t, then
for 0 < e < 2n, e = -j or 2n - -j as cos e is positive in the first and fourth quadrants.
For 2n < 8 < 4n, the solutions are 8 = 2n + -j or 4n - -j,
for 4n < e < 6n, the solutions are e = 4n + -j or 6n - -j,
for 6n < e < 8n, the solutions are e = 6n + ~ or 8n - ~.
for -2n < e < 0, the solutions are e = -2n + -j or 0- -j.
Thus a general solution would be
e = 2nn ± -j where n is any integer.
This could be written as
e = 2nn ± cos- 1 (t)
Thus, the general solution to
cos e = b is
e = 2nn ± cos- 1 b

THE INVERSE TANGENT FUNCTION


Iff is defined as y = tan x then an inverse function exists, given by
f- 1 : x = tan y which, by definition, is equivalent to
y = tan- 1 x (tan to the minus 1, x)
By looking at the graph of y = tan x, it is evident that a restricted domain is needed if the inverse
function is to exist.
The domain off: y = tan x is taken as --I < x < -I.
N .B. The inequality is less than rather than less than or equal to as the curve has vertical asymptotes
at x = -I and x = - -r. The range remains the set of all real y.

Thus, y = tan- 1 x has as domain the set of all real x and the range is --I < y < -r.
In this case, y = --I andy = -I are horizontal asymptotes.

This graph passes through the origin and


when x < 0 it is concave upwards,
when x > 0 it is concave downwards.
195
Thus if,J: y = tan x with D 1 = {x: -~ < x < ~}
R1 = {y: y is any real}
then the inverse function
f- 1 : y = tan- 1 x with Drt = {x: xis any real}
R1 -t = {y: -~ < y < ~}
and its graph is

-,'

To solve the equation


tan e = 1
On the interval
-~ < e < ~. e = -!t
~ < e < 32"' e = n + -![:
32" < e < 52"' e = 2n + -![:
52" < e < 72"' e = 3n + -![:
and so the general solution could be expressed as
e = nn +-![:
or e = nn + tan- 1 (1) where n is any integer, positive, negative or zero.

Thus, the general solution of


tan e = b is
e = nn + tan- 1 b, where n is any integer.

Example:
Find (a) sin-
1
(fi)
(b)
0

Sill -1(-ll)
(c) sin- 1 1 - tan- 1 1.
Solution:
(a) If y = sin-
1
(fi)
smy
0

= J2l
n
y=-
4
(b) Lety = sin- 1 ( -~)
l
siny =
2
y=
n
6
(the result must be on -~ < y < ~)
196
(c) y = sin- 1 (1) - tan- 1 (1)
rc rc
2 4
rc
4

1. Evaluate the following angles as an exact answer in terms of rc.


(a) sin- 1 1 (e) cos- 1 (f) (i) tan-
1
(~)
(b) sin- 1 (f) (f) cos- 1 (1) (j) tan- 1 ( -J'J)
(c) sin- 1 (0) (g) cos- 1 (- ~) (k) sin- 1 (-~)
(d) sin- 1 (-f) (h) tan- 1(0) (1) cos- 1 (-f)
2. Use calculators to evaluate the following, giving answers in degrees correct to three decimal
places.
(a) sin- 1 0·7854 (c) tan- 1 0·8296 (e) cos- 1 ( -0·3286)
1 1
(b) cos- 0·9276 (d) sin- (-0·1714) (f) tan- 1 ( -0·2762)
3. Use calculators to evaluate the following, giving answers in radians correct to four decimal places.
(a) sin- 1 ( -0·2718) (c) tan- 1 50·621 (e) sin- 1 ( -0·2871)
(b) cos- 1 ( -0·9850) (d) tan- 1 ( -10·326) (f) tan- 1 0·005
4. Without calculators or tables evaluate the following:
(a) sin- 1 (sin~) 2
(d) tan- 1 (tan rc)
3
(g) cos [cos-
1
fJ
1
(b) sin- [sin ( -~) J (e) sin [cos-
1
(- ~) J (h) cos [sin- 1 OJ

2
(c) cos- 1 (cos rc)
3
(f) cos (sin -
1
~) (i) sin- 1 1 - sin- 1 ( -1)

5. Use calculators to evaluate the following where possible. Use radians.


(a) sin- 1 (sin 1·50) (c) tan (tan- 1 30) (e) sin (sin- 1 1·50)
1 1
(b) cos- (cos 3·0) (d) cos (cos- 3·0)
Why do error messages result in Parts (d) and (e)?
6. Show that each of the following approximate to the value of;}. Calculators may be used.
(a) tan - 1 (t) + tan - 1 m
(b) 2 tan- 1 Ct) + tan- 1 (~)
(c) 4 tan- 1 (!)- tan- 1 ( 2 i 9 )
7. (a) For each of the following state the domain and range and sketch the graph.
(i) y = sin- 1 (f) (iii) y = 2 sin- 1 x
1
(ii) y = sin- (2x) (iv) y = tan- 1 m
(b) State the domain and range of the following functions:
(i) f: y = tan- 1 ~1 - x 2
(ii) f: y = x sin- 1 (x 2 )
197
(c) Sketch the graphs of the following functions:
(i) y = cos- 1 (cos x)
(ii) y = cos (cos- 1 x)
(iii) y = tan- 1 (tan x)
8. For the following functions:
(i) write down the general solution (in terms of n)
(ii) the set of solutions given by n = -2, - 1, 0, 1, 2.
(a) sin() = ~ (c) tan() = ~ (e) sin () = - J3
2
(b) cos () = ! (d) tan() + 1 = 0.

As with the trigonometrical functions, relationships also exist between their inverse functions.

PROPERTY 1
sin- 1 ( -x) = - sin- 1 x
Proof: Let y = sin 1 ( - x) (o:)
.". -x=siny for-~<y<~
.". x = -siny
=sin ( -y) as sin ( -{}) = -sin()
.". -y = sin- 1 x
y = -sin- 1 x. (/3)
Thus, from (o:) and (/3)
sin- 1 ( -x) = -sin- 1 x.
This result is evident from the graph of y = sin- 1 x.
For example, sin- 1 ( -1) = -~
= - [sin- 1 (1)].
The graph is an odd function and so the general result holds.

PROPERTY 2
cos- 1 ( -x) = n - cos- 1 x
Proof: Let y = cos 1 ( -x) (o:)
.". -x =cosy when 0 < y < n
.". x = -cosy
=cos (n- y) from properties of angles of any magnitude
.". n - y = cos- 1 x
y = n - cos- 1 x. (/3)
Thus, from (o:) and (/3):
cos- 1 ( - x) = n - cos- 1 x.

PROPERTY 3
tan- 1 ( -x) = -tan- 1 (x)
Proof: Let y = tan 1 ( -x) (o:)
.". -x = tany when-~< y < ~
:. x
= -tany
= tan ( -y) as tan (- ()) = -tan()
:. -y = tan- 1 x
y = -tan- 1 x. (/3)
Thus, from (o:) and (/3):
tan- 1 ( -x) = -tan- 1 x.
198
PROPERTY 4
sin- 1 x + cos- 1 x = 1-
Proof: Lety = sin- 1 x (a)
.'. x = sin y when -1- ~ y ~ 1-
= cos (1- - y) as the sine of an angle equals the cosine of its complement .
.'. 1- - y = cos- 1 x ({3)
.'. (a) + ({3) = sin- 1 x + cos- 1 x = 1-

Thus sin- 1 ( -x) = -sin- 1 x


cos- 1 ( -x) = n- cos- 1 x
tan- 1 ( -x) = -tan- 1 x
sin- x + cos- 1 x = 1-
1

Example (i):
Show that tan- 1 (!) + tan- 1 (!) = ;j:.
Solution:
Put x = tan- 1 (!)andy = tan- 1 (!)
.'. tan x = t; tan y = t.
Now tan (x + y) = tan x + tany
1 - tan x tan y
.!_+.!_
- 2 3
-1-txt
5
- 6
-s-
6
= 1
.'. x +y = tan- 1 (1)
=:!:
.'. tan- 1 (t) + tan- 1 (!) = :1:

Example (ii):
For the domain -1 ~ x ~ 1
(a) show that cos (sin- 1 x) = )1 - x 2
(b) show that sin- 1 x = cos- 1 )1 - x 2
Hence evaluate cos (sin- 1 %).
Solution:
(a) Let y = sin- 1 x
.'. sin y = x
Now, the range of this function is --r ~ y ~ -rand so cosy ~ 0.
Now cos 2 y + sin 2 y = 1
.'. cos y = .,/r:-1---st:--.n-;;-2 -Y
= )1 - x 2 (1)
.'. cos (sin- x) = )1 - x .
1 2

(b) From (1):


y = cos- 1 .,/1 - x 2
= sin- 1 x.

Now cos (sin- 1 %)

199
Example (iii):
Without the use of calculators, evaluate the following:
(a) tan - 1 (J2 cos lf)
(b) sin (2 tan - 1 ~)
(c) cos [sin- 1 ( - 153 )]
Solution:
(a) tan - 1 (.j2 cos lf)
= tan-
1
( J2 x ~)
= tan- (1)
1

=!
(b) sin (2 tan- 1 ~) Thus in a right-angled triangle
Let fJ = tan- 1 G)
.'. tan fJ =!
Now sin (2 tan- 1 ~)
= sin 2{3 3
= 2 sin fJ cos fJ
=2·-3_,_5_
J34J34
=g. :. I'= )32 +52
=J34
(c) cos [sin- ( -l3 )]
1

= cos [ -sin- 1 a3)]


Let fJ = sin - 1 153
sin fJ = A
.'.cos [ -sin- 1 {f3 )]
= cos (- {J)
= cos fJ
- 12
- n

Thus x = 12.

1. Show by evaluation that:


(a) sin- 1 G) = cos- 1 (!) = tan- 1 (~:)
(b) sin- 1 (t3) = cos- 1 (g)= tan- 1 U2 )
(c) 2 sin- 1 (~) = sin- 1 (f)
(d) sin-
1
(f)+ cos- 1
(f)=~·
Confirm the results in parts (a) and (b) by drawing a suitable right-angled triangle.
200
2. Without the use of calculators, find an exact value for each of the following.
(a) cos (sin- 1 ~ + cos- 1 ~)
(b) tan [tan - 1 (!) + tan - 1 ( - !) J
(c) cos [cos- 1 (~) + cos- 1 ( -~)]
(d) sin [sin- 1 Ci) + sin- 1 ( -i)J
(e) sin [sin - 1 (~) - cos- 1 Ct)].

3. Evaluate the following as exact answers.


(a) cos [sin- 1 (11 )] (e) cos [tan- 1 Us)]
(b) cos [sin - 1 ( - !~)] (f) sin [2 cos- 1 Ct)J
(c) sin [2 tan- 1 Ci)J (g) tan [2 tan- 1 ( -!)]
(d) sin [cos- 1 (~)] (h) sin [2 cos- 1 {!)].

4. (a) Prove that tan- 1 (i) + tan- 1 (~) = !f.


(b) Prove that sin- 1 (
2
f) = cos- 1 (~). Hence evaluate sin- ~) + sin-
1
(
1
ef).
(c) Provethatsin- 1 (~)+ sin- 1
(v;r) = ~·
5. By setting sin- 1 x = rx, sin- 1 y = f3 and using the expansion
sin (rx + {3) = sin rx cos f3 + cos rx sin /3, prove that
sin- 1 x + sin- 1 y = sin- 1 {x.j1 - y 2 + y.jr:-1---x-.,2 }.
Hence, show that
sin- 1 Ct) + sin- 1 (~) = sin- 1 (1).

6. By using the expansion


t tan rx - tan f3
an (rx - {3) = 1 + tan rx . tan f3
prove that
tan - 1 x - tan - 1 y = tan - 1 ( x - Y ) .
1 + xy

Hence evaluate:
(a) tan- 1 (J3) tan- 1 (fi)
1
(b) tan- (J3)- tan- 1
( -J3).

7. Prove the following results:


(a) cos (sin- 1 x + cos- 1 x) = 0
(b) tan- 1 (x) + tan- 1 (~)=~when x > 0.
Show that this is correct by considering a right-angled triangle with the non-hypotenuse sides
1 unit and x units.
(c) 2 cos- 1 x = cos- 1 (2x 2 - 1)
(d) By setting rx = tan- 1 x; f3 = tan- 1 y
(i) Prove, using the expansion for tan (rx + {3), that tan- 1 x + tan- 1 y = tan- 1 (x + Y)·
1- xy
(ii) Hence write down an expression for 2 tan- 1 x.
(iii) Hence show that 2 tan- 1 (f) + tan- 1 (t) = !f.
(e) Prove that cos- 1 a+ cos- 1 b = cos- 1 [ab- ~. JI=biJ.
(f) (i) Write down an expression for sin 2rx.
(ii) By putting x =sin rx, show that 2 sin- 1 x = sin- 1 2x.j1- x 2 .
201
1. THE DERIVATIVE OF sin- 1 x

y = sin- 1 x
.·. x = siny
Differentiating with respect toy,
dx
dy =cosy
dy 1
But -=-
dx dx
dy
dy 1
· · dx cosy
Now sin 2 y + cos 2 y = 1
.'. cos 2 y = 1 - sin 2 y
. dy 1
= 1- x 2
· · dx J1- x 2
.J
cos y = 1 - x 2
.
1.e. -d (s1n
. -1 x ) 1
= -,==~
dx .)1 - x 2
N.B. (i) The domain of the derived function is -1 < x < 1 and its range is the set of real numbers
greater than or equal to 1 as J 1 - x 2 ~ 1.
(ii} Taking the positive square root ensures that the gradient of the tangent to y = sin - 1 x is
always positive.
(iii) y = sin- 1 xis monotonic, increasing for all x in its domain.

2. THE DERIVATIVE OF cos- 1 x

y = cos- 1 x
,',X= COSy
. dx .
. . dy = -smy
dy 1
dx dx
dy
and sin y = .j1 - cos 2 y
sin y
1 =~

i.e. .!!___ (cos- 1 x) =


dx

(i) Taking the positive square root ensures that the gradient of the tangent toy = cos- 1 xis always
negative.
(ii) Thus y = cos- 1 xis a monotonic decreasing function over its domain -1 ~ x ~ 1, decreasing
from y = n toy = 0.
See Appendix XIV for an alternative derivation.
202
3. THE DERIVATIVE OF tan- 1 x
y = tan- 1 x .
:. x =tan y
dx
-=sec 2 y
dy
dy
· · dx dx
dy
1
- sec 2 y
but sec 2 y = 1+ tan 2 y
1+x
2
=
1
1+
'
t.e. d (t an -1 x ) = - -1- •
-d
x 1 +x2
N.B. (i) For y = tan- 1 x, the value oft is always positive.
1
(ii) Thus y = tan- xis a monotonic increasing function for all real x.

These basic derivatives may be extended to a more general form.

Let y = sin- 1 ~
a
. X .
, , - = Slll y
a
x =a siny
. dx
.. dy =a cosy
' dy 1
· · dx =a cosy and cos 2 y = 1 - sin 2 y
1 x2
= 1-2
a
a2- x2
a2
Ja2- x2
:. cosy=
.
t.e. d (. -1
-d sm - X) = Ja2- x2 .
1 a
x a

5 -d ( cos
· dx
-1 (X))
-
a
-
-
- -;======='
J a 2 1- x 2
y = cos- 1 (~)
X
-=cosy
a
x =a cosy
dx
-a sin y
dy
203
. dy )a2 - x2
· · dx a sin y and siny = ---'-----
a
1

d ( tan _ 1 - = 2
6 .-d
x a a
x) a
+x
2

y = t an -1 - (X)
a
.'. x =a tany
dx 2
dy =a sec y
. dy- ------;;--
. · dx - a sec 2 y and sec 2 y = 1 + tan 2 y
1 x2
= 1 +2
a
a2 + x2
a a2
a2 + x2
. -d
t.e. d ( tan - 1 - =
x a a2
x) a
+ x2

H ence, d ( . -1 )
-d sm x = 1
x 1
y1- x2
-d ( Sill
. -1 X) = _1===:;;:
- ,----;c-
dx a )a2. - x2

dx (cos- 1 x) = 1
= -~(sin- 1 x)
d )1 - x2 dx
d ( -1
dx cos ~
X) = - . ) a2 -
1 d ( . -1
x2 = - dx sm ~
X)
~(tan - 1 x) = -
1-
dx 1 + x2

~ (tan - ~) = 1
a2 : x2 ·

Example:
Differentiate the following:
(a) y = sin- 1 4x
(b) y = x 2 • cos- 1 (1)
(c) y = tan- 1
(
-X --
3
2)
(d) y = esin-1(2x).

204
Solution:
(a) y = sin- 1 4x
. h . f . dy dy du
U smg t e composite unctton ru1e dx = du · dx

dy- 1 . ~(4x)
dx - )1 - (4x) 2 dx
4

(b) y = x 2 • cos - 1 ( ~)
using the product rule,
dy
dx
= x2 • ~
dx
(cos- (~)) + cos- (~) d (x
3
1
3dx
1

2
)

= x2 • -
1
)32 _ x2
+ cos- 1 (~)
3
· 2x

2x cos - 1 (~)
2
= - --;::c=x====::c2
3 )9- x
2
(c) y = tan- 1 (-x-;-)
dy 3
dx 3 2
+ (x - 2) 2
3
9 + x2 - 4x +4
3 + x2 - 4x + 4 ·3
1
x 2
- 4x + 13 9

9 + x2 - 4x +4 3
9
3
x2 - 4x + 13
(d) y = esin- 1(2x)
1
dy = esin-1(2x). d[sin- (2x)]
dx dx
= 1
esin- (2x) . 1 .2
)1 - (2x) 2
2esin-1(2x)
)1-4x 2 •

1. Differentiate the following:


3
(a) sin- 1 x (f) cos- 1 Sx (k) tan- 1 (5x)
(b) sin- 1 3x (g) cos-
1
( ~) (1) tan- 1 (2x; 1)
(c) sin- 1 (x 2) (h) cos- 1 (:X 2) (m) sin - 1 (1 - 2x)

(d) sin-
1
(~) (i) cos- 1 (.jX) (n) tan- 1 ( ~)
( ). _ 1 ( 2x
e sm
3
'7 1) (j) tan- 1 (~) (o) .
Slll
-1 e xS- 3) .
205
2. Find the derivatives of each of the following:
(a) y = sin- 1 (ax+ b) (e) y = cos- 1 (sin x) (i) y = sin- 1 (ex)
(b) y = x sin- x1
(f) y = sin- 1 (sin x) (j) y = tan- 1 (loge x)
1
(c) y = (sin- x) 2
(g) y = cos- 1 .j1=-x (k) y = ecos-1(2x-1)
(d) y = (9 + x ) . tan- (j)
2 1
(h) y = )tan- 1 x (1) y = x sin - 1 x +)1- x 2.

3. Find dy for each of the following:


dx
(a) y = sin- 1 (3x)- cos- 1 (3x) (f) y = {1 + cos- 1 (3x)P
(b) y = {tan- p 1
m (g) y = x(sin- 1 x) 2 - 2x + 2)1 - x 2 sin- 1 x
(c) y =loge {tan- 1 (f)} (h) y = x cos- 1 (2x) - !)1 - 4x 2
(d) y = (x 2 - 2) sin- 1 (f) + 1)4- x 2 (i) y = )a 2 - x 2 +a sin- 1 G), a > 0
1
(e) y = -.-_-1- ( J') y = t an _1 ( 1 2 tan x2 ) .
Sill X -tan x
4. (a) Calculate the gr~dient of the tangent at the origin for:
(i) y = sin- 1 x
(ii) y = tan- 1 x.
(b) Evaluate the gradient of the tangent to
y = sin- 1 (tan x) at x = 0.
(c) Find the first and second derivatives of
f(x) = tan- 1 -f.
(d) Prove that the derivatives of tan - 1 (tan -f) and tan- 1 (sec x + tan x) both equal!.
(e) Find the derivative ofj(x) = (sin- 1 -f) 2 + (tan- 1 x) 2 at x = 1.
5. (a) Find the equations of the tangent and normal toy = sin- 1 (x - 1) at the point(!,~).
(b) Find the points on the graph of y = tan- 1 2x where the tangent line is parallel to the line
13y = 2x- 5.
(c) Show that ifj(x) = sin- 1 (cos x) and g(x) = cos- 1 (sin x), thenf'(x) = g'(x).
(d) Find the equation of the tangent to the curve f(x) = etan-tx at the point where it cuts they .
axis.
6. (a) Over what interval is the graph of y = tan- 1 x
(i) concave upwards?
(ii) concave downwards?
(b) Find the equations of
(i) the tangent
(ii) the normal at the point x = -~on y = (sin- 1 x) 2 .
(c) f(x) = x sin- 1 (x 2)
(i) State the domain and range ofj(x).
(ii) Findj'(x).
(iii) Show that there is a horizontal inflexion at the origin.
(iv) What is the slope of the tangent at x = 1?
(v) Sketch the curve.

As ddX (Sill
. -1 )
X =
1
then J)1-
dx
x2
= sin- 1 x + c
Jl--=-?
d ~. -1 X) 1 then J)a2-
dx =
.
Sill
-1 X+
- C.
dx Sill -; = )a2 - x2 x2 a

206
-d (t an - 1 x ) = -
dx
I -
I + x2
then J+
I
dx 2
x
= tan- 1 x +c

d~ (tan- ~) =
1
a2 : x2 then I a2
dx
+ x2 =-I t an -1 -+c.
a
X
a
of less importance are

d (COS -1 dx
-d X
)
= then = cos- 1 x + c
X
f JI- x 2

X)
-d ( cos -1 - =
dx _ x
=cos 1 - +c
dx a J -
Ja2- x2 a
. -1 X
= -sm -+c.
a

dx . -1 x
+C
I-;=';:;====::::;: =
Ja2 - x2
Sill -
a

f----
a2
dx
+ x2
It
= - an
a
-----------------------"
-1 x
-+c.
a

Example (i):
Find the primitive functions for the following:
I
(a) J4- x2
I
(b) x2 + 4
(c) (3 - x 2 )-t
I
(d) 9 + 2x 2 '

Solution:

(a) IJ4 I
d~ x2 - J22d~ x2

207
(d) I :~xz
9
To bring into the correct form, make the coefficient of x 2 unity

= 2
1 J+
2_
dx
xz
2

=
I2 CfiJ
1 dx
+ x2

= 21 ·~tan
1 -1
f +c
(X)
= ~ tan- (J;) +c.
1

Example (ii):
Evaluate the following definite integrals.
r3 dx
(a) Jo )9 - X2

(b) rvrz 2 dx
Lvrzx + 2

Solution:

(a) f )9 d~ x2 = r)32d~x]
. _1
3
x2

= [
sm -
3 0
= sin- 1 (1) - sin- 1 (0)
=~-0
2
n
2
JZ dx IJZ dx
(b) I-vrzx2 + 2 = -vrz(J'i)z + xz

=
[1J2 t
an
-1 X
J2J-JZ
v'Z

= ~ {tan- 1 (1) - tan- 1 ( -1)}

= ~{~- (-~)}

208
1. Find primitive functions for the following:

~
1 1
(d) x 2 25 (g)
(a) )9- x2 1 + (x + 3) 2
1 1
(b) )25 1_ x2 (e) 3 + x2 (h)
.)36- x 2
1 1 1
(f) (i)
(c) )5 - x2 +5

2. Find the following indefinite integrals:

(a) J x2 dx
+ 9 (e)
f dt
)1 - 9t2
(i) J dx
.)4- (x- 2) 2

(b) Jt_____!!!_
+ X
2 (f) J dt
)1 - 2t 2
(j) J dx
(x- 2) 2 + 16

(c) J 1 +dt7t 2 (g) f a2 +dxb2x2 (k) f )a2 -~:- b)2


(d) f 25 : 4t 2 (h) f .Ja2 ~ b2x2 (1) f dx
(x- b) 2 + a 2

3. Perform the following integrations:

(a)
f dx
)9- 4x2 (c)
f dx
)9 - 4(x - 3) 2 (e)
f dx
25 + 9x 2

(b) f )16 d~ 9x2 (d) f 9 +dx4x 2 (f)


f dx
4(x - 3) 2 + 9

(a) r d~
4. Evaluate the following definite integrals. Leave answers in terms of n.

4 (e) [1 )2 d~ x 2 (i) f~!)1 ~ 4x 2


(b) f )4d~ x2 (f) f 6~ 1 (j) t2 1 + (~x- 1)2
(c) f x2)-~dx
(9 - (g) J: )a2d~ x2 (k) tt )1 ~x 2x2
(d) I! dt (h) rb dx (1) ri dx
Jo ~ Jo b2 + x2 Jo25 + 9x2.
1
5. (a) (i) Find the maximum value of y = - •
1 +x 2
(ii) Sketch the curve.
(iii) Find the area under the curve bounded by the ordinates x = 0 and x = 1.
= ~ between the lines x = - 2 and x = 2.
+4
(b) Find the area under the curve y
X
(c) Find the area bounded by the curve y = (16 - x 2 )-~ enclosed above the x axis by the ordinates
x = 0 and x = 2.

6. (a) The area bounded by the curve y = 1 , the x axis and the lines x = -1 and x = 1,
)1 + x 2
is rotated about the x axis. Find the volume of the solid generated.
(b) The area bounded by the curve y = (1 + 4x 2 )-~, the x axis and the ordinates x = 0 and
x = f, is rotated about the x axis. Find the volume generated.

209
(c) Find the volume of the solid generated when the area under the curve y = (9 - x 2 ) -!,above
the x axis and between x = f
3
and x = 3, is rotated about the x axis.

1
7. Ifj(x) = -r===::o2
J9- x
(a) write down the domain and range ofj(x)
(b) find dy and show that there is a minimum value at (0, t)
dx
(c) sketch the curve
(d) use a calculator to find the area bounded by f(x), the x axis and the lines x = -2 and x = 2.
8. For the functionf(x) = sin- 1 x,
(a) Use the trapezoidal rule with ten sub-intervals to find an approximation to the area under
the curve and between x = 0 and x = 1.
(b) Repeat the process using Simpson's Rule with ten sub-intervals.
Give answers correct to four decimal places.

210
PAPER 57
1. (a) Solve the equation 3 tan 2 8 = 2 sin 8, giving all solutions in the domain oo :::; 8 :::; 360°.
sin 48 - cos 48 + 1
(b) Prove that tan 28 = . .
Sill 48 +COS 48 + 1

2. A spherical balloon is inflated with gas at the rate of 10 m 3 /min. How fast is the radius of the
balloon increasing at the instant when the radius is 2 m?
3. Write down the primitives of:
(a) 2 + !x - x 2 (b) 2[! (c) cos 2x.
4. A particle moves on a line so that its distance s from the origin at time t is given by
s = 2t 3 - 15t 2 + 24t - 5
Find when and where the particle first comes to rest.
5. The three sides of a triangle are 3 em, 5 em and 7 em. Calculate the largest angle of the triangle
and also its area.
6. A is the centre of a given fixed circle. Band Care the centres of any two circles which touch each
other externally and touch the fixed circle internally. Prove that the perimeter of the triangle ABC
is constant.

PAPER 58
1. Solve the equations, for 0 :::; 8 :::; 2n.
(a) sin 28 = cos 28
(b) cos 28 + sin 8 - 1 = 0.
2. Evaluate
(a) L (1 - 2x) 5 dx

(b) reX + 1dx.


Jl X
3. Two circles intersect at A and B. AP and AQ are diameters of the respective circles. Prove that
the points P, B and Q are collinear.
4. A variable line with gradient m passes through the point (1, 2) and intersects the x axis at A (a, o)
and they axis at B (o, b). Find the area of the triangle AOB of least area if both a and bare positive.
5. A particle is moving back and forth along a straight line in simple harmonic motion. When the
particle is 3 em from 0, the centre of oscillation, its velocity is 16 cmfs. The period of the motion
is 1- seconds.
Find the maximum displacement of the particle from 0 and find the velocity of the particle as
it passes through 0.
6. Iftan 8 = t and tan </J = t, show that cos 28 = sin 4</J.

PAPER 59

1. (a) Show that ~ + cos 2~


= cot2 A.
-cos 2
(b) Prove that sin (A+ B). sin (A- B)= sin 2 A- sin 2 B.
211
2. Differentiate:
I
(a) ex sin X (b) (1 + 2x) 3 (c) loge cos x.

3. (a) Sketch the curve y = tex for -1 : : ; ; x : : ; ; 3.


(b) Find the area bounded by the curve y = tex, the x axis and the ordinates at x = 1 and x = 2.
(c) If the area defined in Part (b) is revolved about the x axis, find the volume of revolution
generated.
4. Find all the angles between oo and 360° satisfying the equation
cos e+ fl sin e= I.
What is the general solution?
5. Prove that the exterior angle of a cyclic quadrilateral is equal to the interior opposite angle.
X and Yare two points in the sides AB, AC of a triangle such that XBCY is cyclic. Prove that
the tangent at A to the circumcircle of ABC must be parallel to XY.
6. (a) A particle P 1 is projected vertically with velocity V. Show that the greatest height attained is
2
2
V and the time taken to return to the horizontal plane is V.
2g g
(b) A particle P2 is projected with velocity V at an angle of rx to the horizontal plane. Show that
v2 . 2
the greatest height attained is ~~n rx and the time taken to return to the horizontal plane
. 2V sin rx
IS---
g

PAPER 60
1. Find all the angles 0 : : ; ; 8 : : ; ; 2n satisfying the equation
sin e + 2 cos e = 1.
2. Prove that the angles in the same segment of a circle are equal. Show how to construct a triangle
ABC, given angle A and the length of side a together with the length of the altitude from A. Give
reasons for your construction method.
3. Express tan 28 in terms of tan 8.
A person observes the angles subtended by a tower and its spire to be equal. He knows the
height h of the tower and his distance x from its foot. Prove that the height of the spire is:

( Xx2 +- h2h2) h.
2

4. (a) Write the formula for cos (x + y) and use it to show that cos 2x = 2 cos 2 x - I.
Hence evaluate J: 2
cos xdx.

x +6
(b) Show that ~
2

X+ 4 = 1 2
+~
4
and hence evaluate
X+
J 1
x
~
2
+
6 dx.
+4
0 X
5. The chord PQ of the parabola x 2 = 4ay passes through the focus S. The tangents at P and Q
meet at the point T. Prove that TS is perpendicular to the chord PQ.

t
6. Use Simpson's Rule to find the approximate value of
4
loge xdx.

212
PAPER 61
1. A particle moves in simple harmonic motion. Find the period of the motion if the acceleration
towards the centre of motion is 6 mjs 2 when the particle is 3m from the centre.
2. Find two sets of values for a, b and k such that x 2 - 4x - 8 + k(x 2 - 2x - 5) = a(x - b) 2
for all values of x.
3. (a) Find the equation of the tangent to the curve y = ~ 1 - x at the point where x = 0.
Jr JX
2x + 1
4
(b) Evaluate dx.
1

4. ABCD is a sheet of cardboard 24 em square. Creases PQ and RS are made parallel to AD and
equidistant from the centre of the square and the flaps are folded up to form a triangular prism
with open ends.
(a) If the distance between the creases is 2x em, find an expression for the volume of the prism
in terms of x.
(b) Find the value of x for which the volume of the prism is a maximum.
5. A man borrows $4000·00 at 1·5% per month interest, where the interest is compounded monthly
on the balance owing. If he pays off the loan in equal monthly instalments over three years, what
is the amount of each monthly instalment?
6. (a) Find the angle of intersection of the lines 2x + y = 4 and A
3x = 2y + 1.
(b) In the figure BC = 5 em, LABC = 45°, LACD = 60°,
LADC = 90°. Find the length AD.

PAPER 62

1. Differentiate loge (loge x) and hence evaluate Je' dx .


e X 1oge X
2. (a) ThepointP(a,b)liesontheline6x- y = 1andthepointQ(b,a)liesontheline2x- 5y = 5.
Find the equation of PQ.
(b) A and Pare the points (- 2, 0) and (5, 0). If P divides AB in the ratio 7: 3, find the abscissa
of B.
3. Using the data of the figure and given A
¢ = 28, find the ratio BC: CD.

4. (a) If tan 8 = a, prove that cos 28 + a sin 28 = 1.


(b) Find an acute angle 8 such that 3 sin 8 + 2 cos 8 = 2·5.
5. (a) Write down a primitive function of -sin rex.
(b) Find J
2xJ1 + x 2 dx by using the substitution u = 1 + x2•

213
6. A particle is projected at an angle 8 to the horizontal with a velocity of 40 mjs. If the particle
reaches its maximum height after 3 seconds, find e.

PAPER 63
1. Differentiate (a) sin- 1 5x

2. Using the substitution x + 1 = u, find f


x
2 x +
+ 2x + 2
1
dx.

3. If the velocity of a particle in terms of its displacement is given in metres per second by
v = J3x + 1, where xis the displacement in metres, find
(a) the acceleration of the particle.
(b) the velocity when the displacement is 5.
4. Water is poured into a conical vessel at a ~niform rate of 27 cm 3 js. If the vertical angle of the
cone is 60°, find the rate at which the water is rising when the depth is 9 em.
5. State the largest possible domain of the function
f(x) = JT+X- ~·
What are its greatest and least values?
6. The centres X and Y of two circles of radii 3 em and 4 em are 5 em apart. If the circles intersect
at P and Q, find:
(a) the area of the triangle PXY
(b) the areas of the sectors XPQ and YPQ
(c) the area common to the two circles.

PAPER 64

1 (a) Evaluate JJ o
2
1
4- x2
dx.

(b) Find f sin 2


x cos xdx by using the substitution sin x = u.
2. Find the smallest integer n for which 2n exceeds 1 000 000. Hence find the sum of all integral
powers of2 which lie between 10 and 1000000.
4 1
3. (a) Show that J5
2 + 5 9- 4y-'
~
. a ratrona
rs . 1 num b er.

3 1
(b) Solvej x; 1 < 5.

4. The period of a particle moving with simple harmonic motion is 4 seconds. The particle starts
from rest at a distance of 8 em from the centre of its path. Find the time taken to move 2 em
and the velocity and acceleration at that instant.
5. (a) Find the exact value, as a rational number, of

A~~ ,
4
5

where
2
A=(~y, B=(~t C=(~J.
(b) Solve x - 4x - 45 = 0.
6. (a) For what values of the constant Bare the graphs of the equations
X - 3y = -2
2x +By=
(i) parallel?
3

(ii) perpendicular?
(b) Show that the line 3x + 4y + 10 = 0 is tangent to the circle with centre at the origin and
radius 2 units.
214
Graph of the function y = x4 - 5x 2 + 4 given on a
computer screen.


I
CHAPTER 28
I
The word polynomial is the general word to describe an algebraic expression with many terms.
The algebraic expression 5x4 + 6x 3 + 7x + 8 is a polynomial in x and is the sum of four terms.
The number 5 in 5x4 is the coefficient of x 4 . The coefficient of x 3 is 6 and the coefficient of xis 7.
The term 8, which is independent of x, is called the constant term. The highest power of x occurring
in this polynomial is 4. We say the polynomial is of degree 4.
More formally:

A real polynomial P(x) of degree n is an expression of the form


2 3 1
P(x) =Po + PtX + P2X + P3X + · · · + P,-tx"- + p,x" (p,. =I 0)
where the real numbers p 0 , p 1 , p 2, ... , p, are called the coefficients of P(x).
The degree of P(x) is that of the highest power of x occurring.

(i) P(x) is defined for all real x and is a continuous and differentiable function of x.
(ii) The equation P(x) = 0 is called a polynomial equation of degree n, and those real numbers x
which satisfy the equation are called the real roots of the equation or the real zeros of the corre-
sponding polynomial.
(iii) In the general polynomial p 0 is called the constant term. A polynomial consisting only of a
constant term has degree zero, since Po = p 0 x 0 •
(iv) The polynomial in whichp 0 = p 1 = P2 = · · · = p, = 0 is called the zero polynomial.

1. For the following polynomials give the degree, the coefficient of the highest power of x and the
constant term.
(a) 9 + 6x - 3x 2 + 4x 3 (d) 3 + x - x 2 + 2x 3 - 6x 4
(b) 4x - 2x + 3x - 1
4 2
(e) 2x 5 - 3x 2 + 7x + 1
3
(c) x - x + 2x + 8
5 2 (f) 3x 6 - 2x4 + 7x 2 - 8x.
2. If a polynomial function is defined by P(x) = 3x4 + 2x 3 - x 2 + 4x - 5, find:
(a) the degree of P(x) (c) the constant term
(b) the coefficient of x 2 (d) the value of P(1), P(O), P( -1).
3. Add these two polynomials:
4x 3 + 2x 2 - 7 and 5x4 - 3x 3 - x + 2.
What is the degree of the sum?
4. What is the degree of the product of the two polynomials:
2x 2 + 3x and 4x 3 - 2x 2 + 1?
5. What is the degree of each polynomial answer?
(a) (2x 3 + x 2 - x) + (x 2 - 4x - 6) (c) (3x 3 + 2x- 4)(x 2 - 5x)
(b) (3x 4 - 2x 3 + x - 4) - (3x 4 + 3x 3 - x 2 - 8) (d) (x 2 + x- 2) 2 .
6. For the following polynomials find the values indicated.
(a) P(x) = x 4 - 2x, find P(1), P( -2), P(O)
(b) f(x) = 2x 3 + x 2 - 5x + 4, findf(l),f(0),/(2)
(c) g(x) = 2x4 - 3x 2 + 6x - 8, findf(O),J( -2),/(2).
216
7. Which of the following algebraic expressions are not polynomials?
(a) 6 - 3x 2 + x 3 (c) 2x - x + 1 (e) 4x - 2
(b) 4 + 7x - ~2 (d) 3x4 - 7x - 2 (f) 8x - log x

8. Write down the derivatives of each of the polynomials:


(a) 2 + 6x + 3x 2 (c) 7x - 2 (e) (x 2 + x)(5x - 3)
(b) x 3 - 4x + 3 (d) x 5 - 4x 2 + 3x - 1 (f) (3x 2 + 2) 3 .
9. If P(x) is a polynomial of degree n > 0, show that P'(x) is a polynomial and state its degree.
10. (a) Solve the polynomial equation x 2 - x - 6 = 0.
(b) What are the zeros of P(x) = x 2 - x - 6?

Certain polynomials have graphs which are quite simple to draw. The graphs ofj(x) = 1,/(x) = x,
f(x) = x 2 ,f(x) = x 3 andf(x) = x 4 are shown below.

f(x) f(x)
f(x) c~ X

X X X

The zeros of a polynomial are those values of x for whichf(x) = 0. In terms of the graph of the
polynomial, the zeros are the points where the graph cuts the x axis. If a polynomial is completely
factored into linear factors it is easy to find the zeros which can then be used to sketch the graph
of the polynomial.
Example:
Sketch the graph ofj(x) = x(x- 3)(x + 2).
Solution:
The graph is a positive cubic. f(x)
The zeros are -2, 0 and 3.
Hence the graph cuts the x axis at x = -2, x = 0 and x = 3.
Plotting a few other significant points a sketch graph can be
drawn.

1. What are the zeros of the polynomial


f(x) = (x- 1)(x + 1)(x - 2)?
Using these values and other suitable points sketch the graph of the function.
217
2. Considery = -x 3 - x 2 + 2xwhichcanbewritteninfactoredformasy = -x(x- 1)(x + 2).
(a) What are the zeros of this polynomial function?
(b) By using the zeros of the function and other suitable values of x graph the function.
(c) The coefficient of x 3 in this function is negative. What effect does this have on the shape
of the graph?
3. For f(x) = (x- 1)(x + 2) 2 there is a zero at x = 1 f(x)
and two zeros have come together at x = -2.
Using these zeros and the fact that the coefficient of
x 3 is positive we can draw a rough sketch graph of the
function as shown. Note that where the two zeros occur X
together at x = -2 the graph touches the x axis.
Obtain sufficient additional points and draw the graph
of this function in more detail.

4. Find the zeros of each of the polynomial functions and use them to draw a rough sketch graph
of the function.
(a) f(x) = x(x + 1)(x- 2) (c) f(x) = (x + l)(x- 2) 2
(b) f(x) = (x- 4)(x- 1)(3x + 2) (d) f(x) = (x + 3) 2 (x - 1)
5. Factor these quadratic polynomials, find the zeros and sketch their graphs.
(a) f(x) = x 2 x - 2 (b) f(x) = 2x 2 - x - 15 (c) f(x) = 15 + 2x - x2
6. The quartic polynomial function f(x) = x(x + l)(x - 1)(x - 3) has zeros -1, 0, 1 and 3.
A sketch graph of this function is given which shows these zeros as the x coordinates of the points
of intersection with the x axis.
f(x)

-2 X

Find the zeros of each of the polynomial functions and use them to draw a rough sketch graph of
the function.
(a) f(x) = (x + 2)(x + 1)(x- l)(x - 3) (c) f(x) = x 2 (x- 2) 2
(b) f(x) = x(x + 2)(x - 1) 2
(d) f(x) = -x(x - 5)(x - 2)(x + 3)
7. Consider the polynomial f(x)
f(x) = (x + 2)(x 2 - 3x + 5).
There is one zero off(x) at x = -2 but since x 2 - 3x + 5 has
no zeros, the graph off(x) cuts the x axis only once as shown.
Use calculus to find the turning points and the point of
inflexion and draw a more detailed graph.

218
8. By finding maxima, minima, points of inflexion, etc., sketch the graphs of the following poly-
nomials.
(a) f(x) = 3x - x 3 (d) f(x) = 2x 3 - 3x 2 - 12x + 2
(b) f(x) = x + 6x + 9x + l
3 2
(e) f(x) = x 3 + 3x 2 - 9x
4
(c) f(x) = x - 2x 2
(f) f(x) = x 4 - 8x 2 + 16
When graphing polynomials use any techniques at your disposal. The following facts may also
prove useful.
- ----------- - - -- ---- -------------~---------------------

(i) For very large lxl,


P(x) ~ p,x".
That is, for very large lxl
the graph of the polynomial P(x) approaches the graph of the
term of highest degree p,x".
(ii) A polynomial of odd degree always has at least one real zero. That is, its graph cuts the x
axis at least once.
(iii) At least one maximum or minimum value of a polynomial occurs between any two distinct
real zeros.

9. Sketch the following polynomials:


(a) P(x) = (x + l)x(x - 1) (d) f(x) = 2x 3 - 9x 2 + 27
(b) f(x) = x 2 (x - l)(x - 2) (e) P(x) = x 2 - 6x + 15
(c) g(x) = i(x 2 + x + 1)(x + 1) (f) h(x) = x 2 - x 4 .

DIVISION OF POLYNOMIALS
The process of division for polynomials is similar to long division for integers.
28 For example 483 divided by 17,
17)483 gives a quotient of 28 and a remainder of 7,
34 which means that:
143 483 = 17 X 28 + 7
136
7
In a similar way it is possible to divide one polynomial by another, obtaining a quotient and a re-
mainder, each of which is a polynomial.
For example we could divide 3x 3 - x 2 + 3x + 4 by x 2 + 2x
3x - 7 Each term in the quotient is obtained at each stage by dividing
x2 + 2x)3x 3 - x2 + 3x +4 leading term into leading term.
3x + 6x
3 2
That is 3x 3 + x 2 = 3x
- 7x 2
+
3x + 4 and -7x 2 + x 2 = -7
- 7x 2 14x
-

17x + 4
Thus 3x 3 - x 2 + 3x + 4 divided by x 2 + 2x gives a quotient of 3x - 7 and a remainder of 17x + 14,
which means that:
3x 3 - x 2 + 3x +4 = (x
2
+ 2x)(3x - 7) + (17x + 14)

The division process can be expressed as an identity


P(x) = A(x)Q(x) + R(x)
where A(x) is the divisor, Q(x) the quotient and R(x) the remainder.
N.B. The degree of the remainder must always be less than the degree of the divisor.

219
Example (i):
Divide x 3 - llx + 2 by x- 3 and express the result in the form:
dividend = divisor x quotient + remainder
Solution:
x2 + 3x - 2
x - 3h 3
- 11x +2
x3 - 3x 2
3x 2 - llx +2
3x 2 - 9x
2x +2
2x +6
-4
The quotient is x 2 + 3x - 2 and the remainder is -4, which means that:
x 3 - llx + 2 = (x - 3)(x 2 + 3x - 2) - 4
Example (ii):
Show that f(x) = x 3 + 8x 2 + 8x - 21 is exactly divisible by x + 3 and express f(x) in factored
form.
Solution:
x2 + 5x 7
x + 3h3 + 8x 2 + 8x - 21
x3 + 3x
2

+ 8x- 21
5x 2 + 15x
- 7x - 21
- 7x - 21
Since there is no remainder f(x) is exactly divisible by x + 3.
Thus f(x) = (x + 3) (x 2 + 5x - 7).

1. Carry out the following divisions and express each result in the form P(x) = A(x)Q(x) + R(x).
(a) (5x 2 + 15x + 8) -;-. (x - 2) (f) (x4 - 2x 3 + 2x) -;-. (x - 3)
(b) (x + 7x - 2x + 8) -;-. (x + 3)
3 2
(g) (2x 4 - x 2 + 3x - 5) -;-. (x + 3)
(c) (3x + 9x + 5x) -;-. (x + 2)
3 2
(h) (5x 4 + 3x 3 - 38x 2 + 6x) -;-. (x 2 + 3x)
2
(d) (8x + 4x + 12x - 5) -;-. (2x + 1)
3
(i) (x 4 + x 2 + 1) -;-. (x 2 - x + 1)
(e) (x + 6x + 6x + 2) -;-. (x + 2)
3 2
(j) (2x 4 + 3x 3 + x + 1) -;-. (x 2 + x + 1).
2. Using the division process show that the first polynomial in each example is exactly divisible
by the second polynomial and express the first polynomial as the product of two factors.
(a) (x 3 + 6x 2 + 4x - 16), (x + 4) (f) (x 4 - 4x 3 + x 2 - 6x + 8), (x - 4)
2
(b) (2x + x + 1), (x + 1)
3
(g) (6x 4 - 5x 3 + x 2 + 9x- 3), (3x- 1)
2
(c) (x - 16x + 19x + 18), (x - 2)
3
(h) (3x 4 + 3x 3 - 4x 2 + 6x - 20), (x 2 + 2)
2
(d) (2x - 7x + 9x - 3), (2x - 1)
3
(i) (x4 + 2x 3 + 2x 2 + 4x), (x + 2)
(e) (6x + 11x - 3x - 9), (2x + 3)
3 2
(j) (x 5 + x 3 - 2x), (x - 1).

THE REMAINDER THEOREM


If the polynomial function P(x) = x 3 + x 2 - 8x + 11 is divided by x - 2 we obtain a quotient of
x 2 + 3x - 2 and a remainder of 7.
The working can be set out as follows:
220
x2 + 3x - 2
x - 2h 3 + x 2 - 8x + 11
x3 - 2x 2
3x 2 - 8x + 11
3x 2 - 6x
- 2x + 11
- 2x + 4
7
Thus P(x) = (x - 2)(x 2 + 3x - 2) + 7.
If we give x the value 2, making (x - 2) equal to zero, we obtain
P(2) = (2 - 2) (2 2 + 3 X 2 - 2) + 7
=0x8+7
= 7.
In other words P(2) gives the remainder when P(x) is divided by x - 2.
This result illustrates the following important theorem known as the Remainder Theorem.

If a given polynomial P(x) is divided by (x - a) the remainder is P(a).

The proof of this theorem is indicated in the above example. We now proceed to the general proof.
If a polynomial P(x) is divided by (x - a) we obtain a polynomial quotient and remainder. Since
the remainder must be of lesser degree than the divisor it must be a constant.
That is P(x) = (x - a)(quotient) + remainder.
Let Q(x) be the quotient polynomial and R be the remainder.
Then P(x) = (x - a)Q(x) + R
Putting. x = a we obtain
P(a) = (a - a)Q(a) + R
= 0 X Q(a) + R
=R
Thus P(a) = R which proves the theorem.
Example:
Find the remainder when 2x 3 + 4x 2 + x - 6 is divided by (x + 3).
Solution:
Let P(x) = 2x 3 + 4x 2 + x - 6
The remainder on dividing P(x) by (x + 3) i.e. (x - -3) is given by P(- 3).
P(- 3) = 2 X ( - 3) 3 + 4(- 3) 2 + (- 3) - 6
= -54 + 36 - 3 - 6
= -27.

1. Use the remainder theorem to find the remainder when


x4 - 3x 3 + 2x - 7x + 1
is divided by x - 1. Check your answer by long division.
2. Use the remainder theorem to find the remainder when the first polynomial is divided by the
second.
(a) (x 3 - 3x 2 + 2x - 1), (x - 1) (f) (3x 2 + 7x - 4), (x + 2)
(b) (x - 2x + 5x + 2), (x - 2)
4 3
(g) (2x 3 - 4x 2 + 6x + 10), (x + 1)
3 2
(c) (x + 2x - 3x + 2), (x + 1) (h) (x 3 + 5x 2 - lOx - 4), (x - 2)
(d) (x - 4x + 2x), (x- 1)
7 5
(i) (x4 - 5x 3 + 2x), (x - 1)
(e) (x - 2x - x + 2), (x - 3)
3 2
(j) (x 3 - 3x 2 - 5x + 5), (x - 4).
221
3. Find the remainder whenf(x) = 4x 3 - 3x 2 - 2x - 8 is divided by:
(a) X - 1 (b) X + 1 (c) X - 2 (d) X + 2
4. (a) What is the remainder whenf(x) = x 3 - 2x 2 - x + 2 is divided by (x =-- 2)?
(b) Is (x- 2) a factor ofj(x)?
(c) Dividef(x) by (x- 2).
(d) Writef(x) as the product of three factors.
5. Find k so that when 2x 3 - x2 - x + k is divided by x - 2, the remainder is zero.
6. Find the coefficient m if 3x 2 - mx + 4 is exactly divisible by x + 1.
THE FACTOR THEOREM
The factor theorem follows directly from the remainder theorem. If the remainder is found to be zero
when a given polynomial P(x) is divided by (x - a), then clearly (x - a) is a factor of P(x).
-------------- - ---------------------~

~J_'f<x_:: a)i,af~~)_:_
For any polynomial P(x):
(i) if P(a) = 0 then (x - a) is a factor of P(x)
0 ...

Example (i) :
Show that (x + 2) is a factor of P(x) = x3 - 7x - 6 and hence express P(x) in factored form.
Solution:
P(x) = x 3 - 7x - 6
The remainder on dividing by (x + 2) is P(- 2).
P(- 2) = - 8 + 14 - 6
=0
.". (x + 2) is a factor of P(x).
By division, we find Dividing P(x) by (x + 2)
P(x) = (x + 2)(x 2 - 2x - 3) x 2 - 2x - 3
= (x + 2)(x + 1)(x - 3) x + 2h 3 - 7x- 6
x 3 + 2x 2
- 2x 2 - 7x- 6
- 2x 2 - 4x
- 3x- 6
- 3x- 6
Example (ii):
Find the factors of the polynomialf(x) = x3 + 2x 2 - 5x - 6.
Solution:
Since the set of factors of the constant term 6 is {1, 2, 3, 6}, try f(l),f( -1),/(2),/(- 2),/(3),/(- 3),
f(6),f(- 6) to see if the value is zero in any case.
f(x) = x 3 + 2x 2 - Sx - 6
/(1) = 1 + 2 - 5 - 6 -1= 0
f( -1) = -1 + 2 + 5 - 6 = 0
.". (x + 1) is a factor ofj(x).
The other factor can now be found by division.
Thus f(x) = (x + 1)(x 2 + x- 6) x + 1h 3 + 2x 2 - 5x - 6
= (x + 1)(x + 3)(x - 2) x3 + xz
x 2 - 5x- 6
x2 + x
- 6x- 6
- 6x- 6

222
I. In each case show that the first polynomial is a factor of P(x) and hence find all the factors of
P(x).
(a) (x - 1), P(x) = x + 4x 2 + x -
3
6
(b) (x + 1), P(x) = 2x + 9x 2 - 8x
3
- 15
(c) (x + 2), P(x) = x 3 + 3x 2 + 7x + 10
(d) (x + 3), P(x) = 3x 3 - 4x 2 - 35x + 12
(e) (x - 2), P(x) = 2x 3 + 3x 2 - 18x + 8
(f) (x 3), P(x) = x 4 - 3x 3 + 3x 2 - 7x - 6
(g) (x - 1), P(x) = x 4 - x 3 - 5x + 5
(h) (x + 2), P(x) = x 3 + 2x 2 - 9x- 18.

2. Find the factors of the following polynomials.


(a) x 3 + x 2 + x + 1 (f) x 3 - 4x 2 + x + 6
(b) x - 3x + 3x - 1
3 2
(g) 6x 3 + 7x 2 - 9x + 2
(c) x + x - x - 1
3 2
(h) x 3 + 3x 2 - 4x - 12
3
(d) x - 1 (i) 4x 3 - 24x 2 + 17x + 15
3 2
(e) x + 5x + 8x + 4 (j) 2x 3 + x 2 - 25x + 12.

3. For what value of k is (x - 1) a factor of x 2 - 3x + k?


4. For what value of b is (x + 1) a factor of x 3 - bx + 3?

5. If x 3 + ax + b is divisible by both (x + 3) and (x - 4), find the values of a and b.

6. The factors of x 3 + x2 - 4x - 4 are (x + 2), (x - 2) and (x + 1). What are the zeros of the
polynomial?

7. The zeros of the polynomial P(x) = x3 - 6x 2 + 11x - 6 are 1, 2 and 3. Write the factors of
P(x).

8. Show that the polynomial P(x) = 2x 3 - 8x 2 + 2x + 12 has three distinct real zeros a 1 = -1,
a 2 = 2 and a 3 = 3 and express P(x) in the form P(x) = p(x - a 1 )(x - a 2 )(x - a 3 ).

9. Show that 1 and 2 are zeros of the polynomial P(x) = x4 - 2x 3 + 5x 2 - 16x + 12 and hence
that (x - 1) (x - 2) is a factor of P(x).

10. Factor the polynomial P(x) = 3x 3 - 7x 2 + 4 and hence solve the equation P(x) = 0.

11. Solve the following cubic equations.


(a) x 3 + 2x 2 - 5x - 6 = 0 (d) x 3 - 3x 2 - 6x + 8 = 0
(b) x 3 - 7x + 6 = 0 (e) 2x 3 + 3x 2 - 23x - 12 = 0
(c) x 3 - x 2 - 2x = 0 (f) x 3 + 4x 2 - 7x - 10 = 0

12. If P(x) = (x - a) 2 (x - b), then the polynomial equation P(x) = 0 has one root x = b and a
double root at x = a.
(a) Find all the factors of P(x) = x 3 - x 2 - x + 1 and show that P(x) = 0 has a double
root at x = 1.
(b) Sketch the graph of the polynomial and note the nature of the graph at the double root.

13. If P(x) p(x - a) 3 , then the equation P(x)


= = 0 has a triple root at x = a.
Solve the polynomial equation x 3 + 3x 2 + 3x + 1 = 0 and show that x = - 1 is a triple
root. Illustrate with a sketch graph.

14. If P(x) = 2x 2 + 3x + 6, show that the equation P(x) = 0 has no real roots. Illustrate with a
sketch graph.
223
The above questions illustrate the following general results:
(i) If P(x) has k distinct real zeros a 1 , a 2 , •.• , ak, then (x- a 1 )(x- a 2 ) ••• (x- ak) is a
factor of P(x).
(ii) If P(x) has degree nand n distinct real zeros a 1 , a 2 , ••• , a,, then
P(x) = p,(x - a 1 )(x - a 2 ) ••• (x - a,).
(iii) A polynomial of degree n cannot have more than n distinct real zeros.
(iv) A polynomial equation of degree n has at most n real roots (and may have none).

QUADRATIC EQUATIONS
If the quadratic equation ax 2 + bx + c = 0 has roots o: and f3
then, (x - o:) (x - f3) = 0
i.e. x - (o: + f3)x + o:f3 = 0.
2

. . 'd
T h1s 1s 1 entlca . x 2 + -b x + -c = 0.
. 1 to t he equatwn
a a
Therefore, equating coefficients
b c
0: + f3 = --, o:f3 = -.
a a

CUBIC EQUATIONS
If the cubic equation ax 3 + bx 2 + ex +, d = 0 has roots o:, f3 andy,
then, (x - o:)(x - f3)(x - y) = 0
i.e. x 3 - (o: + f3 + y)x 2 + (o:/3 + f3y + yo:)x - o:f3y = 0.
This is identical to the equation x 3 + ~ x 2 + £ x + q_ = 0.
a a a
Therefore, equating coefficients
o: + f3 +y= - ~, i.e. the sum of the roots one at a time = b
a a

o:f3 + f3y + yo: = £, i.e. the sum of the roots two at a time = £
a a
o:f3y = _q_'
a
i.e. the product of the roots = _q_.
a

QUARTIC EQUATIONS
The method used above can be extended to polynomial equations of any degree.
If the equation ax 4 + bx 3 + cx 2 + dx + e = 0 has roots o:, f3, y and (J then,
o: + f3 + y + (J = -~,
a
i.e. the sum of the roots one at a time =
b
a
o:f3 + o:y + o:o + f3y + f3o + yo = £, i.e.
a
the sum of the roots two at a time =£
a
o:f3y + o:f3o + o:yo + f3yo = _q_'
a
i.e. the sum of the roots three at a time= _q_
a
o:f3yo = !!_, i.e. the product of the roots = !!_.
a a
In general for the equation ax" + bx"-1 + cx"- 2 + dx"- 3 + ... = 0 we have:
224
the sum of the roots one at a time = - ~a
the sum of the roots two at a time = !!_
a
the sum of the roots three at a time = - ~
a
the sum of the roots four at a time=~.
a

Example (i):
If o:, f3 and y are the roots of 2x 3 - 6x 2 + x + 2 = 0 find the value of
(a) t>: + f3 + y (b) o:/3 + f3y +yo: (c) o:f3y (d) (o:- 1)(/3- 1)(y - 1).
Solution:
b 6
(a) o: + f3 + y = -- = - = 3
a 2
c 1
(b) t>:/3 + f3y + yo: = - = -
a 2
d 2
(c) o:f3y = -- = -- = -1
a 2
(d) (t>: - 1)(/3 - 1)(y - 1) = (t>: - 1)(f3y - f3 - y + 1)
= o:f3y - (t>:/3 + f3y + yo:) + (t>: + f3 + y) - 1
=-1-!+3-1
-
-
12•

Example (ii):
If x 3 - 5x 2 + 7x + k = 0 has a double integral root, find k.
Solution:
Let the roots be o:, o: and f3
then 2o: + f3 = 5 (1)
o: 2 + 2o:f3 = 7 (2)
o: 2 f3 = -k. (3)
From (1) f3 = 5 - 2o:
Substitute 5 - 2o: for f3 in (2)
t>: 2 + 2o:(5 - 2o:) = 7
3o: 2 - 1Oo: + 7 = 0
(3t>:- 7)(t>:- 1) = 0
.'. o: = 1 or 2k.
Since o: is integral, o: = 1 is the only solution.
Hence f3 = 3
and from (3) 3 = -k
.'. k = -3.
Example (iii):
Solve the equation x 3 + 6x 2 + 3x - 10 = 0 given that the roots are in arithmetic progression.
Solution:
Let the roots be o: - d, o:, o: + d then,
from the sum of the roots 3o: = - 6
(X= -2

from the product of the roots o: 3 - o:d 2 = 10


i.e. -8 + 2d 2 = 10
d2 =9
d = ±3.
Thus the roots are -1, 2 and 5 for either value of d.
225
1. If a, f3 and y are the roots of the cubic equation 2x 3 - 4x 2 + 6x - 5 = 0 find,
(a) a + f3 + y (b) af3 + f3y + ya (c) af3y (d) (a + 1)(/3 + 1)(y + 1).
2. If a and f3 are the roots of 3x 2
+ 2x - 4 = 0, find values of
(a) a + f3 (b) af3 (c) a2 + af3 + f3 2
3. Find the sum and product of the three roots of the equation x 3 - 5x 2 + 6x + 8 = 0.

4. Form the quadratic equation whose roots are


(a) Sand -2 (b) 3 + J2and3- fl.
5. If x = 3 is a root of the equation 5x 2 + kx - 6 = 0, write down the other root and find the
value of k.
6. One root of 5x 2 + 6x + k = 0 is double the other. Find the roots and the value of k.
7. Find kif the equation 3x 2
- 4x +k = 0 has equal roots.

8. Find kif the sum of the roots of x 2 - (k + 6)x + 2(2k - 1) = 0 is half the product of the roots.
9. If two roots of x 3 + bx 2 + ex + 12 = 0 are 3 and -2, find the values of band c.
10. If a, f3 andy are the roots of 3x 3
6x 2
+ 3x - 1 = 0, find the value of
1 1
(a) __!_
af3
+ _!_ + _!_
f3y ya
(b) - +-1 + -.
a f3 Y
11. Given two roots of 2x 3 - kx 2 +8= 0 are equal, find k.
12. If a, /3, y and (j are the roots of x 4 - 4x 3 + 2x 2 - x +2= 0, find the values of
(a) a + f3 + y + (j (d) af3y(j
(b) af3 + ay + a[) + f3y + f3(j + y[) (e) !a + !f3 + !y + !.
(j
(c) af3y + af3b + ay[) + f3y(j
13. Form the cubic equation whose roots are
(a) 2, 3 and 5 (b) 1, (1 + yi2), (1 - yi2).
14. The product of two roots of the equation 2x 3 - kx 2 - 8x +4 = 0 is 1. Find the value of k.
15. The roots of x 3 + px 2 + qx + r = 0 are 4, -1 and 2. What are the values of p, q and r?
16. Two of the roots of x 3 + qx + r = 0 are 5 and 1.
(a) What is the other root?
(b) What are the values of q and r?
17. If x 3 - 8x 2 + kx - 12 = 0 has one root equal to the sum of the other two, find the value of k.
18. Explain why the roots of the equation x 4 + 6x 3 + px2 + qx - 5 = 0 cannot all be positive.
19. Show that 3x 4
+ 7x 3 - 3x 2 + x = 0 has a zero root.
20. What property is common to all equations of the form x 3 + mx + k = 0?
21. Solve the equation x 3 - 12x 2 + 39x - 28 = 0 if the roots are in arithmetic progression.
22. Solve the equation x 3 - 14x 2 + 56x - 64 = 0 if the roots are in geometric progression.
23. Given that the equation x 3 - 7x 2 + 16x + k = 0 has two equal and integral roots find the
value of k.
24. Given that the equation 2x 3 - x2 - 13x - 6 = 0 has two roots whose product is unity, solve
the equation.
226
We know that there is a formula for finding the roots of a quadratic equation but there are no similar
formulae for finding the roots of equations of higher degree.
Sometimes the roots can be found by factorisation but at other times this is not possible. On such
occasions we can find an approximation to a root by using a numerical method.
We will examine two such numerical methods, 'Halving the Interval' and 'Newton's Method of
Approximation'.

HALVING THE INTERVAL


Suppose f(x) is continuous in the interval x = a to y
x = b. If f(a) and f(b) have opposite signs, then there
is a value x = c between a and b for whichf(c) = 0.
That is x = cis a root of the equationf(x) = 0.
We now find a value for f(x) at the mid-point of the
interval a :::;; x :::;; b. X

That isf(a; b).


Iff (a ; b) = 0, then a; bis a root of the equation.
Iff (-a+- b) < 0, then c hes
. between x a+- band x =b.
= -
2 2
Iff (-a+- b) .
> 0, then c hes between x =a and x = 1-a+- -.
b
2 2
This process halves the interval within which the root lies. Repeated applications will give the root
to any desired degree of accuracy.

Example:
Find a root of the equation x 3 +x - 1 = 0 to two decimal places.

Solution:
Let f(x) = x 3 + x - 1
then f(O) = -1 andf(l) = 1.
Hence there is a root of the equation between 0 and 1. Let this root be x = c.
1(
0 1
; ) = f(0·5) = -0·38, therefore c lies between 0·5 and 1.

f ( 0·5 + 1) = /(0·75) = 0·17, therefore c hes


. between 0·5 and 0·75.
2
f ( 0·5 + 0·75) = /(0·63) = - 0·12, therefore c lies between 0·63 and 0·75.
2
f (0·63
\ + 0·75) = /(0·69) = 0·02, therefore c lies between 0·63 and 0·69.
2
(0·63 + 0·69)
f \ = /(0·66) = -0·05, therefore c lies between 0·66 and 0·69.
2
f (0·66
\ + 0·69) = /(0·68) = -0·01, therefore c lies between 0·68 and 0·69.
2
Thus x = 0·68 is a root of the equation to two decimal places.
This method is very tedious even when working with a calculator but lends itself well to programming
for a computer.
227
1. Use the 'halving the interval' method to show that one root of the equation x 3 + 2x - 8 =0
is approximately 1·67.
2. Find an approximate root for the equation x 3 - 4x +2= 0 in the interval 0 < x < 1 to two
decimal places.
3. Show that there is a root of x4 + 8x - 12 = 0 in the interval 1 < x < 2 and find its value
correct to one decimal place.

NEWTON'S METHOD OF APPROXIMATION


Suppose a continuous functionf(x) has a graph as shown Y
and that x = a is close to a root of the equationf(x) = 0.
The tangent toy = f(x) at x = a has the equation
Y - f(a) = f'(a)
x-a
that is y - f(a) = f'(a)(x - a).
This tangent meets the x axis at the point (a 1 , 0) where
f(a)
a 1 =a-f'(a)' x
If the original value of a was close to the desired root, then
the new value a 1 is even closer. Similarly, by repeated
application
j(a1)
a2 = a1 - f'(a )
1
is an even closer approximation. This process can be continued to obtain the desired degree of accuracy.
Example:
Find an approximate root of the equation x 3 +x - 1 = 0.
Solution:
Let f(x) = x 3 + x - 1
then f(O) = -1 andf(l) = 1.
Hence there is a root of the equation between x = 0 and x = 1.
Let a = 0·5 be an approximate solution
/(0·5)
then a 1 = 0·5 - f'(0· 5) is a closer solution
-0·375
a 1 = 0·5- .
75
= 0·71.
Now 2 0 71 f(0· 7 l)
a = · - /'(0·71)
= 0·71 - 0·068
2·512
= 0·68.
Therefore x = 0·68 is an approximate root of the equation x 3 + x - 1 = 0.
N.B. (i) In general Newton's Method requires fewer steps for a given accuracy than the Halving
the Interval Method. Compare the same example in the text which has been worked by
both methods.
(ii) Care must be taken in applying Newton's Method since it may fail if the initial guess for
the root is not close enough.
(iii) We may use both Newton's Method and the Halving the Interval Method even though
the function is not a polynomial. The function must be continuous in the interval being
considered.
228
1. If x = 1·6 is close to a root of the equation x 3 - 4x +2= 0, find a better approximation
correct to two decimal places.
2. Find a root of the equation x 3 - 3x - 1 = 0 in the interval 1· 5 < x < 2 correct to two decimal
places.
3. By evaluating f(x) = x 3 + x 2 - 5 for x = 0, 1, 2, 3, find an interval which contains a zero.
Use this fact to find an approximate root of the equation x 3 + x 2 - 5 = 0 correct to two decimal
places.
4. Show that the equation x 3 - 2x - 3 = 0 has a root between x = 1 and x = 2. Choose an
approximation for this root and then use two applications of Newton's Method to give a better
approximation.
5. Find a root of the equation x 4 - 5x - 8 = 0 in the interval 2 < x < 3 correct to two decimal
places.
6. Use Newton's Method to find an approximate root of the equation x 3 - 20 = 0 and hence give
J20 correct to two decimal places.
7. Draw sketch graphs of y = 2 sin x andy = x and show that the equation x - 2 sin x = 0 has
a root near x = 2. Find this root correct to two decimal places.
8. The equation ex - x - 2 = 0 has a root close to x = 1·2. Use Newton's Method to find this
root correct to two decimal places.

229
Playing cards arranged in the pattern known as
Pascal's triangle.

i I
CHAPTER 29

Often in previous work you have come across expressions like (a + b) 2 or (1 + x) 3 • In this chapter
we will learn quick ways to expand more complex expressions like (a + b) 6 or (1 - 3x 2 ) 8 • These are
called binomial expressions because there are two terms inside the parentheses. They have many
important applications and in the following chapter we will apply them to probability.

1. Verify by multiplication that (1 + x) 2 = 1 + 2x + x 2 •


2. By multiplying the above result by (1 + x) show that (1 + x) 3 = 1 + 3x + 3x 2 + x 3 •
3. Verify that (1 + x) 4 = 1 + 4x + 6x 2 + 4x 3 + x 4 •
4. Find the expanded form of (1 + x) 5 •
5. Look at the pattern at the side. It gives the coefficients 1 +X 1 1
of x in the expansions listed. (The constant term is the (1 + x) 2 1 2 1
coefficient of x 0 ) (1 + x) 3 1 3 3 1
(1 + x) 4 1 4 6 4 1
(1 + x) 5 1 5 10 10 5 1
See how a triangle is developing. Can you see a pattern of how each line can be formed from
the line above? Write the next two lines of the pattern.
6. Assuming the pattern continues, guess at an expression for (1 + x) 6 • Verify by multiplication
given (1 + x) 5 •
7. Guess the expression for (1 + x) 7 • Check.
8. Assuming the pattern continues write down an expanded form for (1 + x) 8 •
9. How many terms are there in the expanded form of
(a) (1 + x) 5 ? (b) (1 + x) 7 ?
10. How many terms would you expect in the expanded form of (1 + x) 12 ?
11. What is the highest power of x in the polynomial expansion of
(a) (1 + x) 4 ? (b) (1 + x) 8 ?
12. What would you expect the highest power of x to be in the expansion of (1 + x) 11 ?
PASCAL'S TRIANGLE
The pattern of numbers developed in Question 5 of Exercise 29.1 is called Pascal's Triangle, so named
after Blaise Pascal (1623-62) who was involved in the early study of probability.
There are two rules in creating Pascal's Triangle. The outer coefficients are each unity. The inner
coefficients come from the addition of the two adjacent numbers immediately above. Of course, each
row has one extra number from the row above. Note that as (1 + x) 0 = 1 we could begin the triangle
from 1.

The expanded form of (1 + x)" (where n = 2, 3, 4, etc.) is a polynomial in x with (n + 1) terms.


The highest power of x is x". The coefficients are shown by the appropriate line in Pascal's
Triangle.

232
1. Begin with 1
1 1
1 2 1
Continue to write down the first ten rows of Pascal's Triangle.
2. Using your triangle, write down expansions for
(a) (1 + x) 5 (c) (1 + x) 4 (e) (1 + x) 8
(b) (1 + xf (d) (1 + x) 3 (f) (1 + x)9
Keep your triangle for reference in the next set of exercises.

In the expression (1 + x) 3 any complex expression can be substituted for x.


Hence, if(l + x) 3 = 1 + 3x + 3x 2 + x 3
Then (1 + a 4 ) 3 = 1 + 3(a 4 ) + 3(a 4 ) 2 + (a 4 ) 3
= 1 + 3a + 3a + a
4 8 12
3 2 3
Again ( l - 2x)
3 -_ 1 + 3 ( - 2x)
3 + 3 ( -3
2x) + ( -3
2x)
2
8x 3
= 1- 2x + 4x - .
3 27
Note in the last expression how the positive and negative signs alternate.
Example (i):
Expand (1 + 3;y
Solution:

;r c;r
First we notice from Pascal's Triangle that the coefficients will be 1, 5, 10, 10, 5, and 1.

(1+
3
=
1+ 5 +C;) wC;Y wC;Y+
2
+5
3
+
4
C~'Y
5
= 1 15x + 45x + 135x + 405x + 243x
+ 2 2 4 16 32
Example (ii):
Calculate (0·998) 4 correct to 6 decimal places.
Solution:
0·998 = (1 - 0·002) i.e. (1 - x) where x = 0·002
(0·998) 4 = (1 - 0·002) 4
= 1 + 4(- 0·002) + 6(- 0·002) + 4(- 0·002) + (- 0·002)
2 3 4

= 1 - 0·008 + 6(0·000004) + 4(- 0·000000008) +


= 1 - 0·008 + 0·000024 - 0·000000032 + ...
= 0·992024.
Notice how terms beyond the third do not affect the first six decimal places and hence do not need to
be calculated.

1. Write the expanded form of:


(a) (1 + 2x) 3 (c) (1 + x 2 ) 5 (e) (1 + 3x 2 ) 3
(b) (1 - !)4 (d) (1 - 2JX) 2 (f) (1 + ay) 4 .
233
2. Expand in ascending powers of x as far as the fourth term.

(a) (1 + ~J (c) (1- xy 2


3
(b) (1 - 5x) 6 (d) (1 + ax 2 ?.
3. Simplify (1 + x) + (1 - x) by expanding and collecting terms.
5 5

4. By treating 1·01 as (1 + 0·01) find (1·01) 6 correct to five decimal places. Check with calculator.
3
5. Find (1·003) correct to five decimal places.
6. Calculate (a) (0·99) 4 (b) (0·98) 3 to 5 decimal places.
7. Find (a) (1 + y'3) 3 + (1 - y'3) 3 (b) (1 + y'3) 3 - (1 - y'3) 3 •
8. Show that (1 + j2) 4 + (1 - J2)4 is rational. Find its value.
9. If(l - 3j2) 3 = P + Qj2 where P and Q are rational, find P and Q.

10. Expand (1 + ~ J and (1 - JxJ·


11. Look at Pascal's Triangle. Add the coefficients in each row for several rows. Can you see a pattern?
12. Expand (1 + x) 5 • Then let x = 1. Notice that, if x = 1, the right hand side of the expansion is
just the sum of the coefficients. By examining the left hand side can you see why the value is 32?
Can you see a meaning or proof for the pattern in Question 11?
13. Without expanding, what would you expect the sum of the coefficients to be in:
(a) (1 + x) 12 ? (b) (1 - x) 12 ? (c) (1 + x) 9 ?

I4. Expand (a) (1 + ~Y

(b) (~ _ ~2r

THE EXPANSION OF (a + x)n


Study this example.
Example (i):
Expand (a + x) 4
Solution:
x) [a(l + ~)]
4

(a+
4
= = a (1· + ~J
4

So (a+ x) 4 = a 4 (1 + 4~ +
a a a
2
6x 2 + 4x 3 + x 4 )
a
3 4

= a 4 + 4a 3 x + 6a 2 x 2 + 4ax 3 + x 4

The expanded form of (a + x)" has the same coefficients as (1 + x)". There are (n + 1) terms.
The powers of a decrease and the powers of x increase term by term. For any term the sum of
the powers of a and x is n.

Example (ii):
Write out the expansion of (a) ( x + ~y (b) (2x - 3y) 4 .

234
1. Write out the expansion of:
(a) (2x + 1)4 (d) (2a- 3b) 3 (g) (x 2 + 2t
3
5 2
(b) (3x - y) (e) ( x + _:2 ) (h) (ax- by?

(c) (x + ~y (f) (3x - 2) 5 (i)


(JX + JxY·
2. Find the coefficient of x in:
3

(a) (x - 2) 5 (b) (2 + x) 3 (c) (x- 5) 4 •

(y + ~)
6
2
3. Find the term independent of x in •

4. For what value of x are the third and fourth terms in the expansion of (x + 3) 4 equal?
5. If (3 - J2f = P + Q.j2 where P and Q are integers find P and Q.

6. simplify (x + ~r- (x- ~Y.


7. Show that (3 - JS? + (3 + j5) 3 is rational.
8. Expand (x + 2) 4 and with or without fully multiplying out find the coefficient of x 3 in
(x + 1)(x + 2t.
9. Find the coefficient of x 3 in (3 - x) 4 (1 + x).

Expanding expressions using Pascal's Triangle is of limited value because it is tedious to write out
the triangle beyond the first few lines. Hence we will develop a more general formula for binomial
expressions.
We know the expansion of (1 + x)" is a polynomial of degree n in the variable x. Let us represent
the coefficient of xk in the expanded form of (1 + x)" by the symbol "Ck. Thus, for example, the
coefficient of x 2 in (1 + x) 7 would be 7 C2 . For generality include "C0 which of course is 1. Then we
write
(1 + x)" = "C0 + "C1 x + "C2 x 2 + "C3 x 3 + · · · + "C,.x"
and in a similar way
(a+ x)" = "C0 a" + "C1 a"- 1 x + "C2 a"- 2 x 2 + · · · + "C,.x"
where the coefficients are the same as for (1 + x)".
235
PROOF OF THE PASCAL TRIANGLE RELATIONS
1. (a+ x)" = "C0 a'' + "C1a"- 1x + "C2a"- 2x 2 + ... + "C,x".
Let a = 1 and x = 0. This becomes
1" = "C0 . 1" + 0 + 0 + · · . + 0
So "C0 = 1.
Now let a= 0 and x = 1. Clearly "C, = 1.

2. Using the formula, we have:


(1 + x)"-1 = 1 + .. -1c1x + .. -1c2x2 + 0 0 -1ckxk +
0 .. 0 -1c,_1x"-1
0 0 ..

Now (1 + x)" = (1 + x)"- . (1 + x)


1

= (1 + x)"- 1 + x(1 + x)"- 1


So (1 + x)" = (1 + "- 1C 1x + "- 2C 2 x 2 + ... "- 1ckxk + ... "- 1c,_ 1x"- 1)
+ x(1 + ,-1c1x + .. -zczxz + ... + ,-1ck-1xk-1 + n-1ckxk +
+ n-1c,-1x"-1)
On the left hand side the coefficient of xk in (1 + x)" is "Ck.
On the right hand side the coefficient of xk is "- 1ck + "- 1ck_ 1.
Two polynomials can only be equal if coefficients of every term are equal.
Hence "Ck = "- 1 Ck_ 1 + n- 1ck
This is the pattern exhibited in Pascal's Triangle from one row to the next (i.e. from row n - 1
to row n).

Any inner coefficient, such as "Ck in the expansion of (1 + x)" is the sum of two adjacent coef-
ficients "- 1Ck_ 1 and "- 1Ck in the expansion of (1 + x)"- 1. That is, "Ck = "- 1ck_ 1 + "- 1ck.

THE FORMULA FOR nck

To prove the formula for "Ck we will use mathematical induction. We must first guess what we think
the formula is and then verify it.
Consider (1 + x) 4 = 4 C0 + 4 C1x + 4 C 2 x 2 + 4 C3 x 3 + 4 C4 x 4 . We know that
(1 + x) 4 = 1 + 4x + 6x 2 + 4x 3 + x 4 .
So C0 and 4 C 4 are each unity as expected. Also 4 C1 = 4, 4 C2 = 6 and 4 C3 = 4. First 4 C1 = 4
4

gives the idea that "C1 may be n. Also 4 C2 = 6 =


4
~ 3 = n(n ~ 1) if n = 4. We guess that "C2
n(n- 1)
maybe--'-------'-
1.2
4C =
3 4 = 4 x 3 x 2 = n(n - 1)(n - 2) if 11 = 4 .
1x2x3 1x2x3
1 2
We guess that "C3 = n(nl- )(n - ) and in general we guess that
X 2 X 3
n(n - 1)(n - 2) x .. · x (n - k + 1)
"Ck = where there are k factors in both numerator and
1x2x3X .. · X k
denominator. Notice that this general formula gives "C, = 1. We include "C0 = 1 as a special fact.
k = 0 obviously cannot be substituted in the formula.
.. "C _ n(n - l)(n - 2) x ... x (n - k + 1)
P roo f th a t the 1ormu1a k - 1
1 X2X 3 Xoo•XI(
is true for all values of k and n (1 :( k :( n).
236
We will use the method of mathematical induction.
Step I:
Suppose n = 2.
We know that (1 + x) 2 = I + 2x + x 2 •
So 2 C0 = 1, 2 C 1 = 2 and 2 C2 = 1. These agree with the formula fork = 1 or k = 2 (and 2 C0 is
a special case).
So the formula is true for all values of k when n = 2.
Step II:
Let us assume that the formula is true for all coefficients in the expansion of (1 + x)"- 1 . We will
try to prove that the formula will then be true for all coefficients in (1 + x)".
Assuming the formula with (1 + x)"- 1 we have
11-1c _ (n - 1)(n - 2) x . · · x (n - k + 1)
1
k- - I X 2 X ..• X (k - I)
_ (n - 1)(n - 2) x .. · x (n - k)
and II 1 ck =
1 X 2 X · · · X (k)
and we know that
"Ck = "-1ck-1 + n-1ck
(n 1)(n - 2) x · · · x (n - k + + (n
- 1)(n - 2) x · · · x (n - k)
1)
1 X 2 X · · · X (k - 1) 1 X 2 X · • · X (k)
_ k[(n - 1)(n - 2) x · · · x (n - k + 1)] + [(n- l)(n _:_ 2) x · · · x (n- k)]
- 1x2x3x ... xk;
_ [(n - 1)(n - 2) x · · · x (n - k + 1)] [k + (n - k)]
- 1x2x3x ... xk
_ [(n - 1)(n- 2) x · · · x (n - k + 1)] [n]
- 1x2x3x ... xk;
_ n(n - 1)(n - 2) x · · · x (n - k + 1)
- 1x2x3x .. ·xk
This expression for "Ck corresponds to the formula so we have proved that if this formula holds in
the case of (1 + x)"- 1 then it holds in the case of (1 + x)".
There is just one minor point to check. In the above proof k could not equal 1 because that would
produce a zero in the denominator of the expansion for "- 1ck_ 1. So we must treat separately the
case when k = 1.
"Ck = n-1ck-1 + "-1ck
So when k = 1 "C1 = "~ 1 C0 + "- 1 C 1
But "- 1 C0 = 1 (special case) and "- 1 C 1 = n - 1 from the assumption that the formula holds in the
case of (1 + x)"- 1 • Hence "C1 = 1 + (n - 1) = n as expected.
The method of mathematical induction is fulfilled.
Step I:
The formula is true for all coefficients for (1 + x) 2 i.e. x = 2.
Step II:
If the formula is true for all coefficients for (1 + x)"- 1 then it is true for all coefficients in (1 + x)".
Thus if it is true when n = 2 it is true for n = 3, 4, etc.
n(n - 1)(n - 2) x · · · x (n - k + 1)
Hence "Ck = --'------'--'--------'-------'-----'-
1x2x3x ... xk
for all values of k and n (1 ~ k ~ n).
N.B. As a special case we include "C0 = 1.

II

(a + b)" = I "Cka"-kbk
k=O
n(n - 1)(n - 2) x · · · x (n - k + 1)
where "C = --'------'--'------'-------'--,-----'-
k 1x2x3x .. ·xk
N.B. It is the (k + lYh term in the expansion of (a + b)" which is "Cka"-kbk.

237
Example:
(a) Expand (a + b) 6 •
(b) Find 5 C 2 and 7 C3 .
(c) Write down the third term in (2x - y) 5 .
Solution:
(a) (a+ b)G = aG + 6C1asb + 6C2a4bz + 6C3a3b3 + 6C4a2b4 + 6Csabs + b6
_ 6 asb ~a 4 b2 6 X 5 X 4 3b3 6 X 5 X 4 X 3 2b4
-a +6 + 1 X 2 + 1 X 2 X 3a + 1 X 2 X 3 X 4a

+ 61 XX 52 XX 43 XX 43 XX 25 abs + b6
= a6 + 6a 5 b + 15a4b 2 + 20a 3b 3 + 15a2 b 4 + 6ab 5 + b 6
(b) sC _ 5 X 4
2 - 1 X 2

=10
7C _ 7 X 6 X 5
3
-lx2x3
= 35.
N.B. Both numerator and denominator have the same number of factors.
(c) Note that for the third term k = 2
T3 = 5 C2 (2x) 3 ( - y) 2
= 10 X 8x 3 X y 2
= 80x 3y 2 .

1. Write out expansions for:


(a) (a + b) 5 (c) (2x - 3y) 3 (e) (1 + x?
(b) (1 - x) 4 (d) (5x - 1) 3 (f) (3 + j2) 4 •
2. Calculate:
(a) 9 Cz
3. Write out the coefficient of x 3 in:
(a) (x + 2) 5 (b) (3x - y) 4 (c) (2x- 1r.
4. What is the third term in each expansion?
(a) ( x + ~y (b) (1 - x 2 ) 8

5. In the expansion of (1 + x) 8
(a) How many terms are there?
(b) What are the second and second-last terms?
(c) What is the coefficient of x 4 ?
(d) Which term contains x 6 ?
(e) What is the power of x in the fourth term?
6. In the expansion of (3x - 4y) 10 :
(a) How many terms are there?
(b) What are the powers of x and y in the fourth term?
(c) Which term contains x 6 ?
(d) What is the sum of the powers of x andy in any given term?
(e) If a term contains x 3 , what power of y does it contain?
(f) Find the term where the powers of x andy are equal.
7. Find the middle term in (3x - 2) 8 •
238
8. Calculate 9 C 1 , 9 C2, 9 C3 , 9 C4 and 9 C5 • Hence write down the ninth line in Pascal's Triangle.
9. Expand (1 + J3xf and if x = 1 express in the form A + BJ3 where A and Bare integers.
10. Show that (1 + 2J2) 6 = 1593 + 1100J2.
11. Evaluate (j3 + 1) 6 + (j3- 1) 6 .
12. By treating 0·9 as (1 - 0·1) show that (0·9) 8 = 0·4304672 correct to 7 places.
13. Expand (1 - 2x) 6 in ascending powers of x. By considering 0·98 as (1 - 2 x 0·01) find (0·98) 6
to 6 decimal places.
14. Simplify (x + t) 4 - (x - t) 4 •
15. Leaving answers in unsimplified form, find
(a) the last term in (3 - 2y) 20 .
(b) the sixth term in (2x- z) 15 .
(c) the middle term of(5y- t) 20 .
16. What are the coefficients of x 4 and x 5 in (x + 1) 7? Hence find the coefficient of x 5 in
(x- 1)(x + 1) 7 .
17. By noticing that (1 + x) 4 is the same as (x + 1) 4 , or otherwise, write out the terms of (1 + x) 4
in descending powers of x (i.e. beginning with the highest power).
18. Write the first three terms in descending powers of x of the expansion (a+ xt.
19. Without calculation show that 7C 2 and 7C5 are equal and that also 7C3 and 7C4 are equal. Check
by calculation.
20. What is true about "Ck and "Cn-k? Why?
21. By remembering the formula for "Ck or by remembering that numerator and denominator each
have k terms, or otherwise, fill in the last factor missing in 2 °Ck below
20 20 X 19 X • • • X ( )
ck = --~----------~~
1x2x .. ·xk
22. Express 2°Ck-t in a similar way to Question 21.

Remember that the (k + 1Yh term, 7k+ 1 , in (a + b)" is "Cka"-kbk. Consider these examples below.

Example (i):
7
Find the coefficient of x 5 in the expansion of (~ 2 + ~) .

Solution:

lk+t x
= 7ck (x 2)7-k (2)k
= 7Ckx14-2k 2kx-k
= 7 ck 2kx14-3k
For the term with x 5 we have 14 - 3k = 5, so k = 3.
If k = 3 T4 = 7 C3 2 3 x 5
_ 7 X 6 X 5. 3 5
-1x2x3 2 .x
= 280x 5 .
239
Example (ii):
~3 )
8
Find the term independent of x in the expansion of (3x - •

Solution:
7k+1 =
=
sck(3x)8-k
sck38-kx8-kx-3k
(~3 r
= sck38-kx8-4k
For the term independent of x (i.e. with x 0 ) we have 8 - 4k = 0 or k = 2.
Using k = 2, T3 = 8 C23 8 - 2x 0
=~·36
1 X 2
= 28 X 729
20412.
=
N.B. Often an answer can be left unsimplified.

Example (iii):
Find the greatest coefficient in the expansion of (2 + 3x) 20 .
Solution:
In equations like this be careful to realise that it is the (k + 1) 1h term which contains xk. Likewise
be careful to interpret k at the end.
We will compare the (k + 1)1h term 7k+1 with the k 1h term Tk.
1k+1 = 2oCk22o-\3x)k
1k = 2ock_ 12 2o-(k-1>(3x)k-1
= 2ock_ 12 21-k( 3x)k-1

7k+1 = 2ock x (3x)


rk 2ock-1 2
20 X 19 X
----~--~----~~~--~x
• •· X (20- k + 1) 1 X 2 X ••• X (k - 1) (3x)
1 X 2 X k
· •· X 20 X 19 X •· • X (20 - (k - 1) + 1) X 2
(20 - : + 1) X C;)
(21 - k)3x
k X 2
The ratio of the coefficients of 7k+ 1 to 7k exceeds 1 when
63- 3k > 2k
i.e. k < 12·6
So, for values of k of 1, 2, 3, ... , 12,
coefficient of 7k+ 1 > coefficient of 7k
But fork = 13 or greater, coefficient 7k+ 1 < coefficient of Jk. The term with the greatest coefficient
is when k = 12 (i.e. T13 )
T13 = 2oC122s312

N.B. (i) Make sure you understand the calculation (and cancelling) of 2°Ck and 2°Ck-l· See Exercise
29.5, Questions 21 and 22.
(ii) Make sure you understand how the coefficients are getting larger until T13 is reached and
then that they reduce.
(iii) Check the special note below.

In (a + b)"
7k+1 n - k + 1 b
1k = k ·-a
Some people find this rule helpful. Check that it is true.

240
1. What is the coefficient of x 4 in:
2
(a) (x - : 2
0
y? (b) (2x- ~y?
2. Find the term independent of x in:

(a) (x- ~Y (b) (2x


2
+ :3 y 0

2~y
8
3. What is the term independent of x in ( x 2 - ?

4. What is the term independent of x in ( x +x


1)211 ?

5. What is the greatest coefficient in


(a) (1 + x) 6 ? (b) (1 + x) 9 ?
6. What is the ratio of the (r + 1Yh term to the r 1h term in (1 + x) 11 ? Find the value of r for which
the coefficients of the r 1h and (r + 1Yh terms are equal.
7. Find the greatest term in (5 - 4x) 12 if x = i.
8. Find the value of kif the coefficients of xk and xk+ 1 in (3x + 2) 19 are equal. Be careful!
9. By treating (1 + x + x 2) as 1 + (x + x 2) expand (1 + x + x 2) 3 .
10. By treating (1 - 2x + x 2) as 1 - x(2 - x) and hence as (1 - a), where a = x(2 - x), find the
first four terms of(1 - 2x + x 2)\ that is, up to the power x 3 •

Example:
By considering that (1 + x) 211 = (1 + x)11 (1 + x) 11 and by examining the coefficient of X 11 on each
side of the identity, show that
II
211CII = L (11Ck)2
k=O

Solution:
(1 + x)211 = 1 + 211Clx + ... + 211CIIxll + ... + x211.
We note that the coefficient of x" is 2 "C,.
(1 + x)" = 1 + "C1 x + "C2x 2 + ... + "Ckxk + ... + x"
(1 + x) 2 " = (1 + x)"(1 + x)"
2 2
= (1 + "C1 x + "C2x + · · · + x")(l + "C1 x + "C2x + · .. + x")
The terms containing x" on the right hand side will arise from
(1 . x" + "C1 x. "C,_ 1 x"- 1 + · · · + "Ckxk. "C,_kxn-k + ... )
But remember that "C1 = "C11 _1 , 11 C2 = "C11 _2 etc.
So coefficient of x 11 on right hand side = ( 11 C0 ) 2 + (11 C1 ) 2 + . . . + (11 C11 ) 2
II
Hence 2ncll = L ("Ck)2.
k=O

1. By considering that (1 + x) 11 +1 = (1 + x)(1 + x) 11 prove that


u+lc3 = ~~c3 + ~~c2.
241
2. Use the result that (1 + x)" = (1 + x)(l + x)"- 1 and consider the coefficient of x' in both
expressions to prove that
"C, = .. -1c,. + .. -1cr-1·
3. In all parts of this question begin with
(1 + x)" = "C0 + "C1x + · · · + "Ckxk + · · · + "C,x"
(a) Let x = 1. What is "C0 + "C1 + · · · + "C,?
(b) Let x = -1. Prove that "C0 - "C1 + "C2 · · · + ( -1)""C, = 0.
(c) From (b) show that "C0 + "C2 + · · · = "C1 + "C3 + · · ·.
(d) Differentiate both sides of the identity and then let x = 1 and hence find
"C1 + 2"C2 + 3"C3 + · · · + n"C,.
(e) Using the results of (a) and (d) prove that
"C0 + 2"C1 + 3"C2 + · · · + (n + 1)"C, = (n + 2). 2"- 1.
(f) Integrate both sides of the original identity. Don't forget to allow for the constant of integra-
tion and find its value. Hence prove:
"C "C "C "C 2"+ 1 - 1
~0+~1+~2+ .. ·+--"-= .
1 2 3 n+l n+1
I

4. Begin with [(1 + x)"] 2 = (1 + x) 2 " and prove that


(1 + "C1 + "C2 + ... + "C,Y = 1 + 2"C1 + 2"C2 + ... + 2"C2,
5. Find the sum of the coefficients (including "C0 or 1) in the expansions of:
(a) (1 + 2x) 6 (b) (3x + 2y) 5 •
Hint: In the expansion let x = 1, y = 1.
6. Use the identity (1 + x) 4 (1 + x) 3 = (1 + x) 7 to prove that 4
C2 + 4
C1 . 3 C1 + 3 C 2 = 7
C2.
Check the result from the numerical values of 4 C2 etc.

242
The probability of a "royal routine" in poker is 1 in
6tJD 7tl0
The probability of a "full house" is approximately 1 in
l394
CHAPTER 30

Many questions in probability can be done by enumerating the sample space. Sometimes a tree
diagram helps as shown in the following example.
Example:
Four cards labelled 1, 2, 3, and 3 (i.e. two the same) are in one hat and three cards labelled A, Band
C are in a second hat.
(a) How many arrangements are possible (including repetitions) if a card is drawn from each hat?
(b) In how many of these do the cards show 3-A?
(c) What is the probability of a 3 and an A?
Solution:
We draw a tree diagram and enumerate the sample space
First Hat Second Hat Sample Space
(a) Altogether 12 arrangements are

1~:
1A
possible (not all different).
18
(b) 3-A occurs twice.
1C (c) P (3-A) = n(E)
n(S)

2~:
2A
2
28 12
-
1
2C
6

3~:
3A

38

3C

3A

3~: 38

3C

Notice that if there were 20 cards in the first hat and 50 in the second it would be tedious to enumerate
the sample space. So we make use of a rule.

If a selection can be made in r different ways and if a second selection can be made ins different
ways, then the two selections can be made in r x s different ways.

We can call this a fundamental counting principle. It could be generalised.

To find the total number of ways of making several choices in succession multiply the number
of ways each choice can be made.

In the example above the answer to part (a) comes from (4 x 3) and the answer to part (b) comes
from (2 x 1) as there are two cards labelled 3 in the first hat and one labelled A in the second.
244
1. A car number plate in Australia usually has 3 letters followed by 3 digits, assuming all arrange-
ments are allowed, how many are possible? Could everyone in Australia over 18 years own a
car with a different plate? Remember that the same letter or number may be repeated.
2. If you were given 100 guesses what is the probability you could guess someone's number plate?
3. If I was given a random number plate for a new car what is the probability that it would (a) begin
with B (b) have the digits 000?
4. Some years ago number plates in Australia had only 2 letters followed by 3 digits. How many
arrangements were possible (assuming all were allowed)?
5. A telephone dial uses 10 digits including zero. How many different 7-figure telephone numbers
are possible?
6. Some years ago telephone numbers had only six figures. (Some were called by letters but that
doesn't alter the mathematics as there were still only 10 letters.) How many numbers were possible?
Why do you believe the system was changed?
7. A combination lock is made with 4 dials and six numbers on each. How many arrangements are
possible? If it took 5 seconds to try each combination, how long might it take to open the lock
by trial and error if the right answer was not found till the end?
8. What is the probability that a person acting at random would open the combination lock of
Question 7 (a) in 100 tries (b) in 10 minutes?
9. On a restaurant menu there is a choice of 4 different entrees, 5 main dishes and 3 desserts. How
many different meals, consisting of entree, main dish and dessert, are possible?
10. AM radio stations in NSW have a call sign beginning with 2 and followed by 2 letters (e.g. 2BL,
2UW). When will this system need to be changed?
11. In NSW postcodes have 4 digits and always begin with a 2 (e.g. 2001). How many different suburbs
or locations can be accommodated in this system?
12. A restaurant offers these choices:

ENTREE MAIN COURSE DESSERT


Garlic prawns Fillet steak Strawberries
Soup of the day Chicken Apple pie and cream
Oysters Fish

How many different 3 course dinners can be chosen?


If I was late and someone ordered for me what is the probability that they would choose what
I wanted, assuming they knew I did not like oysters?
13. A shirt manufacturer makes shirts in 10 neck sizes and with 3 possible arm lengths. How many
combinations does this give? If this manufacturer also makes his shirts from 30 different styles
or materials, how many different shirt combinations is this altogether?
14. Concerning the shirts of Question 13 what is the probability that one chosen at random will
fit my neck and arm although I may not like the style?
15. A shoe shop keeps sizes of men's shoes from size 5 to size 11, and for each size there are 5 frac-
tional fittings (widths). The shoes also come in black or tan. How many different shoes must the
shopkeeper have to be sure he will suit a particular customer with his style of shoe?
16. A car manufacturer offers 12 paint colours, 4 possible colours of interior trim and the choice
of the deluxe or standard version. How many choices does that give the customer altogether?
If the car dealer has six cars in stock what is the chance he has what a customer wants?
245
17. A poker machine has 4 wheels with 20 symbols on each. How many arrangements are possible?
If there are 2 aces on each wheel in how many ways can all wheels stop on an ace? What is the
probability of 4 aces showing across the dial?
18. A poker machine with 4 wheels and 20 symbols on each wheel has 5 aces on the first wheel, 4
aces on the second and two aces on each of the other wheels. What is the probability of 4 aces
showing?
19. In car number plates (3 letters and 3 digits) how many begin with the prefix AAA? How many
are there with the three letters all the same (e.g. AAA, BBB, etc.)? What is the probability of
receiving such a plate at random?
20. (a) With two packs of cards (52 cards; no joker) in how many ways, taking one card from each
pack, can you choose:
(i) two aces?
(ii) two queens?
(iii) two cards of the same denomination?
(b) What is the probability of choosing two cards of the same denomination when making just
one choice of the two cards?

First compare the following two examples.


Example (i) :
The digits 1, 2 and 3 are each written on a card which is then placed in a box. A card is chosen, read
and replaced. A second card is chosen and read. How many possible 2 digit numbers can be chosen?
Solution:
The first digit can be any of 1, 2 or 3. The second digit can be any of 1, 2 or 3. There are 9 possible
numbers, which you can check for yourself.
Example (ii):
The cards numbered 1, 2 and 3 are placed in the box as before. But this time the first card chosen is
not replaced. How many 2 digit numbers can be made?
Solution:
The first digit can still be chosen in 3 ways. But the second digit can only be chosen from the two
cards left. There are 6 possible 2 digit numbers.
N.B. The difference here is that the numbers 11, 22 and 33 are not possible.

Example (iii):
In horse racing, a trifecta is the name given to choosing the first three horses in a race in correct order.
In how many ways can this be done if there are 10 horses in the race? What is the probability that
a given choice will be correct, assuming all choices are equally likely?
Solution:
Any of the 10 horses in the race can come first. Then any of the remaining 9 horses can come second
and any of the remaining 8 horses can come third.
No. of trifectas (permutations) = 10 x 9 x 8
= 720
Probability = 1/720.
246
1. How many 3 digit numbers can be formed from the five numbers 1 to 5 if each is used only once?
2. In how many ways can 5 boys line up?
3. In how many ways can four letters from the word BRIDGE be arranged if no letter is repeated?
4. In a class of thirty children one prize is awarded for English, another for Science and a third for
Mathematics. In how many ways can the recipients be chosen if no child can receive more than
one prize?
5. What would be the answer to Question 4 if a child is allowed to receive more than one prize?
6. Seven boys are running in a race. In how many ways can the first three places be filled if there
are no dead heats?
7. In how many ways can the first 3 horses in an 8 horse race come home?
8. What is the probability of picking the trifecta in (a) a 12 horse race, (b) a 6 horse race, (c) a 15
horse race? Assume that all the horses are equally likely to win.
9. In a 12-horse race five of the horses really have hardly any chance of gaining a place and for
this question may be disregarded. In how many ways can the first three places be filled from the
other horses? If these horses are all about equally likely to gain places, what approximately are
the chances of picking the trifecta?

If we arrange 4 objects in order the number of permutations is 4 x 3 x 2 x 1. For convenience


we have a notation for this. It is calledfactorial4 and written 4! Thus factorial6 is
6 x 5 x 4 x 3 x 2 x 1 or 720 and 3! means 3 x 2 x 1 which is 6. Note that 0! is defined to be 1.
The number of permutations of n objects r at a time is written "Pr
4 6
P4 = 4 X 3 X 2 X 1 P2 = 6 X 5
= 4! 6 X 5 X 4 X 3 X 2 X 1
4 X 3 X 2 X 1
6!
4!

L
~-~------------------

11 _ " _ n!
I=~e:eral P11
1
- n . and Pr - (n _ r) !

Study these examples.


Example (i):
In how many ways can 6 people be arranged around a circular table if the order around the circle
is all that matters?
Solution:
The first person can sit anywhere. The second person can sit in any of the remaining 5 places. The
third person can sit in any of the remaining 4 places, and so on.
No. of permutations = 5! = 120.
(Note that for n people the answer is (n - 1) !)
Example (ii):
In how many ways can 8 books be arranged on a shelf
(a) if a given book must be first in line?
(b) if two particular books are kept alongside each other?
247
Also, what is the probability that in a random arrangement of the 8 books the conditions (a) and
(b) will be satisfied?
Solution:
(a) Fix the given book in the first position. This leaves 7 books for 7 positions.
No. of permutations = 7!
(b) Consider the two books tied together as if one book. Then there are 7 books which can be arranged
in 7! ways. But of the two books tied together either could come first.
Total permutations = 7! x 2.
For calculating probability note that a random arrangement of the 8 books could be done in 8! ways.
7
Probability of Condition (a) = ; = l
8 8 0

7 2
ProbabilityofCondition(b) = ' 8~ =~
Note that the probability for condition (a) could have been quickly calculated just by noticing that
any one of the 8 books, could be first.
Example (iii):
How many numbers greater than 500 can be made with the digits 1, 4, 6 and 9 if each may be used
only once?
Solution:
Any 4 digit number will exceed 500 and so will 3 digit numbers beginning with 6 or 9.
Permutations with 4 digit numbers = 4 P4 = 24.
For 3 digit numbers.
The first digit may be 6 or 9 (i.e. 2 ways)
Any of the 3 digits left may be second (i.e. 3 ways)
Any of the two remaining digits may be last (i.e. 2 ways)
Permutations of 3 digit numbers = 2 x 3 x 2 = 12
Total permutations = 24 + 12 = 36.

1. Calculate (a) 6 P 3 (b) 8P 2 (c) 5P 5 (d) 5 P 1 •


2. In how many ways can 6 objects be arranged (a) in a line (b) in a circle if only the order matters?
3. A man has 5 flags. In how many ways can he fly them one above the other?
4. A typist has 6 letters and 6 envelopes. In how many ways can the letters be placed one in each
envelope? If she has forgotten which is which what is the probability of being correct?
5. A man has 6 history books and 4 geography books. He wants the history books to come first
on the shelf. In how many ways can the history books be arranged? What about the geography
books? How many total arrangements are there?
6. A punter is fairly sure (aren't they all!) that he can choose the best 3 horses in a race. But he
doesn't know which actually will be first, second, or third. In how many ways can the three horses
concerned be arranged as first, second, and third?
7. The punter above wants to cover all the possible ways that the three horses can fill the first three
places. How many bets must be made to 'box', as they say, the 3 horses?
8. How many bets must be made to 'box' 4 horses (i.e. so that all arrangements are covered for them
to be first, second or third in a race)?
9. In how many ways can 8 boys be arranged if the tallest is first and the shortest last?
10. In how many ways can the letters in the word HOUSE be arranged? In how many of these do
the consonants come at the two ends? In how many arrangements are the consonants together?
248
11. What is the probability that in a random arrangement of the letters of the word MEALS
(a) the consonants will come together?
(b) the vowels will come at the ends?
12. In a random arrangement of the letters of the word NUMBER what is the probability that the
two vowels will come together?
13. In how many ways can 5 mathematics books and 3 science books be arranged on a shelf so that
the books of each subject come together?
14. In how many ways can 5 English books, 3 history books and 2 geography books be arranged
on a shelf (a) with no restrictions? (b) if the books of each subject are kept together? (c) if one
geography book is at one end, the other geography book is at the other end and the books on
each of the other subjects are kept together?
15. How many 5 digit odd numbers can be made from the digits 3, 4, 5, 6 and 7 if each digit is used
once only in a number?
Hint: Fill the last place first.
16. How many numbers between 6000 and 7000 can be made from the digits 5, 6, 7, and 8 if each
digit is to be used once only in any number?
17. How many odd numbers greater than 8000 can be made from the digits 5, 6, 7 and 9? Each digit
is to be used once only in a number.
18. How many numbers less than 5000 can be made from the digits 3, 4, 5 and 6 if no digit is to be
used more than once in any number? (N.B. 1, 2, 3 or 4 digit numbers are allowed.)
19. How many numbers divisible by 5 can be made with the digits 2, 3, 4 or 5 if each digit is not
used more tha'n once?

Example (i):
In how many ways can the letters of the word
(a) HEAR (b) HERE be arranged?
Solution:
(a) There are 4 letters, all different.
Number of permutations = 4 P4 = 4! = 24
(b) If all the permutations of the word hear were written out there would be pairs of arrangements
such as eahr and aehr. In such a pair if the a was changed to a second e the two permutations
would appear the same. It is not difficult to see that having two letters e which are the same simply
halves the number of permutations. Write them out in pairs and check if you wish.
4P 41
Permutations of here = - 4 = _:_ = 12
2! 2!
Notice that although it was said the permutations were halved we have divided by 2! (which is 2).
The point at issue is that the two letters can be arranged among themselves in 2! ways.
Example (ii):
How many arrangements are possible with letters of the word EERIE?
Solution:
Sp 51
Permutations = 3f = j = 20
3
N.B. If the letters e were written as el> e 2 and e 3 there would be 5! permutations. But these 3letters
can be arranged in 3! ways so the number of distinctly different arrangements is reduced.
For the following example remember that in a rowing eight (no cox) four rowers are on the stroke
side and four are on the bow side.
249
Example (iii):
(a) In how many ways can eight boys sit in an eight-oared boat if 3 of the crew can only row on the
stroke side and one particular boy must be on the bow side?
(b) In how many ways can 8 boys sit in this boat without restriction and hence what is the probability
that if the boys sat at random the conditions of part (a) would be met?
Solution:
(a) Firstly, seat the 3 boys who must be qn the stroke side. There are 4 available places.
Number of arrangements = 4 P3 = 24 ways.
Secondly, seat the one boy on the bow side. There are 4 available places.
Number of arrangements = 4 P 1 = 4 ways.
Now seat the remaining 4 boys in the remaining 4 seats. There are no restrictions.
Number of arrangements = 4 P4 = 24 ways.
As any arrangement can exist with any other.
Total permutations = 24 x 4 x 24
= 2304.
(b) Permutations without restriction = 8!
= 40320
P ro b a b1 1ty o meetmg restnctwn = 2304
'l' f . . .
40 320
2
35
= 0·057 (approx.)
N.B. It can be advantageous to leave numerator and denominator in factored form and cancel.

1. In how many ways can the letters of the word READER be arranged?
2. How many permutations are there of the letters in ARRANGE?
3. In how many ways can the letters in THEREAFTER be arranged?
4. In how many ways can 4 red counters, 3 blue counters and a white one be arranged in line?
5. In Morse code how many arrangements are there of 3 dots and 2 dashes?
6. How many car registration plates have the prefix letters AAB in any order and the digits 667
in any order?
7. If the letters m, e, t and e are used then the words meet, mete, and teem can be made. Can you
think of any other meaningful arrangements? If these 4 letters are written in random order what
is the probability of
(a) a meaningful word (b) a nonsense syllable?
8. What is the probability that a car registration plate chosen at random will have a prefix with
the letters X, X, Yin any order, assuming all plates have a 3 letter prefix?
9. In a rowing eight four of the crew can only row on the bow side. How many arrangements are
possible for the crew?
10. In a 4 oared boat (2 on each side) one particular boy wants to sit on the stroke side. What is the
probability that this would occur in random seating?
11. In a railway car, with 3 seats facing the engine and 3 seats with their back to the engine, in how
many ways can 6 people 'be seated if two of them insist on facing the engine?
12. If an ape typed the letters S, E, E and Min any order what is the probability that he would type
the word SEEM?
250
13. An ape uses a simplified typewriter with only the letters A, Band Con it. What is the probability
that in typing 3 letters he would type
(a) AAA? (c) AAB in any order?
(b) all letters the same? (d) two letters the same?

Sometimes we are faced with the task merely of making a selection from given objects and not with
the question of how the selected group is arranged. For example, we may be choosing a pair to play
tennis without any concern as to who will play on the forehand court. In horse racing, the quinella
refers to choosing the first two horses without regard to order, unlike the trifecta where order is
important. To win games of chance like Lotto it does not matter in which order the numbers arise
but only that the correct group has been selected. In all these cases we talk about a combination.

When order is important a selection is called a permutation


when order is not important a selection is called a combination.

Study the example.


Example:
How many pairs of tennis players can be chosen from 4 available players?
Solution:
Label the players A, B, C and D.
If we were seeking permutations the answer would be 4 P2 , or 12. Let us list them:
AB, AC, AD, BA, BC, BD, CA, CB, CD, DA, DB,· DC.
If it was important who played on the forehand court then AB and BA are different arrangements.
But AB and BA are the same combination. So are AC and CA. In fact, the permutations come in
pairs because for each combination (group) there are two arrangements (permutations).
In this case there are just half as many combinations as permutations.
. . 4p2 12 6
Num b ero f comb matwns = T = = .
2
In generalr things can be arranged among themselves in r! ways. Thus:
. . f h' . f "Pr n!
Com bmatwns o n t mgs m groups o r = --;T = ,. !(n _ r)!
The notation used for the number of combinations of n things taken rat a time is "Cr. An alternative
notation sometimes used is (~).

Example (i):
Seven people are available to play tennis. A group of 4 is to be chosen to play. In how many ways
can this be done?
Solution:
Number of ways= 7 C4
7!
4!3!
7 X 6 X 5 X 4 X 3 X 2 X
1 X 2 X 3 X 4 X 1 X 2 X 3
= 35.
251
Alternatively,
Number of permutations= 7P 4 = 7 x 6 x 5 x 4
. . Permutations
Number of combmatwns = !
4
7 X 6 X 5 X 4
l X 2 X 3 X 4
= 35.
Either way there are less combinations than permutations as the 4 players could have been arranged
among themselves in 4! ways.
Example (ii):
In how many ways can a committee of 3 men and 2 women be chosen from 6 men and 5 women?
Solution:
The 3 men can be chosen from 6 men in 6 C3 ways. The 2 women can be chosen from 5 women in 5 C2
ways. Each choice of the group of men can be matched with any of the choices of women.
Total Combinations = 6 C3 X 5 C2
6! 5!
---x---
3!x3! 3!x2!
= 200.

1. In how many ways can a basketball team of 5 players be chosen from 8 available players?
7
2. C3 means the number of combinations of 7 things 3 at a time. Find its value.
8 1 9 4
3. Find (a) C2 (b) °C3 (c) C4 (d) C3.
4. Compare 6 C4 with 6 C2. Also compare 5 C2 with 5 C3.
5. In how many ways can a cricket team of 11 players be chosen from 13 available players?
6. The selectors for a soccer team name a squad of 12 players with one to be omitted. In how many
ways can the team of 11 be formed? Can you see the easier way to do this question? (Refer to
Question 4.)
7. Prove that "C, = "C,_,. Give a verbal explanation of why this is so.
8. Using the formula find "C,. Remember that 0! is defined to be 1. Give a verbal explanation of
what "C, means and hence reason that the derived answer must be correct.
9. A girl wants to invite 9 friends to tea but only has room for 5. In how many ways can she make
the choice?
10. In how many ways can a group of 4 cards be chosen from 52 cards? Using this answer state the
probability of being dealt 4 aces if you are given just 4 cards?
11. In how many ways can you choose 13 cards from a 52 card pack? What then is the probability
that you will get a particular selection such as 13 hearts?
12. In how many ways can 4 books be chosen from 7 books?
13. In how many ways can a committee of 3 liberal and 4 labor MPs be chosen from 8 MPs of each
party?
14. In how many ways can a committee of 5 women and 2 men be chosen from 6 women and 8 men
available for selection.
252
Many games of chance are based on combinations. For example, in the NSW Lotto six numbers
have to be chosen from the 40 available numbers. There are 4 °C6 selections of 6 from 40 and this
gives almost 4 million possible selections. If you know the number of games a person has purchased
(usually in batches of four) then you can calculate the probability for them winning, as all selections
are equally likely.
The theory of combinations also occurs in card games and horse racing. In poker, one can calculate
the probability of being dealt, say, four aces in a hand of five cards by considering the number of
combinations possible. In horse racing the quinella consists of choosing the first two horses in a
race as a group and in either order. Notice that il1 Lotto and card games the selections are equally
likely. However, in horse racing the events are not equally likely and one needs to be cautious in
speaking of probabilities. ·
The questions below expand on these matters. They are not included to teach gambling but merely
to illustrate the mathematics of chance. Actually knowing the mathematics might discourage one
from gambling.

1. If all horses are equal in ability what is the probability of winning the quinella prize in (a) an
8 horse race? (b) a 15 horse race?
2. A punter believes that only 5 horses in a 14 horse race really have any chance of winning or coming
second. How many possible quinella pairings of these horses are there? How many bets must
be made to cover all these ways?
N.B. Whilst this increases the chance of winning, the money won may be less than the amount
expended.
3. In a simple game of Lotto 4 numbers have to be chosen from 10 numbers in a barrel. Show that
the number of combinations is 1 °C4 and evaluate it.
4. Four numbers have to be chosen from 20 numbers. What is the probability that one given entry
will be the correct one?
5. In the NSW Lotto 6 numbers have to be chosen from 40 numbers. In how many ways can this
be done?
6. If a person submits 4 selections for NSW Lotto what is the probability that he will have the
correct solution?
7. If a million people submit 4 entries each to NSW Lotto (see Question 5) about how many correct
solutions would you expect from these entries?
8. In a game of poker the players are dealt a hand of 5 cards. How many such hands are there from
a 52 card pack?
9. In poker, a royal flush contains the ace, king, queen, jack and 10 of one suit. How many royal
flushes are there? By comparing this with the number of possible hands calculate the probability
of being dealt a royal flush in a 5 card hand.
10. If a person played 10 games of poker every week for 50 years show that there is still a low proba-
bility to ever be dealt a royal flush.
11. A straight flush in poker is 5 consecutive cards of the same suit. Excluding royal flushes, how
many are there? What is the probability of being dealt one?
12. In how many ways can you be dealt 4 aces and one other card in a hand of 5 cards? What is the
probability of this occurring. (See Question 8.)
253
13. In how many ways can you be dealt 3 aces (from the available 4) and 2 other cards (from the
available 48 non-ace cards)?
14. In how many ways can you be dealt 3 kings and 2 other cards in a 5 card hand? In how many
ways can you be dealt any three the same (i.e. 3 aces or 3 kings etc.) with 2 other cards? Comparing
this answer with the total number of ways that a 5 card hand can be selected state the probability
of being dealt 3 the same.
N.B. Your answer includes the so-called 3 of a kind, where the other 2 cards are not alike, and
the so-called full house, where the other 2 cards match.
15. In the Australian Soccer Pools 6 numbers must be chosen from 36. In how many ways can this
be done?
16. What is the probability that one given entry will win the pools, assuming that all numbers were
equally likely?
17. A person buys 200 tickets in the pools. What is the chance of success, assuming all numbers
are equally likely?
18. NSW Lotto tickets cost 25 cents each or $1·00 for four. How many tickets can be purchased
for $20 and what is the probability of winning? Neglect agent's fees and use the results of earlier
questions.
19. Whilst one way to increase chances of winning Lotto or Pools is simply to buy more tickets
another way is to make a so-called systems entry. In a System 7 entry a person crosses off 7 num-
bers rather than 6. Show that this is equivalent to buying 7 games.
20. How many groups of 6 can be selected from 8 numbers? If in Lotto or Pools a person has a
System 8 ticket which allows 8 numbers to be crossed off how many choices of 6 numbers is
this equivalent to? By what factor are the chances of winning increased?
21. A normal ticket in the Pools costs 50 cents. A System 9 ticket costs $42·00. By finding out how
many choices of 6 this gives, state if the price appears correctly judged.
22. NSW Lotto advertises that a System 10 ticket is equivalent to 210 games. Is this so? What is the
probability of winning Lotto with such a ticket? (use results of Questions 5 and 6).

Example (i) :
In choosing 3 letters from the word HEATING (and assuming all choices are equally likely) what
is the probability of choosing
(a) 1 vowel (b) 2 vowels (c) at least 2 vowels.
Solution:
(a) If the chosen group has one vowel then it also has two consonants.
The vowels can be chosen in 3 C1 i.e. 3 ways
The consonants can be chosen in 4 C2 i.e. 6 ways.
The total choice can be made in 18 ways.
However, any 3 letters can be chosen from ?letters in 7C3 i.e. 35 ways.
p b bTt f selections with one vowel
ro a 1 1 y 0 one vowe 1 = total selections
18
35'
(b) The 2 vowels can be chosen in 3 C2 i.e. 3 ways. The remaining consonant can be chosen in 4 ways.
Thus 2 vowel words can be chosen in 12 ways.
Probability of two vowels = ~~.
254
(c) If there are to be at least 2 vowels then there may be 2 vowels or 3 vowels.
A 3 vowel word can be chosen in just one way.
As seen in (b) a 2 vowel word can be chosen in 12 ways.
Hence probability of at least 2 vowels = ~~.
N.B. A word with no vowels can be chosen from the 4 consonants in 4 C3 or 4 ways. The probability
for a word with no vowels is
3~. Notice how the probabilities of words with 3 vowels, 2 vowels,
1 vowel and no vowels all add to unity because this exhausts all possibilities. Also notice how
the answer to part (c) could have been obtained by subtracting the probabilities of words of
1 vowel or no vowels from unity.

Example (ii):
A committee of 4 women and 2 men has to be selected from 7 women and 5 men. It is known that
Miss Smith will not serve with Mr Brown, both of whom are available. What is the probability that:
(a) Miss Smith and Mr Brown will both be selected?
(b) One only will be selected ?
(c) Neither will be selected?
(d) Miss Smith and Mr Brown do not have to serve together?
Solution:
Total number of possible committees = 7 C4 x sc2
= 35 X 10
= 350.
(a) If Miss Smith and Mr Brown are selected then that settles two places. We need to select 3 more
women from 6 women and one more man from 4 men.
No. of combinations = 6 C3 x 4 C1
= 20 X 4
= 80
Probability = 38s00 = 38s.
(c) Treating part (c) first we notice that if neither is included it means choosing 4 women from 6 left
and 2 men from 4.
No. of Combinations = 6 C4 x 4 C2
= 15 X 6
= 90
Probability = is00 = is.
(b) This is a complementary event to (a) and (c).
Probability = 1 - 38s - is
= ~~-
Of course, this part could have been done the longer way by finding committees with Miss Smith and
without Mr Brown and with Mr Brown and without Miss Smith and then adding.
(d) If Mr Brown and Miss Smith do not serve together then neither serves or one only serves. By
adding the results of parts (b) and (c) we have
P ro b a b1"l"1ty = 18 + 9
35 35
27
35"
Alternatively, if Miss Smith and Mr Brown do not serve together the probability is given by
Probability = 1 - probability that they are both selected
= 1- 8
35
27
35"
255
1. In how many ways can a committee of 3 Year 12 students, 2 Year 11 students and 1 Year 10
student be chosen from 7 Year 12 students, 5 Year 11 students and 3 Year 10 students? What is
the probability that a particular Year 12 student and a particular Year 11 student are both on the
committee?
2. A detachment consists of 3 sergeants, 4 corporals and 20 privates. In how many ways can a guard
of 1 sergeant, 1 corporal and 3 privates be selected?
3. In how many ways can 4 people be selected to play mixed doubles (i.e. 2 men and 2 women)
from 6 married couples? How many ways can it be done if no husband and wife play in the same
match?
Hint: In the second part find the number of ways of choosing the women and note that in the
choice of the men certain of them will be excluded.
4. A committee of 6 is to be chosen from 8 men and 6 women so as to contain at least 3 men and
2 women. In how many ways can this be done? Also in how many ways can it be done if 2 par-
ticular men refuse to serve together?
5. A poker hand consists of 5 cards from the 52 card pack.
(a) How many selections (hands) are possible (leave unsimplified)?
(b) In how many ways can a hand of 5 black cards be chosen?
(c) What is the probability that a randomly dealt hand will consist of all black cards?
6. What is the probability that a hand of 5 cards will contain
(a) the 4 aces and one other card?
(b) 2 spades and 3 clubs?
(c) 2 jacks. (and three other cards which must not be jacks)
(d) 3 jacks.
(e) at least 2 jacks.
7. There are 4 red, 3 white and 2 blue balls in a bag. In how many ways can 3 balls be withdrawn
(a) if there is no restriction?
(b) if there are to be 2 red balls and one white ball?
(c) if there is to be one of each colour?
8. Find the probability of choosing 3 balls from the bag in Question 7 if (a) they are all to be red
(b) there are two red and one white (c) one is of each colour (d) none is red?

Often the hardest task is to be sure to distinguish whether a situation is a permutation where order
is important or a combination where order is not an issue. Exercise 30.8 is a mixed set. Decide first
which situation applies.

1. In how many ways can the first and second prize winners arise from a class of 25 pupils?
2. The principal wishes to choose a group of 6 prefects from 100 available pupils. In how many
ways can this be done (leave unsimplified)?
3. How many words, including nonsense syllables, can be made from the letters of KEEPER.
4. If a committee of 4 has to be chosen from 4 men and 4 women what is the probability that it
would consist of (a) 2 men and 2 women (b) all women, assuming all choices are equally
likely?
256
5. How many 4 digit numbers can be made by using once each the digits 1, 3, 4 and 5? In how many
of these will the number be even?
6. What is the probability of using each of the 4 digits 1, 3, 4, 5 once and making a 4 digit number
which is (a) even (b) divisible by 5?
7. There is room for just 6 books on a shelf. In how many ways can it be filled from 8 available and
dissimilar books if the order on the shelf is unimportant?
8. In how many ways can host and hostess and 4 visitors be arranged around a round table (a) if
there is no restriction (b) if host and hostess sit next to each other (c) if the hostess sits in a
particular seat?
9. In how many ways can a football15 be selected from 17 available players?
10. In how many ways can a football15 be chosen from 17 players if one particular player is included?
Hence or otherwise, what is the probability of a particular player being chosen when a team of
15 is chosen from 17 equally able players?
11. Normally 12 players are selected for a cricket test and then one is made twelfth man. Obviously
many factors determine who should be omitted but if it were equally likely for all players to be
named twelfth man, what is the probability of a given player being chosen?
12. See Question 11. Assuming the captain, vice-captain, and wicket keeper are certain to play
but all other players are equally likely to play what is the probability for each of the other players
to be named twelfth man?
13. How many even 4 digit numbers can be made from the digits 1, 3, 4 and 6 if (a) repetitions are
allowed (b) no repetitions are allowed?
14. In a simple game of Lotto 3 numbers have to be chosen from 10 available numbers. What is
the probability that a given choice of three numbers will be correct?
15. If 4 men and 2 ladies sit down in line what is the probability that a given lady, Miss A, will sit
next to a given man, Mr B, if all arrangements are equally likely?
16. Repeat Question 15 if the people were seated at a round table.
17. Prove that 7 C5 = 7 C2 and also that 11 C, = 11C11 _,.

18. Find ~~~~,leaving the answer unsimplified. Show that the answer is not 2 and that it is the same
II
211
as C11 •
19. Show that 8 C6 x 6! = 8 P6 . State what 8 C6 and 8 P6 mean in words and hence also give a verbal
explanation of the equation above.

The binomial theorem has a strong link with the theory of combinations and with probability.
Firstly, we draw attention to the fact that the coefficient of xk in the expansion of (a + xl was
earlier denoted as 11 Ck. In fact, it has the same formula as the number of ways of choosing n things
kat a time.
We do not wish to digress to prove this statement here and readers may follow it up. Suffice to
say that (a + x) 11 is really the multiplication of n factors each of which is (a + x). In forming terms
of the expansion either an a or an x will be taken from each parenthesis. In the case of the term con-
taining xk it will also contain a11 -k because if x has been chosen from k factors then a will be chosen
from the remaining (n - k) factors. And this choice can be done in 11Ck ways as the xis being taken
from k factors out of a total of n factors. That is, there are altogether n letters x and they are being
taken k at a time.
257
A reminder.
(a+ x)" =a"+ "C1 a"- 1 x + "C2 a"- 2 x 2 + · · · + "Cka"-kxk + · · · + x"
or (a+ x)" =a"+ na"- 1 x + n(n - l) a"- 2 x 2 + ...
1·2

+ n(n - 1) · · · (n - k + 1) -k k
a" x · · · + x".
1·2 ... k
-------------~---- -------~---------------------

1. Use the binomial theorem to expand (1 + x) 5 • Compare the coefficients with the number of
ways of choosing 5 objects either 1, 2, 3, 4 or 5 at a time.
7 7
2. Prove that 7 C 1 = 7 C6 , 7 C2 = C5 and 7 C3 = C4 . Expand (a+ x) 7 and show that the coefficients
form a symmetric pattern.
3. Prove that "C, = "C11 _,. What does this mean about the coefficients of (a + x)"?
4. Expand (1 + x) 6
in decreasing powers of x (i.e. beginning with x 6
).

5. Show that the coefficient of x 5 in (a+ x) 7 can be written as 7 C 5 a 2 •

There are many occasions where just two possible outcomes exist from an experiment or event. For
example, in tossing a coin there may be a head or a tail. In drawing a card from a pack or in throwing
a die one may achieve the desired objective (e.g. an ace, a 6) or one may fail. When manufacturing
objects they may meet specifications or they may be defective. In this section we look at probabilities
that arise in repeated events of this kind.
Study the examples below.
Example (i):
A die is thrown. What is the probability of achieving a 6 and of failing to do so? What is the proba-
bility of getting 0 sixes, 1 six, or 2 sixes from two independent throws.
Solution:
Probability
Probability of success = p = ~

<
success

Probability of failure= q = 1 - p = ~· success


p

< <
From the tree diagram q fail pq
Prob (6, 6) = p 2 = l 6
Prob (one 6) = 2pq = j~ success qp

Prob (no 6) = q 2 = (t) 2 = U.


fail
Notice that the final probabilities of p 2 ,
2pq, and q 2 are the terms in the binomial q tail
expansion of (p + q) 2 •
Example (ii):
For a given factory, 99% of items manufactured meet specifications and 1% are defective. Draw a
tree diagram showing the probabilities which exist if 3 items are chosen at random. Calculate the
probability that all items will be good or that 1, 2, or all 3 will be defective.
Solution:
Here p = 0·99 and q = 0·01.
Let G stand for good and D stand for defective.
258
1st 2nd 3rd Sample Probability
Item Item Item Points
p3

<::G<:
GGG

GGD {i'q

GOG {i'q
q D< G
q D GOD prf

D<G<:
DGG {i'q

DGD pcf

DOG pcf
q D< G
q D DOD if

From the tree diagram we notice


P (all items good) = p 3 = (0·99) 3
P (1 item defective) = 3p 2 q = 3 x (0·99) 2 x-(0·01)
P (2 items defective) = 3pq 2 = 3 x (0·99) x (0·01) 2
P (all items defective) = q3 = (0·01) 3 .
Notice that the termsp 3 , 3p 2 q, 3pq 2 and q3 arise from the binomial expansion of(p + q) 3 .
Example (iii):
Copy the tree diagram above and add a fourth column. What are the respective probabilities for 4
successes, 3 successes, etc., in 4 independent trials where the probability of success of each event is
p and of failure is q.
Solution:
Probabilities respectively are p 4 , 4p 3 q, 6p 2 q 2 , 4pq 3 and q 4 . Notice the link to (p + q) 4 .

If there exists a series of repeated independent events where the probability of success is p and
of failure is q then the probability of various successes in n trials is given by the terms in the
expansion of (p + q)".
Such events are called Bernouilli trials (after a famous mathematician) and form what is called
a binomial distribution.
In symbols
P (x = r) = "C,p'q"-r where P (x = r) means the probability that there will be r successes in
then events.

1. A coin is tossed 4 times. By considering (p + q) 4 where p = !


and q = ! find the probability of
(a) 3 heads (b) 2 heads. Also draw a tree diagram and check your answer.
2. Consider a series of 4 attempts to throw a six with a die. What is the probability of obtaining
(a) one six? (b) two sixes? (c) no sixes?
3. Three red balls and two blue balls are in a hat. Find the probability of getting (a) 2 red balls
(b) at least 2 red balls, from 3 independent attempts (i.e. with replacement each time).
4. A card is chosen from a 52 card pack and replaced. This is repeated to give 3 trials. What is the
probability of obtaining (a) three aces? (b) one ace?
5. Ten per cent of the time I try to drive a nail in straight I fail. If I try to drive in 10 nails what
is the probability that (a) only one will bend? (b) none will bend?
259
6. What is the probability of 5 heads in 6 tosses of a fair coin?
7. What is the probability of 3 sixes in 5 throws of a die?
8. What is the probability of nailing at least 4 nails straight out of 5 nails if each time the probability
of success is 0·9?
9. A machine manufacturing bolts is known to produce on average one defective bolt per hundred.
What is the probability that just one defective bolt will be made in the manufacture of 10 such
bolts?
10. For the machine in Question 9 what is the probability of
(a) no defective bolts at all in 10 made?
(b) no more than one defective bolt in 10 made?
11. A particular batsman tends to make one scoring stroke every 3 balls. What is the probability
that he will make 2 scoring strokes in a given 6 ball over? In 100 overs how many are likely to
be overs which have two scoring strokes in them?
12. What is the probability of getting at least one six in 4 throws of a die?
13. If there was a prize for anyone who could throw at least one six in 4 throws of a die how many
would win the prize (on average) out of 100 starters?
N.B. This was a very famous gambling situation set up by a Frenchman, Chevalier De Mere.
He made money because he found in practice that he could achieve this slightly more than
half the time. Does your calculation agree?
14. A biassed coin has at probability of landing heads. If the coin is tossed 5 times which is the most
likely event to happen, 5 heads, 4 heads, or 3 heads? Justify your answer.
15. In a family of 4 find the probability that there are (a) all girls (b) 3 girls and 1 boy (c) 2 girls
and 2 boys (d) at least 3 girls. Assume p = ±exactly.
16. Suppose an X-ray picture has a probability of0·9 of showing a certain fracture. If 3 X-ray pictures
are taken at different angles what is the probability that
(a) the fracture shows up in all three pictures?
(b) the fracture remains undetected?
(c) the fracture is detected?

260
1. Simplify (x + 3)(x - 4) - x(x - 1). 21. ABC is a triangle in which B = 135°,
A = 30° and b = 10 em; find a in surd
2. Find the factors: form. ·
(a) x 2 - x - 2
22. Find the equation of a straight line which:
3. Simplify 4x: - 4 x + 1. (a) passes through (- 3, 5) and has a
2x - 4x +2 gradient of i.
(b) is perpendicular to 3x - 2y - 7 = 0
4. If PV = pv(l + 1 ~ 0 } find t if P = 7, and passes through (2, -1).
v = 140, p = 6, v = 130. 23. In a throw with two dice, find the probabil-
ity of scoring (a) either 7 or 11
ove x-5+x+5_
5. Sl - 5- - 5. (b) more than 6.
10
6. If9x = find x. 24. In a family of three children, what is
2\ ,
the probability of there being (a) 3 boys?
2
x 2 - 5x (b) at least 2 boys?
7. Simplify x - ;
x- 3- X •
. m
25 . S1 +y2 - -X- -+-3.
l'f pX 1
8. Find the factors: x+3 x+2
(a) 10 + 3x - x 2
(b) x 2 + 2x - ax - 2a. 26. Simplify (J'? - 2J'3)(J7 + 2J3).
9. Simplify 3J2 + 4J8- J32. 27. Simplify a+* when a= J5- 1.
28. Solve 3x - y = - 5 }
10. Express J33 2- 1
with a rational denom- y = x 2 - 2x- 1
ina tor. 29. Solve lx + 121 = l2x + 61.
11. If 2 = and 3 =
m 0 ' 623 express as m 0 ' 847 ,
30. Solve 2x- 1 I < 4.
a power of m the number (a) 6 (b) 9. 3
1

12. If (2J5 - 3JT0) 2 = a + bJ2, find the 31. In 6ABC, AB = 17 em, AD = 15 em


value of a and b. and LDAB = 132°. Find the area of
13. Solve 3x + 7y = 27} 6ABD.
5x + 2y = 16 32. Find the 30th term of the sequence
14. Solve l3x - II = 5. 6, 9, 12, 15, ...

15. Given m = (J'3-


J2) 2 + J24, find m 33. In an arithmetic sequence T7 = 23 and
in its simplest form. T13 = 41, find the first term and the
b2 common difference.
16. Use the formula e 2 = 1 - 2 to find a
a 34. For the arithmetic sequence
when e = 0·29 and b = 5. 5, 12, 19, 26, ... , find T50 and S 50 .
17. Solve 13 - 2xl < 6. 35. Which term of the geometric sequence
18. Solve (2x- 1)(x + 4) < 0. 3, 6, 12, ... is equal to 768?

19. If tan 0 = f and 0 is acute, find cos 0.


36. If x, x + 4, x + 16, are in geometric se-
quence, find x and the common ratio.
30
20. In 6ABC, a = 7, b = 6, c = 5, find the 37. Evaluate L: (4n - 1).
largest angle of the triangle. n=l

261
12
57. Find the equation of the normal to the
38. Evaluate L5 x 2"- 1 .
curve y = 4x - x 2 at the point where
n=l
x=l.
39. Find the limiting sum of the geometric
series in which a = 25 and r = 0·2. 58. If x 2 + 4x = ax(x + 1) + b(x + 1) + c,
find a, b and c.
40. Solve 2x 2 + 3x - 14 = 0.
59. Find the value of k for which
41. Find the roots in surd form for
x 2 + (k + 1)x + k = 0 has equal roots.
x 2 + 1 = 6x.
60. The sum of a number and its square is
42. If 3 is a root of the equation 132. Find the number.
x 2 + kx - 6 = 0, find the value of k.
61. Find the limiting sum of the series
43. The roots of the equation
16 + 8 + 4 + 2 + ...
2x 2 - 7x + 12 = 0 are o: and /3. Find
(a) o: + f3 (b) o:/3 (c) +
(d) ();2 + {32
t * 62. Express o-37 as a geometric series and
find its limiting sum.
44. Solve for x if 2x 4 - 26x 2 + 72 = 0. 63. At what points on the curve y = ~l
45. Find the value of k for which
x+
is the tangent parallel to the line y = x?
3x 2 - 5x + k = 0 has equal roots.
46. Show that 2x 2 - 3x + 6 is positive for
64. If f(x) = 2x 3
- 4x + 5, find /(2), /'(2),
/"(2).
all values of x.
65. What is the second derivative of(3x - 2) 4 ?
47. For what values of x is (3x - 1)(2 - x)
positive? 66. For what values of x is f(x) increasing if
f(x) = 2x 3 + 6x 2 - 18x + 3?
48. If 2x 2 - 9x + 1 =Ax 2
+ Ex + C, find
A, Band C. 67. Find the centre and radius of the circle
x 2 + y - 4x + 6y + 9 = 0.
2
49. Express 4x 2 - 6x + 5 in the form
A(x - 1) 2 + B(x - 1) + C. 68. Find the primitive of x 2 - 3x + 1.
50. Find the value of k if the roots of the 69. lfj'(x) = 3x 2
- 2x + 1, findf(x).
equation 3x 2 - 15x + k = 0 differ by 3.
dy 1
70. - = 2 + 3, find y.
dx x
51. Find: if:
71. The gradient function of a curve is 7 - 4x
(a) y = 2x + 23 (b) y = (x 2 + 3) 4 and the curve passes through the point
X
(1, 10). Find its equation.
3x
(c) y = 2x - 1 72. What is the focal length of the parabola
x 2 = 20y?
52. Differentiate:
(a) (x - 3)(2x 2 - 2x + 4) 73. Find the equation of the parabola with
(b) )x 2 - 6 focus at (0, 3) and directrix y = -3.

5 3 74. Find the minimum value of2x 2 - 6x + 7.


53. Findf'(x) ifj(x)

54. Find the gradient of the curve


= x
X
; x.
75. Evaluate l 3
(2x 2 - x + l)dx.

y = x 2 (x - 2) at the points where x


and where x = -2.
= 1
76. Evaluate t4
(2x - 1) 2 dx.

55. The curve y = 2x 2 + bx + 8 has a gradi- 77. Find the area under the curve y = x 2 +2
ent of 3 when x = 2. Find b. from x = 1 to x = 3.
56. Find the equation of the tangent to the 78. Find the point of inflexion on the curve
curve y = x 3 - 4x 2 at the point (1, - 3). y = 3x 3 - 3x2 + 3x - 6.
262
79. Find the domain for which the curve 95. Differentiate with respect to x:
y = 4x 2 - x 3 is concave upwards. (a) 3x 2 - 4e 2 x (b) loge 5x
(c) ex loge X
80. For the curve y = 2x 3 - 12x 2 - 5x - 3,
find the equation of the inflexional tangent. 96. Find the length of the arc of a circle of
radius 15 em which subtends an angle 29"
81. Find the stationary points on the curve radians at the centre.
y = x 3 + x 2 + 1 and distinguish between
them. 97. Find the area of the sector in a circle of
radius 20 em where the angle at the centre
82. Given that ~; = 3t 2 - t and that s = 11 is 1·86 radians.

when t = 2, express sin terms oft. 98. A segment of a circle has an area of 50 cm 2
and subtends a central angle of 3" radians.
83. The gradient of a curve is given by 1x = Find the radius of the circle correct to one
decimal place.
1 + 4x - x 2 . What is the equation of the
curve if it passes through the point (3, 0)? 99. Draw sketch graphs of:
(a) y = 4 cos x (b) y = 2 sin 2x
84. Find the equation of the curve given
2 100. Differentiate:
d y = 3x - 1 and when x = 2 dy = 7 (a) sin 3x (b) tan(2x- 1)
~2 '~
(c) 3 cos(~ - x)
andy= 3.
85. Evaluate: 101. Find 1x if y = ecosx.

(a) f 1
5
3dx (b)
f
l

0
3

x 2dx
102. Find the derivative of 2x tan x.
86. Find the area enclosed by the curve 103. Find the primitive ofsin(4x + 2).
y = 6x - x 2 and the x axis.
87. Find the indefinite integrals 104. Evaluate II(2- cos x)dx.

(a) f (6x 2
- : 3 )dx (b) f (3x + 2) 2
dx 105. Find the area under the curve y = 2 sin x
between x = 0 and x = ~·
88. Find f )6x + 1 dx. 106. Show that the derivative of loge cos x is
-tan x.
89. Find the area enclosed by the curve
y = x(x + 2), the x axis and the ordinates A particle moves in a straight line and its
at x = - 1 and x = 1. displacement from the origin is given by
s = 5 - 6t + t 2 , find:
90. Find the area bounded by the curves
y = 3x - x 2 andy = 3 - x. 107. the distance of the particle from the origin
after 2 seconds.
91. Using the trapezoidal rule and three
sub-intervals, find an approximation for 108. the times when the particle is at the origin.

r4 2xdx. 109. the velocity of the particle when t = 5.


110. at what instant the velocity is zero.
92. Using Simpson's Rule with two equal
sub-intervals find an approximation for 111. the acceleration of the particle.
I 3
Iogexdx. 112. The velocity of a particle is given by
3t 2 + 2. If the initial displacement is 10
93. Find /"(4) if f(x) = Iogex. metres from the origin, find the displace-
ment after 4 seconds.
94. Find the volume of revolution if the area
below the graph of y = f between x = 1 113. If .X= -2 and .X= 10 when t = 1, find
and x = 3 is rotated about the x axis. the velocity function.
263
2 20
114. If d 2s
dt
= 3t - 4 find s given that when
'
129. Evaluate I (10- 2n).
u=3
ds 130. How many terms of the geometric series
t = O, s = 0 and dt = 0.
1 + 3 + 9 + 27 + ... are needed to give
115. The population of a town grows according a sum of 3280?
to the formula N = N 0 ek'. If the popula- 131. For a geometric sequence T3 = 18 and
tion grows from 40 000 to 50 000 in 2 years, T6 = 144. Find the first term and the com-
find k and the population after a further mon ratio.
6 years. Give answer to nearest thousand.
132. Insert 3 numbers between 21 and 45 so
116. In a certain vehicle the cost $C of making that the five numbers form an arithmetic
392 sequence.
a specified trip is given by C = 2V + V,
where Vis the average velocity in metres 133. Find the equation of the straight line
per second. Find the most economical which passes through the point (1, 3) and
average velocity. through the point of intersection of the
lines x - 4y + 2 = 0 and 3x + y + 6 = 0.
117. Find the gradient of the tangent to the
3 134. Prove that the line 3x + 4y = 30 is a
curve y = x - at the point (1, -1). tangent to the circle with centre at the
X + 1
origin and radius 6 units.
118. Investigate the nature of the turning points
at each of the stationary values for the 135. Find the perpendicular distance from the
point (2, - 1) to the straight line
curve y = x 3 - 3x 2 .
5x - 12y + 4 = 0.
119. Find values of k for which 5x 2 - 3x +k
is positive definite. 136. Find the locus of a point P(x, y) which
remains equidistant from A(3, 0) and
120. If3x 2 + x + 1 and B( -1, 4).
a(x - l)(x + 2) + b(x + 1) + c
137. Find the equation of the parabola with
are equal for all values of x, find a, band c.
focus (0, 6) and vertex (0, 0).
121. Find the point on the curve
138. Find the coordinates of the focus of the
y = 3 + 4x - x 2 where the tangent IS
parabola x 2 = - 12y + 24.
perpendicular to y = %+ 1.
139. Find the locus of a point P(x, y) which
122. Find the value of k in the equation
moves so that its distance from the fixed
kx 2 - 3x + 2k = 0 if twice the sum of
point (0, 4) is always equal to its perpendic-
its roots is equal to their product.
ular distance from the fixed line y = -4.
123. Show that the expression 2x - 5 - 3x 2 is
140. Differentiate:
negative definite. (a) y = e4x-1 (b) y = x 2 tan 2x
124. If the minimum value of the expression
141. Find the indefinite integrals:
3x 2 + 6x + k is -2, find the value of k.
125. The parabola y = 7 - kx - 2x 2 is sym-
(a) f 3xdx- 2
(b) f e~x dx
metrical about the line x = - 2. Find the
142. Sketchy = cos 2x for 0 ~ x ~ nand find
value of k and the maximum value of the
the area under the curve between 0 and !f.
expression.
1
126. Solve 4x - 18(2x) + 32 = 0. 143. Evaluate { sin nxdx.
127. For what range of values for y will the
144. In Lo.ABC, D and E are point on the sides
series y + y 2 + y 3 + · · · + y" have a
AB and AC such that BD = CE and
limiting sum? Find the limiting sum if
LB = LC = 70°.
y=-i.
(a) Find LA.
128. If the limiting sum of the geometric series (b) Find LADE.
1 + x + x 2 + x 3 + ... is 20, find x. (c) Show that DE is parallel to BC.
264
145. ABC is a triangle right-angled at A. AD
is perpendicular to BC. P is a point on 160. Find x if cos x = f and 0 ~ x ~ 2n.
interval ED such that CP = CA. If
LAPC = 80° show that P A bisects 161. For what value of k does
LEAD. 2x 3 + kx 2 - 2x + 16 give a remainder of
I when divided by x + 5?
146. ~ABC is right-angled at C and AB =
162. Show that x 3 + x - 1 = 0 has a root be-
8 em, AC = 3 em. Find the length of the
tween 0 and 1 and use Newton's Method
median AD.
to find the root correct to 2 decimal places.
147. Find the number of degrees in each angle
163. Find the size of the acute angle between
of a regular decagon (10 sides).
the lines 4x - 3y + 1 = 0 and
148. How many sides has the regular polygon X+ 4y + 1 = 0.
each of whose angles is 156 degrees?
164. Show that the equation
149. Show that the angle formed by the bi- sin 2 x - cos x = 0 has one root in the
sectors of two adjacent angles of a quad- domain 0 ~ x ~ n and find its value cor-
rilateral is equal to half the sum of the rect to 2 decimal places.
remaining angles.
165. Find dA when A = nr 2 , r = 5 and
150. In the quadrilateral ABCD, ADIIBC, the dt
diagonal A C bisects LA and ED bisects ~· = 0·1.
LD. Prove that AB = BC = CD.
151. The roots of the equation 166. A sphere is expanding so that its surface
x 2 5x + 10 = 0 are a and {3. Find the is increasing at the rate of 0·05 cm 2 js. Find
values of the rate of increase of the volume, at the
(a) a2 + {32 (b) a2{3 + af32 instant when its radius is 10 em.
(c) a3 + {33. 167. Find the volume of revolution when the
152. In how many different ways can 2 boys .J
portion of the curve x = 1 - y 2 between
and 3 girls be chosen from a group of y =!andy= 1 isrotatedabouttheyaxis.
6 boys and 9 girls? 168. In how many ways can seven boys
A, B, C, ... be arranged in a row so that
153. Expand (x + ~r in descending powers A and B never sit next to each other?
of x. 169. If two circles touch each other, show that
the line joining their centres passes through
154. Prove that 3x - 2x 2 - 2 is negative for the point of contact.
all real values of x.
170. Two circles touch internally at X, and a
155. Given that a, f3 andy are the roots of the line XYZ is drawn to meet the circles at
equation x 3 - 2x 2 + 4x + 7 = 0, find Y and Z. Prove that the radii to the points
the value of (a + 1)({3 + l)(y + 1). Y and Z are parallel.
156. Find the values of a, band c if, 171. If lrl
< 1, find the sum to infinity of the
n3 =a(n - l)n(n + 1) + b(n - 1) + c. series a + ar + ar 2 + · · ·. Hence express
157. Find the area bounded by the curve the recurring decimal4·2S as a fraction.
y = (x - l)(x - 3) and the x axis. 172. From 6 Australians and 5 New Zealanders
a committee of 4 is to be formed. In how
158. A body starts from rest and after t seconds
many ways can this be done if the com-
its velocity in metres per second is given
mittee is to contain at least 2 Australians?
by v = 4t + 5t 2 • Find how far it goes in
the fifth second. 173. Find the term independent of x in the
expansion of (x 2 - ~) 6 .
159. Show that the line x +y = 1 is a tangent
to the circle 174. A particle is projected vertically with a
x 2 + y 2 - 8x + 2y + 15 = 0. velocity of 20 mjs. Show that the time to
265
20
reach its greatest height is g and that the 187. Differentiate:
greatest height is 2 ~ 0 .
(a) sin 2 x (b)~.
COS X
175. A particle is vibrating under simple har-
monic motion with a period of n. Given 188. Evaluate:
dx
that the maximum velocity of the particle
1~ cos xdx
f
lo
(a) (b) ~-.
is 20 mmjs, find the amplitude of the X- 5
6
vibration and its maximum acceleration.
189. Given that 11 is a positive number indicate
176. Given that (a) the largest
3 2
x - 4x + 7x = _2 _
3X. + 4 + _B_ (b) the smallest
x- I X .
x- I, of the following numbers:
find R. 3-lf, 31f, 3", 3-".
177. ABC is a triangle, with LABC = 90°. A 190. (a) On the same diagram sketch the graphs
line parallel to AB cuts AC in L and BC of y = e 2 x andy = e-x.
in M. If AL = 8 em, BM = 6 em, (b) Find the angle between the curves at
MC = 9 em, find LC. their point of intersection.
178. Find the coefficient of x 3 in the expansion 191. Give three inequalities satisfied by every
of (2x + 3) 6 . point in the interior of a triangle with
vertices (0, 0), (0, 3) and (2, 0) and such
179. If2x 3 + ax 2 - 8x- 3isexactlydivisible that no point outside the triangle satisfies
by 2x + I, find a. When a has this value all three inequalities.
find all the factors of the polynomial.
192. Find all values of x in the domain
180. What is the gradient of the curve 0 :::;; x :::;; 2n which satisfy
y = x - ~at the point (1, -1)? For what
2 sin 2 x + 5 sin x + 2 = 0.
values of x is the gradient greater than 3?
193. Find the stationary points of the curve
181. Without using tables or a calculator find
y = x 4 - 18x 2 + 32. Make a sketch
the value of:
graph of the curve.
<J3 + 1)4 + <J3 -
1)4.
Hence find two consecutive whole num- 194. Find the maximum value of 3x(1 - x).
bers between which (J3 + 1) 4 lies.
195. If 11 is a positive integer, prove by mathe-
182. The points A, Band Care equally spaced matical induction that sn - 1 is divisible
around the circumference of a circle of by 4.
radius r. Find an expression for the area
of the triangle ABC. 196. P divides AB internally in the ratio 2: I.
Q divides AB externally in the same ratio.
183. For the curve y = ax 2 + bx + cit is given If A and B are the points ( -1, 2) and
that: (2, -4), what are the coordinates of P
and Q?
y = 2 anddy = 2 when x = I.
dx
197. Find from first principles the gradient of
Show that a = c.
the curve y = 2 - x 2 at the point (2, - 2).
184. There are 3 pairs of socks in a drawer. 198. Show that in any geometric series with a
Each pair is a different colour. If 2 socks common ratio -t, the ratio of the sum to
are selected at random, what is the proba- infinity to the sum of the first 3 terms is
bility that they are a matching pair? 27:28.

185. Show that ~ + cos~~ = cot2 A. 199. Find the area of the segment cut off from
-cos the parabola y = x(1 - x) by the line
X= 4y.
186. Find the term independent of x in the
expansion of (x + _!__)
2x
8
. 200. Find y if h = sin 2 x.

266
~ 208. (a) Find lim sin 5x

I:
201. (a) Find the primitive of x
3 8 x->O 3x
(b) Ifj(x) = cos- 1 ( -x) + cos- 1 (x)
(b) Evaluate (1 + cos x) 2 dx. show thatj'(x) = 0.
What type of a function isf(x)?
Sketch the graph ofj(x).
202. Differentiate each of the following:
(a) y = sin (3x + 4) 209. Find the volume of the solid generated
(b) y = X loge x + cos 2 x when the graph of y = tan x is rotated
about the x axis from x = -t to x = t.
(c) y = J9- x 2
Leave your answer in terms of n.
(d) y = sin x
X 210. (a) Find the derivative of x sin 2x.
X- 1 (b) Thus use this result to evaluate the
(e) y = --1.
x+
definite integral L~ x cos 2xdx, correct
203. Show that the acute angles between the to three decimal places.
pairs of lines
x - y + 2 = 0, y = - 3x + 4; and 211.
x- 4y + 2 = 0,2y = 9x- 1
are equal.

204. (a) Find the derivative of


y = loge (1 + ex).
(b) Hence find the equation of the tangent
at the point where x = 0.
(c) Find the derivatives of
0 is the centre of the circle and PT is a
(i) y = loge ( 1 +ex)
_ ex tangent touching the circle at T.
1
(ii) y = esin2 x. PO cuts the circle at Q. TS is a diameter.
If L TPO = 60°, find the size of
(i) L TOP (ii) L PQS.
205. Prove that
Give full reasoning for your answers.
tan (45° + e) - tan (45° - e)
= 2 tan 28. 212. In a parallelogram ABCD, P is any point
on the side AB. The side BC is produced
206. (a) Evaluate the following integrals using to an arbitrary point Q and a point R is
the given substitution chosen to complete a parallelogram P BQR.
x2 Prove that the angles DAB and PRQ sum

1
3 I
(i) ~dx, u = (1 + x) 2 to two right angles.
o V1 +X
213. (a) If
(ii) l\/4 - x 2 dx, X = 2 sin 8.
f(x) (x 2 2) sin- 1 ~ + ~ J4- x 2

I
= - ,

(b) Find (i) (l dx ) 2


show thatf'(x) = 2x sin- 1 ; .

(") I
X oge X

11
(loge x)2 dX.
X
. d
(b) F m IJ1- 2xdx
x4
.
usmg t he su bstttu-
.

tion u = x 2 •
207. State the domains of the following func-
tions: 214. (a) Find correct to three decimal places
(a) f(x) = JX=l the area bounded by the graphs of
(b) f(x) = loge X X+ 4y = 5 Y = i.
(c) f(x) = sin- 1 x (b) Find the minimum value of
(d) f(x) = cos- 1 x f(x) = e 2 x - 2ex - 12x. Give your
(e) f(x) = loge (.jX). answer correct to three decimal places.
267
215. Show that the volume of the solid gener- 223. Find the quotient and remainder when
ated when f(x) = sec x is rotated about x 2 - 2x + 1 is divided by x + 2.
the x axis between x = 0 and x = %is the x2 +
f
- 2x 1
approximate value of 2e. (Correct to two Hence find ' ' dx.
x+2
decimal places.)
216. (a) Ifj(x) = x 3 + 5x 2 - 22x + 16 show 224. A triangle AOC, right angled at 0, is
that f(x) is divisible by x - 1 and by placed so that A is on the 0 Y axis and C
X - 2. on the x axis. The hypotenuse AC has
(b) Find the three roots of the equation a length of one unit and the angle ACO
x3 + 5x 2 - 22x + 16 = 0. is 8 radians, OA = y units and OC = x.
(a) Express X andy in terms of e.
217. (a) Write down the factors of
(b) Show that the area of the triangle AOC
x 2 - 3x + 2.
is given by A =!sin 28.
(b) Find the values of a and b so that
(c) Show that the maximum possible area
x 4 + x 3 + x 2 + ax + b is divisible
is !unitl.
by x 2 - 3x + 2.
218. A glass bowl is made by rotating the pa- 225. If x +2 is one factor of the expression
rabola x 2 = 4y about they axis. Find the px 3 - x2
- 13x - 3p, find p and then
volume of water in such a bowl when it has fully factorise the expression.
a depth of 12 em.
219. (a) Sketch the graph of y = x 2 - 2x. 226. (a) (i) State a geometrical fact concerning
(b) On the same diagram, by taking the the angle between a tangent and
absolute values of the ordinates, sketch a chord through the point of con-
the graph of y = 2 - 2xl. lx tact.
(ii) State a fact about tangents drawn
220. (a) Define the meaning of lxl. to a circle from an external point.
(b) Solve the equation lx - 21 = 1 - 2x. (b) TA and TB are tangents drawn to a
(c) Sketch, on the same diagram, the circle from an external point T. If
graphs of y = lx - 21 and LATE= 40°, find the size of the
y = 1 - 2x. Use these graphs to con-
L APB. Give a full justification for
firm your solution to part (b). your answer.
221. Integrate each of the following using the A
indicated substitution:

(a) f 2x(x
2
+ 2)dx; u = x
2
+2
T

(b)f Sx dx;u=4x 2 - I
J4x 2 - 1

(c)
f cos xdx
J! + sm. x ; u = 1 + sm x
.

(d) f(1 - sin x) 4 cos xdx;


227. (a) Prove that angles in the same segment
of a circle are equal.
(b)
u = 1 -sin x.
222. A fisherman sets 5 crab traps overnight.
For each trap the probability of catching
a crab is 0·7.
(a) What is the probability that he has
caught exactly 3 crabs?
(b) Find the number of crabs that he is
most likely to catch. What is the proba-
bility of this catch?
268
PQRS is a cyclic quadrilateral and A For what values of k will the roots
is any point on PQ. A circle through be real?
the points P, A and R cuts RS produced (b) (i) If o:, {3 are the roots of
at B. 2x 2 + 7x - 9 = 0, find o: + {3;
Prove that BAIISQ. o:{3.
(ii) Find 3(o: + {3); 9o:f3 and hence
228. Ifj(x) = x4(x - iJ when -1 ~ x ~ 8,
write down the equation whose
(a) Where does this curve cut the x axis?
roots are 3 times those of the
(b) Find the stationary points and hence
original equation.
find the absolute maximum and mini-
mum values for f(x) over the given 234. (a) Find the coefficient of x 7 in the ex-
interval. pansion of (2x + x 2 ) 5 .
(c) Sketch the curve. (b) Find the coefficient of the term in-
dependent of x in the expansion of
229. (a) Find the equation of the line through (x + i)lo.
the point (6, -1) and perpendicular
to the line y = 2x - 1. 235. (a) In how many ways can 2 objects be
(b) Find the equation of the line through chosen from 8?
the point (- 2, 3) which is parallel to (b) If two pairs of vertices of an octagon
the line 6x - 2y = 1. are joined by straight lines, some will
(c) Write down the equation of the line be diagonals and some sides of the
through the point (- 2, 3) which is octagon. Find the number of diagonals
perpendicular to the line x = 1. in an octagon.
(d) Show that these three lines are collinear (c) How many diagonals are found in a
at a point P. Find the coordinates of P. dodecagon (12 sides)?
230. (a) If A is an acute angle whose tangent is 236. (a) By means of the quotient rule show
15
8 and B is an obtuse angle whose sine
1
that the derivative of f(x) = -.-- is
is !, find without using a calculator, smx
the value of sin (A + B). --.-- = -cosec x cot x.
COS X
2
(b) Write down a formula for the ex- sm x
pansion of cos (o: + {3). Hence derive (b) Use the quotient rule to find the de-
a formula for cos 2o: in terms of sin 2 o:. rivative of tan x.
(c) Prove that ~ + cos o:
2
- cos 2o:
= cot2 o:.
(c) Show that if f(x) = sin x sin 2x then
f'(x) = 2 sin x cos 2x + sin 2x cos x.
Prove that this is identically equal to
231. (a) Use Simpson's Rule with 5 function
2 sin x (2 - 3 sin 2 x).
values to find an approximation to
tan x- 1
the area under f(x) = ~1 - x 2 from (d) Ifj(x) = , show that
sec x
x = 0 to x = 1. Give your answer
f'(x) = cos x + sin x.
correct to three decimal places.
(b) Check your answer by the integration 237. (a) Prove that the opposite sides of a
ofj(x) = ~1 - x 2 , using the substitu- parallelogram are equal.
tion x = sin 8. (b) In a square ABCD, Xis the mid-point
What is the percentage error made of BC; Y the mid-point of CD. If AX,
using Simpson's Rule? BY meet at T, prove that
232. Solve the following equations for
(i) l::,.ABX =
6.BCY
(ii) TXCY is a cyclic quadrilateral.
0 ~ 8 ~ 2n. Give answers in terms of n
(where obvious) or otherwise correct to 238. (a) Show that
three decimal places. _2_+_3_= 5x+ 8
(a) 5 cos 2 8 + 2 sin 8 - 2 = 0. x +4 x - 2 x 2 + 2x - 8 ·
(b) 3 cos 8 - 4 sin 8 = - 3. Hence show that
233. (a) For what value of k does the quadratic + 8 dx
5x
equation kx 2 + 9x + k = 0 have
equal roots?
J
=
x2 + 2x- 8 ~
loge {(x + 4) 2 (x - 2) 3 } + k.
269
(b) Show that use mathematical induction to prove that
sec 2 x 11
9 + tan 2 x = 9 cos 2 x + sin 2 x · Sn=n+I'
Hence by means of the substituion
242. In a certain state half the population is
u = tan x, evaluate

t 1

9 cos 2 xd: sin x·


2

239. Eggs are packed into cartons of 12, all


male and half female. From statistics it is
found that 1 in 10 is left-handed (the same
for both male and female) and that the
probability of being pregnant, if female,
the eggs being selected at random. As they is 1 in 100.
are checked if 2 or more eggs have cracks If a person is chosen at random from
in their shell the carton is rejected and the the population, find the following proba-
eggs used for egg powder. If 2% of the bilities:
eggs are cracked, what is the probability (a) male and left-handed
that the carton is rejected? (b) male and pregnant
(c) female and pregnant
240. In photography the amount of light reach- (d) male or pregnant
ing a film is of vital importance to the (e) female or left-handed.
exposure. On a camera fitted with a focal
plane shutter, when the shutter button is 243. (a) Show that the equation of the chord
pressed, the shutter takes 20 milliseconds joining the points x = x 1 and x = x 2
to open and then shuts at the end of 70 on the parabola x 2 = y is
milliseconds. The attached electronic flash y = xx 1 + xx 2 - x 1 x 2 .
has an output measured in millions of (b) If the parabolas y = x 2 and
lumens, which varies during the duration 2y = (x - a) 2 intersect at the points
of the flash according to the following P and Q where P is at x = x 1 ; Q at
table: x = x 2 , show that the abscissa of these
[t = time after pressing the shutter points is given by the equation .
2 2
(milliseconds), f(t) = light output in mil- x + 2ax - a = 0. Hence show that
lions of lumens.] x 1 + x 2 = -2aandthattheequation
of PQ is y + 2ax - a 2 = 0.
(c) Show that the line PQ is a tangent to
t f(t) t f(t) the parabola x 2 = - y.

0 0 45 1·1 244. In a local tennis tournament the first prize


5 0·1 50 1·3 is $20 000; the second $18 000; the third
10 0·3 55 1-4 $16 000 and so on.
15 0·7 60 1-3 (a) What is the value of the n'h prize?
20 1·0 65 1·0 (b) If there are 8 players, what would be
25 1·2 70 0·8 the total prize money needed?
30 1·0 75 0·6 (c) If the total prize money available was
35 0·9 80 0·3 $110 000, what could be the total num-
40 1·0 85 0·2 ber of players in the tournament if the
90 0·0 player coming last was to receive
$2000?
Use Simpson's Rule to find the total light . l'f
245. (a) SImp I y 1 + 1
------:=----
reaching the film while the shutter is open. ()2 + 1)2 ()2 - 1)2.
70 (b) In an experiment an object moves
i.e. Calculate approximately

241. For the series


f
20
f(t)dt. from an initial point 0 along the
positive OX axis.
After t seconds it has travelled a
__ 1 __ 1 1 distance s metres will an equation of
8
n-1 x2+2x3+3x4+ motion given by s = t 3 + 6t. Find,
+ 1 (i) the velocity after 2 seconds.
n(n + 1)' (ii) the acceleration after 2 seconds.
270
(c) Find the middle term in the binomial Hence, on the interval
expansion of (1 + xt. oo : : :;
8 :::::; 360°, solve the equation
246. (a) A High School class of 30 pupils is 3 cos 8 + 4 sin 8 = 2. Give answers
correct to two decimal places.
to elect a captain, vice-captain and
(c) Find the numerical value of
secretary. In how many ways is this
cos 2 A + cos 2 (120° + A)
possible?
(b) A tennis team of 4 players is to be + cos 2 (120° - A).
selected from a squad of 10. 250. (a) Write an expression for sin 28 in
(i) How many different teams may be terms of 8.
selected so that they are seeded (b) A stepladder LMN stands in the form
from No. 1 to No. 4? of an isosceles triangle with the legs
(ii) How many teams without regard LM = LN = 3 m. One leg extends to
to order may be selected? X to hold a tray. XL = 0·25 m and
(c) In a right-wrong test of 10 questions MN= 1·5m.
a student makes a random guess at (i) By applying the sine rule to
the answers. To pass the test he has LLMN, show that cos 8 = !.
to score at least 8 out of 10 correct. (ii) Find the vertical height of X above
What is the probability of passing? the ground. (Answer correct to
(Give your answer correct to three three decimal places.)
decimal places.) X
247. The circle x 2 + y 2 = a 2 is rotated about
the y axis to generate a sphere. Find
(a) the volume of the spherical cap cut
off by the planes y = 1, y = a.
(b) the volume of water in the sphere if
it is filled
(i) to a depth of~ units
3
(ii) to a depth of ; units.

248. If rx is the acute angle between the lines 251. Solve for x: 2x 2 + 28 = 15x.
y = m 1 x + c 1 and y = m 2 x + c 2 , write 4
down the formula for tan rx. 252. (a) Evaluate 2T3.
A point P has coordinates (t 2 , 2t). A (b) Simplify j2 X ifl X 1'2·
line l passes through P and is parallel to
253. Express 0·07 as a fraction.
the x axis; a line m passes through P and
has the equation y = !t x + t. A third line 254. (a) Express with a rational denominator
6
n passes through P so that the line m bisects J5- J2'
the angle between the lines l and n.
(a) Find the equation of n; (b) Simplify x 2 - ~if x = 2- JS.
X
(b) Show that 11 meets the x axis at a point
Q which is independent oft; 255. The sum of the cubes of the integers from
(c) Prove that the length of PQ is equal
1 to 11 is given by the formula .
n 2 (n + 1) 2
to the distance of P from the line 4
X= -1. Using this formula find the sum of the
249. Prove that: cubes of the integers from 21 to 30 in-
clusive.
(a) (cosec A + cot A) 2 =11 -cos
+cos A
A.
256. Given E = tmv 2 find m to two decimal
(b) If a cos 8 + b sin 8 =A cos (8 - rx), places if E = 200 and v = 6.
write down the value of A and tan rx
in terms of a and b. 257. Solve for real x: j3x- 4j = j2x- 1j.
271
258. A chord 24 em long is drawn in a circle 273. How many terms must be taken of the
of radius 13 em. What is the perpendicular geometric progression 4, 6, 9, ... so that
distance of the chord from the centre? the sum will exceed 2000?
259. The area of a trapezium is 60 cm 2 and 274. Differentiate with regard to t
the parallel sides are 24 em and 16 em. a cos 2t - b sin 3t.
What is the distance between these parallel
sides? 275. If y = x 3 cos x find dx.
260. A cart wheel has 30 spokes. If they are
equally spaced what is the angle between 276. Find the indefinite integrals of
each neighbouring pair? (a) 2 sin 3x (b) 5e- 2 x.

261. What kind of quadrilateral is it if: 277. Integrate:


(a) both pairs of opposite angles are
equal? (a)
i dx
J 4 + 6x (b) f) 1
9- x 2
dx.
(b) opposite angles are supplementary?
(c) the diagonals bisect at right angles? . d
278. F lll f COS X
cos 2x.
+ Slll X
dX.
262. BX and BY are the interior and exterior
bisectors of L ABC (i.e. BY bisects the dy 1
279. If dx = + x 2 and y = 0 when x = 1
angle formed by producing AB). Prove 1
that BX and BY are perpendicular. prove that x = ~ + tan y.
- tany
263. If a pair of dice is thrown what is the
probability of:
(a) a double 6 (b) a total of 5?
280. Find f~~(x + sin x)dx.

264. In a family of 6 what is the probability 281. Simplify:


of 4 girls and 2 boys? (assume p = ! 2
(a) loge x
exactly.)
loge X
265. Two sides of a parallelogram are 6 em 282. Differentiate with respect to x:
and 10 em and the angle between them (a) loge (x 2 - 5) (b) (2x- 1) 8 •
is 120°. Find the length of each diagonal
of the figure.
283. Find the area under the graph y = x ~ 1
266. The diagonals of a rhombus are 12 em
from x = 2 to x = 3.
and 16 em. What is its area?
267. Solve the equation sin 2() = sin () for 284. Differentiate with respect to x:
values of() between 0 and 2n inclusive. (a) y = sin- 1 (2x + 1) (b) cos x.
X
268. Write down an expression for sin 2() and
for cos 28 in terms of ratios of() and hence 285. Find sin- x + cos- x (0 ~ x ~ 1).
1 1

2 tan() 286. Find the first and second derivatives with


prove that tan 2() = ().
1 -tan 2 respect to x of y = x 2 ex. Hence or other-
269. The line 2x + by = 8 passes through the wise find the maximum and minimum
point (2, -1). Find b. values of this expression.

270. Solve l2x +51 ?:: 3. 287. Find the equation of the parabola with
focus at (0, 4) and directrix y = -4.
271. An equilateral triangle ABC with each
side of length 6 em is inscribed in a circle 288. Write the first 3 terms in the expansion
(i.e. A, Band C lie on the circle.) Find the of (1 + x) 8 and hence obtain an approxi-
radius of the circle. mation for (1·01) 8 . Comment on the
accuracy of your approximation.
272. The probability that a missile will hit a
target is t. What is the probability that 289. What is the term independent of x in
the target will not be hit if 3 missiles are
fired in quick succession?
(x 2
_ ~y?
272
290. Simplify (.j2 + 1) 6 - (.j2- 1) 6 . ground and its height s m after t seconds
is given by
291. Which of the following are factors of
s = 20t- 5t 2
2x 3 + x 2 - 15x - 18 :
What is the greatest height reached?
(a) X+ 3 (b) X+ 2 (c) X - 3
(d) 2x- 3 (e) 2x + 3? 298. A projectile is fired at an initial velocity
of 80 mjsec at an angle of 30° to the
292. In how many ways can the batting order
horizontal. Assuming acceleration due to
of an 11 man cricket team be arranged?
gravity is 10 m/s 2 , find the time taken to
293. In Morse code there are 2 signals, a dot reach maximum height and also the hori-
and a dash. Show that there are enough zontal range.
arrangements by taking these signals, 1,
299. Given that the tangent to a circle is per-
2, 3 or 4 at a time to represent all letters
pendicular to the radius drawn from the
of the alphabet.
point of contact, prove that two tangents
294. 7 girls are chosen for a netball tour so as drawn to a circle from an external point
to have 2 reserves. In how many ways can are equal.
a team of 5 be chosen?
300. A 20 year old man puts $1000 in the bank
295. What is the chance of throwing a six 3 and receives 10% compound interest each
times in 5 throws of a die? year (i.e. the value of his money increases
by 10% each year). He forgets about the
296. Find the vertex and focus of the parabola
investment and remembers it when he is
y = x 2 + 1.
60 years old. Show that he may be pleas-
297. A stone is thrown upwards from level antly surprised.

273
PAPER 65
1. (a) Calculate the sum of the 40 terms of the arithmetic progression 5, 10, 15, 20, ... 200.
(b) Calculate the sum of all the whole numbers from 1 to 200 which are not multiples of 5.
2. A particle moves in a straight line so that after t seconds its distance from a fixed point on the
line is given by s = t 3 - 6t 2 + 9t. Obtain an expression for the velocity of the particle and
hence calculate the values of t when the particle is at rest. What is the distance of the particle
from the fixed point at these times?
3. Show that the expression x 2 + 2x + 4 is positive for all real values of x.
Prove that the expression has a minimum value when x = - 1 and find this value.
Draw a sketch graph of the function for values of x between -4 and 3.
4. An angle A lies between 90° and 180° such that sin A = l Find the value of:
2 cos A - 3 tan A
sin A +cot A
5. (a) What deduction can be made if you are told that (x- m) is a factor of
P(x) = x 3 + 2x 2 - 14x- 3?
(b) Solve the equation x 3 + 2x 2 - 14x - 3 = 0.
6. The base of a triangle is 45 em, and the base angles are 57° and 65°. Find the area of this triangle.

PAPER 66
1. (a) Calculate the values of x at the points where the line y = 3 cuts the curve y = 4x - x 2 and
hence calculate the area enclosed by the curve and the line y = 3.
(b) If the area enclosed by the curve y = 4x - x 2 from x = 0 to x = 1, the x axis and the ordinate
x = 1 is rotated about the x axis, calculate the volume of the solid generated.
2. Write down without proof the expansion of (a + b)" where n is a positive integer. (Include the
general term.) Find the middle term of (x - i) 8 .
3. The sum of the first two terms of a geometric progression is -4 and the sum of the fourth and
fifth terms is 108. Calculate the common ratio and the third term.
4. Write a formula for the number of arrangements that can be made from n things taken rat a
time.
A boy has to place 7 birds in 7 cages, one in each cage, but 2 of the birds are too large for 3 of
the cages. Find the number of possible arrangements.
5. (a) Show that a 3 + b 3 = (a + b) 3 - 3ab(a + b).
(b) Given the quadratic equation x 2 + px + q = 0 whose roots are a and {3, find a 3 + {3 3 in
terms of the coefficients.
6. What is the locus of the mid-points of parallel chords of a circle? In a circle, centre 0 and radius
4 em, two parallel chords AB and CD are drawn of lengths 6 em and 4 em respectively. Calculate
the distance between the mid-points of AB and CD if:
(a) they lie on opposite sides of 0
(b) they lie on the same side of 0.
274
PAPER 67
1. The 14th term of an arithmetic progression is 20 and the product of the first two terms is 1. Find
the sum of the first 10 terms.
2. State the general term in the expansion of (1 + x)" where n is a positive integer.
Find the coefficient of x 3 in the product of (1 + 2x) 2 and (1 - 3x) 6 •
3. The lengths of the two altitudes of a parallelogram are 2 m and 3 m, and the perimeter of the
parallelogram is 20m. Find the lengths of the sides of the parallelogram and also its angles.
4. Find the value of
3+ (J5- 1)3
( J5 + 1)
JS-1 J5+1
5. Prove that "C, = "Cn-r·
In how many ways can a team of 4 be chosen from 7 men, if 2 of the men say that both of them
must play, or neither?
6. Solve the equations for 0° ~ (} ~ 360°.
(a) 4 cos 2 (} = 1
(b) 3 tan 2 (} = 2J3tan(} - 1.

PAPER 68
1. (a) Calculate the maximum value of the function 5 - 2x - 3x 2 •
(b) A particle moves in a straight line from rest and after t seconds its velocity is given by ( 1Ot - t 2 )
m/s. Calculate the distance which the particle travels in the first 6 seconds.
2. The area bounded by the x axis, the ordinates x = 2 and x = 3, and the curve y = x - i is
rotated about the x axis. Calculate the volume of the solid generated.
3. EGis a chord and ED is a diameter of a circle. The tangent to the circle at D meets EG produced
at F.
(a) Prove that the triangles DEG and FDE are similar.
(b) If 2DF = 3DE, calculate the ratio DE: DG.
4. Given that log" N log 10 N + log 10 a:
=
1
(a) Deduce that log" 10 = - - - .
1ogto a
(b) Find the value of loge 10.
5. Find the sum of n terms of the geometric progression 3, 9, 27, ... and determine how many
terms must be taken so that the sum may exceed 10000.
6. Solve the equation 2 tan 8 + 3 cot 8 = 7, giving the four smallest positive values of 8.

PAPER 69
1. (a) Write down the first three terms and the general term of the expansion of (a + b)", for n a
positive integer.
(b) Find the middle term of (x - i) 2 ".
1
2. Differentiate: (a) (1 + 3x) 2
(b) x sin 2x.

3. Evaluate: (a) 1~ sin 2xdx (b) I4 41 dx.


1 X
4. (a) Find the equation of the parabola with vertex (1, -1) which passes through the origin and
whose axis is parallel to they axis.
275
(b) Findmaximumandminimumpointsandpointsofinflexionforthecurvey = x 3 - 6x 2 + 12
and sketch the curve.
(c) Given 2 2 x = 1024, solve for x.
5. Show that the equation x 3 - 5x + 1 = 0 has a root near x = 2. Use Newton's Method to find
an approximation to this root correct to two decimal places.

6. Show that tan- 1 x + tan- 1 y = tan- 1 t+ y and hence prove that


- xy

t an
_1 2a - b
!i
+ t an _1 2b !i- a _ 7r_
- .
y3b y3a 3

PAPER 70
1. The sides AB, BC and CA of a triangle have equations x - 2y + 8 = 0, x +y = 2 andy = mx
respectively. If the origin is the mid-point of AC, find the value of m.
2. Two particles A and B move along a straight line so that their displacements from the origin at
timet are given by xA = 9t 2 - t 3 and x 8 = 27t + 6 where xis in metres and tis in seconds.
(a) Find the velocity of each particle when t = 1.
(b) When do the particles have the same velocity?
(c) What is the acceleration of particle A when t = 0? When is the acceleration zero?
(d) What is the maximum positive displacement of particle A?
3. A point moves so that its distance from the x axis is always half its distance from the line x = 2.
Find the equation of the locus of this point.
4. (a) Find the gradient of the curve y = tan x at the point where x = 1-.
(b) Differentiate loge cos x and hence evaluate f: tan xdx.

5. In a given population study, the population N is given by N = 200 + Aek 1


• Initially N = 300
and after 3 seconds N has grown to 500.
Find: (a) the values of A and k
(b) the population after 5 seconds.
6. A point moves along the curve y = 3x 2 + 2x + 4. The abscissa of the point changes at the
constant rate of 3 units per second. At what rate is the ordinate increasing when x = 4?

PAPER 71
1. Show that for all values of p the line 9p 2 y - 6px + 1 = 0 is tangent to the parabola y = x 2 •
Find in terms of p the coordinates of the point of contact.

2. Find the value of loge x and!!_ when x = e. Hence find the point of intersection of the graphs of
X

y = loge x andy = !!.-. Find also an expression in terms of e for the tangent of the acute angle
X
between the curves at this point of intersection.
3. The position of a particle at timet seconds is given by x = 4 + e 31 where x metres is its distance
from a fixed point 0. Find:
(a) its initial position, velocity and acceleration
(b) an expression for the acceleration in terms of x.
4. A worker invests $1000 at the beginning of each year in a superannuation scheme. If the interest
is paid at the rate of9% p.a. on the investment (compounded annually), how much will his invest-
ment be worth after 20 years?
276
5. If rx and f3 are the roots of the equation x 2 - (m 2 + n~)x-+ mn = 0 and rx + f3 = 2rxf3, show
that m = n.
6. Find any turning points or points of inflexion for the curve y = 7 + 3x - x 3 and sketch the
curve.

PAPER 72
1. ABCD is a cyclic quadrilateral in which ABIIDC. If 0 is the centre of the circle containing the
quadrilateral, LAOD = 130°, L CBD = 20° and the side DC is produced to E, find the size of
LBCE.
2. Prove that the equation of the tangent to the parabola x 2 = 4ay at the point with parameter p is
px y - ap 2 = 0. The tangent at P, a variable point on the parabola x 2 = 4ay, meets the
parabola x 2 = -4ay at Q and R. Show that the locus of the mid-point of QR is the parabola
3x 2 = -4ay.
3. (a) Given that sin 8 -=f. 0, eliminate 8 from the equations:
a sin 28 = b sin 8
c cos 28 = d cos 8.
(b) Find the derivatives of: (i) loge (cos x) (ii) esin x.
4. A particle moves in a straight line, its displacement from the origin being given as a function of
time by x = cos 2 t.
(a) Find the velocity when t = t 2 •
(b) Find an expression for the acceleration as a function of displacement.
5. Find the value of k for which 9x 4 - 25x 2 + lOkx - k 2 is divisible by both x - 1 and x + 2.
With this value of k find the roots of the equation.

6. (a) Iff (6 - 2x)dx = 8, find the value of a.

(b) What is the volume of the solid generated by the revolution about the x axis of the positive
arc of the parabola y = x(l - x)?

PAPER 73
1. (a) If a 2 + b2 = 14ab, prove that 2 log (a + b) = log 16 + log a + log b.
10
(b) Find the coefficient of x 4 in the expansion of (2x - _!_)
2x

2. Find any stationary points, points of inflexion and the asymptotes for the curve y = x + ~.
Sketch the curve.

3. (a) Evaluate correct to three decimal places . JK 6 COS 8. d8.


2 + 3 Sin 8

t
0
2
(b) Using the substitution x 2
+ 1 = u, evaluate (x 2 : ) dx.
12
4. (a) Without using a calculator or tables find the exact value of tan 75°.
(b) Simplify sin (A + B) .
cos A cos B
5. A committee of3 is formed from 4 boys and 5 girls, each person having an equal chance of selection.
What is the probability that:
(a) the committee will comprise all boys?
(b) the committee will contain 1 boy and 2 girls?
(c) a particular girl will be on the committee?
277
6. (a) Calculate the volume of the solid generated when the region in the first quadrant bounded
by the curve y = eX, the line x = loge 3 and the coordinate axes, is rotated about the x axis.
(b) If y =~,show that sin xdy + y cos x = I.
smx dx

PAPER 74
. x 2 + 5x 1- k · .
1. Rearrange the equatwn = - - as a quadratic m x and find:
7x + 9 · 1 + t(1
(a) the value of k for which one root is zero
(b) the value of k for which the sum of the roots is zero.
2. (a) Show that cos 4A cos 3A + sin 4A sin 3A =cos A.
(b) Evaluate correct to three decimal places J: sec 2 xdx.

3. (a) The gradient of a curve at a point (x, y) on the curve is JX -


2. If the curve pass through
the origin, find its equation.
(b) Find the perpendicular distance from the point (- 3, 2) to the line 4x - y + 5 = 0.
x 2 - 3x + 6
4. Differentiate: (a) loge --;;-2 - - - - (b) tan x + ~ tan 3 x.
x + 2x- 5
5. Obtain the sum ton terms of the geometric series:
1+ x xz + x3 +
I + x + (1 + x) 2 (1 + x) 3 · · •

and show that if x is positive the sum never exceeds 1 + x.


6. (a) Write down the (r + lYh term and hence the r1h term of the binomial expansion of (3x + 4) 27 •
In this expansion two consecutive terms have equal coefficients. Which are these terms?
(b) Find the cubic equation whose roots are the reciprocals of the roots of
x 3 + 3x 2 + 2x + 1 = 0.

PAPER 75
1. (a) Write down an exact value for sin (180° - 60°).
(b) In a triangle ABC, the bisector of the angle A meets BC at L. By using the sine rule in each
of .6.s ABL, A CL find :
(i) an expression for BL in terms of the angle ~ , side c and L ALB.

(ii) an expression for LC in terms of the angle ~ , side b and L ALC.


BL c
Hence prove that LC = b.

2. (a) Write down the binomial expansion for (1 + x) 6 •


(b) By differentiating both sides of this identity find the expansion for (1 + x) 5 •
(c) If the term (1 + x) 6 and its series expansion are both integrated, what value would be given
to the constant of integration so that an expansion for (1 + x) 7 results.
3. (a) Find the remainder when x 3 - 2x 2 + x - 1 is divided by (x - 1).
)! Ji-b X )I
(b) I f - - = - - , show that-+-= 1.
x-a x a b
(c) If the quadratic expression ax 2 + bx + c = 0 has a remainder of 4 when divided by (x + 1)
and/(- 2) = 9 while/(- 3) = 20. Find the values of a, b, c.
(d) By completing the square show that the least value of the expression in Question 3(c) is
approximately 3·7. For what value of x does this occur?
278
4. (a) Prove that angles in the same segment of a circle are equal.
P The triangle PQR is an acute angled triangle inscribed
in a circle with PT as a diameter.
TS is a chord drawn paralled to QR and PS cuts
M QR at N.
QL is perpendicular to PR and cuts PS atM.
Prove that (i) L QPT = LSPR. (Hint: Join TR.)
(ii) LMNR = 90°.
(iii) LMNR is a cyclic quadrilateral.

5. (a) (i) Write down an expression connecting sec 2 e and tan 2 e.


2
(ii) Show that (3/6 - 2) = 58 - 12J6.
(b) If tan e = ~(2/6 - 3) show that sec e = ~(3J6 - 2).
Hence show that these values for sec e and tan e satisfy the equation
2 sec e - 3 tan e = 1.
6. A cylindrical tin open at the top is to be constructed so that its volume is 64 cm 3 .
If the cylinder has height h and radius r., show that:
64
(a) h = - 2
nr
(b) the area of material required is nr 2 + 1 ; 8
(c) the minimum surface area occurs when r =i= 2·73 em.

PAPER 76

1. (a) Find I JX 2 x(l


dx
+
JXx) 2
using the substitution u = 1 + JX.

(b) Find I xdx


)16 - x 2
by substituting x = 4 sin e.
(c) Use the substitution u = x 2 to evaluate Lfii x sin 2 (x 2 )dx.

2. (a) Use mathematical induction to prove that the sum of the cubes of the first n natural numbers
11

is given by I n 3 = {n(n +
2
. 1)}
i=l 2
(b) (i) Expand as a binomial series (1 + N0 ) 5 . Hence write down its numerical value.
Check result by direct use of your calculator.
(ii) If $5000 is invested for 5 years at 10% interest, to be compounded annually, find the
interest received at the end of 5 years.
(Use the result that total amount = P(1 + r)" where Pis the principal invested; r the
interest as a percentage and n the number of years.)
3. (a) If IX, f3 are the roots of the equation 3x 2 - 2x + 1 = 0 find:
(i) IX + f3 (ii) IX/3 (iii) IX 2 + f3 2 (iv) IX 2 f3 2 ,
Hence write down an equation whose roots are the squares of those of the original equation.
(b) Ifj(x) = x 4 + 4x 3 - 3x 2 + 2x + 1 is divided by (x - 2) write down the quotient function
Q(x) and the remainder.
4. (a) Write down expressions for sin x, cos x and tan x in terms oft where t = tan~·
(b) Write an expression for sec 2 ~in terms of tan 2 ~·
279
X ~ ~
(c) If t =tan- find- and hence show that dx = - - -2 •
2 dx 1+ t
(d) Hence, by making suitable substitutions in terms oft, show that
2 _(1
f dx
5 + 4 cos x = 3 tan
1 x)
3 tan 2 + c ·
(e) Evaluate correct to three decimal places J~ :x .
0
5 + COS X

5. In any acute-angled triangle ABC with sides a, b, and c:


(a) Write down a formula for cos A in terms of a, b, c.
(b) Express cos 2 A in terms of cos 2A.
(c) Hence write down an expression for 2 cos 2 ~in terms of cos A.
(d) Use these results to show that
~ _ J(a + b + c)(b + c - a)
cos 2 - 4bc

= N where s = ~(a + b + c).


6. (a) What is the gradient of the line y = 1?
(b) A light ray travelling along the line y = 1 meets a circular mirror with equation x 2 = 4 + y2
at a point in the first quadrant.
(i) What is the angle of incidence between the light ray and the tangent at the point of contact?
(ii) Assuming that the angle of reflection is equal to the angle of incidence, by using your
knowledge of Euclidean geometry (or otherwise), derive the equation of the reflected ray.

PAPER 77
1. (a) Evaluate the following integrals:
(i) J:(cos x- sin x)dx

(ii)r
. .)1
(x- 1)(x + 1)dx

(111
3

0
3dx
- 92"
+X
(b) By using the substitution u2 = x + 1 find the primitive of x.JX+l.
cosec 8
2. (a) Prove that .
cos 8 + sm 8 tan 8
= cot 8.
(b) For what value of k does the quadratic equation x 2 + 2(k + 2)x + 9k = 0 have equal roots.
(c) f(x) = x 2 - p(x - 1) is divisible by (x - 2). Find p.

3. (a) Write down the general term in the expansion of (x + D"·


(b) (i) Find the term independent of Xin the expansion of r
(X + ~
(ii) Find the term independent of x in ( x + L) 12

(c) Show that the value of the greatest term in the expansion of(3 + 2x) 9 , when x = 2, may be
expressed as 7 x 36 x 2 11 .
4. (a) Prove that the angle in a semicircle is a right angle.
280
(b) PQRS is a rectangle inscribed in a circle.
A chord PX is drawn so that PX = PS.
(i) What can you say about the line PR? Why?
(ii) Prove that RX = RS.

5. (a) If cos 8 = g and 8 is acute, find an exact value for cos 28.
(b) Prove that cos 38 = 4 cos 3 8 - 3 cos 8. [Use the expansion of cos (2A + A).]
(c) By means of the substitution x = cos 8, when 0 ~ 8 ~ 2n, find the three roots of
12x 3 - 9x - 1 = 0. (You must work in radians.) Give answers correct to three decimal
places.
6. y A door arch is in the shape of a parabola as in the diagram.
The base has a width of 4 m and the maximum height is 5 m.
(a) Using the standard parabola which has an equation of the
form y = ax 2 + bx + c, show that the equation of the arch
isy = -ix 2 + 5.
(b) Find the area of the door opening.

PAPER 78

1. (a) Evaluate the definite integral (i) r 1 dx 0

Jo )4- xz
(ii) 1" az dx+ xz
0
(b) What is the equation of the tangent toy= 3 sin- 1 2x at x = 0?
Sketch the graph of y = 3 sin- 1 2x from x = -t to x = t.
1
(c) Show that the derivative of y = sin- 1 (x - 2) is
.J4x- 3- x 2
1
Hence, find the area under the curve y = , from x = 2 to x = 2·5.
)4x- 3- x 2
u3 1
2. (a) Show that--= u 2 - u + 1- - - .
1+ u u+ 1
(b) By means of the substitution u = xl show that
~du.
8 2

J 1 1
x! 1 dx
+ X3
= 3f
1+ U 1
Hence, evaluate the given integral, correct to three decimal places.
3. Consider the functionf(x) = x(1 - 2x) 20 •
(i) Show thatf(x) ;:;:: 0 when x ;:: 0 and thatf(x) < 0 when x < 0.
(ii) Show thatf'(x) = (1 - 2x) 19 (1 - 42x) and hence thatf(x) has a maximum at x = lz and
a minimum at x = t.
281
(iii) By using the substitution u = 1 - 2x and using the fact that - f f(x)dx = lb f(x)dx, show

that the area between the graph of f(x), the x axis and the ordinates at x = 0, x = ! is 1 l 48
units 2 •
4. (a) Prove that the tangents drawn to a circle from an exterior point are equal.
(b) A An incircle in 6ABC touches the sides of
6ABC at X, Y and Z.
(i) List 3 pairs of equal line segments. Give
your reasons.
(ii) If s = !(a + b + c), prove that
BX + XC + AZ = s.
What is the length of AZ in terms of s
and the sides?

~)
211
5. (a) Show that the general term in the expansion of ( x 2 - , where n is a positive integer, is

2nck(x2)k ( -~Y'o-k
(b) What value of k (in terms of n) results in the power of x being equal to 6?
(c) What is the coefficient of x 6 in the expansion of ( x 2 - ~ y 2
?

6. (a) (i) Write down an expression for cos 2e in terms of cos e.


(ii) Form the identity for cos 2e in terms of sin e.
(iii) Show that tan e sin 2e = 1 - cos 2e.
(b) If x = cos 2e show that dx = -2 sin 2ede. Use this substitution to evaluate

It J~ ~ ~
0 X
dx.

PAPER 79
1. (a) (i) lff(x) = cos 2 x - sin 2 x, findf'(x).
(ii) If g(x) =cos 2x find g'(x) and show thatf'(x) = g'(x).
Hence, what connection is there betweenf(x) and g(x)?
(b) If ¢(x) = cos 2 x + sin 2 x, find ¢'(x) and hence show that ¢(x) is a constant.
By substituting x = n, show that the value of the constant is 1.
2. A test consists of 6 questions, each with three possible answers given. (Only one answer is correct.)
If a student makes a random guess at each question:
(a) What is the probability that he will have exactly 4 correct answers?
(b) Find the most likely number of questions that the student will have correct. What is this
probability?
(c) If 4 or more questions correct is taken as a pass, what is the probability that the student
passes?
3. (a) Prove that the angle between a tangent and a chord at the point of contact is equal to the
angle in the alternate segment.
(b) A tangent from an external point C touches a circle at B. BD is any chord of the circle and
BE a chord parallel to CD.
Prove that (i) 6BCDIII6EDB
(ii) BD 2 = EB . DC.
282
4. (a) Find the values of 111 so that the line 4111
2
y - 4111x + 1 = 0 is a tangent to the parabola
y = xz.
Find in terms of 111 the coordinates of the point of contact.
(b) If P(2t, t 2 ) is a point on the parabola y = ix 2 ;
(i) Write down the equation of the normal at P.
(ii) If Q(3, 0) lies on the parabola find an expression for PQ 2 and show that it has a minimum
value when t = 1.
(iii) Hence show that the normal at the point t = 1 passes through the point Q.
5. (a) State the sine rule.
(b) T A hill which has uniform slope is inclined at 14·2° to the
horizontal. From the bottom of the hill, A, the elevation of
T, the top of a tower TB standing on the hill, is 25·5°.
On moving 50 m up the hill to a point C, the elevation of T
is 55·5°.
Find: (i) LTAC
(ii) LATC
(iii) TA
(iv) the height of the tower TB.
6. A mathematician has her swimming pool in the shape of a semi-circle given by y = .)4 - x 2 ,
joined to the parabola y = 2(x 2 - 4).
(a) Sketch a diagram of the pool superimposed on a rectangular axes system.
Find the maximum width and length of the pool.
(b) Show that the surface area of the pool is (2n + 21t) m 2 • (For integration of the circle equation
use the substitution X = 2 sin 8.
(c) Assume the pool is symmetrical about a vertical plane through they axis and that the sides
are vertical.
(i) Complete the following table where d is the depth of the pool in metres for the corres-
ponding x value (the pool has the same depth at all points for the given value of x), lis
the length of the pool for the given x value and A is corresponding area of crosssection.
(lis in metres and A in square metres.)

x(m) 0 0·5 1·0 1·5 2·0

d(m) 3 2·5 2 1·5 1·0

/(m) 9·44 7·73 0

A(m 2 ) 15·46 0

(ii) Use Simpson's Rule with 5 function values to find the volume of the pool in cubic metres,
correct to one decimal place.

PAPER 80
1. (a) In an arithmetic progression the 9th term is twice the 4th term. Show that the 11th term is
three times the 3rd term.
(b) A geometric progression has a = 10 and r = t. Find the difference between the limiting sum
and the sum of the first 6 terms.
(c) A ball is dropped from a height of 90 em and bounces up to a height of 60 em and then keeps
rebounding always to a height t of that from which it fell. What is the total distance through
which it will travel until it stops?
283
2. (a) The vertical (unequal) angle of an isosceles triangle is 50°. What is the size of each base angle?
(b) What is the size of each interior angle of a regular polygon with 9 sides?
(c) ABC is an isosceles triangle with AB = AC. BA is produced to D and AX is drawn to bisect
the LDAC. Prove that AX is parallel to BC.
3. The 3 points 0(0, 0) A(4, 2) and B(4, 4) form a triangle. What is the gradient of the median from
B to the mid-point X of OA? Prove that the medians of the triangle are concurrent and find the
point Gin which they meet. Also find the ratio BG: GX.
2 .h
4. (a) D 1'ffierentlate
. y = -
X -
- w1t respect to x.
x+ 2
(b) If y = e-x sin x prove that~;: + 2~ + 2y = 0.
(c) For y = x +!sin 2x find any stationary value in the domain 0 ~ x ~ 2n and show whether
it is a maximum or a minimum or a point of inflexion.
5. (a) Prove that (p 2 + q2 )x 2 - 2(p + q)x + 2 = 0 has no roots unless p = q.
(b) Write down the term independent of xin the expansion of (x 2~,) 18 .
2
-

6. (a) A particle starts from rest at the origin and moves so that its acceleration is given by
a = 6t - 3t 2 where t is the time in seconds. Find:
(i) the maximum distance the particle moves from the origin
(ii) the time when the particle returns to the origin.
(b) A projectile is fired with initial velocity V mjs at an angle of rx to the horizontal. If the ground
is level find the expression for the horizontal range of the projectile and show that the maximum
2
· v m where g 1st
range 1s- · he acce1eratwn
· d ue to grav1ty.
·
g

PAPER 81
1. (a) Find the equation of the tangent to the curve y = e 2 x at the point (0, 1).
(b) Consider the lines whose equations are 3x - y - 3 = 0 and ax + 2y + 4 = 0. For what
value of a:
(i) are the lines parallel?
(ii) are the lines perpendicular?
(iii) do the lines meet in the point (2, 3).
2. (a) ABCDisarhombusandthediagonalsACandBDmeetatO.AB = 10cmand LEAD= 140°.
(i) What is the size of LAOB?
(ii) Find LABD. Give reasons.
(iii) Find the area of the rhombus and the length of each diagonal.
(b) If in the figure above a circle is drawn with centre 0 and radius OA to pass though A and C
and it cuts AB at X and BC at Y, find the size of LXOY.
3. (a) Find the maximum and minimum values of y = 2x 3 + 3x2 - 12x + 1.
(b) Show that one root of the equation 2x 3 + 3x 2 - 12x + 1 = 0 lies between x = 0 and x = 1.
(c) Use Newton's Method to find an approximate value for the root between x = 0 and x = 1,
correct to two decimal places.
4. (a) State the domain and range for y = sin- 1 x.
(b) Sketchy= sin- 1 x; -1 ~ x ~ 1.
(c) Differentiate y = sin- 1 x + cos- 1 x with respect to x. Explain your result.
(d) Differentiate y = x tan- 1 x - ! log (1 + x 2 ) with respect to x and hence or otherwise find
Jo1 tan-1 x.

284
5. (a) Five coins are tossed simultaneously. Find the probability of:
(i) five heads
(ii) three heads
(iii) at least three heads.
(b) A biassed coin has a probability of 1of landing heads. What is the most likely event to occur
if it is tossed five times (i.e. 5 heads, 4 heads, etc.).
6. (a) Show that 9x 2 - 5x + 4 is positive for all values of x and find the values of k for which
9x 2 - kx + 4 is positive.
(b) Prove by mathematical induction
2 + 4 + 8 + 16 + ... + 2" = 2(2" - 1).

PAPER 82
1. (a) Differentiate with respect to x:
(i) (1 - 4x 3 ) 4 (ii) Jsin x +1 (iii) log (1 + x 2 ).
(b) Find the equation of the tangent at (1, 2) to the curve y = 2x 3 - 3x 2 + x + 2.
2. A(O, 0), B(2, 5) and D(6, I) are vertices of a parallelogram ABCD. Find:
(a) the vertex C (c) the angles of the figure
(b) the length of the diagonal BD (d) the point of intersection of the diagonals.
3. (a) Two tangents are drawn from a point P 20 em from the centre of a circle whose radius is
12 em. Find their length.
(b) If the tangents meet the circle in A and B find the length of AB.
4. (a) Where does the line y = 13 - x meet the curve y = 25 - x 2 ?
(b) What is the area enclosed by the curve y = 25 - x 2 and the line y = 13 - x?
(c) The region bounded by the curve y = cos x and the axes is rotated about the x axis. Find
the volume of the solid of revolution.
5. A farmer already has an existing straight fence of considerable length. Using this fence as one
boundary he wishes to make a rectangular enclosure by erecting the other three sides to form a
rectangular paddock. If the farmer has 60 m of fencing available for these other three sides what
is the best shape to make the rectangle so as to enclose the maximum area?
6. Two people each write down one of the integers from 1 to 10. What is the probability
(a) both numbers are even? (c) both numbers are the same?
(b) both numbers are less than 5? (d) the numbers add up to 6?

PAPER 83
1. (a) Differentiate y = tan- 1 (x 2 + 1) with respect to x.
(b) Integrate J :x+
2x 1
dx.

2. Where does the curve y = sin x intersect the curve y = cos x in the interval 0 : : :; x : : :; n. Find
the area enclosed by the curves y = sin x, y = cos x and the x axis. Also find the volume of the
solid of revolution if this region is rotated about the x axis?
3. Two unequal circles with centres 0 and I intersect in A and B. The diameters of the circles drawn
from A are AOP and AIQ. BP and BQ are joined.
(a) Find the size of LABP?
(b) Prove that B, P and Q are collinear.
(c) Prove that Olis parallel to PQ.
(d) Prove that OJ is the perpendicular bisector of AB.
285
4. (a) Find the maximum and minimum values for y = x 3 - 3x 2 - 9x + 4.
(b) Show that the tangents to the above curve at (0, 4) and (2, -18) are parallel.
(c) Find the point on the curve where the tangent is parallel to the line y = 6 - 12x.
(d) Use Newton's Method to find an approximate root close to x = 0·5.
(e) Roughly sketch the curve.
5. (a) There are 100 tickets in a raffle and Mrs Smith buys just two tiCkets. What is the probability
that Mrs Smith wins:
(i) first prize?
(ii) both first and second prize?
(iii) either first or second prize (or both)?
(b) In the expansion of (4a- 5b) 12 which is the term which contains b3 ? Evaluate it if a= 2
b = 1. Answer may be left as a product of factors provided it is simplified.
6. A projectile is fired at an angle a to the horizontal and with a speed V m/s. Assume the acceleration
due to gravity is 10 m/s 2 and the ground is level. If the projectile returns to the ground after
4 seconds and its range is 100m find Vand a. Also find the maximum height attained.

PAPER 84
1. (a) Differentiate with respect to x:
(i) (x2 + 3)s
(ii) ex sin 2x
... ) log x
(111 --.
X
(b) Find the second derivative of e-x cos x.

2. (a) Find I' sin ~dx.

(b) Find the area bounded by the curve y = e 2 x they axis and the line y = e 2 •
3. (a) Write down the exact values of sin~ and sin !f and also cos f and cos !f and using the formula
for sin (A + B) find an exact value for sin i~.
(b) Find the total perimeter and area of a sector of a circle of radius 10 em containing an angle
between the radii of 60°.
4. A, B, C and Dare 4 points in order on a circle so that AB is parallel to DC.
(a) Show that arc AD = arc BC.
(b) Suppose AB = DC. Show that ABCD is a parallelogram.
(c) Further, given AB = DC and ABCD is a parallelogram prove that LABC is a right angle.
5. (a) A coin is tossed 5 times. What is the probability that
(i) the first 3 results will all be heads?
(ii) 3 of the 5 tosses will land heads?
(b) A and B toss a coin alternatively with A first then B then A, and so on.
(i) What is the probability A will gain a head with his first throw?
(ii) What is the probability that A will fail and B will succeed in throwing a head on his
first throw?
(iii) What is the probability that A will succeed on his second throw after the first throws
were failures?
(iv) What is the probability that A will succeed on either his first or second throws (assuming
B fails)?
(v) Prove that the probability that A will be first to succeed is i.
6. The acceleration of a particle moving along the x axis is given by the expression a = -ie-x.
The particle begins from the origin at time t = 0 with velocity 1 unit per second. Show that the
formula for velocity is v = e--I and hence show that x = 2 loge (1 + -!).
286
Before proceeding to find the derivative of sin x we note that

. sin h
(a) II m - - = 1 (already proved in Chapter 20)
11--+0 h

We will now establish two further results.

sin 2 h 1 - cos 2 h
(b) (from the Pythagorean result sin 2 h + cos 2 h = 1)
IT h2
sin 2 h (1 - cos h)(1 + cos h)
IT h2
sin 2 h (1 - cos h)
-,-----------,-- · - 2- -
1 + cos h h - h2
(dividing each side by (1 + cos h))
2
i.e. 1 - cosh = 1 (sin h)
2
h 1 +cosh h

. (1 - cos
IIm
11--+0
2
h
h) - 21
-
.
( Smce as h -t 0, cos h -t
sinh
1 and -h- -t 1)

(c) We need a formula for sin(x + h). A simple derivation of this formula follows.
A Area ~ABC = !cb sin A
= !cb sin(x + h)
also area ~ABC = !Jcp + !kq
c b Hence !cb sin(x + h) = !kp + !kq
k
. . k p k q
.. sm(x + h) = b · --;; + 7; · b

We now proceed to find the derivative of sin x.


!!___( · ) _ 1. sin(x + h) - sin x
sm x - 1m h
dX 11--+0

. sin h cos x + sin x cos h - sin x


= 1lm - - - - - - - , - - - - - - - - - -
11--+0 h

= . (sin
11m - -
h) cos x + sm. x (coshh - 1)
11--+0 11

= . (sin
hm -h- cos x
h) + .
sm x . h
(cos h- 1)
12
~0 1

= cos x as h -t 0 - h
sin- -t 1 cos h2 - 1 -t --
1)
( ' h ' h 2
287
Therefore :X (sin x) =

Assume that all functions defined in this appendix are continuous over the same interval. Thus they
are differentiable and integrable.
Let u be a function of x (---> u = g(x)
Let y be a function of u (---> y = F(u)
= F[g(x)].
The composite function rule for differentiation states that
dy dy du
- == - · -
dx du dx
= F'(u) . g'(x). (a)
Now, if y = F(u) is a primitive of some function f(u),
then y = F(u) = f f(u)du }
([3)
and F'(u) = f(u).
Now from (er:) 1
= f(u) . g'(x)
x = f[g(x)] . g'(x)
.'. y is a primitive off[g(x)] . g'(x)
i.e. y = Jf[g(x)]. g'(x)dx

.·. from ([3) f f[g(x)] . g'(x)dx = Jf(u)du


where u = g(x) and du = g'(x)dx
du
= dxdx.
Thus, the integral has been transformed by the substitution
u = g(x) and du = g'(x)dx
du
=-·dx
dx
With a suitable function 'g' a simpler integral should result.

Proof: Let y = f(x)


Let Llx, Lly be small corresponding increments of x andy .
. Lly . Llx = 1
· · Llx Lly
Take the limits of both sides as Llx and so, also, Lly approach zero
i.e. lim (Lly · Llx) = lim 1
Llx Lly
L>x--+0 l>x--+0
l>y--+0

1. (Lly)
'lm-·Im-=
• • t.x--+0Llx
1. (Llx)
Lly
1
Ay-->0

I.e. dy. dx = 1.
dx dy
288
Now sin- 1 x + cos- 1 x = 1.
Differentiate both sides with respect to x
~(sin- 1 x) + ~(cos- 1 x) = ~ (!!_)
dx dx dx 2
Fx + x2
dd (cos- 1 x) = 0
x
as 1 is a constant.

. d( -1 ) 1
.. -d
x
COS X = ) 1- x2
.

289
(e) -2e-> + C (f) X3 + ±e2x + C
(g) -6e4 -·' + C (h) -j,el-6x + C
(i) -ke-2x + C
2. (a) 2ex' + C (b) e·' - ±ex' + C
(c) f.re1x'+1 + C (d) j,e2x'+I + C
(e) ±ex' - ±e-x' + C (f) -e-x' + C
3. (a) 6·362 (b) 0·17S (c) 3-627
4. e 2 - 1 ~ 6·389 square units
S. 7·2S4 square units 6. 2·S62 square units
8. 0·8S9 square units 9. 10·036 cubic units
I. (a) t, -h (b) 16, 32 I 0. 49·1SO cubic units 11. I square unit
(c) 1024 (d) increases rapidly 12. O·OS9 units 2 ; 0·0141 units 3
(e) No (f) 0
2. (a) No (b) approaches 0
(c) (0, 1) (d) larger
(e) 2" I. (a) 1·6094 (b) 1·94S9 (c) 0·32SO
s. (0, 1) 7. 1·1, l·S, 2·6 (d) 2·3026 (e) 0 (f) 0·9933
8. 0·9 2. (a) 6·9078 (b) -4·60S2 (c) 4·248S
(d) 8·8S37 (e) -0·3S67 (f) -4·9619
3. (a) 1·3863 (b) 3·29S8 (c) 0·40SS
(d) 2-4849 (e) 2·8904 (f) 2·6027
I. (a)
(c)
1·2214
0·606S
(b) S·3388
(d) 0·0247
4. (a) 7 CbH (c) t
S. 0 6. 0 7. No
(e) 100·4277 (f) 67-2381 8. No 9. No, No 10. 2·7
3. (a) 4·48 (b) Yes II. Symmetrical about y = x.
13. 0 < X < 00; - 00 < y < 00
14. log a IS. loge e = I
16. (a) 1·0986 (b) -0·693
I. (a) 3e 3 ·' (b) sesx- 6 (c) 7e 7 x+S
17. (a) 6 (b) 64
(d) -4e- 4 ·' (e) 2e 2x (f) ex + lOx
18. (a) 4 (b) 81
(g) 3 - x- 2 - e-x (h) !(ex + e-·')
19. 2
(i) -2e 1 -x
20. (a) 3 (b) 7·S (c) 10 (d) 1
2. (a) 30e 6 x (b) 2xe·'' (c) 1 + e7 -·'
21. (a) 0·693 (b) 2
(e) - 8xe- 4 x' (f) _l_e./X
2J'X
3. (a) 4e 2x (b) 9e 3 x+ 2 (c) e1-x
4. tee - e- 1 ) s. e
2
7. y - 3x - 1 = 0; y - 3x - 3 = 0 I. (a) .!. (b) .!. + 2 (c) -
10. 90° 12. y = e·' + 1 X X X

(d) 2e2x +~ (e) 3x ~ 1


(f) _2_
2x 3

(g) ~
2x
(h)
3
s - ;- (i) ~
I. (a) ex(l + x) (b) 3ex(l + x) X
(c) (2 - x)e-x (d) (3x + 6x)e-'
2
1
(e) (2x + l)e>x (f) (I- x)e
(j) .!l. (k) 7
(I) 2x
X X
ex(x - 1) -ex __ 1___ 1_ ___ I_
(g) 2
(h) (ex + 1)2 3 4
X x + I x +4 2(x + 9)
. 2- ex+ e-x 2 I I I
(I) (e x _ 1)2 S. (a) 2x + I + x - S (b) ;- - x + I
2. (a) (4x + 2)ex' (b) (2 + x)e·' (c) (2x
2
4)e-x 3 I
3. se-x; se-x (c) x + 6 (d) 2(x + 4)
4. l7e 6 , 21
I I I 6
S. y = 3x + 3; y = 2x + 2; y = x + 1 meet at ( -1, 0). (e) x +I+ x - I - x +2 (f) 3x +2
8. (a) Minimum (0, 1) (b) y--> oo
1 -2 -3
9. (a) A = (1, e- 1) B = (2, 2e- 2) C = ( -1, -e) 6. (a) x (b) (2x - I) (c) -;-
10. (a) ex(! + x); (2 + x)ex
2 2x3
(b) ( -1, -e- 1) is minimum. 7. (a) I + In x (b) 6x In (x - 2) + (x _ )
(c) (-2, -2e- 2 ) (d) Yes 2
(e) very large (f) small, negative (c) In x
_ ~. 21n x 9. _I_
8
X X xln x

(b) -±e-2x + C 10. (a) (3x : 1)2 (b) .!.


I. (a) !e4 ·' + C X
(c) ~eax+b + C (d) !e 3 x e-x + c 13. (a) y = x - I (b) 3y x = 3 log 3 - 3

291
14. X= 2; 2y - X = 2Jn 4 2
16. (a) min (1, I) (b) max (e, e- 1); inf. (ef, ie-l)
(c) min (I, I); inf (2, In 2 + !) (a) 2·70481 (b) 2·71692 (c) 2·71815
17 . (a) I - x In x (b) -e-x(x In x + I)
xex x(ln x) 2
(c) ~+ex In x (d) - 2ex
X (e·' + !)(ex - I) I. (a) - 2e-2x (b) _I_
(e) 3 J +X
I I 2x 5
19.--+--or
x+3 x 3 x 2 -9
-- (d) (3x + 5)e·' (e) 5x +2
20. -2 < X < 00; X I; I 2. y = 3e 3 x- 2e 3 ; -2e 3
21. x = e 3. (a) !e 2 x+ 3 + C (b) tIn (5x - 4) +C
(c) 2ln (x 2 + I) + C
e6- e2
4.--
2
I. (a) tinx +C (b) In (x + 2) + C 5. e 2 + e- 2 - e - e- 1 ; area = e 2 + e- 2
(c) In (4x - 3) + C (d) iln(6x -7) + C square units.
(e) x 3 + x 2 - 5 In x + C (f) iln (2x I)+ C e.fX
8. ( a ) - (c) 3ln x + 3
2. (a) In (x 2 + I) + C (b) ! In (x 2 - 3) + C 2JX
! In (x 2 - 4x + I) + C (d) -In(!- x 2 ) + C
~
(c) I - x2 (f)
(e) In (ex + I) + C (f) ! In (x 4 + 3) + C (d) (x I) x(x 2 + I)
6 I I
3. (a) 5x + 7 In x + - + C 9. (a) !On 4 In 3) (b) 7! + 41n 2 (c)---
X 3 3e
1 I 10. (e - 1), (~ - 1) II. In 5 square units
(b) -x 3 + 2ln x - - 3 + C
3 3x 12. (e - 1), ~(e - I) 13. !{e 3 - e- 3 )
(c) ie
2
x' - In + C x 14 n(e4 - I)
16. 55 log 10 2
4. (a) I - log 2 (b) e 2 + 1 . 2
(c) ! In 5 (d) ! In 5 17. (a) max(l,!)min(-1, -!)
6. - x - 2ln (I - x) + C (b) approaches 0 (d) ! In 5
7. 2x - 3ln (x + 4) + C 18. ! In 5
9. (a) In 5 (b) 1 19. t<Jn4+ln5-ln8)=%ln2·5
11.4ln2 12. (3, 0); 12 - 3ln 31
13. I +log!= 1 log2 14. y = In (x + I) + 4
15. n cubic units 16. !In 11 !In 3
17. nln2 19. (e - I)
20. ~(e 2 - I) 21. !(e 4 1)
eY
22. x = -;if(e 2 - 1)
2 I. (a) i radians (b) -!f radians (c) ~ radians
(d) ¥radians (e) ¥radians (f) If radians
(g) ¥radians (h) l.f radians (i) if radians
(j) fa radians (k) -~radians (I) n
radians
(m) --¥radians (n) \"-radians (o) 325{ radians
I. 0·693, 1·099 2. 100 In 10 == 230·26
2. (a) 90° (b) 540° (c) 240°
3. (a) 4 log a . a4 x (b) ax[l2x + (6x 2 + !)log a]
(d) !5° (e) 225° (f) 720°
(c) 4·'(5x In 4 + 5) (d) Y(x In 3 - l) (g) 2° (h) 105° (i) 70°
x2
(j) 112·5° (k) 255° (I) 45°
1 (m) -40° (n) 300° (o) -157·5°
4.ln5.(5");--(5")+C
In 5 3. (a) 85·9437° (b) 114·5916" (c) 68·7549°
1
5. - -(JQx) + C 6. _}_ = 10·10 (d) 2·8648° (e) 157·5634° (f) 17·1887°
In 10 In 2 (g) -180-4817" (h) 359·8175°
4. (a) 0-4887 radians (b) 1·3439 radians
(c) 1·9722 radians (d) 2-3911 radians
(e) 2·0333 radians (f) 1·6991 radians
I. (a) 0·6678 (b) 1·7584 (c) 2·2628 (g) 5·2098 radians (h) -5·5441 radians
2. (a) 0·2900 (b) 0·7637 (c) 0·9827 n ,[3 I 5n I 1
4. (a) 1·924 (b) -1·229 (may be written 2·771) 5
' (a) 3'2'2 (b) 4'- ,fl'- J2
(c) I
5n -,[3 1 2n ,[3 1
5. ( a ) - -
1
(b) ~ + 4x log 10 x (c) 3'_2_'2 (d) 3'2' -2
xlog 10 In 10
(c) 1 - In 10 log 10 x (d) _ 2 _ (e) 3n _!.._ _ _!.._ (f) 7n,,f3)
x 2 In 10 x log 10 4, J2' J2 3 2 2
(e) lOx - 7 0,[3 , -,[3
6. (a) 120 , (b) 540°, 0, 0
(5x 2 - 7x)log 10 2
292
(c) 480°, J3, -,[3 (d) 270°, -l,doesnotexist
2
I. (b) -2n, -n, 0, n, 2n
(e) 5W,~, ~ (f) 225°, ~,I 2. (a) -2n, -n, 0, n, 2n
7. (a) 0·8660 (b) 0·7071 (c) 2·5722 3. n
(d) 0·5646 (e) 0·8855 (f) 3·0777
(g) 0·2588 (h) 0·6247 (i) 0·8090
(j) -1·5574 (k) -1·4142 (I) -7·0153
8. (a) 30° (b) 60° 2. 3
3.(a)x=0,1·9 (b)x 0, 2·28 (c) x = 0, 2·48
(c) (1') }2n ra d'1ans; 11 3n rad'mns; ("')
(") 5n ra d'mns
111 (j 4. (b) (i) X = 0, 1·9 (ii) X = 0, 2-48
4
(d) 30°, 108° (iii) X = 0· 52, 2-62
5. X = 0, 1·24
6. X= 0·79

I. (a) 38·4845 sq. em; 21·9911 em


(b) 6·125 em (c) 10·7188 sq. em I. 6 cos 6x 2. 3 sec 2 x
2. (a) 8 em (b) 12·5 em (c) 10·56 em 3. -4 sin 4x 4. t cos tx
(d) 30 em (e) 13-666 em 5. n cos nx 6. -2 sin (2x + 5)
3. (a) 18·6 em (b) 1·8 em (c) 2·932 radians 7. 3sec 2 (3x- I) 8. 2cos2x
(d) 4-603 em (e) 6·731 em (f) 49·114 em 9. 2 sin(3 - 2x) 10. 8 sec 2 2x
4. 66 em II. b cos bx 12. -b sin(bx + c)
5. (a) 1·4193 radians (b) 1·8445 km 13. cosx+sinx 14. -2sinx+3sec 2 x
6. (a) 39 cm 2 (b) 52 cm 2 (c) 52·36 cm 2 15. 3 cos 3x - 2 sin x 16. t sec 2 1 - I
(d) 130·90 cm 2 (e) 26·18 cm 2 (f) 104·72 cm 2 17. -3 sin 3x- 2 cos 2x 18. 2 sec 2 2x + 4 cos 4x
(g) 117-81 cm 2 (h) 32·29 cm 2 19. 6 cos(3x - 4) 20. 4 sin(t - x)
7. (a) 10·5 em (b) 5-498 em 21. sin x + x cos x 22. cos 2 x - sin 2 x
8. (a) 2-618 radians (b) 150 degrees 23. x(2 tan x + x sec 2 x) 24. ex(sin x + cos x)
9. (a) 176·715 cm 2 (b) 12·315 cm 2 (c) 2·34 radians 25. x 2 (3 cos 2x - 2x sin 2x) 26. cos x . e'1"x
(d) 0·524 radians (e) 6·83 em (f) 10 em 27. 2 sin~ + x cos~ 28. e·'(cos x - sin x)
(g) 8·210 m 2 (h) 9·35 em (i) 224·8 degrees 29. 2 cos x sec 2 2x - tan 2x sin x
10. (a) 0·4712 m (b) 0·2827 m 2 COS X
30. --.-2 - = -cosec x cot x
II. (a) 12·5 cm 2 (b) 7·5 cm 2 (c) 22·5 cm 2 Sin X
12. (a) 2-810 cm 2 (b) 2·265 cm 2 (c) 31-695 cm 2 sin x sec 2 x 2
31. - - - =sec x tan x 32. - - - = cosec x
13. (a) 102·047 cm 2 (b) 16·350 cm 2 ; 16 cm 2 2
COS X tan 2 x
14. 72° ex(sin X - cos x)
15. (a) 29·3215 m (b) 205·25 m 2 33 34. 3 sin 2 x cos x
· sin 2 x
2
16. (a) 3-4336 cm (b) 3-4336 cm 2
17. 8n = 25·13274 em 35. -4 sin 2x cos 2x 36. 2 cos 2x = 2 cot 2x
sin 2x
. 2(cos x + x sin x) 38. -sin x = - tan x
37
cos 2 x COS X
39. 2e 2 x(sin 2x + cos 2x) 40. e-x(sec 2 x - tan x)
I. (a) I; 2n (b) 2; 2n (c) 3;2n (d) 4; 2n 41. -2 sin (2x + t) 42. 2x(sin 2x + x cos 2x)
2. (a) t (b) 5 . -2 COS X e4 x(tan x - sec 2 x)
43 44. -~--,..--~
3. (a) I; 4n (b) I; t (c) I; lf (I + sin x) 2 tan 2 x
4. (a) ¥ (b) 6n (c) 2 x cos x - 2 sin x . - sin 2x
5. (a) I; 2n (b) I; 2n 45 . 46
(c)2;n (d)l;2n 3
X
~
yCOS 2x
6. (a) x = 0, 2n (b) X = t, ¥- . 2 cos 2x cos 3x + 3 sin 2x sin 3x
(c) x = 0, 'J, lf, n 47
cos 2 3x
2
48 . 3 sec 3x = . 3
tan 3x s1n 3x . cos 3x

I. I
2. (a) x = 0, 2n (b) x ;= n (c) x = t, ¥-
3. (a) 2; 2n (b) 5; 2n (c) I; n I. (a) -tcos 2x + C (b) t sin 5x + C
(d) I; lf (e) I; 2n (f) I; 4n (c) tan x + C (d) -tcos 3x + C
(g) I; 2n (h) I; 2n (i) 3; n (e) 2tan~ + C (f) - t cos (2x + 3) + C
5. 2n; 5 (g) -sinG- x) + C (h) sin x + cos x + C
(i) t tan (4x + 3) + C
(j) t sin 3x - t cos 2x + C
(k) x - t sin 3x + C (I) cos (n- x)
I. n 2. t 3. 2n (m) .f. + t cos (2x - 3) + C

293
(n) x + cos G- x) + C 7. 22" 10. 6 em
( o) t tan 2x +! cos 4x + C II. 134°
2. (a) I (b) I (c) I
(d) n (e) -1 (f) 2
(g) ~ (h) ~ (i) 4
3. (a) 2 (b) I (c) t I. 115°, 70° 2. 50°, 93°
(d) i (e) 4 - t)2 3. 47", 25°, 68°, 40°, 87" 4. 95°, 73°
4. (b) n cubic units 5. 110°, 48°
5. !

1. 108°
I. (a) -'f sin¥ (b) 2 sin x cos x
2. 40°, 20°, 40°, 140°
(c) n sec 2 nx (d) 2e 2 x(sin 2x + cos 2x)
3. 18°, 18°
(f) 3x cos 3x - sin 3x
(e) -3 tan 3x 5. ljn or 10·47 em
3x 2
6. 90 em, '¥-em
2. (a) -25 sin 5x (b) 16 cos 4x
7. ¥em
(c) 2(cos 2 x - sin 2 x) 8. 15°, 45°, 120°, I: 2
4. 0; 0 9. 45°, 60°, 75°
5. X = 1'!i'O radians; 1ao cos ts'o
0

10. 20°
6. (a) -!cos 3x + C (b) !tan4x + C
11. ~
(c) *sinnx + C 12. 11·1 em
7. (a) t (b) I + !f
(c) t- -J'i
8. fo sec 2 %
10. e- I 2. 9·8 em
II. j
4. 4·6 em
12. loge sin x + C 7. X+)'= 90
13. (a) 0·7071 (b)t 11. 19·6 em
(c) 2·1213 (d) 0·9239 12. 12 em
15. (a) I (b) t (c) 0
14. )r 2 + R 2
16. ¥
17. t
18. *~ 2·5465 square units
19. I, -1
2. 6·9 em 3. 1·25 em
20. !f 4. 8, 10,2 em
21. 0·6165 square units

I. (a) 64° (b) 40°, 25°


(c) 35o (d) BP = BT
2. (a) 64° (c) PT/IQR
3. 72°
I. 10 em 2. 3 em 4. 52°, 70°, 58°
3. 12 em 4. 8 em 5. 112°, 48°, 20°
5. 10·5 em 6. 13·4 em 6. 58°
7. !Ocm 8. 20 em
9. 24 em 10. 18 em
II. 24 em 12. 22·6 em
13. 8 em 14. 10 em

I. 34° 2. 84°, 42°


3. 52°, 26° 4. 180°, 90°
5. 2x 0 , X 0 , equal 6. t1°, t1°
7. 55°, 250°, 125" 8. 144°, 216°, 108°, 180° 1. (a) 10(2x + 1)4 (b) ~
9. (a) 63°, 11 r (b) 142° (c) 180°
(e) opposite angles supplementary (c) -sin xecosx (d)~
2
x +1
2. (a) ! (b) 290 (c) 5
3. a= 16, b = 12 or a= -5!, b = Ij
I. 62° 2. 52° 4. 76°42', 76°42', 103°18', 103°18'; 146 cm 2
3. 67" 4. 102° 5. (1, 1), minimum; (- 2, 28), maximum;
5. 30° 6. 80° ( -t, I4t), point of inflexion.

294
4. (a) s = 8 (b) s = -1, s = 0
(c) Im (d) -6 mjs, 2 mfs
1. (a) 3 sin 2 x cos x (b) ex(l + x) 2 (e) I 3 (f) 2 mfs 2
-12 2 2 5. (a) 24·5 m (b) 29·4 mfs
(c) ( x _ l)s (d) cos x - sin x (c) 9·8 mjs 2 (d) 78-4 m, 39·2 mfs
3
2. -16! 6. (a) 3m (b) v = 21 - 4
3. A = 2, B = 3, C = 4 (c) -2 mjs, 2 mjs, opposite directions
4. 2x + y- 5 = 0 (d) after 2 seconds
5. 1t square units; f cubic units (e) at I = I and 1 = 3, the body passes through the
origin in opposite directions at the two times.
7. 2 units per second per second, 19 units per second
8. 38 mjs, 30 mfs 2
1. y = x3 + x + 2 9. (a) X = 0, X = 4, X = 21
2. (a) 1545 (b) 3(2 12 - 1) (b) 17m (c) 8 m/s
2
3 dy - 12 24. d y - 96 36 (d) I = 1, I = 2 (e) X = 1, X = 4
· dx - x 3 - x 4 ' dx 2 - x 5 - x 4 (f) a = -10 + 121, 0· Hi m/s
10. (a) 12 mfs
y = 0 I'f X= 131 ;dy
- = 0 I'f X= 0 or 2 ,.
1
11. (a) 17m/s (b) 8m/s 2
dx
d2y (c) 3 s, 39m
- = 0 if x = 0 or 2%
dx 2
4. (a) 8·4 em (b) 50·3 cm 2 (c) 4·0 cm 2

1. (b) 2 mfs (c) returns when I = f


(d) I= if, a= -4; I=¥, a= 4
2
(b) 2(x + 3x + 1) 2. I= Vl. I=¥; X= 3·73, X= 1·5
1. (a) 3x 2 + lOx - 1 3. (a) 1m (b) 1·41 m
(2x + W
(c) 1 m/s, 0 mjs, -I m/s
(c) -3 cosec 3x cot 3x (d) _2._ (d) -1·37 mfs 2 , 0 mfs 2
e2x
4. (b) 1·7 mfs
2. (a) x 3
x + tx + C
-
2
(b) ix 1 + ~x 1 + C
(c) ~log. (5x - 3) (d) X +loge X
3. 13-443
4. (a) 2 (b) -5 (c) -~ (d) 14
1. (a) 54 units per second (b) s = 21 3
6. (a) t (b}i
(c) 54 units (d) after 2 seconds
2.x=21·5
3. (a) v = -61 (b) s = -31 2 + 27
1.112;1100 (c) after 3 seconds, -18 metres per second
2. I square unit 4. s = 228
,ff 5. s = 12t
3. (a) x = ±T (b) a = 2, b = 3, c = I 6. (a) 43 units per second (b) 93 units
7. (a) s = -41 (b) s = -21 2 + 18
4. 101·3 km
(c) after 3 seconds, v = -12
5. (a) log. 2 - I =:= -0·307
(b) t(e 4 - e2 ) - I =:= 22-605 8. after 10 seconds, 266% m
9. (a) 68 m (c) 20m
(c) 2
10. 6 units
11. 35m
12. 300m 3
1. (a) e 2 - I =:= 6·389 square units 13. 30·6 m 2
(b) f(e 4 - I) =:= 84·192 cubic units 14. 6·1 m 2 /s
2. (a) 6225 (b) 3t
3. 3x + y +I= 0
4. r = _!Q_ =:= 1-4 metres 1. 4·8 mm 2 /s
4+n
5. 4·003 square units 2. 240000mm 3 fs 240cm 3 /s
3. 0·64n = 2·01
4. 40n = 125·66
5. 20000n cm 3 fs
6. 0·24 cmfs
7. 4·33 cm 2 /s
8. 6 units per second
9. (a) 1 5. mm/s
1. I= 0, I= t (b) 8 mm 2 /s
2. 320 m/s 2 10. (a) 0·04 cmfs
3. I= 2 (b)8cm 2 /s

295
11. (a) 1 6 Aorr = 0·0002 cmjs N
(b) 1 cm 3 js
12. - 3A2 = -3
4m·
13. (a) 12·06 cm 3 js
(b) 5·03 cm 2 js
14. (b) io mjmin
15. fz- em/min 0 12 5 10 20 t

6. 66·3°C
7. (a) 115·0° (b) 1·363 h (c) 250°
1. (a) 12 000 (b) 29 500 (approx) 8. (a) 49·90°C (b) 27·73°C
2. (a) 81 000 (approx) (b) 116 000 (approx) 9. (a) 56·25°C (b) 221·81 minutes
3. (a) 1977 (b) 807000 (approx) 10. (a) 20·625°C (b) 1·677 minutes
4. (a) 160 g (approx) (b) 102 g (approx) 11. (b) 22·6 minutes
(c) 81 g (approx) 12. (a) V = 490(1 - e-o·o2t) (b) 88·82 mjs
5. (a) 9000°K (b) 6800°K (approx) (c) 490 mjs
6. (b) (i) 0·9 years (approx); 1·1 years (approx) 13. (a) X = 50 + 30e-O·Olt (b) 66·5 kg
(ii) 366 g (approx) 14. (b) 48·75
15. (b) 4292
16. (b) k =
t; 86
(c) P
1. (a) 1·926 kg (b) 2-473 kg (c) 13-86 days 100
2. (a) 2600 (b) 2931 (c) 1Hyears
3. (a) -0·0223143 (b) 35·8 g (c) 31·06 years
4. (b) (i) 1620 years (ii) 948 years
10
5. $3297-44
6. (a) 4 mjs (b) 1·79 mjs (c) 0·03 mjs
7. 42000
8. (a) 0·0896048 (b) 1·3 X 10 7
9. (a) 8488 kgjm 2 (b) 1653 kgjm 2
10. (a) 7·79 g (b) 1386 seconds
11. (a) 0·0126022 (b) 97 000 (approx)

I. (a) (i) 500 (ii) 2741 I. (a) a = 1; x = 2t} (b) a = 2; x = 4t }


y = (2 y 2t2
(b) (i)
(c) (i)
0·693 (ii) 2·749
400 (ii) 0·57 (iii) 4161 (c) a=!;x=!t}
y = ±t2
(d) a=!;x=
y = !t2
t}
(d) (i) 650 (ii) 5
(e) (i) 480 (ii) 1 (iii) 4674, 20 530, 867 980 (e) a= -1;x = -21} (f) a= -4; x -8t}
y = -(2 y = -412
2. (a) (i) N = 200 + Ae 0 ' 751 (ii) 3046
(b) (i) 685 (ii) 494 (iii) 266 (iv) 205 2. (a) x 2 = 8y (b) x 2 = 12y (c) x 2 = 16y
(v) 200 (d) x 2 = y (e) x 2 = -16y (f) x 2 = -2y
(g) x 2 = -!y (h) x 2 = -4ay
N
800 3. (a) (i) I -3 -2 -1 0 I 2 3

X -3 -2 -1 0 1 2 3

1 1
y 4! 2 2 0 2 2 4!
0 12 5 10 20
(ii)
3. (i) 284 (ii) 447 (iii) 3425 (iv) 179 007
as t --> rf:), N--> rf:)
4. (a) (i) 159 (ii) 132 (iii) -105
(b) 4·159
(c) as t---> rf:),N--> -rf:)

5. t 0 1 2 5 10 20

N = f(t) 40 42·2 43-9 47·1 49·2 49·9

as t --> rf:), N---> 50 from below.

296
(iii) x 2 = 2y
(iv) (0, 0); (0, t)
(v) Y = -t I. (a) x - 2y + 4 = 0 (b) 3x - 4y + 12 = 0
2. (a) (i) 3x - 4y + I 0
(b) (i) f -3 -2 -I 0 I 2 3 (b) (i) 4 (ii) 4x - 3y + 6 = 0
(iii) ( -!, i) (iv) t = -t
X -18 -12 -6 0 6 12 18 3. (b) 4a units
4. (b) ! (c) p = -q
y -27 -12 -3 0 -3 -12 -27 5. (a) 4x - 3y + 3a = 0

(ii) -18 -12 -6 12 18

I. (a) x + y + I = 0; x- y + 3 = 0
(b) x- 2y- I = 0; 4x + 2y- 9 = 0
(c) 2x- y 2 = 0; x + 2y- 6 = 0
-12 (d) 2x- y- 4 = 0; x + 2y- 12 = 0
(e) X + )' + 2 = 0; X - )' + 6 = 0
-18 (f) X-)'- 3 = 0; X+)'- 9 = 0
(g) 2x - y = 0; x + 2y - 5 = 0
-24 (h) 5x + y + 6 = 0; x - 5y + 22 = 0
(i) 2x - y 4a = 0; x + 2y - 12a = 0
-30
(j) 4px- 16y- ap 2 = 0;
64x + 16py - ap 3 - 32ap = 0
(iii) x 2 = - 12y 2. (a) (i) 2x + y + 2 = 0; 2x - y - 2 = 0
(iv) (0, 0); (0, - 3) (ii) X - 2y + 6 = 0; X + 2y - 6 = 0
(v) y = 3 (iii) (0, - 2) (iv) (0, 3)
4. (a) x + 2y - 4 = 0 (b) (i) X- 2y- 2 = 0; X+)'+ 4 = 0
(ii) 2x + y - 9 = 0; x - y + 12 = 0
(iii) ( -2, 2) (iv) (-I, 11)
(c) (i) x- y I = 0; x + y + I = 0
(ii) X + )' - 3 = 0; X - )' + 3 = 0
(iii) (0, -I) (iv) (0, 3)
(d) (i) 2x - y - 12 = 0; y = 0
(ii) X + 2)' - 36 = 0; X = 0
(iii) (6, 0) (iv) (0, 18)
3. (a) (4, I) (b) (-6, 3) (c) (10, 5); t =I.
4. (a) m < -8 u m > 0 (b) m = 0, -8
(c) -8 < m < 0
3 5. (a) 3x - y - 27 = 0
(b) 9x - 3y + 19 = 0; parallel
6. (a) (8, 8); ( -4, 2)
(b) 2x - y - 8 = 0; x + y + 2 = 0
(c) (2, -4)
8. (d) (i) (t, -I) (ii) (1, 2t)
-3 3
9. y=x-a;y= -x-a;y= -x+3a;y=x+3a
10. (a) (2, I); (4, 4)
(b) X+)'- 3 = 0; X+ 2y- 12 = 0
(c) (-6,9)
-3 II. (a) (i) 2x - y - 4 = 0; 2x + 4y + I = 0
(ii) (!!, -1)
(b) (i) X- 2y + 12 = 0; X-)'+ 3 = 0
(c) y = x2 - I (ii) (6, 9)

I. (b) (i) 3x - 2y - 2 = 0 (ii) x - y = 0


(iii) 6x - y + 2 = 0 (iv) x - 2y - 2 = 0
(v) 2x - 3y - 3 = 0 (vi) y = I
2. (a) x- y = 0 (b) (0, 0); (8, 8)
(c) y = 0; 2x- y- 8 = 0
3. (a) 3x- 4y + 16 = 0 (b) (16, 16); ( -4, I)
(c) (6, 8t)
-1
4. (a) (i) y = 4 (ii) x - 4y + 8 = 0 (b) (8, 4)
5. (b) 3x+4y-4=0 (c) (l,i);(-4,4)

297
(d) (i) (4, 0) (ii) (0, I) (iii) (~, -I) 10. (b) (i) !rq sin (a + {J)
6. (a) x - 4y + 12 0 (b) (6, 4!);( -4, 2) (ii) !rl sin a, !ql sin fJ
(c) (1, 3!) (d) (i) (I,!). (iii) I = r cos a, I = q cos fJ

I. (b) (i) x + py - ap = 0 (vi) rhombus I. (a) (i) tt (ii) -is (iii)¥;


13. (c) (i) q2 + qr + 2 = 0 (b) (i) m (ii) m (iii) m
14. (a) (ap(l - p 2 ), a(2p 2 + I)) (c) (i) tt (ii) -is (iii) ¥-;
15. (b) (i) (ap,O) (ii) a(p 2 + l)t,a(p 2 +I) (d) (i) m (ii) m (iii) m
(e) (i) m (ii) -m (iii) -m;
(f) (i) m (ii) -m (iii) -m
.J3
(g) ( ! ) - (11") 1
-- (iii) -J3;
p- 2x 2 2
I. (a) y = - - · (b) y = 0, the x axis.
2p 1112 12
(h) (i) ____3!!!!_ (ii) - (iii) ____3!!!!_
3. Parabola, x 2 a(4y - a) t> + 1112 /2 + 1112 1112- t>
10. Parabola, x 2 = a(y - 3a) I
II. (b) Parabola, (x - 1) 2 = 2(y - !) 2. (a) (b) J2 (c) J2 (d) J3
2 4 4 2
12. (a) y = 0 (b) x 2 = 3(y 3)
15. (b) x 2 = a(y - 3a) (e) J2 (f) - J3 (g) J3 (h) 2- J2
2 2 2 2

(i) 1 (j) J3
3
2 2 tan 5A
4. (a) 2 cos 0 (b) cos 4A
(c) I - tan 2 5A
(d) I - sin 20 (e) cos 6A (f) 1 + sin 20
I. (a) sin a cos fJ + cos a sin fJ
(b) cos a cos fJ + sin a sin fJ g cos 20
() •20-
- - sm (h) 2 sin~ cos~
(c) sin 2x cosy + cos 2x sin y 2 2 2 2
(d) cos 3x cos 2y + sin 3x sin 2y 2 tan~
(e) tan 2x - tan y (f) tan 3A + tan 2A (i)
2
1 + tan 2x tan y 1 - tan 3A tan 2A 1 - tan 2 ~
(g) 2 sin 3A cos B (h) 2 sin 2x sin y 2
2. (a) cos (a - {J) (b) sin (a + {J) 5. (a) cos 20 (b) sin 20 (c) 1 (d) cos 2A
(c) sin (2a - {J) (d) cos 2(A + B) (e) tan 2A (f) cos A (g) cot 2 0 (h) tan x
(e) sin (2F - E) (f) sin (A + 40°) 7 ( ) (') 30 = 3 tan 0 - tan 0.
3

(g) tan (3A + 2B) (h) tan (2a - 40°) . a I tan 1 3 tan 2 0 '
(i) tan 10 (j) - sin 45 cos 30 = 4 cos 3 0 3 cos 0
(k) sin 7A (I) cos A (d) sin 30 = 3 sin 0- 4 sin 3 0;
3 . (a) .j6 J2 (b) .j6 - J2 (c) 2 + J3 cos 30 = 4 cos 3 0 - 3 cos 0
4 4 8. (a) ~- ! (b) £- (c) n
(d) ! - ~ (e) -£- (f) fi
(d) 2 J3 (e) .j6 + J2 (f) -(2 + J3) 9. (a) !(x - sin x) + c
4
(b) -!sin 4 x + c
(g) ~ (h) !2 (i) J3
2 (c) !x + fa- sin 2(ax + b) + c
(j) I (d) tx- +.,sin 2(ax +b)+ c
4. (a) 2 sin A cos B (b) 2 cos A cos B 10. (a) ¥- units 3 (b) ¥- units 3 (c) n units 2
(c) -2sinxsiny (d) 2cos2xsiny (d) £- + 1 (e) 'g units 2
(e)sin (Q - R) (f) cosy ll.(a) sin 2x = 2 sin x cos x; (i) -!cos 2x + c
(ii) i - i-f sin 4x + c (iii) 'g + t
(g)
5. (a)
sin 2A
(i) ~ (ii) ¥s-
3
(iii) *
(h) tan A + tan B.
(iv) m (b) !(1 - cos 4x)
(c) }x - ! sin 2x + i-f sin 4x + c
(b) (i)
4
~ 0+ (ii)
4
~ 0+
3
; 66·87°, 6·8r (d) ir
12. (a) sin 30 = 3 sin 0 - 4 sin 3 0; -f2(8 - 5,j2)
(c) (i) .j6 ~ J2 (ii) .j6 ~ J2; 75°, 15° (b) :I(9n - 16) units 3 ;
(c) (i) (,j2- 1) units 2 (ii) :I units 3
(d) (i) m (ii) , , (d)xcosx;:I-1 (e)40sin20;!
(iii) N; a - fJ =
2
39·31°; a+ fJ = 84·55°
a = 61·93°; fJ = 22-62°
(e) (i) -!, £; ft, -¥
(ii) 216·8r, 112-62°, 104·25° 2
(iii) ~, -*,
-N; 104·25° l. (a) U (b) i_ (c) J3 (d) .!2
13 3 II
8. (b) (i)
Pi
f, -2
1
(d)
I
3
I
(e)-
21
(f) 55
3

298
2. (a) 6° (b) 26° (c) 15° (d) 15° 3. (a) 90°,270°, 48·19°, 311·81°
(e) 27o (f) 45° (b) 90°
3. (a) 0·32 (b) 0·14 (c) 1·43 (d) 0·79 (c) 0°, 180°, 14·04°, 194·04°
(e) 0·91 (f) 0·86 (g) 1·05 (h) 0·32 (d) 90°, 210°, 330°
4. (a) 61·93° (b) 18·43° (c) 10·30° (d) 0° (e) 60°, 300°, 48· W, 311-81 o
(e) 90° (f) 45°, 225°, 153-43°, 333-43°
5. (a) 38°, 100°, 42° (b) 52°, 33°, 95° (g) 35·05°, 215·05°, 99·95°, 279·95°
(c) 90°, 45°, 45° (d) 27°, 90°, 63° (h) no solution
(e) 90°, 45°, 45° (i) 270°, 30°, 150°
6. (a) 23°, 112°, 45° (b) 90°, 45°, 45° (j) 50· w, 129·86°, 334·26°, 205·74°
(c) 7°, 98°, 75° (d) 52°, 8T, 42° 4. (a) 90°, 270°, 30°, 150° (b) 0°, 360°, 120°, 240°
7. (a) 68°, 68°, 45o (c) 30°, 150°, 270° (d) 60°, 300°, 180°
(b) 82°; 82°; isosceles; t; t; bisects the angle Q (e) oo, 180°, 78·46°, 281·54°
(c) (i) 34·5°; 34·5°; equal; (ii) 54·0° (f) 45°, 225°, 116·57°, 296·57°
8. (a) x + 3y - 5 = 0 (g) 0°, 180°, 52·24°, 127·76°, 232·24°, 307-76°
(b) 3x + y - 18 = 0; x - 3y + 4 = 0 (h) 22-98°, 157·02°, 219·82°, 320·18°
9. (a) (i) (1, I) (ii) 71·57° (i) 9·W, 80·2T, 99·W, 170·2T, 189·74°, 260·27°,
(b) (ii) 1·23 radians 279·74°, 350·27°
10. (a) No; +0·12°; 19-44° (j) 0°, 180°, 360°
11. 30° 12. I ; perpendicular 5. (a) 115·333°, 318-40T (b) 90°, 323·130°
(c) 94·867°, 219·894° (d) 86·565°, 326·565°
(e) 72-412°, 220·208° (f) 270°
(g) 123-648°, 354-424° (h) 67-380°
I. (a) tan 8 (b) sin 8 (c) cos 8 6. (a) 1·129, 3-868 (b) 5·262, J.811
(d) ___!__ (e)~
I
(f)
J3
-T
(c) 5·760 (d) 0·777, 4·219
(e) 2-474, 6·161 (f) 0·955
J3 J2 (g) 1·076, 4-418 (h) o·78s m
2
2. (a) (1 - 1)2 (b) I + 21 - 1 (i) 0·351, 2·203, 3·492, 5·344
12 +I 2
1 +I (j) no real solution
2
(c) 1 (d) 1 + 21 - I
1 + 1
2

(e) 2(21 - I) (I + 2) (f) !


1 + I
2
I
I + 21- 12 I+ I
(g) 12 + 21 - 1 (h) 1-=t
2 2
(i) 1 + 1 (') 2(1 + 2)
I - 12 J 12 + 1
3. (a) (i) I + 12 (") 1 + 12 (" ') 1 - 12
21 II 1 - 12 11l -2-1-
2 I. S = Ha +b)
(b) (i) (I + 1) (ii) !_
I - 12 2 2. (a)~
ex
(b) -2 tan 2x
(d) 1 + 12 ( ) 8
(a - b)1 2 + (a + b) g tan (c) (6x)e 3 x'+ 4
- b ± .) b 2
- a 2
+c 2 3. 1500cm3 /s
(h) (i) - - - - - - 4. (a) !(e 4 - e- 2 ) (b) 0
a+c 5. y 2x 3 - 2x 2 + x - 2
4. (a) 0, ~, 2n (b) 'j, ~ (c) 1·23, lf 6. pq 2
= -I; x = a(y - 3a)
(d) 0·93, 5·11 (e) 2·21, ~ (f) 2·21, 2·50
(g) 0·64, 0·93 (h) 1·18, 4·07

I. (a) 2·45 radians


(b) (i) 49 em (ii) 490 cm 2
I. (a) 45°, 135° (b) 30°, 330° 2. II = 9; II = 16
(c) 30°, 60°, 210°, 240° (d) 90° 3. 8 = 45°, 135°, 225°, 315°
(e) 15°, 105°, 195°, 285° I I
(f) 70°, 110°, 190°, 230°, 310°, 350° 4. - J2 <X< J2
(g) 60°, 120°, 240°, 300°
(h) 90°, 270° (i) 90°, 270° Maximum at(~, 2 f)
(j) 33-75°, 56·25°, 123-75°, 146·25°, 213·75°, 236·25°,
2
303·75°, 326·25° Minimum at (_ ___!__ - J2)
2. (a) 0·72, 5·56 (b) H8, 5·94 \ J2' 3
(c) 2-30, 3-98 (d) 0·52, 2-62, 3-67, 5·76 12log 10 . .
5. mmutes ~ 40 nun
(e) 0·79, 3-93 (f) 1·05, 2·09, 4·19, 5·24 log2
(g) 0·79, 2-36, 3-93, 5·50 (h) 0·73, 2-41, 0·34, 2-80 6. 48 m/s; 125t m

299
4. (a) ( -t, #) max, (3, -18) min
(b) 10 54 " cubic units
b c
I. a+ f3 = --,a/3 = -;k
a a
±3 5. (a) sin x + x cos x (b) _2_
cos 2x
3. (a) (0, 10), maximum 6. 2 min 35·7 s after the engines stopped
(,[2, 6), minimum
(- ,[2, 6), minimum
(b) I square unit

4. -i
5. (a) S
unit/; econd
1
= -[2a + (n - l)d], S
a r" _
= ( 1
),
2 r 1
11
(b) S = ; I; 7 (b) x4 I I
I. (a) x 3 + x 2 + x + c; - +3x•+c
4 2x 2

(c) ~x 3 + ~xt + 2.JX + c


I. (a) ~ (b) !s (c)# (d) (5x + W + c
4. 18°26' 15
5. ~(e 2 4e + 5) square units, approx. 2·38 square units
- (e) !(2x - 5)1 +c (f) !(3x - 2)1 + c
6. 33·6 hours (g) 4 + log, x + c (h) i log, x + c
(i) t log, (2x + 3) + c (j) fesx + c
(k) !e2x-3 +C (I) -ie-4x+3 + c
2. (a) log, (x 2 + I) + c (b) log, (x 2 - 3x + 5) + c
I. r = 0·135 (c) !Jog, (x 2 - 1) + c (d) t log, (x 5 - 5) + c
2. I mjs (e) ~log, (x 2 + 5) + c (f) log, (ex + I) + c
-3ab 3b- a (g) log, (sin x + 1) + c (h) log, (sin x + cos x) + c
3
' x = 5a- 15b' a (i) log, (ex - e- 2 x) + c
4. (a) (i) sin 2x + 2x cos 2x 3. (a) -tcos 3x + c (b) 2 sin (!x) + c
(ii) x- I (c) t tan (5x + 4) + c (d) 2 sin x t sin 2x +c
-!sin 2x + c 2x + c
2
.Jx - 2x (e) (f)
5. b 2 = ac (g) sin- 1 J + c (h) sin- 1 ! + c
6. (a) (0, 3), y = -3 (i) ttan- 1 1' + c (j) !tan- 1 ~ + c
(b) 4x- y- 2 0 . -1 -X+ - 2 + c
(k) sm (I) stan
1 -1 X - 3
-5- +c
3
4. (a) 2t (b) 2 (c) t (d) TI:
I. (a) A = 325 (e) 4 (f) ;'j (g) i log, 5 (h) 25t
(b) A = 2, B = 3, C = 4 (i) H U) ~ (k) ~ (I) 2t
2. 15 square units (m) t(e - I) (n) 0 (o) log, 3
3. (a) 2x 3 - 3x + C
Jx 5. (a) log, x; x(log, x - I) +c
(b)~+ c I
(b) ; 0·345 (c) It
(c) -cos 4x +C )x + 9
2

4 (d) 0·119 (e) 0·128


4. 7-47
5. (4, 0)
6. 105 850 (Unless otherwise stated answers are in square units)

I. (a) I (b) log, 2 = 0·693


(c) e- I 1·718 (d) !f = 0·785
I. (a) 3x - I (e) log, 10 = 2·303 (f) 130~
x+3 (g) 2 (h) 2-350
(b) e = 199°28' or 340°32'
(i) ~ = 1·047 (j) 1!=3·142
2. 29·69 m
3. (%, 0), r ~
2. (a) x = 0; H (b) X= 2; 16
(c) x = ;'[; 1 (d)x=2;6t
y 2 = 5x- 4
4. 26m
(e) x = -t, x = I; 3! (f) X = H; ~ + 2J3
3. (a) 9 (b) 4! (c) 2t
5. (a) (x - W = Y + ± (d) 2! (e) ~ - t (f) 18
(b) t, (t, -t), y = -t (g) 2 (h) 0·114 (i) 18·135
(j) 3(n - 2)
4. (a) (i) n (ii) n
I. 16 em (b) g(x) = x(log, x - I) + c
2. 2 square units (c) log, {x + .Jx2 + 9} + c
3. 20g, 10·18g (d) (i) f; (ii) I (iii) y = x - I (iv) t
300
. I
(e) (1) y = -x + -.jJ - -rc (iii) 1·543
2 2 6
(f) 2)2
(g) (i) 224 (ii) 97
I. I = I, X= -2 and I = 5, x = -34
= ~,y* =
7
(h) (i) x* : (ii) $12·38
2. (a) x 2
(b) ;( = I~ m/s
(c) .\' = ±
m/s 2
(Answers are in cubic units unless otherwise indicated. In 3. X= 50
general they are left as exact answers) 4. When I = I, X = 2
When I = 2, x = 0
I. (a) ¥ (b) .flrc When 1 = 3, .\' = -16
(c) ~ (d) ~(e - I) 5. (a) When 1 = 2 (taking 1 as positive)
(e) T\;(rc - 4 + 2)2) (f) W' (b) X = 16, .\' = -12
(g) !rcm 2 h 3 (h) 32 ;gsrr 6. (a) 53! m
(i) 8rc (j) 4rce (b) 20 m/s
2. (a) 8rc (b) 12r
(c) 4rc(2 log. 2 - I) (d) 18rc
(e) ~ (f) w I. .\' 3
(g) ~ (h) 32rc =
3. (a) (i) 2rcah 2 (ii) 288rc cm 3 2. 14 m/s 2
(b) (iii) 468rc cm 3 3. (a) I m/s 2
4. (a) 5 W (b) (iii) 134·04 (b) -2m
(c) ±4 m/s
4. (a) v = 2Jx + 9
(b) x = -9 m
I. (a) ! sin (2x + 3) + c (b) ! tan (5x + I) +c (c) v = +6 m/s
(c) ±log. (4x - I) + c (d) !e 5 x+ 2 + c 5. v = J6x + 54, ±8·3 m/s
6. v = 6·1 m/s
I
(e) sm
4
. - 1 [4(x - I
--
3
+ c -J>l 7. v = 2·8 m/s
8. v = 1·5m/s
(f) _l tan-1 5(x + 7) + c 9. v = ± 1·4 m/s
20 4
10. (c) v = 1 + 2, 7 m/s
3. (a) ~(3x + 1)1 + c (b) 2J2x + I + c II. (a) x = 12 + 51
(c) 2,)2x + I, + c (d) ~(x 3 - I) + c (b) v = 21 +5
(e) !(I + x 2 )' + c (f) t(x 2 + 4)1 + c
I ' I
(g) (2x - I)' + c (h)--- -+ c
3 4 sin 2 2x
I. 5 seconds, 30i m
4. (a) 2 sin JX + c (b) -i cos 3 21 + c 2. 0·3 second
(c) 2Jx + 3 + c (d) ~)x +3+c 3
3. 176·4 m, 58·8 m/s
3x 2 - 6x + 4 2 '
+c
3cx- 2)'(x + 4) + c
(e) · ( -·x) 3 (f) 4. 31·3 m/s, 3·2 seconds
3 2
5. (a) 5·1 m
2(x + l)t(3x - 2)
(b) 0· 37 s, 6·4 m/s
(g) 15 +c
1·68 s, -6·5 m/s
(h) -log. (cos x) +c 6. 27·5 m, 49 m/s downwards
JX (J') - 2(cos x >'' 7. height 241·6 m, 40·8 m/s upwards
2 +c +c
(1') 2 sm
. -1
3 8. 7·2 seconds, -43·6 m/s
5. (f) i sin- 1 G) + !,.)9 - x + c 2
9. 1·48 seconds after the first stone is let fall
10. The objects meet 6 seconds after the first was projected
at a height of 117·6 m
II. 98 m
I. (a) 4~ (b) 6! (c) 3-fs (d) 3-fs 12. 15·06 m
(e) (i) 2 (ii) 2 (f) 5t (g) 9!
(h) 9! (i) fz (j) 3#
2. (a) 2·120 (b) 2·333 (c) 0·504 (d) 0·732
(e) 2·162 (f) 0·828 (g) 0·644 (h) 0-414 I. height 15·2 m, range 105·2 m, max. range 121·5 m
(i) 0·924 (j) 0·400 2. 80 m 3. 60°35'
3. (a) 8·0 (b) ! (c) (ii) -0·806 4. After 6·2 seconds with a horizontal range of 62 m
(d) 1·557 (e) (ii) -4·216 5. 626 m
(f) (i) 54·8 units 2 (ii) 54·8 units 2 6. 4·04 seconds, 20 m
4. (a) (ii) 0·280 (b) 0·15 units 2 7. 19·6 m
3
(c) (i) 3·75 units 2 (ii) ~" units 3 8. 4·03 m approx.
(d) (i) 8~ units 2 9. V = 31·3 m/s, o: = 45o

301
10. V = 16 mjs, a = 51 °23' (b) y =X- I, {-1 ~X~ 3}, {-2 ~ y ~ 2}
II. V = 328 mjs, a = 13°30'
12. 512 m
13. -
v2 m, 15° and 75°
g
14. 45°

I. X =20 cos 30° . I


y =20 sin 30° . 1 !gl 2
x =34·6 m, y = 0·4 m
2. (a) X 40 cos 35° . I
y = 40 sin 35° . 1 - !g1 2
gx2
(b) y = x tan 35o - 3200 c~s2 35o
(c) 153m
3. X =60 cos 40° . I, (c) y = -!x, { -8 ~ x ~ 8}, {-4 ~ y ~ 4}
y 60 sin 40° . 1 - !gt 2 , 7·9 seconds
4. 61 m/s, 62 m

I. v = ±8
2. (a) T =fin (b) T= 2f\: -1

4n
3. (a) 40 cmjs (b) mjs
3
4. ±2J7 cmjs, -6 cmjs 2
5. 2J3 mjs
6. -" -2n m/s 2
.'i: = J'ln
mjs
7. (a) 0·84 seconds,
(b) -,[5 mjs, -2 mjs 2
8. 4·3 cmjs, 2·7 cmjs 2
9. 4·2 cmjs, 1·6 cmjs 2 (d) y = -;(X= -xt, { 8 ~ x ~ 8}, { -2 ~ y ~ 2}
10. 4·6 m
12. n seconds, 20 cmjs 2
13. (a) a=¥ (b) I second
14. 6n seconds, 9 em
15. (a) 12·6 cmjs, 0 cmjs 2
(b) 0 cmjs, 26· 3 cmjs 2
(c) 9·4 cmjs, 17·6 cmjs 2
16. 10.10 a.m.
17. 2 ·007 seconds

I. (a) y = 2x, {0 ~ x ~ 4}, {0 ~ y ~ 8}


(e) y = 2 - x; {-I ~ x ~ 5}, {- 3 ~ y ~ 3}

f= f'

302
(f) y x2 - 1, {0 ,;;; x ,;;; 2}, { - 1 ,;;; y ,;;; 3}

1
I. (a) (i)
2-yx
r;:; (ii) X= y2 (iii) 2JX
(b) (i) 5x 4 ; (ii) X = y!; (iii) _I_
5x4
1 2
(c) (i) · (ii) x = 2 y ;
2J2- x'
-1 3 (iii) -2J2- X

-1 (d) (i) 2x; (ii) x = J.YTI; (iii) _!_


2x
2. (a) x=y 3 ; (b) 3y 2
3. (a) x = e3 Y; (b) 3e 3 Y· _!_. (c) y =!log
3 x· _!_
3- X , 3x' e' , 3x
2. (a) y = - -, {all real x};
2
(b) y = Jx + 2, {x;;, -2}
4. (a) (i) x = y2 +I; (ii) 2y;
2 ~;
(d) y *'-
(c) y =}, {x # 0, all reals};
2, {all reals, x # 0}
3. (a) y = log. x
(b) (i) X )'

(c) (i) x = 27y- y 3 ;


+ )' 5 ; (ii) I + 5y 4 • _ I _4,
, 1 + 5y '

(ii) 27 - 3y 2 ; I
3(3 - y)(3 + y)
5. (a) -_2, (b) -~.d.
2.» 3
f
X=~; 1.
1
2
6. (a) X y(9- )' );t. (b)
1- y

I. (a) f CbH (c) 0


(d) -~ (e) {! (f) 0
(g)¥ (h) 0 (i) {!
(b) y = log 10 x; {x > 0}; {all reals} (j) -~ (k) -{! (I) ¥
(c) y = JXTI; {x;;, -!}; {y;;, 0} 2. (a) 51·758° (b) 21 ·936° (c) 39-679°
(d) y = 3 2x; (d) -9·869° (e) 109· 184° (f) - 15-440°
3. Answers in radians
(a) -0·2753 (b) 2-9682 (c) 1·5510
(d) -1 ·4743 (e) -0·2912 (f) 0·0050

4. (a) ~ (b) -~ (c) 2n


3
(d) 2n (e) J_ (f) J_
3 J2 J2
(g) J3 (h) 1 (i) n
2
5. (a) 1·50 (b) 3·0 (c) 30·0
(d), (e) outside domain of function
7. (a) (i) -2 ,;;; x ,;;; 2; -~,;;; y,;;; ~;

x-4
4. (a) y = -·- ; (b) ( -4, -4)
2
5. (a) (i) {x;;, 0}; (ii) y = .ft; (iii) {x ;;, 0}
(b) (i) {x ;;, 0}; (ii) y = JX=l; (iii) {x ;;, 1}
(c) (i) {x;;, 2}; (ii) y = JX + 2; (iii) {x ;;, 0}
(d) (i) {x ;;, 0}; (ii) y = :;;x; (iii) {x ;;, 0}
(e) (i) {x;;, !}; (ii) y 2 - 2y - X 0;
(iii) {x;;, -1}

303
(ii) -t ~X ~ t; -~ ~ Y ~ f; (ii)

-1

(iii)

(iii) -1 :;:; x:;:; 1; -rc :;:; y :;:; rc;

8. (a) (i) e = nrc + ( -1)"(~)


(ii) {-If, -¥, ~' ~' .9f-}
(b) (i) e = 2nrc ± ~
(ii) {-l~n, _lj'r, -lj\ -¥, -'j,},¥,~,
1jn, l~n}
(c) (i) e= nrc + 'k
(ii} {-1Jn, -~,~,J;, l~n}
(d) (i) e = nrc - ~
(ii) { -~, -~, -~, ¥, -"?f}
(e) (i) e
= nrc + (-1)"( -~)
(iv) - oo < x < oo; -1 < y < 1 (ii) { -lf, -lf-, -j-, ¥, ¥}

2. (a) 0 (b) 0 (c) -1 (d) 0 (e) 0


3. (a) !¥- (b) /r (c) ¥s (d) ! (e) H
(f) ¥s (g) -% (h) ¥s
4. (b) 't
6. (a) ~ (b) lf
7. (d) (ii) tan- 1 (~)
1-x
(f) (i) sin 2a 2 sin a cos a

7. (b) (i) -1 :;:; X :;:; 1; 0 :;:; )' :;:; ~


(ii) -1 :;:; X :;:; 1; -1 :;:; )' :;:; 't.
~1 ~ 9x
(c) (i) 3
1. (a)
(b)
J1=7 2

2x
(c)
~1- x 4
(d) ~25 1_ xz
1 (f) -5
(e)
~2 + x- x2 ~1 - 25x 2
-1 (h) -2x
(g)
~16- x 2 JI=7
-1
(i)
(j) 16 : x 2
2~x(1 - x)
5 3
(k) (I)
1 + 25x 2 2x 2 -2x+5

304
(m) -1
Jx(1 - x)
(n) I
-I
+ x2 (c) sin- 1 }s + c (d) 1tan- 1 ~ + c
5 5

tan- fl +
I 1 X
(o) I (e) J3 c (f) 3 tan- 1 3x + c
)4 + 3x- x2
2. (a) a (b) sin- 1 x + _ _
x_ (g) tan- 1 (x + 3) + c (h) -sin- 1 ~ + c
6
)1- (ax+ W ~
1 (1") I
(c) 2sin- x (d) 3 + 2x tan- 1 (}) J5 tan- J5 +
I X
c
J1=7
(e) -1 (f) I 2. (a ) I tan -1 X +c (b) 2 tan- 1 2x + c
3 3
(g) 1 (h) 1
2Jx(1 - x) 2(1 + x 2 ))tan- 1 x (c) J7I tan- J?t
1
+ c (d) _1_ tan- 1 3!. + c
10 5
(i) ex (j) I 2
(e) ~ sin- 1 31 + c (f) ~ sin- 1 fit + c
~ x{1 +(log. x) }
-ecos- 1 (2x-1)
(I) sin- 1 x I bx h) I . _1 bx
(k) --;=;c;====;:- (g) - tan- 1 __:_ + c ( bSill --; +C
Jx(l - x) ab a
3. (a) ~ (b) 6(tan-1 f)2 (i) sin- 1 x- 2 + c (J") I tan -1 -X -
-2 + c
2
vI - 9x 2 4+x 2 4 4
(d) 2x sin - 1 ~
2 (k) sin- 1 x- b + c It -1 --+c
X - b
(c) (I) -an
(4 + x 2 ) tan- 1 f 2 a a a
1 2
b) I . _1 3x
(e) -1 (f) -9(1 + cos- 3x)
3. (a ) I sm
. _1 2x +c ( sm +c
(sin- 1 x) 2 . Jl=7 )1 - 9x 2 2 3 3 4
(g) (sin- 1 x) 2 (h) cos- 1 2x I . _ 2(x - 3) I 2x
(c) sm 1 - -- + c (d) - tan- 1 __:_ + c
(i) Jaa+x
-x (j) 2
2 3
(e) _1_ tan_, 3x + c
6
1t
3
_1 2x 6
4. (a) (i) I (ii) I; (b) I 15 5
(f) 6 an - -+
3
c

(c) 4: x2; (4 ~4;2)2 (e) n( f + ~) 4. (a) 7f:


8
(b) 7f:
6
(c) 7f:
2
(d) 7f:
6

5. (a) y - 7f: = 2_
6 J3 2 '
(x - ~) ·
y - 7f: =
6
- J3 (,
2 .
-3)
2
(e) 7f:
2
(f) 3n
2
(g) 7f:
2
(h) !!_
4b
(b) (J3, 1·2898),(-JJ, -1·2898) (d) y = x +I (i) 7f: (j) 7f: (k) J2n (I) !!_
6. (a) X < 0; X> 0 2 2 8 60
5. (a) (i) 1
(b) y ~~ = - ,;;n(x + \~} (ii)

y - 1!2 = J2 (x- + J._)


16 1! • J2
(c) (i) -I ~ x ~ I; _'/[: ~ y ~ 7f:
2 2
1
(ii) Jl=-7 sin- (x
2
) + 2x
2

)1- x4
(iv) vertical
(v)
(iii) 7f: units 2
4
(b) 4n units 2 (c) ~ units 2
6. (a) ~ units 3 (b) ~ units 3 (c) ~ units 3
I (b)
7. (a) - 3 < X < 3, )' ~ - ; X
3 (9 - x 2 ) 2
(c)
y

.L
3

0
I. (a) sin- 1 ~ + c (b) sin- 1 ~ + c
3 5

305
(d) 1·4595 units 2
8. (a) 0·5793 units 2 (b) 0· 5744 units 2
5 _ 2e cos-• (2x)
1. (a) )1 - 25x 2 (b) )1 - 4x 2
2. ! Jog (x 2 + 2x + 2) + c
3. (a) 1·5 mjs 2 (b) 4 mjs
4. tcm/s
5. 1 ~X~ 1, .j2., -.j2
6. (a) 6
1. (a) e = 0°, 30°, 150°, 180°, 360° (c) 6·65 cm 2
2. trr
mjmin or approx. 19·9 em/min
3. (a) 2x + -f - 4
+ c (b) 4xl + c
(c) ! sin 2x + c
4. t = 1, s 6 l.(a):J (b)tsin 3 x+c
15 2. 1 048 560
5. 120° , 6·5 cm 2 or 4 cm 2 J3 3. (a) -17 (b) -3 < x < 3f
4. 0·46 seconds, 8·3 cmjs, 14·8 cmjs 2
6. Perimeter of !:c,ABC is equal to the diameter of the
5. (a) h (b) x = ± 3
fixed circle and so is constant
6. (a) (i) B = -6 (ii) B = f

1. (a) 8 = i, ¥, 1f, l~n


(b) e = 0, if, -'f, n, 2n
2. (a) 0 (b) e
4. l:c,AOB is 4 square units
5. a 5, v = 20 cmjs
1. (a) 3; 4; 9 (b) 4; 4; -1
(c) 5; 1; 8 (d) 4; -6; 3
-6 (e)5;2;1 (f)6;3;0
2. (a) ex(sin x + cos x) 2. (a) 4 (b) -1
(b) (1 + 2x) 4
(c) -5 (d) 3, -5, -9
(c) -tan x
3. (b) A = !(e 2 - e) (c) V i(e 4 - e2) 3. 5x4 + x 3 + 2x 2 - x - 5; 4
4. e 60°; e = 60° ± n(360°) 4. 5
5.(a)3 (b)3 (c) 5 (d) 4
6. (a) -1, 20, 0 (b) 2, 4, 14
(c) -8, 0, 24
1. e= :r or 5-64 7. (b) (c) (f)
2 tan e 8. (a) 6 + 6x (b) 3x 2 - 4
3. tan 28 =
I - tan 2 e (c) 7 (d) 5x 4 - 8x + 3
2 (e) 15x 2 + 6x - 3 (f) 18x(3x 2 + 2) 2
4. (a) cos (x + y) = cos x cosy - sin x sin y; n + 9. degree n - I
8
(b) 1 + tan- 1 ! = 1-464 10. (a) x = 3 or x = -2 (b) 3, -2
6. 2·16 approx.

1.1,-1,2
1. .Ji.n
seconds 2. (a) 0, 1, -2
2. a = -!, b -1, k = -1! or a= -t, b = -2, (c) the graph begins in a negative quadrant and ends
k = -1! in a negative quadrant
3. (a) x + 3y - 3 = 0 (b) 11! 4. (a) 0, -1,2 (b) 4, 1, -f
4. (a) V 24x(l44 24x)t (b) x = 4 (c) -1, double zero at 2
5. $144-61 (d) 1, double zero at -3
6. (a) 41°11' (b) 11·8 em 5. (a) (x- 2)(x + 1);2and -1
(b) (2x + 5)(x - 3); -2! and 3
(c) (5- x)(3 + x); 5 and 3
6. (a) -2, -1, 1, 3 (b) 0, -2, 1, 1
1. log. 3 (c) 0, 0, 2, 2 (d) 0, 5, 2, -3
2. (a) x +y - 6= 0 (b) 8 7. ( -t, 1~), maximum; (1, 9), minimum;
3. 1·158 ct, 9#), point of inflexion
4. (b) WC12' 8. (a) (1,2)maximum;(-1, -2)minimum;
5. (a) cos nx + c (0, 0) point of inflexion
n (b) (-3, 1)maximim;(-1, -3)minimum;
6. 47"18' (- 2, - 1) point of inflexion

306
(c) (O,O)maximum;(l, -l)and(-1, -!)minimum, (i) (x 5)(2x + 1)(2x - 3)
(j) (x- 3)(x + 4)(2x 1)
1 --5) and ( - 1 --5) pomts
( -.[3'
- . of m
. fl exwn
.
9 J3' 9 3. k = 2
(d) ( -1, 9) maximum; (2, -18) minimum; 4. b = -2
ct' 4t) point of inflexion 5. a= -13, b = -12
(e) (-3, 27) maximum; (1, -5) minimum; 6. -2, 2, -1
(- 1, 11) point of inflexion 7. P(x) = (x - l)(x - 2)(x - 3)
(f) (0, 16) maximum; (2, 0) and (- 2, 0) minimum; 8. P(x) = 2(x + l)(x - 2)(x - 3)
X = J, X = 2, X = -~
(~, 7~) and ~, 7~) points of inflexion
10.
(- 11. (a) x = -1 or 2 or -3
(b) x = 1 or2or -3
(c) x = 0 or -1 or 2
(d) x = 1 or -2 or 4
1. (a) P(x) = (x - 2) (5x + 25) + 58 (e) x = 3 or -tor -4
(b) P(x) = (x + 3)(x 2 + 4x- 14) + 50 (f) x = 2 or -1 or -5
(c) P(x) = (x + 2)(3x 2 + 3x - 1) + 2 12. (a) P(x) = (x - 1) 2 (x + 1)
(d) P(x) = (2x + 1)(4x 2 + 6)- 11
(e) P(x) = (x + 2)(x 2 + 4x - 2) + 6
(f) P(x) = (x- 3)(x 3 + x 2 + 3x + 11) + 33
(g) P(x) = (x + 3)(2x 3 - 6x 2 + 17x- 48) + 139 1. (a) 2 (b) 3
(h) P(x) = (x 2 + 3x)(5x 2 - 12x -2) + 12x (c) 2! (d) 8!
(i) P(x) = (x 2 - x + l)(x 2 + x + 1) 2. (a) -1 (b) -4
(j) P(x) = (x 2 + x + 1)(2x 2 + x - 3) + (3x + 4) (c) 1~ (d) -2#
2. (a) (x + 4)(x 2 + 2x - 4) 3. 5, -8
(b) (x + 1)(2x 2 - x + 1) 4. (a) x 2 3x - 10 0 6x + 7 = 0
(c) (x 2)(x 2 - 14x- 9) 5. k = -13
(d) (2x- l)(x 2 - 3x + 3) 6. -!, -t k = ~~
(e) (2x + 3)(3x 2 + x - 3) 7. k = It
(f) (x - 4)(x 3 + x - 2) 8. k = 7
(g) (3x - 1)(2x 3 - x 2 + 3) 9. b = -3, c = -4
(h) (x 2 + 2)(3x 2 + 3x- 10) 10. (a) 6 (b) 3
(i) (x + 2)(x 3 + 2x) 11. k = 6
(j) (x - l)(x4 + x 3 + 2x 2 + 2x) 12. (a) 4 (b) 2 (c) 1 (d) 2 (e)!
13. (a) x 3 - 10x 2 + 31x - 30 = 0
(b) x 3 - 3x 2 + x + 1 = 0
14. k = 1
1. -6 15. p = -5, q = 2, r = 8
2. (a) -1 (b) 12 (c) 6 (d) -1 16. (a) -6 (b) q = -31, r = 30
(e) 8 (f) -6 (g) -2 (h) 4 17.k=19
(i) -2 (j) 1 18. !1.{3yi5 = -5. Therefore at least one root must be
3. (a) -9 (b) -13 (c) 8 (d) -48 negative.
4. (a) 0 (b) yes 19. !1.{3yi5 = 0. Therefore at least one root must be zero.
(c) (x - 2)(x - l)(x + 1) 20. Sum of the roots is zero.
5. k = 10 21. x = 1 or 4 or 7
6. m = -7 22. x 2 or 4 or 8
23. k = 12
24. x = -tor -2 or 3

1. (a) (x - l)(x + 2)(x + 3)


(b) (x + l)(x + 5)(2x - 3)
(c) (x + 2)(x 2 + x + 5)
(d) (x + 3)(x- 4)(3x- 1) 2. X= 0·54
(e) (x- 2)(x + 4)(2x 1) 3.x=1·2
(f) (x - 3)(x 3 + 3x + 2)
(g) (x - l)(x 3 - 5)
(h) (x + 2)(x - 3)(x + 3)
2. (a) (x + l)(x 2 + 1) J. X= 1·68
(b) (x - 1) 3 2. X 1·88
(c) (x - l)(x + 1) 2 3. X= 1·43
(d) (x - l)(x 2 + x + 1) 4. X= 1·89
(e) (x + l)(x + 2? 5. X= 2·07
(f) (x + l)(x- 2)(x 3) 6. X= 2·71
(g) (x + 2)(3x - 1)(2x - 1) 7. X= 1·90
(h) (x 2)(x + 2) (x + 3) 8. X= 1·15

307
4. 1 + 5x + 10x 2 + 10x 3 + 5x 4 + x 5
6. 1 + 6x + 15x2 + 20x 3 + 15x4 + 6x 5 + x 6
7. 1 + 7x + 2lx 2 + 35x 3 + 35x4 + 2lx 5 + 7x 6 + x 7
8. 1 + 8x + 28x 2 + 56x 3 + 70x4 + 56x 5 + 28x 6 + 8x 7 + x 8
9.(a)6 (b)8
10. 13
8
11. (a) x 4 (b) X
12 . .\'11

These answers occur in various questions of Exercise 1.

2 3 4
1. (a) 1 + 6x + 12x 2 + 8x 3 (b) 1 - 2x + -3x - x- + -x
2 2 16
(c) 1 + 5x 2 + 10x4 + 10x6 + 5x 8 + x 10 (d) 1 - 4JX + 4x
(e) 1 + 9x 2 + 27x4 + 27x 6 (f) 1 + 4ay + 6a 2 y 2 + 4a 3 y 3 + a4 y 4
4x 2x 2 4x 3
2. (a) 1 + - + - + - + .. · (b) 1 - 30x + 375x 2 - 2500x 3 + . · ·
3 3 27
lOx 40x 2
80x3
(c) 1 - - + - - - + ...
3 9 27
3. 2 + 20x 2 + 10x4
4. 1·06152
5. 1·00903
6. (a) 0·96060 (b) 0·94119
7. (a) 20 (b) 12y'3
8. 34
9. p = 55, Q = -63
4 6
10.
4 6 4
(a) 1 +- + -2 + -3 + -4
1
(b) 1 - - + -- - 4 + -12
x x x x JX x xJX x
13. (a) 212 (b) 0 (c) 2 9
14 . (a) 1 + ~ + 902 + 270 + 405 + 243
x x x3 x4 x5

(c) 22-4
1 7J'5.

1. (a) 16x4 + 32x 3 + 24x 2 + 8x + 1 (b) 243x 5 - 405x 4 y + 270x 3 y 2 - 90x 2 y 3 + 15xy4 - y5
(c) xs + 10x3 + 40x + 80 + 80 + 32 (d) 8a 3
- 36a b 2
+ 54ab 2
- 27b 3
x x3 x5
~6 2 3 1
(e) .:< + 3x + 2 + 6 (f) 243x 5 - 810x4 + 1080x 3 - 720x 2 + 240x - 32
X X
(g) x 8 + 8x 6 + 24x4 + 32x 2 + 16 (h) a
3
x
3
3a 2 x 2 by + 3axb 2 y 2 - b3y 3
( 1') x 2 + 4 x+
. 6 +;:+x
4 1
2

2. (a) 40 (b) 1 (c) -20


3. 15
4. 2
5. p = 193, Q = -132
,2 8
6. 8.\ + 2
X
7. 144
8. 32
9. 42

308
1. (a) a 5 + 5a 4 b + !Oa 3 b 2 + !Oa 2 b 3 + 5ab 4 + b 5 (b) I - 4x + 6x 2 - 4x 3 + x 4
(c) 8x - 36x y + 54xy - 27y
3 2 2 3
(d) 125x 3 - 75x 2 + !5x - I
(e) I + 7x + 2!x 2 + 35x 3 + 35x 4 + 2!x 5 + 7x 6 + x 1 (f) 81 + 108,}2 + 108 + 24,}2 +4= 193 + 132,}2
2. (a) 36 (b) 5 (c) 56 (d) II
3. (a) 40 (b) -!08y (c) 140
4. (a) !5x 2 (b) 28x 4 (c) 540
5. (a) 9 (b) 8x, 8x 1 (c) 70 (d) T1 = 28x 6
1 3
6. (a) II (b) x y (c) T5 (d) 10
(f) - 1 °C5 . 3 5 . 45 x 5 y 5 = -62705664x 5 y 5
7. 70. 34 .2 4 • x 4 = 90720x 4
8. I, 9, 36, 84, 126, 126, 84, 36, 9, I
9. 568 + 328,)3
11. 416
13. I - !2x + 60x 2 - 160x 3 + 240x4 - !92x 5 + 64x 6 ; 1·885842
2 216
14. 24x + - 2
X
10 310 10
5 ·Y
15. (a) 22oy2o (c) 2oc10 ·
10
X
16. 35x4 , 2!x 5 , 14
17. x 4 + 4x 3 + 6x 2 + 4x + I
18. x 9 + 9x 8 a + 36x 1 a 2
19. Equal by symmetry of each line in Pascal's Triangle (or otherwise)
20. Equal. As above
21. (20 - (k - I)) = 20 - k + I = 21 k
. 20 X 19 X • · • X (22 - k)
22
I X 2 X ••. X (k - I)

1. (a) 210 (b) -192 (c) -448


2. (a) -20 (b) 10c4. 26 . 34 (c) 8 C6 . 26
3. T13 = 1BC122-12

4. Tn+1 = 2"C,.
5. (a) 20 (b) 126 (c) 17

6. 11 - r + I . x;,. = n + I
,. 2
7. 12 C4. 5 8 . 84 . r 4
8. k =II
9. I + 3x + 6x 2 + 7x 3 + 6x4 + 3x 5 + x 6
10. I - 8x + 28x 2 - 56x 3 •••

3. (a) 2" (d) II. 2"- 1


5. (a) 36 (b) 5 5

10. Only when more than 676 stations are required


11. 1000
12. 18, I in 12
I. 17 576000, yes 13. 30, 900
2. linl75760 14. I in 30
3. (a) I in 26 (b) I in 1000 15. 70
4. 676000 16. 96, I in 16
5. 10000000 17. 160000, 16, I in 10000
6. I 000000. More numbers were needed. 18. I in 2000
7. 1296, I h 48 min. 19. 1000, 26000, I in 676
8. (a) /2°9°6 (b) /o0s = -/4 20. (a) (i) 16 (ii) 16 (iii) 208
9. 60 (bh~

309
13. 3920
14. 168
1. 60
2. 120
3. 360
4. 24360 I. (a) is (b) t6s
5. 27000 2. 10, 10
6. 210 3. 210
7. 336 4. I in 4845
8. (a) tAo (b) 1~0 (c) 2l3o 5. 3 838 380
9. 210, 2 lo 6. About I in 950000
7. About I
8. 2598 960
9. 4, I in 649 740
1. (a) 120 (b) 56 (c) 120 (d) 5 10. Probability I in 25 approx.
2. (a) 720 (b) 120 II. 36, I in 72 193 approx.
3. 5! = 120 12. 48, 54 i4s
4. 6!, 7~0 13. 4512
5. 6! ways, 4! ways, 17 280 14. 4512, 58 656, if approx.
6. 3! = 6 15. 1947792
7. 6 16. I in 1947 792
8. 24 17. I in 9739 approx.
9. 6! 720 18. 80, I in 47980 approx.
10. 120, 12, 48 20. 28, 28, 28
11. (a) -fo (b) io 21. Price correct. Equivalent to 84 games.
12. t 22. yes. I in 18 278
13. 2 X 3! X 5! = 1440
14. (a) 10! (b) 5! X 3! X 2! X 3!
(c) 5! X 3! X 2 X 2
15. 72 I. 1050, 6 in 35
16. 6 2. 13 680
17. 4 3. 225, 90
18. 52 4. 2170, 1825
26c
19. 16 5. szcs'. z6cs'. __
52Cs
s

6. (a) 1 in 54145 (b) 1 in 117 approx.


(c) 1 in 25 approx. (d) I in 576 approx.
I. 360 2. 1260 (e) 1 in 24 approx.
3. 151200 4. 280 9
7. (a) C3 = 84
5. 10 6. 9 (c) 24
7. (a) I in 4 (b) 3 in 4 8. (a) 1 in 21 (b) 3 in 14
8. 3 in 26 3 (c) 2 in 7 (d) 5 in 42
9. 576
10. I in 2
II. 144
12. I in 12 I. 600
2. tooc6
13. (a) -f.r (c) ~ CdH
3. 120
4. (a) 18 in 35 (b) I in 70
5. 24,6
I. 56 6. (a) i (b) t
2. 35 7. 28
3. (a) 28 (b) 120 (c) 126 (d) 4 8. (a) 120 (b) 48 (c) 120
6 5 5
4. C4 = 6 Cz = 15; Cz = C3 = 10 9. 136
5. 78 10. 120, H
6. 12 11. n.
7. Choosing r objects, leaving (n - r) behind. It would be 12. ~
the same as choosing out the (n - r) objects and leaving 13. (a) 128 (b) 12
r behind. 14. 1~0
8. Choosing the total group can only be done in one way. 15. t
9. 126 16. t
10. 270 725, 27o1ns 18. 2n!
II. 52 C 13 = 635014000000 approx. n!n!
I in 635 014 000 000 approx. 19. Every combination of 8 things 6 at a time can be ar-
12. 35 ranged in 6! ways to give the number of permutations

310
28. J
x 1 -1, y = 2 or x = 6, y = 23
29. x =\6 or -6
I. I + 5x + IOx 2 + 10x 3 + 5x4 + x 5 3o. - 5t\ < x < 6t
2. a 7 + 7a 6 x + 21a 5 x 2 + 35a 4 x 3 + 35a 3 x 4 + 31. 94·75 m 2
21a 2 x 5 + 7ax 6 + x 7 32. 93
3. They form a symmetric pattern from each end 33. a 5, d = 3
4. x 6 + 6x 5 + 15x4 + 20x 3 + I5x 2 + 6x + 1 34. T50 = 348, S 50 = 8825
35. T9
36. x = 2, r = 3
37. 1830
I. (a) t 38. 20475
2. (a) m Cb) N6 Cc) 6N6
1
39. 31·25
3. (a) {24s (b) Ns 40. x = 2 or 3±
4. (a) Cn) 3 (b) 3 . n.. c-rt) 2 41, X= 3 ± 2J2

5. (a) (O·W (b) (0·9) 10 42. k -I


6. f2 43. (a) '} (b) 6 (c) n (d) t
7. Nis 44. x = ±2 or x = ±3
8. 0·92 approx. 45. k = 2.fr
9. (0·99) 9 X (0·1) 46. a> 0, ~ < 0
10. (a) (0·99) 10 (b) (0·99) 9 X (1·09) 47. ! <X< 2
II. 8
°
2 4 3 , 33 approx.
48. A= 2, B -9, C = I
12. Nl6 ~ o·5177 49. 4(x - 1) 2 + 2(x - I) + 3
13. 52 50. k = 12
14. 4 heads and 3 heads have equal probability of ,S4 3 ° 51. (a) 2 - x 3
6
(b) Sx(x
.2
+ 3) 3 ) -3
(c ( x _ !) 2
15. (a) -h (b) rt = t (c) fo = i (d) -ft 2
16. (a) 0·729 (b) 0·001 (c) 0·999
52. (a) 6x 2 - I6x + 10 x_
(b) _ _
.Jx 2
- 6
I
53. 5 + -2
X
54. 1; 20
55. b = -5
56. 5x+y - 2 = 0
I. -12 57. X+ 2y - 7= 0
2. (a) (x - 2)(x + I) (b) 5(x + 2y)(x - 2y) 58. a= I, b = 3, c -3
3. 2x- I 59. k =I
X- 2
60. II or -12
4. I= 2%4 61. 32
5. X= 15 62. H
6. X= -It 63. (0, 0) and ( -2, 2)
7. 1- X 64. /(2) = 13,/'(2) 20, /"(2) = 24
x-5 65. 108(3x 2) 2
8. (a) (5 - x)(2 + x) (b) (x + 2)(x - a) 66. x < -3 and x > I
9. 7J2 67. (2, -3), radius = 2
10. J3+I
68. y =
x3
3 -
3
2X
2
+X +C
II. (a) 6 ml-4 7 (b) 9 = ml-6 94
12. a= IIO,b = -60 69. f(x) = x 3
- x
2
+x +C
13. 2, )' = 3 I
14.
X=
x = 2 or -I!
70. y = 3x - x+ C
15. 111 = 5 71. y = 5 + 7x - 2x 2
16. a= ±5·22 72.5
17. -It<x<4t 73. x 2 = 12y
18. -4 <X< t 74. 2t
19. t 75. 15!
20. 78°28' 76. 57!
21. a= 5J2 em 77. 12f
22. (a) 2x 3y + 21 = 0 (b) 2x + 3y - I = 0 78. (!, -5~)
23. (a) ~ (b) n 79. X< I!
24. (a) ! (b)t 80. 29x + y - 13 = 0
-2x- 5 81. (0, 1), minimum; ( -f, 1}\r), maximum
25 2
' x + 5x + 6 12
26. -5
82. s = 13 - 2 +5
5)5- 3 x3
27. ___:__ _ 83. y = x + 2x 2 - ~ - 12
4

311
x3 x2 142. ! square unit
84. y = -
2
- -
2
+ 3x - 5
*
143. ~ 0·637 square units
144. (a) 40° (b) 70°
85, (a) 12 (b) !
86. 36 square units 146. AD= J9f
2
em~ 4·8 em
I
87. (a) 2x 3 + 2 + C (b) k{3x + 2) 3 + C 147. 144°
X
88. ~.J(6x + I? + C 148. 15
89. 2 square units 151. (a) 5 (b) 50 (c) -25
90. 1! square units 152. 1260
91. 21 square units 153. x 4 + 4x 2 + 6 + 4x- 2 + x- 4
92. 1·290 square units 154. a < 0, 1'1 < 0
93. --h 155. 0
94. JJ; cubic units ~ 8· 378 cubic units
156. a = 1, b = 1, c = 1
95. (a) 6x 8e 2x (b) ~ (c) ex(loge X + ~) 157. I! square units
96. 10·472 em 158. 119tm
97. 372 cm 2 160. ~or
1
J"
98. 49·7 em 161. k = 9
100. (a) 3 cos 3x (b) 2 sec 2 (2x 1) (c) 3 sinG- x) 162. X = 0·68

101. -sin xecosx 163. 67°10'


102. 2(tan x + x sec 2 x) 164. X= 0·90

103. -!cos (4x + 2) 165. n


104. n- 1 166. icm 3 /s
167. ~cubic units
105. 2 square units
dy sin x 168. 3600 ways
106. - = - - - = -tan x
dx cos x
171. soo = 1".,4·28 = 4-H
172. 265 ways
107. - 3 units
173. 15
108. I = 1 and 1 = 5
175. a= 10, x = 40 mmfs 2
109. 4 units/second
176. R = 4
110. I= 3
177. 12 em
111. 2 units/second/second
178. 4320
112. 82 metres
179. a= -3, (2x + l)(x + l)(x- 3)
113 . .X= -21 + 12
13 180. 3, -1 < X < 1
114. s = - - 21 2 181. 56, 55 and 56
2 2
115. k = 0·1116; 98000 182. -3.J3- r square units
4
116. 14 metres/second
184. t
117. 1
118. (0, 0), maximum; (2, -4), minimum 186. ¥
187. (a) 2 sin x cos x = sin 2x
119. k > !o
120. a = 3, b = -2, c = 9 (b) COS X + X sin X
cos 2 x
121. (3!, 4!)
122. k = 3 188. (a) 1 (b) log. 5 o!= 1-6
189. (a) 3" (b) r"
123. a < 0, 1'1 < 0
124. k = 1 190. (b) 71 °34'
125.k=8;15 191. x > 0, y >0, 3x + 2y- 6 < 0
126. x = 4 or 1 192. 1[-, ~~-
127. -1 <y < 1;3 193. (0, 32), ( -3, -49), (3, -49)
128. X= j! 194. i
129. -234 196. P(l, -2), Q(5, -10)
130. 8 197. -4
131. a= 4!; r = 2 199. 1~8
132. 21,27, 33,39,45 200 . .:: _ sin 2x + C
133. X - y + 2 = 0 2 4
134. Perpendicular distance from (0, 0) to 3x + 4y = 30 is 201. (a) ! log. (3x - 8) + c (b) !f + .J3 - 1
equal to the radius. 202. (a) 3 cos (3x + 4) (b) 1 + log.x - sin 2x
135. 2 units (c) x (d) X COS X - sin X
136. X y + 1= 0 .J9- ,,2 x2
137. x 2 = 24y 2
138. (0, -1) (e) (x + 1)2
139. x 2 = I6y ex
204. ( a ) - - (b) x 2y + 2 log. 2 = 0
140. (a) 4e4 x-l (b) 2x(tan 2x + x sec 2 2x) 1 +ex

141. (a) ! log. (3x- 2) +C ~


(b) ex
3
+C (c) (i) ~
1 - e2x
(ii) sin 2x . e' 1"
2
x

312
206. (a) (i) 1t (ii) n (ii) -¥, -¥; 2x 2 + 21x - 81 0
234. (a) 80 (b) 8064
(b) (i) ---=-!___ + c (ii) !(loge x) 3 +C 235. (a) 28 (b) 20 (c) 54
loge X
207. (a) x ~ I (b) X> 0 236. (b) sec 2 x
(c) -I ~ x ~ I (d) -1 ~ X ~ I 238. 0·16
(e) x > 0 239. 0·023
208. (a) i (b) constant = n 240. sst million lumens
209. 2n(l - ~) units 3 242. (a) -io (b) 0 (c) 2 60
210. (a) 2x cos 2x + sin 2x (b) 0·102 (d) 15do (e) fo-
211. (i) 30° (ii) 165° 244. (a) 2000(11 - n) (b) $104000
213. (b) sin- 1 (x 2 ) + c (c) 10
214. (a) 0-489 units 2 (b) -10·183 245. (a) 6
216. (b) {1, 2, -8} (b) (i) 18 m/s (ii) 12 mfs 2
217. (a) (x- 2)(x- I) (b) a= -25,b = 22 (c) 6x 2
218. 288n cm 3 246. (a) 24 360
220. (b) -1 (b) (i) 5 040 (ii) 210
(c) (c) 0·055
247. (a) 'J(2a 3 3a 2 + I) units 3
3 3
(b) (i) 24'
5
(ii) 2]4"
2
248. (a) 2tx y(t - I) - 21 = 0
249. (b) A = .J a 2 + b2 ; tan a ~;119·552°, 346·708°
(c) ~ a
250. (a) 2 sine cos e (b) (ii) 3·147 m
251. 3!, 4
252. (a) ir (b) 2
253. io
-1 3 254. (a) 2J2 + 2J5 (b) -8JS
255. 172125
256. 11·11
257. 3, I
258. 5 em
221. (a) !Cx 2 + 2) 2 + k (b) 2)4x 2 - I + k 259. 3 em
(c) 2.J1 + sin x + k (d) -tcJ - sin x) 5 + k 260. !2°
222. (a) 0·309 (b) 4; 0·360 261. (a) parallelogram
223. x- 4; 9; 4- 4x + 9ioge (x + 2) + c (b) cyclic quad
224. (a) X = COS 8, )' = sin 8 (c) rhombus
263. (a) +,;
225. 2; (x + 2)(x - 3)(2x + 1)
226. (b) 70°
228. (a) 0; i (b) (0, 0); (1, -i); 100; -¥
264.
265.
*
14 em, .J76
em
CbH

(c) y 266. 96 cm 2
267. 0, 2n, i,-¥
269. -4
270. x ~ -4 or x ~ - I
271. 2/3 em ~ 3-46 em
2n.R
273. 14
X
274. -2a sin 2t - 3b cos 3t
275. 3x 2 cos x - x 3 sin x
276. (a) -1 cos 3x + C (b) -1e-2x
277. (a) ! log (4 + 6x) (b) sin- 1 ~
278. sin x + cos x + C
1!2
280. - - - I
8
281. (a) 2 (b) e" (c) a
-4 2
282. (a) x (b) 16(2x- 1) 7
-5
283. log 2
229. (a) x + 2y - 4 =0 (b) 3x - y +9 = 0
(d) (-2,3)
1 (b) -X sin X - COS X
(c) y = 3 284. (a) - - --
)-x2- x x2
230. (a) -ll
231. (a) 0·771 units 2 (b) 0·785 units 2 ; 1·78% 285. ~
232. (a) ~; 3·785, 5·640 (b) n; 1·288 286. (x 2 + 2x)eX, (x 2 + 4x + 2)ex
233. (a) H; -1 ~ k ~
(b) (i) -±;-1
1 (0, 0) min.; ( -2, ~)max.
313
287. x 2 = !6y
288. 1·0828, other terms may effect 4th place only.
289. 84 I. (a) a"+ "C1 a"- 1 b + "C2 a"- 2 b2 ; "C,a"-'b'
290. 140,}2 (b) 2"C,( -I)"
291. (b), (c), (e) -6
292. II! 2. (a) (I + x)' (b) sin 2x + 2x cos 2x
3
293. 30 possibilities 3. (a) I (b) ! log, 4
294. 21 4. (a) (x - 1) 2 = (y + I)
295. Nis (b) maximum at (0, 12), minimum at (4, -20), point
296. (0, I); (0, I!) of inflexion at (2, - 4)
297. 20m (c) 3-322
298. 4 sec, 320)3 m 5.x=2·!3
300. $1000(1·1) 40 = $45 259

I. m = -t
2. (a) 15 m/s, 27 mfs (b) I = 3
(c) 18 m/s 2 ; when t = 3 (d) 108m
3. X - 2y- 2 = 0
4. (a) 4 (b) log, 2 ~ 0·69
I. (a) 4100 (b) 16000 5. (a) A = 100, k ~ 0·366 (b) 820 approximately
2. v = 31 2 121 + 9 6. 78 units per second
1 = I or t = 3 ; s = 4 or s 0
3. x 2 + 2x + 4 = (x + 1) 2 + 3; minimum value of 3
when x = -I
-H
4.
5. (a) P(m) = 0
I. (3> 9~ 2)
-5 + ,j2T 2 tanB=~
(b) x = 3 or x = -; · e2 - I
3. (a) x = 5, .\: = 3, x = 9 (b) 9x- 36
6. 907 cm 2
4. $55 765
6. maximum at (1, 9)
minimum at (- 1, 5)
inflexion at (0, 7)
I. (a) It square units (b) ~3f cubic units
2. a"+ "C1 a"- 1 b + "C 2 a"- 2 b 2 + · · · + "C,a"-'b' + · · ·
+b";ll20
3. r -3, T3 = 18 I. 85°
4. "P, = _,_z!_ 3. (a) b 2 c- 2a 2 c = abd
(11- r)! (b) (i) -tan X (ii) cos x. e' 1"x
1440 arrangements 4. (a) -t (b) 2 - 4x
5. 3pq- p 3 5. k = 2, (x - l)(x + 2)(3x - 1)(3x - 2)
6. A diameter which is perpendicular to the parallel chords 6. (a) a= 2or4 (b) fa
(a) (2.}3 + j7) ~ 6·11 em or
(b) (2.}3 - /7) ~ 0·82 em

I. (b) -1920
2. minimum at (3, 6),
I. nt or 55-fs maximum at ( -3, -6),
2. "C,x'; -72 no inflexion,
3. 4 em, 6 em, 4 em, 6 em, 30°, 150°, 30°, 150° asymptotes at x = 0 and y = x
4. 18 3. (a) 0·187 (b) %
5. 15 4. (a) 2 + .j3 (b) tan A + tan B
6. (a) 60°, 120°, 240°, 300° (b) 30°, 210° 5. (a) fr (b) # (c) t
6. (a) 4n cubic units

I. (a) 5! (b) 108m


2. 4· 5 n cubic units I. (a) I (b)i
3. (b) JT3:
3 2. (b) 1·155
4. 2-3026
3. (a) y = ix!- 2x (b) ~units
3(3"- I) 17y
5. 2 ;9
4. (a) 2 2x- 3
2x + 2
(b) sec 4 x
6. 26°34', 71 °34', 206°34', 251 °34' x - 3x + 6 x 2 +2x-5

314

5. (1 + x) [1 - (t : x)J 2. (b) 4·284
4. (b) (i) AZ = AY; BZ = BX; CX = CY (ii) :, -a

If x is positive _x- < I. Therefore the limiting sum 5. (b) k =


6
+3 211 (c) 673 596
1+x
is I+ x
2
6. (a) (i) 2 cos fJ - I (ii) 1 - 2 sin 2 fJ
6. (a) T 16 and T17 2-,[3 n
(b) x 3 + 2x 2 + 3x + I = 0
(b) - 2 - +6

I. (a) (i) -2 sin 2x (ii) -2 sin 2x;f(x) =g(x)


I. (a) J} (b) 0
· A b ' A 2. (a) 0·082 (b) 2; 0·329 (c) 0·099
(b) BL =
c sm 2 . LC = sm 2
sin LALB' sin LALC 4. (a) all realm; (__!_,
2m 4m
~)
2. (a) 1 + 6x + 15x 2 + 20x 3 + 15x4 + 6x 5 + x 6
(b) 1 + 5x + IOx 2 + 10x 3 + 5x4 + x 5
(b) (i) X + )'I - 13 - 21 = 0
(ii) 14 + 41 2 - 121 + 9 = 0
(c) + a b c
3. (a) -I (c) 3, 4, 5 (d) -i 5
' (a) sin A = sin B = sin C
e
5. (a) (i) sec 2 = 1 + tan 2 e
(b) (i) 11·3° (ii) 30° (iii) 66 m (iv) 13-34 m
6. (a) 4 m; 10m
(c) (i) 10; 4-82; 30; 23-6; 7·23 (ii) 61·4 m 3
1
I. (a) - - - + c (b) -)16 - x 2 + c
1+ JX
(c) t
I. (b) ft (c) 450 em
2. (b) (i) 1·61051 (ii) $3052·55 (b) 140°
2. (a) 65°
3. (a) (i) i (ii) t (iii) -~
3. t,{~, 2), 2 : I
(iv) t; 9x 2 + 2x + I = 0 4
(b) x 3 + 6x 2 + 9x + 20; 41 4. (a) (x + 2)2
21 1 - 12 21
4 · (a) 1 + 12 ; 1 + 12 ; 1 - 12
(c) inflexions at (f, f), (¥-, ¥).
2
Note that y' = cos 2x + I is never negative
(b) sec f = 1 + tan f
2
5. (b) 18 C6 Ct) 12
(e) 0·215 (ii) after 4 seconds
6. (a) (i) 6! units
b2 + c2 - a2
5. (a) cos A = - - - - V 2 sin 2o:
2bc (b)---
g
(b) cos 2 A = 1 + cos 2A
2
(c) 2 cos 2 1 = 1 + cos A
6. (a) 0 (b) (i) 60° (ii) ,fix - y - 2 = 0 I. (a) y = 2x + 1
(b) (i) -6 (ii) i (iii) -5
2. (a) (i) 90° (ii) 20°
(iii) 64·26 cm2 , 18·79 em, 6·84 em
I. (a) (i) J2 - 1 (ii) 4 (iii) t (b) 100°
(b) %(x + l)t - t(x + I)~ + c 3. (a) ( -2, 21) max; (1, -6) min
2. (b) {1, 4} (c) 4 (c) 0·09
3. (a) "Ckx 2 k-n (b) (i) "C ~ (ii) W 4. (a) -1 ~x~ 1; -f~y~f
4. (b) (i) diameter (c) 0; sin- 1 x + cos- 1 x = fwhich is a constant
5. (a) - 2 ~ 9 (c) 0·917, -0·113, -0·804 (d) tan- 1 x; t - log 2 t
6. (b) 13! m2 5. (a) (i) -h. (ii) 1\,- (iii) t
(b) 4 H and 3 H equally likely
6. (a) k ;;. 12 or k ~ -12

I. (a) (i) ~ (ii) f.-


(b) y = 6x (c) ~ units 2
I. (a) (i) -48x2 (1 - 4x 3 ) 3
(ii) COS X
2)1 +sin x
(iii)~2
I +x
(b) X - y + I = 0
2. (a) (8, 6) (b) 4J2 ~ 5·7
(c) 75°58', 104°2' (d) (4, 3)
3. (a) 16 em (b) 19·2 em

315
4. (a) (4, 9), ( -3, 16) (b) 571; square units 5. (a) (i) fr, (ii) 49150 (iii) 4Wo
2
(c) ~ cubic units (b) 12c3(4a)9( -5W, -55 x 89 x 53
2
5. 300m by 150m 6. V = ,)1025, a= tan- 1 0·8, 20m
6. (a) ! (b) 1s (c) to (d) -fa

I. (a) (i) 10x(x2 + 3) 4 (ii) ex(2 cos 2x + sin 2x)


...) I -log x (b) 2e-x sin X
(111 2

~: 2 ~log (2x 2 +
X
I. (a) x 4 + +2 (b) I)+ C
2. (a) 2 (b) !e 2 + !
2.
(4n, J2I ) , 2 - y R2 ·
square umts; (n4 - 2n) cu b'1c umts·
2
3 a!_!_ y'3 _!_ J2+J6
. ( ) 2' J2' 2 'J2' 4
3. (a) 90° (b) 30·47 em, 52·36 cm 2
4. (a) (3, -23) min. (-I, 9) max. 5. (a) (i) i (ii) ft
(c) (1, -7) (d) 0-4 (b) (i) ! (ii) ! (iii) i (iv) i

316

You might also like